67% found this document useful (9 votes)
8K views291 pages

Krishna's Question Bank Analytical Geometry

This document contains solutions to 4 problems related to polar equations of conics. Problem 1 proves that the reciprocal of the product of distances from the foci to points on perpendicular focal chords is constant for any conic. Problem 2 shows that the perpendicular focal chords of a rectangular hyperbola are equal. Problem 3 expresses the sum of reciprocals of distances from a focus to points on a chord subtending a right angle at the focus in terms of the eccentricity. Problem 4 proves that the locus of midpoints of all focal chords of a conic is a conic of the same type.

Uploaded by

ijoky
Copyright
© © All Rights Reserved
We take content rights seriously. If you suspect this is your content, claim it here.
Available Formats
Download as PDF, TXT or read online on Scribd
67% found this document useful (9 votes)
8K views291 pages

Krishna's Question Bank Analytical Geometry

This document contains solutions to 4 problems related to polar equations of conics. Problem 1 proves that the reciprocal of the product of distances from the foci to points on perpendicular focal chords is constant for any conic. Problem 2 shows that the perpendicular focal chords of a rectangular hyperbola are equal. Problem 3 expresses the sum of reciprocals of distances from a focus to points on a chord subtending a right angle at the focus in terms of the eccentricity. Problem 4 proves that the locus of midpoints of all focal chords of a conic is a conic of the same type.

Uploaded by

ijoky
Copyright
© © All Rights Reserved
We take content rights seriously. If you suspect this is your content, claim it here.
Available Formats
Download as PDF, TXT or read online on Scribd
You are on page 1/ 291

is h na's

Kr uestion
Bank

Analytical Geometry
(For B.A. and B.Sc. I year students of All Colleges affiliated to Allahabad State University)

As per Allahabad State University Syllabus


(w.e.f. 2017-2018)

By

A. R. Vasishtha Hemlata Vasishtha


Retired Head, Dep’t. of Mathematics M.Sc. (Gold Medalist), Ph.D.
Meerut College, Meerut (U.P.) C.C.S. University, Meerut (U.P.)

KRISHNA Prakashan Media (P) Ltd.


KRISHNA HOUSE, 11, Shivaji Road, Meerut-250 001 (U.P.), India
Jai Shri Radhey Shyam

Dedicated
to
Lord

Krishna
Authors & Publishers
B rief C ontents
Dedication..............................................................(III)
Brief Contents........................................................(iv)

Chapter-1: Polar Equation of a Conic...................................(G-01—G-32)

Chapter-2: Systems of Co-ordinates......................................(G-33—G-39)

Chapter-3: Direction Cosines and Projections.................(G-40—G-50)

Chapter-4: The Plane..................................................................(G-51—G-73)

Chapter-5: The Straight Line.................................................(G-74—G-132)

Chapter-6: The Sphere..........................................................(G-133—G-166)

Chapter-7: The Cone..............................................................(G-167—G-195)

Chapter-8: The Cylinder........................................................(G-196-G-205)

Chapter-9: The Central Conicoids...................................(G-206—G-235)

Chapter-10: The Paraboloids...............................................(G-236—G-243)

Chapter-11: Generating Lines.............................................(G-244—G-256)

Chapter-12: The Plane Sections of Conicoids.................(G-257—G-277)

Chapter-13: Reduction of General Equation of


Second Degree.................................................(G-278—G-288)

(iv)
Krishna's

ANALYTICAL GEOMETRY
C hapters

1. Polar Equation of a Conic


1.

1. Systems of Co-ordinates
2.

1. Direction Cosines and Projections


3.

1. The Plane
4.
1. The Straight Line
5.

1. The Sphere
6.

1. The Cone
7.

1. The Cylinder
8.

1. The Central Conicoids


9.

1. The Paraboloids
10.

11. Generating Lines


10.

1. The Plane Sections of Conicoids


12.

1.
13. Reduction of General Equation of
Second Degree
Chapter-1 G-3

Polar Equation of a Conic

Comprehensive Problems 1
Problem 1: If PSP ′ and QSQ ′ be two perpendicular focal chords of a conic, prove that
1 1
+ is constant.
SP . SP ′ SQ . SQ ′
(Meerut 2004, 06B; Rohilkhand 06; Kanpur 07, 16;
Kumaun 09, 13, 15, 16; Bundelkhand 14; Kashi 14)
Solution: Proceeding as in Example 2, we have for the focal chord PSP ′,
1 1 + e cos α 1 − e cos α 1 − e2 cos2 α
= ⋅ = …(1)
SP . SP ′ l l l2
1
Replacing SP by SQ , SP ′ by SQ ′ and α by π + α in (1), we have for the focal chord
2
QSQ′,
1 − e2 cos2  π + α 
1
1 2  1 − e 2 sin2 α
= = …(2)
SQ . SQ ′ l2
l2
Adding (1) and (2), we have
1 1 2 − e2
+ = which is constant.
SP . SP ′ SQ . SQ ′ l2
Problem 2: Prove that the perpendicular focal chords of a rectangular hyperbola are equal.
Solution: Let PSP ′ and QSQ ′ be two perpendicular focal chords. Then proceeding as
in Example 2 part (iii), we have
2l 2l
PP ′ = , QQ ′ = ⋅
1 − e2 cos2 α 1 − e2 sin2 α
[Give complete proof here]
In the case of a rectangular hyperbola, e = 2.
2l 2l 2l
∴ PP ′ = =− = (in magnitude),
2
1 − 2 cos α cos 2α cos 2α
2l 2l
and QQ ′ = = ⋅
1 − 2 sin2 α cos 2α
∴ PP ′ = QQ ′.
Problem 3: A chord PQ of a conic whose eccentricity is e and semi-latus rectum l subtends a
right angle at the focus S, show that
2 2 2
 1 − 1 +  1 − 1 = e ⋅
   
 SP l   SQ l  l2
(Kumaun 2012)
G-4

Solution: Let the equation of the conic be


l / r = 1 + e cos θ. …(1)
It is given that the chord PQ subtends a right angle at the focus S. Therefore if the
1
vectorial angle of P is α, then that of Q is π + α.
2
l
∴ = 1 + e cos α
SP
= 1 + e cos  π + α 
l 1
and
SQ 2 
= 1 − e sin α.

2 2
∴  1 − 1 +  1 − 1
   
 SP l   SQ l 
2 2
1 + e cos α 1 1 − e sin α 1
=  −  +  − 
 l l   l l
2 2
e cos α   − e sin α 
=   +  
 l   l 
e2 e2
= (cos2 α + sin2 α ) = ⋅ Proved
2
l l2
Problem 4: PSP ′ is a focal chord of a conic. Prove that the locus of its middle point is a conic of
the same kind as the original conic.
Solution: Let the equation of the conic be
1 / r = 1 + e cos θ. …(1)
Let the vectorial angle of P be α, then that of P ′ is π + α.
Since P, P ′ lie on (1), therefore
l / SP = 1 + e cos α ,
and l / SP ′ = 1 − e cos α.
Suppose M is the mid-point of PP ′.
If the polar coordinates of M are ( r1, α ), then
1
r1 = SM = SP − MP = SP − ( SP + SP ′ )
2
1 1 l l 
= ( SP − SP ′ ) =  − 
2 2  1 + e cos α 1 − e cos α 
− le cos α
= ⋅
1 − e2 cos2 α
Generalizing i. e., replacing r1 by r and α by θ, the locus of the point M ( r1, α ) is
r (1 − e2 cos2 θ) = − le cos θ. …(2)
The equation (2) represents a conic as can be easily seen by changing it to cartesians.
Multiplying (2) by r, we get
r2 − e2 ( r cos θ)2 + le ( r cos θ) = 0 …(3)
G-5

Changing (3) to cartesian coordinates, we have


x2 + y2 − e2 x2 + le . x = 0
or x2 (1 − e2 ) + y2 + lex = 0, …(4)
which is an equation of second degree in x and y and so it is the equation of a conic.
Now we are to prove that (1) and (4) are conics of the same kind.
We consider the following cases :
Case I: If e = 1 i. e., the conic (1) is a parabola, the equation (4) becomes
y2 + lx = 0 or y2 = − lx
which is also a parabola.
Cases II and III: The equation (4) may be written as
2
 le  y2 l2 e2
x +  + =
 2 (1 − e2 )  1− e2
4 (1 − e2 )2
which is an ellipse or a hyperbola according as e < 1 or > 1 i. e., according as the conic (1) is
an ellipse or a hyperbola.
Remark: Cases I, II and III can also be proved as follows :
Comparing (4) with the general equation of second degree
ax2 + 2hxy + by2 + 2 gx + 2 f y + c = 0, …(5)
we have a = 1 − e2, h = 0, b = 1.
We know that (5) represents an ellipse or a parabola or a hyperbola according as
h2 − ab < or = or > 0.
For the equation (4), we have
h2 − ab = 0 − 1 (1 − e2 ) = e2 − 1.
∴ The equation (4) represents an ellipse, a parabola or a hyperbola according as
e2 − 1 < or = or > 0 i. e., according as e < 1 or = 1 or > 1 which is also the condition for
the conic (1) to represent an ellipse, a parabola or a hyperbola. Hence the equations (1)
and (4) represent conics of the same kind.

Problem 5: A circle passing through the focus of a conic whose latus rectum is 2l meets the conic
in four points whose distances from the focus are r1, r2 , r3 and r4 , prove that
1 1 1 1 2
+ + + = ⋅
r1 r2 r3 r4 l

Solution: Proceed as in problem 8 of this Problem Set and get the equation (3). The
four values of r given by the equation (3) are the focal distances of the four points of
intersection of the circle and the conic.
Let the four values of r given by (3) be r1, r2 , r3 and r4 .
the coeff. of r in (3)
Then, r2 r3 r4 + r1r3 r4 + r1r2 r4 + r1r2 r3 = − …(4)
the coeff. of r4 in (3)
constant term
and r1r2 r3 r4 = ⋅ …(5)
coeff. of r4
G-6

Dividing (4) by (5), we get


1 1 1 1 coeff. of r 8a2 l 2
+ + + =− = = ⋅ Proved
r1 r2 r3 r4 constant term 4a2 l 2 l
Problem 6: If the circle r + 2a cos θ = 0 cuts the conic l / r = 1 + e cos (θ − α ) in four points,
find the equation in r which determines the distances of these four points from the pole. Show that if
the algebraic sum of these four distances is equal to 2a, the eccentricity is equal to 2 cos α.
Solution: The equation of the conic is
l / r = 1 + e cos (θ − α )
or l / r = 1 + e cos α cos θ + e sin θ sin α …(1)
and the equation of the circle is
r = − 2a cos θ. …(2)
The radii-vectors of the points of intersection of (1) and (2) are given by
 2 
 l − 1 − e cos α  − r   = e sin α  1 − r  
    
 r  2a    4a2  
  
 4a − r2 
2
or [2al − 2ar + er2 cos α ]2 = 4r2 a2 e2 sin2 α  

2
 4a 
or r4 ( e2 cos2 α + e2 sin2 α ) − 4ae cos α r3
+ r2 (4a2 − 4e2 sin2 α + 4ael cos α ) − 8a2 lr + 4a2 l2 = 0
or e2 r4 − 4ae cos α . r3 + r2 (4a2 − 4e2 sin2 α + 4ael cos α )
− 8a2 lr + 4a2 l2 = 0 …(3)
The equation (3) gives the distances of the four points of intersection of (1) and (2)
from the pole. Let these distances be r1, r2, r3 and r4 .
4ae cos α 4a cos α
Then r1 + r2 + r3 + r4 = = ⋅
e2 e
Now by question, the algebraic sum of these four focal distances r1, r2 , r3 and r4 is 2a.
Hence 2a = (4a cos α ) / e, or e = 2 cos α.
Problem 7: Show that the equation of the directrix of the conic l / r = 1 + e cos θ corresponding
l 1− e2
to the focus other than the pole is =− e cos θ.
r 1+ e2 (Kumaun 2013)
Solution: Let the focus S be taken as the
pole and the axis SZ as the initial line SX.
Let Z ′ M ′ be the directrix corresponding to
the focus S′ (other than the pole S).
We are required to find the equation of
Z ′ M ′.
Consider a point P with coordinates ( r, θ)on
Z ′ M ′,
so that SP = r, ∠ ZSP = θ
and ∠ Z ′ SP = π − θ.
G-7

We have ZZ ′ = 2a / e and SZ = l / e.
b2 a2 (1 − e 2 ) l
Also l= = = a (1 − e 2 ), so that a = ⋅
a a 1− e2
2l
∴ ZZ ′ = ⋅
e (1 − e 2 )
2l l l 1+ e2
Now SZ ′ = ZZ ′ − SZ = − = ⋅ ⋅
2
e (1 − e ) e e 1− e2
Also SZ ′ = SP cos ( π − θ), from ∆ Z ′ SP.
l 1+ e2 l 1− e2
∴ ⋅ = − r cos θ or =− e cos θ.
2
e 1− e r 1+ e2
This is the required equation of the directrix Z ′ M ′.
We have derived the equation of the directrix Z ′ M ′ considering the conic to be an
ellipse. In a similar way we can derive the equation if the conic is a hyperbola.
Problem 8: A circle of given radius passing through the focus S of a given conic intersects it in A,
B, C, D; show that SA . SB . SC . SD is constant. (Kumaun 2012)
Solution: Referred to the focus S as the pole and the axis as the initial line let the
equation of the conic be
l / r = 1 + e cos θ. …(1)
The equation of any circle of given radius a and passing through the pole (i. e., the focus)
is
r = 2a cos (θ − θ1 )
or r = 2a cos θ cos θ1 + 2a sin θ sin θ1. …(2)
The focal distances of the points of intersection where these meet are given by
eliminating θ between (1) and (2). From (1),
cos θ = ( l − r ) /( re ).
Putting it in (2), we get
l−r   l− r
2

r = 2a cos θ1 + 2a sin θ1 1 −   
re  re  
2
 r − 2a  l − r  2 2  ( l − r )2 
or    cos θ1 = 4a sin θ1 1 − 2 2 
  re    r e 
or e 2 r4 + 4ear3 cos θ1 + 4ar2 ( a − el cos θ1 − e2 asin2 θ1 )
− 8a2 lr + 4a2 l 2 = 0. …(3)
The equation (3) is of degree four in r and so it gives four values of r. These four values of r
are SA, SB, SC and SD i. e., the radii vectors of the four points of intersection of (1) and
(2).
the constant term in (3) 4a2 l 2
∴ SA . SB . SC . SD = =
the coefficient of r4 in (3) e2
which is constant i. e., independent of θ1.
G-8

Comprehensive Problems 2
Problem 1: Prove that the condition that the line l / r = A cos θ + B sin θ
may touch the conic l / r = 1 + e cos (θ − α ) is
A2 + B2 − 2 e ( A cos α + B sin α ) + e2 − 1 = 0.
(Meerut 2009, 10B; Avadh 13)
Solution: Suppose the given line
l / r = A cos θ + B sin θ …(1)
is a tangent to the given conic at the point whose vectorial angle is β. Then (1) should be
identical with the tangent to the given conic at the point β whose equation is
l / r = cos (θ − β ) + e cos (θ − α )
or l / r = (cos β + e cos α ) cos θ + (sin β + e sin α ) sin θ …(2)
Comparing (1) and (2), we have
cos β + e cos α sin β + e sin α
1= =
A B
or A − e cos α = cos β, and B − e sin α = sin β.
Squaring and adding, we get
( A − e cos α )2 + ( B − e sin α )2 = 1
or A2 + B2 − 2e ( A cos α + B sin α ) + e2 − 1 = 0,
as the required condition.
l
Problem 2: Prove that the line = cos (θ − α ) + e cos (θ − γ ) is the tangent to the conic
r
l
= 1 + e cos (θ − γ ) at the point for which θ = α.
r
Solution: The line intersects the conic where cos (θ − α ) = 1
or θ − α = 2nπ, where n is any integer
or θ = α + 2nπ.
For every integral value of n, θ = α + 2nπ represents the same point on the conic whose
vectorial angle is α. Thus the given line meets the conic only at the point α and hence is a
tangent to the conic at the point for which θ = α.
Problem 3: Show that the two conics l1 / r = 1 + e1 cos θ and l2 / r = 1 + e2 cos (θ − α )

will touch one another if l12 (1 − e22 ) + l22 (1 − e12 ) = 2l1l2 (1 − e1e2 cos α ).
(Bundelkhand 2006, 07, 08; Purvanchal 07,10)
Solution: Let the given conics touch one another at the point whose vectorial angle is β.
The equations of the tangents at the common point ‘β’ to the two conics are
l1 / r = cos (θ − β ) + e1 cos θ, …(1)
and l2 / r = cos (θ − β ) + e2 cos (θ − α ). …(2)
The equations (1) and (2) may be written as
l1 / r = (cos β + e1 ) cos θ + sin β sin θ, …(3)
and l2 / r = (cos β + e2 cos α ) cos θ + (sin β + e2 sin α ) sin θ. …(4)
G-9

The equations (3) and (4) should be identical. Hence comparing them, we have
l1 cos β + e1 sin β
= =
l2 cos β + e2 cos α (sin β + e2 sin α )
or ( l1 − l2 ) cos β = − l1e2 cos α + l2 e1,
and ( l1 − l2 ) sin β = l1e2 sin α.
Squaring and adding, we get
( l1 − l2 )2 = l12 e22 + l22 e12 − 2l1l2 e1e2 cos α

or l12 (1 − e22 ) + l22 (1 − e12 ) = 2l1l2 (1 − e1e2 cos α ),


as the required condition.
Problem 4: PSP′ is a focal chord of a conic; prove that the angle between the tangents at P and
 2 e sin α 
P ′ is tan −1   where α is the angle between the chord and the major axis.
2 
 1− e 
(Meerut 2005, 06; Kanpur 06, 08, 14; Purvanchal 07, 13)
Solution: Referred to the focus S as the pole and the axis as the initial line, let the
equation of the conic be
l / r = 1 + e cos θ. …(1)
Since the focal chord PSP ′ is inclined at an angle α to the major axis, therefore if the
vectorial angle of P is α then that of P ′ is π + α.
The tangent at P is
l / r = cos (θ − α ) + e cos θ
or l = r cos θ cos α + r sin θ sin α + er cos θ
or l = x cos α + y sin α + ex [Changing to Cartesians]
or l = (cos α + e ) x + y sin α.
∴ m1 = the slope of the tangent at P
cos α + e
=− ⋅
sin α
Similarly, m2 = the slope of the tangent at P ′
cos (α + π ) + e
=−
sin (α + π )
e − cos α e − cos α
=− = ⋅
− sin α sin α
Now if β is the angle between the tangents at P and P ′, then
m ~ m2 ( e − cos α ) /sin α + ( e + cos α ) /sin α
tan β = 1 =
1 + m1 m2 1 + {( e − cos α ) /sin α} { − ( e + cos α ) /sin α }
2e sin α 2e sin α
= = ⋅
sin2 α − ( e2 − cos2 α ) 1 − e2
2e sin α 
∴ β = tan−1  ⋅
 1 − e2 
G-10

Problem 5: Show that the locus of the point of intersection of two tangents to the parabola
l / r = 1 + cos θ, which cut one another at a constant angle α is the hyperbola, l / r = cos α + cos θ.
(Rohilkhand 2007)
Solution: The given parabola is
l / r = 1 + cos θ. …(1)
Consider two points β and γ on (1). The tangents at these points are
l / r = cos (θ − β ) + cos θ …(2)
or l = (cos β + 1) x + y sin β …(2′)
and l / r = cos (θ − γ ) + cos θ …(3)
or l = (cos γ + 1) x + y sin γ. …(3′)
cos β + 1
∴ m1 = the slope of (2′) = −
sin β
1
2 cos2 β
2 1
=− = − cot β
1 1 2
2 sin β cos β
2 2
cos γ + 1 1
and m2 = the slope of (3′) = − = − cot γ.
sin γ 2
It is given that the angle between the tangents (2) and (3) is α. Therefore using the
m − m2
formula tan α = 1 , we have
1 + m1m2
1 1
− cot β + cot γ
tan α = 2 2 = tan  β − γ
⋅
1
1 + cot β cot γ
1  2 
2 2
∴ α = ( β − γ ) / 2. …(4)
Now to find the locus of the point of intersection of the tangents (2) and (3), we have to
eliminate β and γ between (2), (3) and (4). Equating the values of l / r from (2) and (3),
we have
cos (θ − β ) + cos θ = cos (θ − γ ) + cos θ,
or cos (θ − β ) = cos (θ − γ ) or θ − β = − (θ − γ )
[∵ If θ − β = θ − γ , then β = γ which is not so]
1
or 2θ = β + γ or θ = ( β + γ ). …(5)
2
Adding (4) and (5), we get
θ + α = β.
Putting β = θ + α in (2), we get
l / r = cos α + cos θ, …(6)
which is the required locus of the point of intersection of the tangents (2) and (3).
The equation (6) may be written as ( l sec α ) / r = 1 + sec α cos θ which is the equation of
a conic whose eccentricity is sec α. Since sec α > 1, therefore the conic is a hyperbola.
G-11

Problem 6: Prove that the portion of the tangent intercepted between the conic and the directrix
subtends a right angle at the corresponding focus.
or
Let the tangent at any point P on a conic whose focus is S meet the directrix in K, show that the
angle PSK is a right angle.
Solution: Let the equation of the conic referred to the focus S as the pole be
l / r = 1 + e cos θ. …(1)
The equation of the directrix corresponding to the focus S is
l / r = e cos θ. …(2)
Let the vectorial angle of any point P on the conic be α. The tangent at P is
l / r = cos (θ − α ) + e cos θ. …(3)
Now the vectorial angle θ of the point of intersection K of the tangent (3) and the
directrix (2) is given by
cos (θ − α ) + e cos θ = e cos θ
[Obtained on eliminating r between (2) and (3)]
or cos (θ − α ) = 0.
∴ θ ~ α = 90 ° i. e., ∠ PSK = 90 ° .

Problem 7: PSP ′ is a focal chord of the conic. Prove that the tangents at P and P ′ intersect on
the directrix.
Solution: Let the equation of the conic be l / r = 1 + e cos θ.
Suppose the focal chord PSP ′ is inclined at an angle α to the initial line so that the
vectorial angles of P and P ′ are α and π + α respectively. The tangents at P and P ′ are
l / r = cos (θ − α ) + e cos θ …(1)
and l / r = cos {θ − ( π + α )} + e cos θ
or l / r = − cos (θ − α ) + e cos θ. …(2)
The locus of the point of intersection of the tangents (1) and (2) is obtained by
eliminating α between (1) and (2).
Adding (1) and (2), we get
2 l / r = 2 e cos θ i. e., l / r = e cos θ
which is the equation of the directrix. Hence the tangents (1) and (2) intersect on the
directrix.
Problem 8: Two conics have a common focus; prove that two of their common chords pass
through the intersection of their directrices.
Solution: Let the equations of the two conics having a common focus be
l / r = 1 + e1 cos θ, …(1)
and L / r = 1 + e2 cos (θ − α ). …(2)
The directrices of (1) and (2) respectively are
l / r = e1 cos θ, L / r = e2 cos (θ − α ).
Now changing (1) to cartesians, we have
G-12

l = √ ( x2 + y2 ) + e1 x
or ( l − e1 x )2 = ( x2 + y2 ),
making the equation rational.
Again on transforming to polars, we have
( l − e1 r cos θ)2 = r2
2
 l − e cos θ − 1 = 0.
or  1  …(3)
 r 
The equation (3) is thus the polar equation of the conic (1) put in the form which when
transformed to cartesians gives rational cartesian equation of the conic.
Similarly the equation (2) can be written as
2
 L − e cos (θ − α ) − 1 = 0. …(4)
 2 
r 
Now any curve passing through the points of intersection of the two conics (3) and (4) is
given by
 l 
2   L 2 
 − e1 cos θ − 1 + λ   − e2 cos (θ − α ) − 1 = 0. …(5)
 r    r  
Clearly if λ = − 1, (5) gives two lines, namely
− e1 cos θ = ±  − e2 cos (θ − α ) ,
l L
r r 
which clearly pass through the point of intersection of the two directrices
l / r − e1 cos θ = 0 and L / r − e2 cos (θ − α ) = 0.
Since the straight lines passing through the points of intersection of the conics (1) and
(2) are their common chords, therefore the common chords of (1) and (2) pass through
the intersection of their directrices.

Problem 9: If the tangent at any point of an ellipse makes an angle α with its major axis and an
angle β with the focal radius to the point of contact, show that e cos α = cos β.
Solution: Let the equation of an ellipse with focus S as pole be l / r = 1 + e cos θ.
Suppose the vectorial angle of any
point P on the ellipse
l / r = 1 + e cos θ is φ. The tangent at
the point ‘φ’ is
l / r = cos (θ − φ) + e cos θ
or l = (cos φ + e ) x + sin φ y.
…(1)
Since (1) makes an angle α with the
major axis i. e., with the initial line,
therefore the slope of (1) is tan α.
G-13

cos φ + e
∴ tan α = − ,
sin φ
sin α cos φ + e
or =−
cos α sin φ
or sin α sin φ = − cos α cos φ − e cos α
or cos (α − φ) = − e cos α
or cos β = − e cos α …(2)
[From the figure, φ + β = α]
If we consider the focal radius through the other focus i. e., if we take l / r = 1 − e cos θ as
the equation of the ellipse, we shall get
cos β = e cos α. …(3)
In view of (2) and (3), we have in either case
e2 cos2 α = cos2 β.
Problem 10: QR, a chord of the conic l / r = 1 − e cos θ, subtends a constant angle 2α at its
focus S, and SP, the bisector of the angle QSR , meets QR in P. Show that the locus of P is the conic
( l cos α ) / r = 1 − e cos α cos θ.
Solution: The given conic is
l / r = 1 − e cos θ, …(1)
the focus S being at the pole.
A chord QR of the conic (1) subtends a
constant mangle 2α at the focus S i. e.,
∠ QSR = 2α. The bisector of the angle QSR
meets QR in P ; therefore
∠ QSP = ∠ PSR = α.
Let the polar coordinates of the point P be
( r1, β ), so that SP = r1 and ∠ XSP = β. The
vectorial angles of the extremities Q and R of
the chord QR are β − α and β + α
respectively.
We know that the equation of the chord joining the points ‘θ1’ and ‘θ2 ’ of the conic
l / r = 1 + e cos θ is
l / r = e cos θ + sec (θ1 − θ2 ) cos θ − (θ1 + θ2 ).
1 1
2  2 
Increasing each vectorial angle by π in this equation, the equation of the chord of the
conic l / r = 1 − e cos θ joining the point ‘θ1’ and ‘θ2 ’ on it is
l / r = − e cos θ + sec (θ1 − θ2 ) cos θ − (θ1 + θ2 ) ⋅
1 1
.…(2)
2  2 
Putting θ1 = β − α and θ2 = β + α in (2), we get the equation of the chord QR of the conic
(1) as
l / r = − e cos θ + sec α cos (θ − β ). …(3)
But the chord (3) passes through the point P ( r1, β ). Therefore
G-14

l / r1 = − e cos β + sec α cos ( β − β ) = − e cos β + sec α. …(4)


Replacing r1 by r and β by θ in the equation (4), the locus of the point P ( r1, β ) is
l / r = − e cos θ + sec α or ( l cos α ) / r = 1 − e cos α cos θ,
which is a conic.
Problem 11: Prove that two points on the conic l / r = 1 + e cos θ whose vectorial angles are
e +1 α β
α and β respectively will be the extremities of a diameter if = tan tan ⋅
e −1 2 2
Solution: The given conic is
l / r = 1 + e cos θ. …(1)
Let the points P and Q be the extremities of a diameter of (1). Let the vectorial angles
of P and Q be α and β respectively.
The tangent at P is
l / r = cos (θ − α ) + e cos θ
or l = (cos α + e ) x + sin α y, changing to cartesians.
∴ Slope of the tangent at P
= − (cos α + e ) /sin α. …(2)
Similarly the slope of the tangent at Q
= − (cos β + e ) /sin β. …(3)
If PQ is a diameter, then the tangents at P and Q should be parallel and hence their
slopes are equal i. e.,
cos α + e cos β + e
− =−
sin α sin β
or cos α sin β + e sin β = sin α cos β + e sin α
or e (sin β − sin α ) = sin (α − β )
1 1 1 1
or 2 e cos ( β + α ) sin ( β − α ) = 2 sin (α − β ) cos (α − β )
2 2 2 2
1
− cos (α − β )
1 1 e 2
or e cos ( β + α ) = − cos (α − β ) or = ⋅
2 2 1 1
cos ( β + α )
2
Applying componendo and dividendo, we have
1 1
− cos (α − β ) + cos ( β + α )
e +1 2 2
=
e − 1 − cos 1 (α − β ) − cos 1 ( β + α)
2 2
1 1
cos (α − β ) − cos ( β + α )
= 2 2
1 1
cos (α − β ) + cos ( β + α )
2 2
1 1
2 sin α sin β
= 2 2 = tan 1 α tan 1 β.
1 1 2 2
2 cos α cos β
2 2
G-15

Problem 12: If POP ′ be a chord of a conic through a fixed point O, prove that
1 1
tan P ′ SO tan PSO is constant, S being a focus of the conic.
2 2

Solution: Let the conic be l / r = 1 + e cos θ, the


focus S being at the pole. Let O be the fixed point
( a, α ). Suppose the vectorial angles of the ends
P and P ′ of any chord POP ′ passing through the
fixed point O are β and γ respectively. Then
∠ P ′ SO = γ − α and ∠ PSO = α − β.
The equation of the chord PP′ is
l / r = e cos θ + sec ( β − γ ) cos θ − ( β + γ ) .
1 1
2  2 
Since it passes through the point O ( a, α ),
therefore
l / a = e cos α + sec ( β − γ ) cos α − ( β + γ )
1 1
2  2 
 1
cos α − ( β + γ ) 
 2  l
or = − e cos α = k, say,
1 a
cos ( β − γ )
2
where k is a constant because l, a, e and α are all constants
cos  (α − γ ) + (α − β )
1 1
2 2 
or =k
 1 1
cos  (α − γ ) − (α − β ) 
2 2 
1 1 1 1
cos (α − γ ) cos (α − β ) − sin (α − γ ) sin (α − β )
or 2 2 2 2 =k
1 1 1 1
cos (α − γ ) cos (α − β ) + sin (α − γ ) sin (α − β )
2 2 2 2
1 1
1 − tan (α − γ ) tan (α − β )
2 2 k
or = ⋅
1 1 1
1 + tan (α − γ ) tan (α − β )
2 2
Applying componendo and dividendo, we have
1 1
2 tan (α − γ ) tan (α − β )
2 2 k −1
=
2 k +1
1 1 1− k
or tan ( γ − α ) tan (α − β ) =
2 2 1+ k
1 1
or tan P ′ SO tan PSO = constant.
2 2
G-16

Problem 13: If the tangent from a point P to the conic l / r = 1 + e cos θ subtends the fixed angle
β at the focus, prove that the locus of the middle point of SP is a conic of eccentricity e sec β.
Solution: The conic is
l / r = 1 + e cos θ, …(1)
the focus S being at the pole.
Let PT be the tangent to the conic (1) from a point P. Let the polar coordinates of P be
( r ′, α ).
If M be the middle point of SP, the polar coordinates of M are ( r ′ /2, α ).
Since the tangent PT subtends an angle β at the focus S i. e., ∠ PST = β, therefore the
vectorial angle of the point of contact T is α + β.
Now the tangent to (1) at the point ‘α + β’ is
l / r = cos (θ − α − β ) + e cos θ. …(2)

Since the straight line (2) passes


through the point P ( r ′, α ), therefore
l / r ′ = cos (α − α − β ) + e cos α
= cos β + e cos α

l /2
or = cos β + e cos α. …(3)
r ′ /2
To obtain the locus of the point
M ( r ′ /2, α ), we replace r ′/2 by r and α
by θ in the equation (3).

Therefore the locus of the point M is


l /2
= cos β + e cos θ = cos β (1 + e sec β cos θ)
r
( l sec β ) / 2
or = 1 + e sec β cos θ,
r
which is a conic of eccentricity e sec β.

Comprehensive Problems 3
Problem 1: Prove that the centres of the four circles circumscribing the four triangles formed by
the four tangents drawn to a parabola at points whose vectorial angles are α, β, γ , δ lie on another
circle which passes through the focus of the parabola.
Solution: Let α, β, γ and δ be the vectorial angles of the four points A, B, C and D
respectively on the parabola
l / r = 1 + cos θ.
G-17

There are four tangents at the four points A, B, C, D of the parabola. To form a triangle,
we require only three tangents (since a triangle has three sides). Three tangents out of
four can be chosen in 4 C3 i. e., 4 ways. Hence four triangles are formed by the four
tangents.
Now from the equation (1) of Example 12 [give complete proof here], the polar
coordinates of the centre of the circle circumscribing the triangle formed by the
tangents at A, B, C are
 l sec α sec β sec γ , α + β + γ  ⋅
 
4 2 2 2 2 
Similarly the coordinates of the centres of the other three circum-circles are
 l sec α sec β sec δ , α + β + δ  , etc.
 
4 2 2 2 2 
These four points clearly lie on the circle whose equation may be put as
α β γ δ α + β + γ + δ
r = sec sec sec sec cos  θ −
l
⋅
4 2 2 2 2  2 
This is the equation of a circle passing through the pole i. e., the focus of the given
parabola.
Problem 2: If the tangents at any two points P and Q of a conic meet in a point T, and if the
chord PQ meets the directrix corresponding to S in a point K , prove that ∠ KST is a right angle.
Solution: Let the equation of the conic with focus S as the pole be
l / r = 1 + e cos θ. …(1)
Let α, β be the vectorial angles of the points P and Q on (1). The equations of the
tangents at P and Q are
l / r = cos (θ − α ) + e cos θ, …(2)
and l / r = cos (θ − β ) + e cos θ. …(3)
The vectorial angle of the point of intersection T of the tangents (2) and (3) is
(α + β ) / 2. [See equation (4), article 17]
∴ ∠TSZ = (α + β ) / 2.
The equation of the chord PQ is
= sec (α − β ) cos θ − (α + β ) + e cos θ.
l 1 1
…(4)
r 2  2 

The equation of the directrix ZK


corresponding to the focus S is

l / r = e cos θ. …(5)

To find the vectorial angle of the point of


intersection K of (4) and (5) putting the
value of l / r from (5) in (4), we have
G-18

(α − β ) cos θ − (α + β ) + e cos θ


1 1
e cos θ = sec
2  2 

(α − β ) cos θ − (α + β ) = 0


1 1
or sec
2  2 

cos θ − (α + β ) = 0.


1 1
or [∵ sec (α − β ) ≠ 0]
 2  2
1 1
∴ θ − (α + β ) = ± π.
2 2
But from the figure we observe that for the point K the vectorial angle θ is less than
1
(α + β ). So for the point K, we have
2
1 1
θ − (α + β ) = − π
2 2
1 1 1 1
or θ = (α + β ) − π i. e., ∠ KSZ = (α + β ) − π.
2 2 2 2
Now ∠ KST = ∠TSZ − ∠ KSZ

= (α + β ) −  (α + β ) − π  = π.
1 1 1 1
2 2 2  2
Therefore ∠ KST is a right angle.
Problem 3: PQ is a variable chord of a conic having S for focus and angle PSQ is constant.
Prove that the locus of the point of intersection of tangents at P and Q is a conic having S for a focus
and the corresponding directrix is common with the given conic.
Solution: Let the given conic be
l / r = 1 + e cos θ, …(1)
the focus S being at the pole.
Let α, β be the vectorial angles of the points P and Q respectively.
Let T ( r ′, θ ′ ) be the point of intersection of the tangents at P and Q. Then we have from
article 17
α+β
θ′ = …(2)
2
α − β α + β ⋅
= cos 
l
and  + e cos   …(3)
r′  2   2 
[Derive these results here]
Now if ∠ PSQ = 2γ (say), where γ is a constant, then
α − β = 2γ . …(4)
The locus of T will be obtained by eliminating α and β between (2), (3) and (4).
Substituting the values of α + β and α − β from (2) and (4) in (3), we have
l / r ′ = cos γ + e cos θ′
or ( l sec γ ) / r ′ = 1 + ( e sec γ ) cos θ ′.
∴ The locus of T ( r ′, θ′ ) is ( l sec γ ) / r = 1 + ( e sec γ ) cos θ,
G-19

which is a conic whose focus is the pole S, and the equation of its directrix corresponding
to the focus S is
( l sec γ ) / r = ( e sec γ ) cos θ i. e., l / r = e cos θ,
which is also the directrix of the given conic (1). Hence the result.
Problem 4: Prove that, if chords of a conic subtend a constant angle at a focus, the tangents at
the ends of the chord will meet on a fixed conic and the chord will touch (or envelope) another fixed
conic.
Solution: Let the given conic be l / r = 1 + e cos θ, the focus S being at the pole. Suppose
PQ is a variable chord of the given conic and let ∠ PSQ = 2γ. Proceeding as in Problem 3,
the equation of the locus of the point of intersection T of the tangents at P and Q is
( l sec γ ) / r = 1 + ( e sec γ ) cos θ,
which is a fixed conic and this proves the first part of the question.
Now the equation of the chord PQ is
l / r = sec (α − β ) cos θ − (α + β ) + e cos θ
1 1
2  2 
or  1 
l / r = sec γ cos θ − (α + β ) + e cos θ [∵ α − β = 2γ ]
 2 
( l cos γ ) / r = cos θ − (α + β ) + ( e cos γ ) cos θ
1
or
 2 
which clearly touches ( or envelopes) the conic whose equation is
( l cos γ ) / r = 1 + ( e cos γ ) cos θ.
This is a fixed conic because γ is a constant.
Problem 5: Prove that the locus of the point of intersection of tangents at the extremities of
perpendicular focal radii of a conic is another conic having the same focus.
Solution: Suppose PS and QS are two perpendicular focal radii of the conic
l / r = 1 + e cos θ, the focus S being at the pole. Let α and β be the vectorial angles of
P and Q respectively. Then here α − β = π / 2 or if α − β = 2γ , then γ = π / 4.
Now proceeding as in Problem 3, the locus of the point of intersection T of the tangents
at P and Q is
( l sec γ ) / r = 1 + ( e sec γ ) cos θ
1 1
or ( l sec π ) / r = 1 + ( e sec π ) cos θ
4 4
or ( l 2 ) / r = 1 + ( e 2 ) cos θ.
This is the equation of a conic with focus at the pole. Thus this conic has the same focus
as of the given conic.
Problem 6: Prove that the radius vector TS of the point of intersection of tangents at P and Q
bisects the angle between the radii vectors of P and Q (i.e., between PS and QS).
Solution: Let α, β be the vectorial angles of P and Q respectively.
Proceeding as in article 18, if θ′ is the vectorial angle of the point of intersection T of the
tangents at P and Q, then
1
θ′ = ∠TSX = (α + β ).
2
G-20

∴ ∠TSQ = ∠TSX − ∠ QSX


1
= (α + β ) − β
2
1
= (α − β )
2
and ∠ PST = ∠ PSX − ∠TSX
1
= α − (α + β )
2
1
= (α − β ).
2
1
∴ ∠TSQ = ∠ PST = ∠ PSQ.
2
This proves the required result.

Problem 7: A chord PQ of a conic subtends a constant angle 2γ at the focus S and tangents at P
1 1 2 cos γ 2 sin2 γ
and Q meet in T ; prove that + − = ⋅
SP SQ ST l
Solution: Let the equation of the conic with focus S as the pole be
l / r = 1 + e cos θ. …(1)
If α, β are the vectorial angles of P, Q respectively and T ( r ′, θ ′ ) be the point of
intersection of the tangents at P and Q then proceeding as in article 10, we have
α+β l α −β α+β
θ′ = and = cos + e cos …(2)
2 r′ 2 2
Clearly α − β = 2γ and r ′ = ST . …(3)
Now the polar coordinates of the points P and Q are ( SP, α )and ( SQ, β )respectively. The
points P and Q lie on (1), therefore,
l / SP = 1 + e cos α, and l / SQ = 1 + e cos β. …(4)
Making use of the results (3) in (2), we have
1
l / ST = cos γ + e cos (α + β )
2
1
or (2 l cos γ ) / ST = 2 cos2 γ + 2 e cos γ cos (α + β ). …(5)
2
Adding both the relations given in (4) and subtracting (5) from their sum, we get
l l 2 l cos γ
+ − = 2 + e ( cos α + cos β ) − 2 cos2 γ
SP SQ ST
1
− 2 e cos γ cos (α + β )
2
1 1
= 2 + 2 e cos (α + β ) cos (α − β ) − 2 cos2 γ
2 2
1
− 2 e cos γ cos (α + β )
2
G-21

1
= 2 − 2 cos2 γ [2nd and 4th terms cancel as γ = (α − β )]
2
= 2 (1 − cos2 γ ) = 2 sin2 γ.
1 1 2 cos γ 2
∴ + − = sin2 γ.
SP SQ ST l
Problem 8: Show that the locus of the intersection of two perpendicular tangents one drawn to
each of the two parabolas with a common focus whose axes are neither coincident nor perpendicular
is a conic.
Solution: Take the common focus of the two parabolas as the pole. Let one parabola be
l / r = 1 + cos θ, …(1)
the axis of the parabola being taken as the initial line.
Suppose the axis of the other parabola is inclined at an angle φ to the initial line i. e., the
axis of (1). Then its equation is l ′ / r = 1 + cos (θ − φ). …(2)
Note that φ ≠ 0 and φ ≠ π / 2.
Suppose P is a point ‘α’ on (1) and Q is a point ‘β’ on (2).
The equation of the tangent to the parabola (1) at the point ‘α’ is
l / r = cos (θ − α ) + cos θ …(3)
or l = (1 + cos α ) x + y sin α,
changing to cartesians.
1 + cos α 1
Its slope m= − = − cot α.
sin α 2
The equation of the tangent to the parabola (2) at the point ‘β’ is
l ′ / r = cos (θ − β ) + cos (θ − φ) …(4)
or l ′ = (cos β + cos φ) x + (sin β + sin φ) y,
changing to cartesians.
cos β + cos φ
Its slope m′ = −
sin β + sin φ
1 1
2 cos ( β + φ) cos ( β − φ)
=− 2 2
1 1
2 sin ( β + φ) cos ( β − φ)
2 2
1
= − cot ( β + φ).
2
According to the question, the tangents (3) and (4) are to be perpendicular.
1 1
∴ mm′ = − 1 i. e., cot α cot ( β + φ) = − 1
2 2
1 1 1 1
or cos α cos ( β + φ) + sin α sin ( β + φ) = 0
2 2 2 2

or  1 1
cos  α − ( β + φ) = 0.
2 2 
G-22

1 1 1
∴ α − ( β + φ) = ± π, or α − β = φ ± π. …(5)
2 2 2
It is required to find the locus of the point of intersection of (3) and (4). Hence we are to
eliminate the variables α, β between (3) , (4) and (5).
Adding (3) and (4), we get
( l + l ′ ) / r = cos (θ − α ) + cos (θ − β ) + cos θ + cos (θ − φ)

= 2 cos θ − (α + β ) cos (α − β ) + 2 cos (θ − φ) cos φ


1 1 1 1
 2  2 2 2

= ± 2 cos θ − (α + β ) sin φ + 2 cos  θ − φ cos φ.


1 1 1 1
 2  2  2  2
[Using (5)]
1
Dividing by sin φ and transposing, we get
2
(l + l ′ )
cosec φ − 2 cos  θ − φ cot φ = ± 2 cos θ − (α + β ) ⋅
1 1 1 1
r 2  2  2  2 
…(6)
Subtracting (4) from (3), we have
( l − l ′ ) / r = cos (θ − α ) − cos (θ − β ) + cos θ − cos (θ − φ)

= 2 sin θ − (α + β ) sin (α − β ) − 2 sin (θ − φ) sin φ


1 1 1 1
 2  2 2 2

= ± 2 sin θ − (α + β ) cos φ − 2 sin  θ − φ sin φ.


1 1 1 1
 2  2  2  2
[Using (5)]
1
Dividing by cos φ and transposing, we get
2
(l + l ′ )
sec φ + 2 sin  θ − φ tan φ = ± 2 sin θ − (α + β ) ⋅
1 1 1 1
r 2  2  2  2 
…(7)
Subtracting (6) from (7) and then adding, we get
2
 l + l ′ cosec φ − 2 cos  θ − φ  cot φ 
 r  
2  2 2 
2
l+ l′ φ φ φ
+  sec + 2 sin  θ −  tan  = 4.
 r 2  2 2
This is the equation of the required locus. This represents a conic, since if it is changed
into cartesian coordinates we get a second degree equation in x and y.

Problem 9: P, Q , R are three points on the conic l / r = 1 + e cos θ the focus S being the pole.
The tangent at Q meets SP and SR in M and N so that SM = SN = l. Prove that the chord PR
touches the conic l / r = 1 + 2e cos θ.
G-23

Solution: The equation of the conic with


focus S as the pole is
l / r = 1 + e cos θ.
Suppose α, β, γ are the vectorial angles of the
points P, Q, R respectively.
The equation of the chord PR joining the
points α and γ is
l 1 1
= sec ( γ − α )cos {θ − ( γ + α )}
r 2 2
+ e cos θ.
…(1)
It is given that the tangent at Q meets SP and SR in M and N respectively and
SM = SN = l. Therefore the polar coordinates of M and N are ( l, α ) and ( l , γ )
respectively.
The tangent at Q is
l / r = cos (θ − β ) + e cos θ. …(2)
The points M ( l , α ) and N ( l, γ ) lie on (2). Therefore
1 = cos (α − β ) + e cos α,
and 1 = cos ( γ − β ) + e cos γ.
These equations may be written as
(cos β + e ) cos α + sin β sin α − 1 = 0, …(3)
and (cos β + e ) cos γ + sin β sin γ − 1 = 0. …(4)
Solving (3) and (4) for cos β + e and sin β, we have
cos β + e sin β 1 1
= = =
sin γ − sin α cos α − cos γ cos α sin γ − cos γ sin α sin ( γ − α )
cos β + e sin β
or =
1 1 1 1
2 cos ( γ + α ) sin ( γ − α ) 2 sin ( γ + α ) sin ( γ − α)
2 2 2 2
1
= ⋅
1 1
2 sin ( γ − α ) cos ( γ − α )
2 2
1 1
∴ cos β + e = cos ( γ + α ) sec ( γ − α ), …(5)
2 2
1 1
and sin β = sin ( γ + α ) sec ( γ − α ). …(6)
2 2
Now the equation (1) of the chord PR may be re-written as
1 1
l / r = sec ( γ − α ) cos θ cos ( γ + α )
2 2
1 1
+ sec ( γ − α ) sin θ sin ( γ + α ) + e cos θ
2 2
G-24

or l / r = (cos β + e ) cos θ + sin θ sin β + e cos θ [Using (5) and (6)]


or l / r = cos θ cos β + sin θ sin β + 2 e cos θ
or l / r = cos (θ − β ) + 2 e cos θ,
which is the equation of the tangent at the point ‘β ’ to the conic l / r = 1 + 2 e cos θ.
Hence the chord PR touches the conic
l / r = 1 + 2 e cos θ.

Problem 10: If PQ is the chord of contact of tangents drawn from a point T to a conic whose
focus is S, prove that
(i) ST 2 = SP . SQ, if the conic is a parabola;
1 1 1 PSQ
(ii) − = sin2 ,
SP . SQ ST 2 b2 2
if the conic is a central conic and b is its semi-minor axis.
Solution: (i) Let the equation of a parabola be l / r = 1 + cos θ, …(1)
the focus S being at the pole.
Let α, β be the vectorial angles of P, Q respectively.
l / SP = 1 + cos α = 2 cos2 1 α i. e., SP = ( l / 2) sec2 1 α, 
Then 2 2 …(2)

l / SQ = 1 + cos β = 2 cos2 1 β i. e., SQ = ( l / 2) sec2 1 β.
2 2 
Now let T ( r ′, θ′ ) be the point of intersection of the tangents at P and Q. Then
1 1
ST = ( l / 2) sec α sec β. [See article 17, equation (6)]
2 2
1 1
∴ ST 2 = ( l2 / 4) sec 2 α sec 2 β = SP . SQ , using (2).
2 2
(ii) Let the equation of a central conic be l / r = 1 + e cos θ, …(1)
the focus S being at the pole.
Let α, β be the vectorial angles of P, Q respectively.
Since P and Q lie on (1), we have
l / SP = 1 + e cos α, l / SQ = 1 + e cos β.
l2
Multiplying, = 1 + e (cos α + cos β ) + e2 cos α cos β
SP . SQ
1 1
= 1 + 2 e cos (α + β ) cos (α − β ) + e2 cos α cos β. …(2)
2 2
Also clearly ∠ PSQ = α − β. …(3)
Now suppose T ( r ′, θ′ ) is the point of intersection of the tangents at P and Q. Then
proceeding as in article 10, we have
1 1
l / r ′ = l / ST = cos (α − β ) + e cos (α + β ). …(4)
2 2
Now from (2) and (4), we have
l2 /( SP . SQ) − l2 /( ST 2 )
G-25

1 1
= 1 + 2e cos (α + β ) cos (α − β ) + e2 cos α cos β
2 2
1 1 1 1
− cos2 (α − β ) − e2 cos2 (α + β ) − 2e cos (α + β ) cos (α − β )
2 2 2 2
= 1 − cos (α − β ) − e 2 cos (α + β ) − 2 cos α cos β 
2 1 1 2 2 1
 2  2  2 
1 1
= sin2 (α − β) − e2 {1 + cos (α + β ) − 2 cos α cos β }
2 2
1 1
= sin2 (α − β) − e2 {1 + cos α cos β − sin α sin β − 2 cos α cos β }
2 2
1 1
= sin2 (α − β) − e2 {1 − (cos α cos β + sin α sin β )}
2 2
1 1
= sin2 (α − β) − e2 {1 − cos (α − β )}
2 2
1 1 1 1
= sin2 (α − β) − e2 . 2 sin2 (α − β ) = (1 − e2 ) sin2 (α − β ).
2 2 2 2
1 1 (1 − e2 ) 2 1
∴ − = sin (α − β ). …(5)
SP . SQ ST 2 l2 2
Now in a central conic, we have
l = semi-latus rectum = b2 / a, and b2 = a2 (1 − e2 ).

1− e2 1− e2 a2 (1 − e 2 ) b2 1
∴ = = = = ⋅ …(6)
2 4 2 4 4
l b /a b b b2
Making use of the results (3) and (6) in (5), we have
1 1 1 1
− = sin2 PSQ.
SP . SQ ST 2 b2 2

Comprehensive Problems 4
Problem 1: Show that three normals can be drawn from a point (ρ, φ) to a parabola.
Solution: Let the parabola be l / r = 1 + cos θ. …(1)
The equation of the normal to (1) at the point ‘α’ is
l sin α 1
⋅ = sin (θ − α ) + sin θ. [See Corollary of article 23]
1 + cos α r
If it passes through the point ( ρ, φ), we have
l sin α 1
⋅ = sin ( φ − α ) + sin φ
1 + cos α ρ
1 1
2l sin α cos α
or 2 2 ⋅ 1 = sin φ cos α − cos φ sin α + sin φ
2 1 ρ
2 cos α
2
G-26

1
or ( l / ρ) tan α = sin φ (1 + cos α ) − cos φ sin α
2
1 1 1 1
or ( l / ρ) tan α = 2 sin φ cos2 α − 2 cos φ sin α cos α.
2 2 2 2
2 1
Dividing both sides by cos α, we have
2
1 1 1
( l / ρ) tan α sec 2 α = 2 sin φ − 2 cos φ tan α
2 2 2
1 2 1 1
or l tan α (1 + tan α ) = 2 ρ sin φ − 2 ρ cos φ tan α
2 2 2
3 1 2 1 1
or l tan α + 0 . tan α + ( l + 2 ρ cos φ) tan α − 2 ρ sin φ = 0.
2 2 2
…(2)
1 1
This equation being a cubic in tan α gives three values of tan α and hence three val-
2 2
ues of α. Thus there are three points on the parabola the normals at which pass through
the point ( ρ, φ). Hence, in general three normals can be drawn to a parabola from a point.
Problem 2: If the normals at three points of the parabola r = a cosec2 1 / 2 θ whose vectorial
angles are α, β and γ meet in a point whose vectorial angle is φ, prove that 2φ = α + β + γ − π.
Solution: The given parabola is
1
r = a cosec2 θ,
2
1
or 2a / r = 2 sin2 θ, or 2a / r = 1 − cos θ.
2
The normal to this parabola at the point ‘ λ’ on it is
− 2a sin λ 1
⋅ = sin (θ − λ ) − sin θ. [See remark after article 23]
1 − cos λ r
If it passes through the point ( ρ, φ), we have
− 2a sin λ 1
⋅ = sin ( φ − λ ) − sin φ
1 − cos λ ρ
1 1
− 2a . 2 sinλ cos λ
or 2 2 = ρ [sin φ cos λ − cos φ sin λ − sin φ]
1
2 sin2 λ
2
1
or − 2a cot λ = ρ [ − sin φ (1 − cos λ ) − cos φ sin λ ]
2

2a = ρ tan λ sin φ . 2 sin2 λ + 2 cos φ sin λ cos λ  ⋅


1 1 1 1
or
2  2 2 2 
2 1
Dividing both sides by 2 cos λ, we have
2
a sec 2 λ = ρ tan λ sin φ tan2 λ + cos φ tan λ 
1 1 1 1
2 2  2 2 
G-27

a 1 + tan2 λ  = ρ sin φ tan3 λ + ρ cos φ tan2 λ


1 1 1
or
 2  2 2
3 1 2 1 1
or ρ sin φ tan λ + (ρ cos φ − a) tan λ + 0 . tan λ − a = 0. …(1)
2 2 2
1 1
This equation being a cubic in tan λ gives three values of tan λ and hence three
2 2
values of λ. Thus there are three points on the parabola the normals at which pass
1 1 1
through the point (ρ, φ). If these points are α, β, γ , then tan α, tan β, tan γ are the
2 2 2
1
roots of the cubic (1) in tan λ .
2
By the theory of equations, we have for the cubic (1),
1
Σ tan α = − (ρ cos φ − a) / ρ sin φ,
2
1 1
Σ tan α tan β = 0,
2 2
1 1 1
tan α tan β tan γ = a / ρ sin φ.
2 2 2
Now from trigonometry, we have
1 1 1 1
Σ tan α − tan α tan β tan γ
 1 1
tan  α + β + γ  =
1  2 2 2 2
 2 2 2  1
1 − Σ tan α tan β
1
2 2
{ − ( ρ cos φ − a) / ρ sin φ} − ( a / ρ sin φ)
=
1−0
( ρ cos φ − a) a − ρ cos φ + a − a
=− − = = − cot φ
ρ sin φ ρ sin φ ρ sin φ

= tan  π + φ ⋅
1
 2 
1 1 1 1
∴ α + β + γ = π + φ, or α + β + γ = π + 2φ
2 2 2 2
or 2φ = α + β + γ − π.
Problem 3: If the tangent and normal at any point P of a conic meet the transverse axis in T
1 1
and G respectively and if S be the focus, then show that − is constant.
SG ST
Solution: Let the conic be l / r = 1 + e cos θ, …(1)
the focus S being at the pole. Let the vectorial angle of P be ‘α’.
The equations of the tangent and normal to (1) at the point P are
l / r = cos (θ − α ) + e cos θ, …(2)

le sin α 1
and ⋅ = sin (θ − α ) + e sin θ …(3)
(1 + e cos α ) r
respectively.
G-28

The tangent (2) meets the axis i. e., the


initial line in T where r = ST and θ = 0.
Putting these values in the equation (2), we
have
l / ST = cos (0 − α ) + e cos 0.
∴ 1 / ST = ( e + cos α ) / l.
…(4)
The normal (3) meets the axis in G where
r = SG, and θ = π.
Putting these values in the equation (3), we have
le sin α 1
⋅ = sin ( π − α ) + e sin π.
(1 + e cos α ) SG
1 (1 + e cos α )
∴ = ⋅ …(5)
SG le
Subtracting (4) from (5), we get
1 1 1 + e cos α e + cos α 1 − e2
− = − = = constant.
SG ST le l le
Problem 4: If the normals at α, β, γ, δ on the conic l / r = 1 + e cos θ meet at a point
(ρ, φ) ( i. e., these normals are concurrent), prove that
2
α β γ δ 1 + e
tan tan tan tan +   = 0,
2 2 2 2 1 − e

and α + β + γ + δ − 2φ = (2n + 1) π i.e., an odd multiple of π radians.


(Rohilkhand 2010; Kanpur 16)
Solution: The given conic is l / r = 1 + e cos θ. …(1)
The equation of the normal to (1) at any point ‘ λ’ on it is
le sin λ 1
⋅ = sin (θ − λ ) + e sin θ.
(1 + e cos λ ) r
If it passes through the point ( ρ, φ), we have
le sin λ = ρ (1 + e cos λ ) [sin φ cos λ − cos φ sin λ + e sin φ]. …(2)
2
2t 1− t 1
Now putting sin λ = and cos λ = , where t = tan λ ,
1+ t2 1 + t2 2
the equation (2) becomes
 e (1 − t2 )   1− t2 
= ρ 1 + + e sin φ
le 2t 2t
2  sin φ. − cos φ ⋅
1+ t  1 + t2   1+ t2
1+ t 2

or 2 let (1+ t ) = ρ [1+ e + t (1 − e )] [sin φ (1 + e ) − 2 t cos φ − t2 sin φ (1 − e )]
2 2

or ρ (1 − e )2 sin φ t4 + 2 { le + (1 − e ) ρ cos φ} t3 + 0 . t2
+ 2 { le + (1 + e ) ρ cos φ} t − ρ (1 + e )2 sin φ = 0. …(3)
G-29

1
λ . Thus there are
The equation (3) being of degree 4 in t gives four values of t i. e., of tan
2
four points on the conic say, α, β, γ and δ, the normals at which pass through the point
( ρ, φ).
Let t1, t2 , t3 , t4 be the roots of the equation (3), where
1 1
t1 = tan α, t2 = tan β,
2 2
1 1
t3 = tan γ, t4 = tan δ.
2 2
2 { le + (1 − e ) ρ cos φ}
Then s1 = Σ t1 = − ,
ρ (1 − e )2 sin φ

s2 = Σ t1t2 = 0, [∵ The coeff. of t 2 in (3) = 0]


− 2 { le + (1 + e ) ρ cos φ}
s3 = Σ t1 t2 t3 = ,
ρ (1 − e )2 sin φ

− ρ (1 + e )2 sin φ (1 + e )2
and s4 = t1 t2 t3 t4 = =− ⋅
ρ (1 − e )2 sin φ (1 − e )2
The last relation may be written as
1 1 1 1 (1 + e )2
tan α tan β tan γ tan δ = − ⋅
2 2 2 2 (1 − e )2
This proves the first part of the question. From trigonometry, we have
α β γ δ  s1 − s3
tan  + + +  =
 2 2 2 2  1 − s2 + s4

− 2 { le + (1 − e ) ρ cos φ} 2 { le + (1 + e ) ρ cos φ}
+
2
ρ (1 − e ) sin φ e (1 − e )2 sin φ
=
2
(1 + e )
1−0 −
(1 − e )2

2 × 2 ρe cos φ (1 − e )2
= ⋅
ρ (1 − e )2 sin φ − 4e
π
= − cot φ = tan  + φ ⋅
2 
α β γ δ  π
Thus tan  + + +  = tan  + φ ⋅
 2 2 2 2   2 
α+β+ γ+δ π
∴ = nπ + + φ, where n is any integer
2 2
or α + β + γ + δ − 2φ = (2n + 1) π,
which is an odd multiple of π radians.
G-30

Problem 5: If the normal at L, one of the extremities of the latus rectum of the conic
l / r = 1 + e cos θ, meets the curve again at Q , show that
SQ = l (1 + 3e2 + e4 ) / (1 + e2 − e4 ).

Solution: The conic is


l / r = 1 + e cos θ, …(1)
the focus S being at the pole.
In the figure LSL′ is the latus rectum.
∴ ∠ LSX = π / 2.
Hence the coordinates of L are
( l, π / 2). The equation of the normal at
the point L is (see article 23)

1
le sin π
⋅ = sin  θ − π  + e sin θ
2 1 1
1 + e cos π 
1 r  2 
 
 2 
le
or = − cos θ + e sin θ. …(2)
r
To find the vectorial angles of the points of intersection L and Q of (1) and (2), we solve
(1) and (2) for θ. So substituting for l / r from (1) in (2), we have
(1 + e cos θ) e = − cos θ + e sin θ
or e + ( e2 + 1) cos θ = e sin θ.
Squaring, e2 + ( e2 + 1)2 cos2 θ + 2e ( e2 + 1) cos θ = e2 sin2 θ = e2 (1 − cos2 θ)

or { e2 + ( e2 + 1)2} cos2 θ + 2e ( e2 + 1) cos θ = 0


or {( e4 + 3e2 + 1) cos θ + 2e ( e2 + 1)} cos θ = 0.
∴ Either cos θ = 0 i. e., θ = π / 2 which corresponds to the point L,
or ( e4 + 3e2 + 1) cos θ + 2e ( e2 + 1) = 0

i. e., cos θ = − 2e ( e2 + 1) /( e4 + 3e2 + 1) …(3)


which corresponds to the point Q.
Again Q lies on (1).
∴ l / SQ = 1 + e cos θ,
where SQ is the radius vector of Q and cos θ is given by (3).
 2e ( e2 + 1) 
= 1 + e  −
l
Hence 
SQ  e 4 + 3e2 + 1 
l 1 + e2 − e4 l (1 + 3e2 + e4 )
or = or SQ = ⋅
SQ e4 + 3e2 + 1 1 + e2 − e4
G-31

H ints to O bjective T ype Q uestions

Multiple Choice Questions


1. See article 11.
2. See article 14, corollary 2.
3. See article 15.
4. See article 10, corollary 1.
l
5. The conic = 1 + e cos θ is an ellipse if e < 1.
r
6. See article 9, result 3.
7. See article 9, result 4.
l
8. The conic = 1 + e cos θ is an ellipse if e < 1. Here e = −5 i. e., e < 1.
r
9. See article 9 result 3.
l
10. The conic = 1 + e cos θ is a hyperbola if e > 1.
r
l
11. If e < 1, the conic = 1 + e cos θ is an ellipse.
r
12. The equation of the tangent at the point ( r1 , θ1 ) of the conic
l l
= 1 + e cos θ is = e cos θ + cos (θ − θ1).
r r
See article 14.
l
13. The slope of the tangent to the conic = 1 + e cos θ at the point ‘α’ on it is
r
e + cos α
− ⋅
sin α
See article 14, corollary 3.
14. The locus of the point of intersection of two perpendicular tangents to a conic, is
called the director circle of the conic. See article 18.
15. The equation of the normal at a point ‘α’ on the conic
l le sin α 1
= 1 + e cos θ is ⋅ = sin (θ − α ) + e sin θ.
r 1 + e cos α r
See article 23, equation (5).
l
16. The conic = 1 + e cos θ is a parabola if e = 1.
r

Fill in the Blank(s)


1. A conic is the locus of a point which moves so that its distance from a fixed point
is in a constant ratio to its perpendicular distance from a fixed straight line. See
article 1.
G-32

2. The length of the semi-latus rectum of the conic


l
= 1 + e cos θ is l.
r
So the length of the latus rectum is 2 l. See article 10.
3. The polar equation of a conic with its focus as the pole and its axis inclined at an
l
angle α to the initial line is = 1 + e cos (θ − α )
r
where l is the length of the semi-latus rectum of the conic and e is the eccentricity
of the conic. See article 12.
4. The locus of the foot of the perpendicular from the focus on any tangent to a
conic (ellipse or hyperbola) is called the auxiliary circle of the conic. See article
16.
5. The polar equation of the chord of contact of the point ( r ′, θ ′ ) with respect to the
conic
 l 
= 1 + e cos θ is  − e cos θ 
l l
− e cos θ ′ = cos (θ − θ ′ ).
r  r   r′ 
See article 21.

True or False
l
1. The conic = 1 + e cos θ is a hyperbola if e > 1.
r
l
Here e = 3 i. e., e > 1, so the conic = 1 + 3 cos θ is a hyperbola.
r
2. The equation of the tangent to the conic
l
= 1 + e cos (θ − α ) at the point ‘β’ on it is
r
l
= e cos (θ − α ) + cos (θ − β ).
r
See article 14, corollary 1.
3. See article 18, equation (7).
4. The equation of the polar of a point ( r ′, θ ′ ) with respect to the conic
 l 
= 1 + e cos θ is  − e cos θ 
l l
− e cos θ ′ = cos (θ − θ ′ ).
r  r   r′ 
See article 21, equation (4).
5. See article 13, equation (5).
❍❍❍
G-33
Chapter-2
Systems of Co-ordinates

Comprehensive Problems 1
Problem 1: (i) Find the locus of a point P which moves in such a way that its distance from
the point A ( u, v , w ) is always equal to a.
(ii) A , B, C are three points on the axes of x , y and z respectively at distances a, b , c from the
origin O ; find the co-ordinates of the point which is equidistant from A , B, C and O.
Solution: (i) Let the co-ordinates of a point P be (α, β, γ ). Given the distance between
P and A( u, v, w ) is a.
Then PA = a
or (α − u)2 + (β − v )2 + ( γ − w ) = a.
On squaring both sides,
(α − u)2 + (β − v )2 + ( γ − w )2 = a2
or α2 + β2 + γ 2 − 2αu − 2βv − 2γw + u2 + v2 + w2 − a2 = 0.
Hence the locus of P is
x2 + y2 + z 2 − 2 xu − 2 yv − 2wz + u2 + v2 + w2 − a2 = 0.
(ii) Given the three points on the axes of x, y and z respectively at a, b, c from O.
Z

C (0, 0, c)
P (x, y, z)

O X
A (a, 0, 0)

B (0, b, 0)
Y
Then co-ordinates of points are A( a, 0, 0 ), B(0, b, 0 ) and C(0, 0, c ).
Let the co-ordinates of a point P be ( x, y, z )which is equidistant from A, B, C and O..
Then PA = PB = PC = PO. …(1)
Here PA = PB.
or PA2 = PB2 .
G-34

∴ ( x − a)2 + ( y − 0 )2 + ( z − 0 )2 = ( x − 0 )2 + ( y − b )2 + ( z − 0 )2
or x2 + y 2 + z 2 − 2ax + a2 = x2 + y2 + z 2 − 2by + b2
or 2by − 2ax + a2 − b2 = 0 …(2)
2 2
and PA = PC
or ( x − 0 )2 + ( y − b )2 + ( z − 0 )2 = ( x − 0 )2 + ( y − 0 )2 + ( z − c )2
or x2 + y2 + z 2 − 2by + b2 = x2 + y2 + z 2 − 2cz + c2
or 2cz − 2by + b2 − c2 = 0 …(3)
2 2
and PA = PO
( x − a)2 + ( y − 0 )2 + ( z − 0 )2 = ( x − 0 )2 + ( y − 0 )2 + ( z − 0 )2
or x2 + y2 + z 2 − 2ax + a2 = x2 + y2 + z 2
a
or −2ax + a2 = 0 or x = ⋅
2
Putting the value of x in equation (2), we get
b
y= ⋅
2
Putting the value of y in equation (3), we get
c
z= ⋅
2
∴ The co-ordinates of P are  , ,  which is equidistant from A, B, C and O.
a b c
 2 2 2

Problem 2: (i) Show that the points A (0 , 1, 2), B(2, − 1, 3) and C(1, − 3, 1) are the vertices
of an isosceles right angled triangle.
(ii) Show that the points (1, 2, 3), (2, 3, 1) and (3, 1, 2) form an equilateral triangle.
Solution: (i) Let A, B and C be points (0, 1, 2), (2, − 1, 3)and C(1, − 3, 1)respectively.

Then, AB = (0 − 2)2 + (1 + 1)2 + (2 − 3)2 = 4 + 4 + 1 = 3

BC = (2 − 1)2 + ( −1 + 3)2 + (3 − 1)2 = 1 + 4 + 4 = 3

CA = (0 − 1)2 + (1 + 3)2 + (2 − 1)2 = 1 + 16 + 1 = 3 2.

Here AB = BC so ∆ ABC is isosceles


and AB + BC = CA2
2 2

or 9 + 9 = 18
So, ∆ ABC is right angled triangle, ∠ B is right angle.
∴ The given points A, B and C represent an isosceles right angled triangle.
(ii) Let A, B and C are points (1, 2, 3), (2, 3, 1) and (3, 1, 2) respectively.

Then AB = (1 − 2)2 + (2 − 3)2 + (3 − 1)2 = 1 + 1 + 4 = 6


G-35

BC = (2 − 1)2 + (3 − 1)2 + (1 − 2)2 = 1 + 4 + 1 = 6

and CA = (1 − 2)2 + (2 − 1)2 + (3 − 1)2 = 4 + 1 + 1 = 6.


∴ AB = BC = CA .
So, ∆ ABC is an equilateral triangle.

Problem 3: (i) Find the co-ordinates of the point which divides the join of (2 , 3, 4) and
(3, − 4, 7) in the ratio 2 : − 4. (Meerut 2003)
2 2 2
(ii) Find the ratios in which the sphere x + y + z = 504 divides the line joining the points
(12, − 4, 8) and (27, − 9, 18).
Solution: (i) Let the co-ordinates of the required point be ( x, y, z ), then by article 10,
we have
2 (3) − 4 (2) 6 − 8
x= = = 1;
2−4 −2
2 ( −4) − 4 (3) −20
y= = = 10 ;
2−4 −2
2 (7) − 4 (4) 14 − 16 −2
z= = = = 1.
2−4 −2 −2
Hence the required point is (1, 10 , 1).
(ii) Let the sphere x2 + y2 + z 2 = 504 meet the line joining the given points in the
point ( x1, y1, z1 ). Then
x12 + y12 + z12 = 504 . …(1)
Now suppose the point ( x1, y1, z1 ) divides the join of the points (12, − 4, 8) and
(27, − 9, 18) in the ratio λ :1.
27λ + 12 − 9λ − 4 18 λ + 8
Then x1 = , y1 = , z1 = ⋅
λ +1 λ +1 λ +1
Putting the values of x1, y1, z1 in (1), we get
(27λ + 12)2 ( − 9λ − 4)2 (18 λ + 8)2
+ + = 504
( λ + 1)2 ( λ + 1)2 ( λ + 1)2
or 9 (9λ + 4)2 + (9λ + 4)2 + 4 (9λ + 4)2 = 504 ( λ + 1)2
or 14 (9λ + 4)2 = 504 ( λ + 1)2
or (9λ + 4)2 = 36 ( λ + 1)2 .
∴ 9λ + 4 = ± 6 ( λ + 1).
Taking + ive sign,
9λ + 4 = 6λ + 6 or 3λ = 2 or λ = 2 /3
and taking − ive sign,
2
9λ + 4 = − 6λ − 6 or 15λ = − 10 or λ=− ⋅
3
G-36

Thus the sphere divides the line joining the given points (12, − 4, 8) and (27, − 9, 18) in
the ratios (2 / 3) : 1 i. e., 2 : 3 and ( −2 / 3) : 1 i. e., 2 : − 3.
Hence the given sphere divides the line joining the given points internally in the ratio
(2 / 3) i. e., 2 : 3 and externally in the ratio (2 / 3):1 i. e., 2 : 3.

Problem 4: (i) Using distance formula show that the points A (3, 2, − 4), B(5, 4, − 6) and
C(9, 8, − 10 ) are collinear. Find the ratio in which B divides AC. (Kumaun 2007)
(ii) Find the ratio in which the line joining the points A(2, 3, 4)and B( −3, 5, − 4)is divided by
the yz-plane. Also, find the point of intersection.
Solution: (i) Given points A (3, 2, − 4), B(5, 4, − 6) and C(9, 8, − 10 )

Then AB = (3 − 5)2 + (2 − 4)2 + ( −4 + 6)2 = 4 + 4 + 4 = 2 3

BC = (5 − 9)2 + (4 − 8)2 + ( −6 + 10 )2 = 16 + 16 + 16 = 4 3

and CA = (9 − 3)2 + (8 − 2)2 + ( −10 + 4)2 = 36 + 36 + 36 = 6 3.

Here AB + BC = CA .
Therefore given points are collinear.
Now Let B divide AC in the ratio λ :1.
 9λ + 3 8λ + 2 −10 λ − 4 
Then co-ordinates of B are  , , .
 λ +1 λ +1 λ +1 
But the co-ordinate of B are (5, 4, − 6).
9λ + 3
∴ =5
λ +1
1
or 9λ + 3 = 5λ + 5 or 4λ = 2 or λ= ⋅
2
∴ B divides AC in the ratio 1 : 2.
(ii) Let the line joining points A(2, 3, 4) and ( −3, 5, − 4) is divided by the yz-plane at
point P ( x, y, z ) in the ratio λ :1.
Then the co-ordinates of P are
 −3λ + 2 5λ + 3 −4λ + 4 
 , , 
 λ +1 λ +1 λ +1 

but in yz-plane the co-ordinate of x is zero


−3λ + 2 2
then = 0 or −3λ + 2 = 0 or λ= ⋅
λ +1 3
Therefore yz-plane divides the line AB in the ratio 2 : 3 and the co-ordinates of
P are
 0, 19 , 4  .
 
 5 5
G-37

Problem 5: Three vertices of a parallelogram ABCD are A (3, 4, − 1), B(7, 10 , − 3) and
C(8, 1, 0 ). Find the fourth vertex D.
Solution: Given the three vertices of a A B
parallelogram ABCD are A(3, 4, − 1), B(7, 10, − 3)
and C(8, 1, 0 ). Let the co-ordinates of D be
( x, y, z ).
Then O is the mid-point of diagonals AC and BD. O
3+8 7+ x
∴ = ⇒ x=4
2 2 D (x, y, z) C
4 + 1 10 + y
= ⇒ y = −5
2 2
−1 + 0 −3 + 3
and = ⇒ z = 2.
2 2
∴ Co-ordinates of D are (4, − 5, 2).

Problem 6: What are the perpendicular distances of the point ( x , y, z ) from the coordinate
axes ?
Solution: Let P ( x , y, z ) be any point in the
space. From P draw PN perpendicular to the
xy-plane. Draw NA and NB perpendiculars
from N toOX and OY respectively. Join
PA , PB and ON. Draw PC perpendicular
from P to z-axis.
Thus OA = x = NB , NA = y, PN = z .
Length of perpendicular from P to OX
= PA = PN 2 + NA2

= z 2 + y 2,

length of perpendicular from P to OY


= PB = PN 2 + NB2 = z 2 + x2

and length of perpendicular from P to OZ


= PC = ON = OA 2 + NA2 = x 2 + y 2.

Problem 7: Find the ratio in which the xy-plane divides the join of A ( − 3, 4, − 8) and
B (5, − 6, 4). Also find the point of intersection of the line with the plane. (Kanpur 2016)
Solution: Let the xy-plane divide the line joining the points A ( − 3, 4, − 8) and
B (5, − 6, 4) in the ratio λ :1 in the point R. Then the coordinates of the point R are
G-38

 5λ − 3 − 6λ + 4 4λ − 8 
 , , ⋅ …(1)
 λ +1 λ +1 λ +1
Since the point R lies on the xy-plane, so the z-coordinate of R is zero.
4λ − 8
∴ = 0 ⇒ 4λ − 8 = 0 ⇒ λ = 2 .
λ +1
Hence the xy-plane divides the join of A and B internally in the ratio 2 : 1. Putting λ = 2 in
(1), the required coordinates of the point R are (7 / 3, − 8 / 3,0 ).

Problem 8: The mid-points of the sides of a triangle are (1, 5, − 1), (0, 4, − 2)and (2, 3, 4). Find
its vertices.
Solution: Let A ( x1, y1, z1 ), B ( x2 , y2 , z2 ) and C ( x3 , y3 , z3 ) be the vertices of the
given triangle and D (1, 5, − 1), E (0, 4, − 2) and F (2, 3, 4) be the middle points of BC, CA
and AB respectively. Then

x2 + x3 y + y3 z + z3
= 1, 2 = 5, 2 = −1
2 2 2
i. e., x2 + x3 = 2, y2 + y3 = 10, z2 + z3 = − 2, …(1)
x3 + x1 y3 + y1 z + z1
= 0, = 4, 3 = −2
2 2 2
i. e., x3 + x1 = 0,
y3 + y1 = 8, z3 + z1 = − 4 …(2)
x1 + x2 y + y2 z + z2
and = 2, 1 = 3, 1 =4
2 2 2
i. e., x1 + x2 = 4, y1 + y2 = 6, z1 + z2 = 8. …(3)
Adding (1), (2) and (3), we get
2 ( x1 + x2 + x3 ) = 6, 2 ( y1 + y2 + y3 ) = 24, 2 ( z1 + z2 + z3 ) = 2

i. e., x1 + x2 + x3 = 3, y1 + y2 + y3 = 12 , z1 + z2 + z3 = 1. …(4)

Subtracting successively (1), (2) and (3) from (4), we get


x1 = 1, y1 = 2 , z1 = 3 ; x2 = 3, y2 = 4, z2 = 5 ;
x3 = − 1, y3 = 6, z3 = − 7.
Hence the vertices of the given triangle are
A (1, 2, 3), B (3, 4, 5) and C ( − 1, 6, − 7).
G-39

H ints to O bjective T ype Q uestions

Multiple Choice Questions


1. The coordinates of the centroid of the triangle ABC are
 2 + 3 + 0 , −1 + 3 + 1 , 0 + ( −3) + 4  i. e.,  5 , 1, 1  ⋅
   
 3 3 3  3 3 
2. Suppose C divides AB in the ratio λ : 1. Then the co-ordinates of C are
 3 λ + 2 −2 λ + 3 2 λ + 4 
 , , ⋅
 λ +1 λ +1 λ +1 
But the coordinates of C are given as (6, − 17, − 4).
3λ+2 4
∴ =6 ⇒ 3λ + 2=6λ + 6 ⇒ 3λ = −4 ⇒ λ = − ,
λ +1 3
−2 λ + 3 4
= − 17 ⇒ − 2 λ + 3 = − 17λ − 17 ⇒ 15 λ = − 20 ⇒ λ = −
λ +1 3
2λ+4 4
and = −4 ⇒ 2λ + 4= −4λ −4 ⇒ 6λ = −8 ⇒ λ = − ⋅
λ +1 3
4
So, C divides AB in the ratio − : 1 i. e., −4 : 3.
3

Fill in the Blank(s)


1. See article 10.
2. See article 9, Corollary 3.
3. The distance between the points A ( −2 , 1, − 3) and B (4 , 3, − 6)
= {4 − ( −2)}2 + (1 − 3)2 + { − 3 − ( −6)}2 = 62 + ( −2)2 + 32
= 36 + 4 + 9 = 49 = 7.
4. Let the yz-plane divide the line joining the points A ( −2 , 4 , 7) and
B (3, − 5, 8) in the
ratio µ :1 in the point R. Then the coordinates of the point R are
 3µ − 2 − 5µ + 4 8µ + 7 
 , , .
 µ +1 µ + 1 µ + 1 
Since the point R lies on the yz-plane, therefore the x-coordinate of R is zero.
3µ − 2 2
∴ = 0 ⇒ 3µ − 2 = 0 ⇒ µ = ⋅
µ +1 3
2
Hence, the yz-plane divides the join of A and B internally in the ratio : 1 i. e., 2 : 3.
3
❍❍❍
G-40

Chapter-3
Direction Cosines and Projections

Comprehensive Problems 1
Problem 1: Find the d.c.’s of a line whose direction ratios are −1, 2, − 1.
Solution: We have ( −1)2 + (2)2 + ( −1)2 = 6.
Hence the d.c.’s of the given line are
−1 2 −1
, , .
6 6 6

Problem 2: Find the direction cosines of the line which is equally inclined to the axes.
(Gorakhpur 2005; Kanpur 11; Bundelkhand 13; Purvanchal 13)
Solution: Let the direction cosines of the line be l, m, n. Since the line is equally
inclined to the coordinate axes, therefore
l 2 = m2 = n2 .
But l 2 + m2 + n2 = 1.
∴ 3l 2 = 1 or l2 = 1 / 3
or l = ± 1/ 3.
Similarly m = ± 1 / 3, n = ± 1 / 3.
1 1 1
Hence the direction cosines of the lines are ± ,± ,± ⋅
3 3 3
There will be eight such lines, one lying in each octant. The direction cosines of the line
lying in the positive octant are 1 / 3,1 / 3,1 / 3.

Problem 3: Find the direction cosines l, m, n of two lines which are connected by the relations
l − 5m + 3n = 0 and 7l 2 + 5m2 − 3n2 = 0.
(Meerut 2010, 12; Purvanchal 13)
Solution: The given relations are
l − 5m + 3n = 0 or l = 5m − 3n …(1)
2 2 2
and 7l + 5m − 3n = 0. …(2)
First we eliminate l or m or n between (1) and (2).
G-41

To eliminate l between (1) and (2), putting the value of l from (1) in (2), we get
7 (5m − 3n)2 + 5m2 − 3n2 = 0
or 180 m2 − 210 mn + 60 n2 = 0
or 6m2 − 7mn + 2n2 = 0 or 6m2 − 4mn − 3mn + 2n2 = 0
or 2m (3m − 2n) − n (3m − 2n) = 0 or (2m − n) (3m − 2n) = 0.
m 1 m 2
∴ 2m = n or 3m = 2n i. e., = or = ⋅
n 2 n 3
l m
Now from (1), = 5 ⋅ − 3. …(3)
n n
m 1
When = , we have from (3)
n 2
l 1 1
=5× −3= − ⋅
n 2 2
m 1 l 1
From = and = − , we see that the d.c.’s of one line are given by
n 2 n 2

l m n l2 + m2 + n2 1
= = = = ⋅
−1 1 2 2 2
( − 1) + 1 + 2 2 6

1 1 2
∴ The d.c.’s of one line are l = − ,m= ,n= ⋅
6 6 6
Again when m / n = 2 / 3, we have from (3)
l 2 1
=5× −3= ⋅
n 3 3
m 2 l 1
From = and = , we see that the d.c.’s of the other line are given by
n 3 n 3
l m n l2 + m2 + n2 1
= = = = ⋅
1 2 3 2 2
1 +2 +3 2 14
1 2 3
∴ The d.c.’s of the other line are l = ,m= ,n= ⋅
14 14 14
Hence the required d.c.’s of the two lines are
1 1 2 1 2 3
− , , and , , ⋅
6 6 6 14 14 14
Problem 4: Find the direction cosines l, m, n of the two lines which are connected by the
relations l + m + n = 0 and mn − 2nl − 2lm = 0.
(Gorakhpur 2005; Kanpur 07; Purvanchal 08; Kumaun 13)
Solution: The given relations are
l + m + n = 0 or l = − m − n …(1)
and mn − 2nl − 2lm = 0. …(2)
Putting the value of l from (1) in the relation (2), we get
mn − 2n ( − m − n) − 2 ( − m − n) m = 0 or 2m2 + 5mn + 2n2 = 0
G-42

or (2 m + n) ( m + 2 n) = 0.
m 1
∴ =− and − 2 .
n 2
From (1), we have
l −m − n m
= = − − 1. …(3)
n n n
m 1 l 1 1
Now when = − , (3) gives = − 1 = − ⋅
n 2 n 2 2
m n l n
∴ = and =
1 −2 1 −2
l m n √ ( l2 + m2 + n2 ) 1
i. e., = = = = ⋅
1 1 −2 √ {12 + 12 + ( −2)2 } √ 6
1 1 −2
∴ The d.c.’s of one line are , , ⋅
√6 √6 √6
m l
Again when = − 2 , (3) gives = 2 − 1 = 1.
2 n
l m n √ ( l2 + m2 + n2 ) 1
∴ = = = = ⋅
1 −2 1 √ {12 + ( −2)2 + 12 } √ 6
1 −2 1
∴ The d.c.’s of the other line are , , ⋅
√6 √6 √6
Problem 5: If P, Q, R, S are four points with co-ordinates (3, 4, 5), (4, 6, 3), ( −1, 2 , 4),
(1, 0, 5) respectively, then find the projection of PQ on RS. Also find the projection of RS on PQ.
(Agra 2001)
Solution: To find the projection of PQ on RS, we should find the d.c.’s of RS.
The direction ratios of RS are
1 − ( −1), 0 − 2 , 5 − 4 i. e., 2 , − 2 , 1.
Also RS = √ [{1 − ( −1)}2 + (0 − 2)2 + (5 − 4)2 ] = √ (4 + 4 + 1) = 3.
2 2 1
∴ d.c.’s of RS are , − , ⋅
3 3 3
Hence the projection of PQ on RS is (See article 6)
= l ( x2 − x1 ) + m ( y2 − y1) + n ( z2 − z1)
2 2 1 2 4 2 4
= (4 − 3) − (6 − 4) + (3 − 5) = − − = − ⋅
3 3 3 3 3 3 3
Again to find the projection of RS on PQ, we should find the d.c.’s of PQ.
The direction ratios of PQ are 4 − 3, 6 − 4, 3 − 5 i. e., 1, 2 , − 2 .
Also PQ = √ {(4 − 3)2 + (6 − 4)2 + (3 − 5)2 } = √ (1 + 4 + 4) = 3.
1 2 2
∴ The d.c.’s of PQ are , , − ⋅
3 3 3
∴ Projection of RS on PQ is [See article 6]
1 2 2 2 4 2 4
= {1 − ( −1)} + (0 − 2) − (5 − 4) = − − = − ⋅
3 3 3 3 3 3 3
G-43

Problem 6: Prove that sin2 α + sin2 β + sin2 γ = 2, where α, β, γ are the angles which the
given line makes with the positive directions of the axes. (Agra 2001)
Solution: Since cos α, cos β, cos γ are the direction cosines of the given line, therefore
cos2 α + cos2 β + cos2 γ = 1.
∴ (1 − sin2 α ) + (1 − sin2 β ) + (1 − sin2 γ ) = 1
or sin2 α + sin2 β + sin2 γ = 3 − 1 = 2.

Comprehensive Problems 2
Problem 1: If points P, Q are (2, 3, − 6),(3, − 4, 5), then find the angle between OP and OQ ,
where O is the origin.
Solution: The origin O is the point (0, 0, 0 ).
The direction ratios of OP are 2 − 0, 3 − 0, − 6 − 0 i. e., 2, 3, − 6.
We have 22 + 32 + ( − 6)2 = 7.
∴ The d.c.’s of OP are 2 / 7, 3 / 7, − 6 / 7.
The direction ratios of OQ are 3 − 0, − 4 − 0, 5 − 0 i. e., 3, − 4, 5.

We have 32 + ( − 4)2 + 52 = 50 = 5 2.
3 −4 5
∴ The d.c.’s of OQ are , , ⋅
5 2 5 2 5 2
If θ is the angle between OP and OQ, then
cos θ = l1 l2 + m1 m2 + n1 n2
− 4   6  5 
+ ⋅ 
2 3 3
= ⋅  + −   
7 5 2 7  5 2   7  5 2 
6 12 30 36 18 2
= − − =− =− ⋅
35 2 35 2 35 2 35 2 35
 18 2 
∴ θ = cos − 1  − ⋅
 35 
Problem 2: Prove that the line joining the points (1, 2, 3) and ( −1, − 2, − 3) is perpendicular to
the line joining the points ( −2, 1, 5) and (3, 3, 2).
Solution: The d.r.’s of the line joining the points (1, 2, 3) and ( −1, − 2, − 3) are
1 + 1, 2 + 2, 3 + 3 i.e., 2, 4, 6
and the d.r.’s of the other line joining the points ( −2, 1, 5) and (3, 3, 2) are
−2 − 3, 1 − 3, 5 − 2 i.e., −5, − 2, 3.
Both lines are perpendicular if
a1a2 + b1b2 + c1c2 = 0. [ By condition of ⊥ ]
G-44

∴ 2 × ( −5) + 4 × ( −2) + 6 × 3 = 0
or 0 = 0.
∴ Both given lines are perpendicular to each other.

Problem 3: Show that the three points A (6, − 7, − 1), B(2, − 3, 1) and C(4, − 5, 0 ) are
collinear.
Solution: Given three points A (6, − 7, − 1), B(2, − 3, 1) and C(4, − 5, 0 ).
The d.r.’s of line AB are
6 − 2, − 7 + 3, − 1 − 1
or 4, − 4, − 2
and the d.r.’s of line AC are
2 − 4, − 3 + 5, 1 − 0
or −2, 2, 1.
We see that d.r.’s of the two lines AB and AC are proportional because we have
4 −4 −2
= = each ratio being equal to −2.
−2 2 1
∴ Lines AB and AC are parallel.
But both lines AB and AC pass through the point A . So AB and AC are on the same
straight line. Hence the points A, B and C are collinear.

Problem 4: Prove that the three lines drawn from a point with direction cosines proportional to
1, − 1, 1 ; 2 , − 3, 0 and 1, 0, 3 are coplanar.
Solution: Let a, b, c be the direction ratios of the normal to the plane in which the two
concurrent lines with direction ratios 1, − 1, 1 and 2 , − 3, 0 lie.
Clearly these lines will be perpendicular to this normal. Hence applying the condition
for perpendicularity of two lines, we have
1 . a + ( −1) . b + 1 . c = 0, …(1)
and 2 . a + ( −3) . b + 0 . c = 0. …(2)
a b c a b c
Solving these, = = or = = ⋅
0 + 3 2 − 0 −3 + 2 3 2 −1
Again the third concurrent line with d.r.’s 1, 0, 3 will lie in this plane if the normal with
d.r.’s 3, 2 , − 1 is also perpendicular to this third line.
We have (3) (1) + (2) (0 ) + ( −1) (3) = 0, showing that the lines with d.r.’s 3, 2 , − 1 and
1, 0, 3 are perpendicular. Hence the three given lines are coplanar.

Problem 5: Show that the lines whose direction cosines are given by the equations
2l + 2m − n = 0, and mn + nl + lm = 0 are at right angles. (Kanpur 2008)
Solution: From 2l + 2m − n = 0, we have n = 2l + 2m. …(1)
Putting this value of n in mn + nl + lm = 0, we get
m (2l + 2m) + (2l + 2m) l + lm = 0
or 2l2 + 5lm + 2m2 = 0 or ( l + 2m) (2l + m) = 0.
∴ l = − 2m and 2l + m = 0.
G-45

When l = − 2 m, from (1), n = − 2 m.


l m n √ ( l2 + m2 + n2 ) 1
∴ = = = = ⋅
2 −1 2 √ {(2)2 + ( −1)2 + (2)2 } 3
2 1 2
∴ The d.c.’s of one line are ,− , ⋅
3 3 3
Again when 2 l = − m, from (1), n = m.
l m n √ ( l2 + m2 + n2 ) 1
∴ = = = = ⋅
−1 2 2 √ {( −1)2 + (2)2 + (2)2 } 3
1 2 2
∴ The d.c.’s of other line are − , , ⋅
3 3 3
The lines will be at right angles if l1l2 + m1m2 + n1n2 = 0.

l1l2 + m1m2 + n1n2 = ⋅  −  +  −    +     = 0.


2 1 1 2 2 2
We have
3  3  3  3  3  3
Hence the lines are at right angles.

Problem 6: Prove that the acute angle between the lines whose direction cosines are given by the
relations l + m + n = 0 and l 2 + m2 − n2 = 0 is π /3. (Meerut 2005B, 13)
Solution: The relations giving the d.c.’s of the two lines are
l + m+ n=0 …(1)
and l 2 + m2 − n2 = 0. …(2)
From (1), n = − ( l + m). Putting this value of n in (2), we get
l 2 + m2 − ( l + m)2 = 0 or 2lm = 0 or lm = 0.
∴ l = 0 or m = 0.
When l = 0, we have from (1), m = − n and when m = 0, we have from (1), l = − n.
∴ The direction ratios of the two lines are 0, − n, n and − n, 0, n or 0, − 1, 1and − 1, 0, 1.
If θ is the acute angle between the two lines, then
|(0 ) ( − 1) + ( − 1) (0 ) + (1) (1)| 1 1
cos θ = = = ⋅
2 2 2
02 + ( − 1)2 + 12 ( − 1)2 + 02 + 12
∴ θ = π /3
Problem 7: If l1, m1, n1 and l2 , m2 , n2 be the direction cosines of two mutually perpendicular
lines, show that the direction cosines of the line perpendicular to both of them are
m1n2 − m2 n1, n1l2 − n2 l1, l1m2 − l2 m1 . (Kumaun 2007)
Solution: Let l, m, n be the sines of the line perpendicular to each one of the two given
lines.
Then ll1 + mm1 + nn1 = 0 …(1)
and ll2 + mm2 + nn2 = 0. …(2)
Solving (1) and (2) by the method of cross multiplication, we get
G-46

l m n l 2 + m2 + n2
= = =
m1n2 − m2 n1 n1l2 − n2 l1 l1m2 − l2 m1 Σ ( m1n2 − m2 n1 )2
1
= , where θ is the angle between the given lines
sin θ
1
= = 1, because the given lines are given to
sin ( π / 2)
be perpendicular to each other.
∴ l = m1n2 − m2 n1, m = n1l2 − n2 l1, n = l1m2 − l2 m1 .

Problem 8: If a variable line in two adjacent positions has direction cosines l, m, n and
l + δl, m + δm, n + δn,
show that the small angle δθ between the two positions is given by
(δθ)2 = (δl )2 + (δm)2 + (δn)2 . (Avadh 2010)
Solution: Since l, m, n and ( l + δl ),( m + δm), ( n + δn)are the actual direction cosines, we
have
l 2 + m2 + n2 = 1 …(1)
2 2 2
and ( l + δl ) + ( m + δm) + ( n + δn) = 1. …(2)
Subtracting (1) from (2), we get
2l δl + 2m δm + 2n δn + (δl )2 + (δm)2 + (δn)2 = 0
or (δl )2 + (δm)2 + (δn)2 = − 2 ( l δl + m δm + n δn). …(3)
Since δθ is the angle between two lines whose d.c.’s are l, m, n and l + δl, m + δm, n + δn,
we have
cos δθ = l ( l + δl ) + m ( m + δm) + n ( n + δn)
= ( l 2 + m2 + n2 ) + l δl + m δm + n δn
1
=1− {(δl )2 + (δm)2 + (δn)2 }. [Using (3)]
2
1
∴ {(δl )2 + (δm)2 + (δn)2 } = 1 − cos δθ
2
δθ
or (δl )2 + (δm)2 + (δn)2 = 2 (1 − cos δθ) = 4 sin2  
2
2
δθ
= 4 ⋅  
2
 ∵ δθ being small, we have sin δθ = δθ 
 2 2 2 
= (δθ)2 .

Hence (δθ)2 = (δl )2 + (δm)2 + (δn)2 .


G-47

Problem 9: Show that the angle between any two diagonals of a cube is cos −1 (1 / 3).
(Garhwal 2001)

Solution: First find the d.c.’s of the four diagonals of the cube as in Example 10.
If θ is the acute angle between the diagonals OP and AE whose d.c.’s are
1 1 1 1 1 1
, , and − , , respectively, then
3 3 3 3 3 3
 1 ⋅ − 1  + 1 ⋅ 1 + 1 ⋅ 1 
cos θ =  = 1
 
 3  3 3 3 3 3 3
or θ = cos −1 (1 / 3).
Similarly the angle between each one of the other pairs is cos −1 (1 / 3).
Hence the angle between any two diagonals of a cube is cos −1 (1 / 3).
Problem 10: If ( l1, m1, n1 ), ( l2 , m2 , n2 ), ( l3 , m3 , n3 ) are the direction cosines of three
mutually perpendicular lines, then find the direction cosines of a line whose direction cosines are
proportional to l1 + l2 + l3 , m1 + m2 + m3 , n1 + n2 + n3 and prove that this line is equally
inclined to the given lines.
(Kanpur 2002)
Solution: Since l1, m1, n1 ; l2 , m2 , n2 ; l3 , m3 , n3 are the d.c.’s of three mutually
perpendicular lines, we have
l1l2 + m1m2 + n1n2 = 0, l2 l3 + m2 m3 + n2 n3 = 0, 

l3 l1 + m3 m1 + n3 n1 = 0. 
…(1)
Also 
l12 + m12 + n12 = l22 + m22 + n22 = l32 + m32 + n32 = 1. 
Now we have √ [( l1 + l2 + l3 )2 + ( m1 + m2 + m3 )2 + ( n1 + n2 + n3 )2 ]
= √ [( l12 + m12 + n12 ) + ( l22 + m22 + n22 ) + ( l32 + m32 + n32 )
+ 2 {( l1l2 + m1m2 + n1n2 ) + ( l2 l3 + m2 m3 + n2 n3 )
+ ( l3 l1 + m3 m1 + n3 n1 )}]
= √ [1 + 1 + 1 + 2 (0 + 0 + 0 )] [Using the relations (1)]
= √3.
Thus the required d.c.’s of the line are given by
l1 + l2 + l3 m1 + m2 + m3 n1 + n2 + n3
, , ⋅ …(2)
√3 √3 √3
Let θ be the angle between the lines whose d.c.’s are l1, m1, n1 and those given by (2). Then
we have
cos θ = l1 . ( l1 + l2 + l3 ) / √ 3 + m1 . ( m1 + m2 + m3 ) / √ 3
+ n1 . ( n1 + n2 + n3 ) / √ 3
= (1 / √ 3) [( l12 + m12 + n12 ) + ( l1l2 + m1m2 + n1n2 ) +
( l3 l1 + m3 m1 + n3 n1 )]
= (1 / √ 3) [1 + 0 + 0 ] . [Using the relations (1)]
G-48

∴ θ = cos −1 (1 / √ 3).
Similarly the angle between each of the lines with d.c.’s l2 , m2 , n2 ; l3 , m3 , n3 and
the line with d.c.’s given by (2) is cos −1 (1 / √ 3).
Problem 11: The direction cosines of two straight lines, inclined at an angle θ are l1 , m1 , n1
and l2 , m2 , n2 . Show that the direction cosines of the bisector of the angle between them are
l1 + l2 m + m2 n + n2
, 1 , 1 .
2 cos(θ /2) 2 cos (θ /2) 2 cos(θ /2)

Solution: Let l, m, n are the d.c.’s of the angle B


bisector between the lines OA and OB. Angle
between OA and OP; OB and OP are same and
θ
equal to , then θ/2
2
θ O P
cos = ll1 + mm1 + nn1 …(1) θ/2
2
θ
cos = ll2 + mm2 + nn2 . …(2)
2
Given l1, m1, n1 and d.c.’s of line OA and
A
l2, m2, n2 are d.c.’s of line OB .
On adding eq. (1) and eq. (2)
θ
l ( l1 + l2 ) + m( m1 + m2 ) + n( n1 + n2 ) = 2 cos
2
     
l + l  m + m   
l 1 2  + m  1 2 + n n1 + n2 = 1.
or    …(3)
θ
 2 cos  θ
 2 cos  θ
 2 cos 
 2   2   2
But l, m, n are d.c.’s of line OP.
∴ l2 + m2 + n2 = 1
or l ⋅ l + m ⋅ m + n ⋅ n = 1. …(4)
Thus eq. (3) and eq. (4) are just same if
l m n 1
= = = ⋅
l1 + l2 m1 + m2 n1 + n2 1
θ θ θ
2 cos 2 cos 2 cos
2 2 2
l + l m + m2 n1 + n2
∴ l = 1 2,m= 1 ,n= ⋅
θ θ θ
2 cos 2 cos 2 cos
2 2 2
So, d.c.’s of the angle bisector are
l1 + l2 m1 + m2 n1 + n2
, , .
θ θ θ
2 cos 2 cos 2 cos
2 2 2
G-49

Problem 12: If two pairs of opposite edges of a tetrahedron are perpendicular, then prove that
the third pair is also perpendicular. (Garhwal 2003)
Solution: Let OABC be the tetrahedron, where O, A, B and C are (0, 0, 0 ), ( x1, y1, z1),
( x2 , y2 , z2 ) and ( x3 , y3 , z3 ) respectively.
The d.r.’s of OA, OB and OC are x1, y1, z1 ; x2 , y2 , z2 and x3 , y3 , z3 respectively. Also
the d.r.’s of BC, CA and AB are
x3 − x2 , y3 − y2 , z3 − z2 , ; x1 − x3 , y1 − y3 , z1 − z3
and x2 − x1, y2 − y1, z2 − z1 respectively.
Let the edge OA be perpendicular to the opposite edge BC .
Then x1 ( x3 − x2 ) + y1 ( y3 − y2 ) + z1 ( z3 − z2 ) = 0. …(1)
Also if the edge OB is perpendicular to the opposite edge CA, then
x2 ( x1 − x3 ) + y2 ( y1 − y3 ) + z2 ( z1 − z3 ) = 0. …(2)
Adding (1) and (2), we have
x3 ( x1 − x2 ) + y3 ( y1 − y2 ) + z3 ( z1 − z2 ) = 0, …(3)
which shows that the third pair of opposite edges i. e., OC and AB is also perpendicular.

H ints to O bjective T ype Q uestions

Multiple Choice Questions


1. We have
sin2 α + sin2 β + sin2 γ = (1 − cos2 α ) + (1 − cos2 β ) + (1 − cos2 γ )
= 3 − (cos2 α + cos2 β + cos2 γ ) = 3 − 1 = 2 .
2. Direction ratios of the line joining the points (0, 0, 0 ) and (1, 1, 1) are
1 − 0, 1 − 0, 1 − 0 i. e., 1, 1, 1.
We have 12 + 12 + 12 = 3.
1 1 1
∴ d.c.’s of this line are , , ⋅
3 3 3
3. The equations of x-axis are y = 0 and z = 0.
4. See remark of article 2.
5. See article 4.
6. See remark of article 2.
7. (1, 1, 0 ) is a point in the co-ordinate plane xy since its z co-ordinate is zero.
8. Direction ratios of the line joining the points (1, − 1, 0 ) and (0, 0, 1) are
0 − 1, 0 + 1, 1 − 0, i. e., −1, 1, 1.
We have ( −1)2 + 12 + 12 = 3.
1 1 1
∴ d.c.'s of this line are − , , ⋅
3 3 3
G-50

9. See Problem 2 of Comprehensive Problems 1.


10. See article 5.

Fill in the Blank(s)


1. See article 2.

2. We have 22 + 32 + ( −6)2 = 7.
∴ Direction cosines of the line whose direction ratios are 2 , 3, − 6 are
2 3 6
, ,− ⋅
7 7 7
3. See Corollary 2 of article 7.
4. See article 8, Theorem 1, part (i).
5. See article 8, Theorem 1, part (iii).

True or False
1. See article 8, Theorem 1, part (ii).
2. See article 2.
3. See article 5.
4. See article 6, Theorem 1, Remark.
❍❍❍
G-51

Chapter-4
The Plane

Comprehensive Problems 1
Problem 1: (i) Reduce the equation of the plane x + 2 y − 2z − 9 = 0 to the normal form
and hence find the length of the perpendicular drawn from the origin to the given plane.
(ii) Find the perpendicular distance from the origin to the plane 2 x + y + 2z = 3. Find also
the direction-cosines of the normal to the plane.
Solution: (i) The equation of the given plane is
x + 2 y − 2z − 9 = 0.
Bringing the constant term to the R.H.S., the equation becomes
x + 2 y − 2z = 9. …(1)
[Note that in the equation (1) the constant term 9 is positive. If it were negative, we
would have changed the sign throughout to make it positive.]
Now the square root of the sum of the squares of the coefficients of x, y, z in (1)
= √ {(1)2 + (2)2 + ( −2)2 } = √ 9 = 3.
Dividing both sides of (1) by 3, we have
1 2 2
x+ y − z = 3. …(2)
3 3 3
The equation (2) of the plane is in the normal form lx + my + nz = p.
1 2 2
Hence the d.c.’s l, m, n of the normal to the plane are , , − and the length p of the
3 3 3
perpendicular from the origin to the plane is 3.
(ii) The equation of the plane is
2 x + y + 2z = 3.
To reduce it to the normal form dividing by
√ (4 + 1 + 4) = √ 9 = 3, we get 2 x / 3 + y / 3 + 2z / 3 = 1.
Hence the perpendicular distance of the plane from the origin is 1 and the
direction-cosines of the normal to the plane are 2 / 3, 1 / 3, 2 / 3.

Problem 2: (i) Find the equation to the plane through P(2, 3, − 1) at right angles to OP.
(Meerut 2007B)
(ii) Find the equation of the plane passing through the points (2, 2, − 1),(3, 4, 2) and (7, 0, 6).
G-52

Solution: (i) Here the lines OP is normal to the required plane. Direction ratio of the
line OP joining O(0, 0, 0 ) and P(2, 3, − 1) are 2 − 0, 3 − 0, − 1 − 0 i. e., 2, 3, − 1.
Hence the required equation of the plane passing through the point P(2, 3, − 1) and
perpendicular to OP is
2( x − 2) + 3( y − 3) + ( −1)( z + 1) = 0
or 2 x + 3 y − z = 14 .
(ii) The equation of any plane passing through the point (2, 2, − 1) is
a ( x − 2) + b ( y − 2) + c ( z + 1) = 0. …(1)
If the plane (1) passes through the points (3, 4, 2) and (7, 0, 6) also, then
a (3 − 2) + b (4 − 2) + c (2 + 1) = 0 i. e., a + 2b + 3c = 0 …(2)
and a (7 − 2) + b (0 − 2) + c (6 + 1) = 0 i. e., 5a − 2b + 7c = 0. …(3)
Solving (2) and (3) by cross-multiplication, we have
a b c a b c
= = or = =
14 + 6 15 − 7 − 2 − 10 20 8 − 12
a b c
or = = ⋅
5 2 −3
Since the R.H.S. of (1) is 0, therefore putting the proportionate values of a, b, c in (1),
the equation of the required plane is
5 ( x − 2) + 2 ( y − 2) − 3 ( z + 1) = 0
or 5 x + 2 y − 3z = 17.

Problem 3: (i) The foot of the perpendicular from the origin to a plane is (4, − 2, − 5). Find
the equation of the plane.
(ii) O is the origin and A is the point ( a, b, c ). Find the equation of the plane through A and at
right angles to OA.
Solution: (i) Let A(4, − 2, − 5) be any point on the plane. According to Question OA
is perpendicular to the plane then OA is normal to plane whose d.r.’s are
4 − 0, − 2 − 0, − 5 − 0
i.e. 4, − 2, − 5
then the required equation of the plane passing through the point A(4, − 2, − 5) and
perpendicular to OA is
4 ( x − 4) − 2 ( y + 2) − 5 ( z + 5) = 0
or 4 x − 2 y − 5z = 45 .
(ii) Here the line OA is normal to the required plane. Direction ratios of the line OA
joining O (0, 0, 0 ) and A ( a, b, c ) are a − 0, b − 0, c − 0 i. e., a, b, c.
Hence the required equation of the plane passing through the point A ( a, b, c ) and
perpendicular to OA is
a ( x − a) + b ( y − b ) + c ( z − c ) = 0
or ax + by + cz = a2 + b2 + c2 .
G-53

Problem 4: A plane makes intercepts 9, 9 /2, − 9 /2 upon the co-ordinate axes. Find the length
of the perpendicular from origin on it.
9 −9
Solution: The equation of the plane which cuts off intercepts 9, , from the
2 2
co-ordinate axis is
x y z
+ + =1 or x + 2 y − 2z = 9. …(1)
9 9 /2 −9 /2

Dividing by 12 + (2)2 + ( −2)2 = 3 both sides


x 2 y 2z 9
+ − =
3 3 3 3
which is the equation of plane in normal form
lx + my + nz = p.
9
Then the length of perpendicular from the origin to this plane = p = =3.
3

Problem 5: A plane meets the coordinate axes at A , B and C such that the centroid of the
triangle ABC is (1, − 2, 3). Find the equation of the plane.
Solution: Given G is centroid of ∆ ABC whose co-ordinates (1, − 2, 3). The plane meets
the co-ordinate axes at A, B and C.
Let the co-ordinates be A ( a, 0, 0 ), B(0, b, 0 )
and C (0, 0, c ) respectively. Z
a+0 +0
Then 1 = ⇒ a=3
3 C (0, 0, c)
0+ b+0
−2 = ⇒ b = −6
3
0+0+ c
3= ⇒ c = 9.
3 O G
∴ Eq. of plane in intercept form
(0, b, 0) B
x y z A
+ + =1 (a, 0, 0)
3 −6 9
or 6 x − 3 y + 2z = 18. Y

Problem 6: Find the equation of the plane perpendicular to the line segment from ( −3, 3, 2) to
(9, 5, 4) at the middle point of the segment.
Solution: The end points of the given line segment are ( −3, 3, 2) and (9, 5, 4). The d.r.’s
of the line segment are 9 − ( −3), 5 − 3, 4 − 2 i. e.,12 , 2 , 2 . The co-ordinates of the middle
point of the line segment are
 1 (9 − 3), 1 (5 + 3), 1 (4 + 2) i. e., (3, 4, 3).
 
2 2 2 
G-54

Thus the plane is to pass through the point (3, 4, 3) and d.r.’s of normal to the plane are
12 , 2 , 2 . Therefore the required equation of the plane is
12 ( x − 3) + 2 ( y − 4) + 2 ( z − 3) = 0
or 6 ( x − 3) + ( y − 4) + ( z − 3) = 0
or 6 x + y + z = 25.

Problem 7: Find the equation to plane through the points (1, 1, 0 ), (1, 2, 1), ( −2, 2, − 1).
(Agra 2006)
Solution: Given three points on the plane are
(1, 1, 0 ), (1, 2, 1) and ( −2, 2, − 1).
The eqn's of plane passing through three points is
x y z 1
x1 y1 z1 1
= 0. [Formula]
x2 y2 z2 1
x3 y3 z3 1

∴ Eq. of plane passing through three given points is


x y z 1
1 1 0 1
= 0.
1 2 1 1
−2 2 −1 1

Apply
R2 → R2 − R1
R3 → R3 − R2
R4 → R4 − R3
x y z 1
1− x 1− y − z
1− x 1− y −z 0
= 0, or 0 1 1 =0
0 1 1 0
−3 0 −2
−3 0 −2 0

or (1 − x )( −2) − (1 − y ) 3 − z(3) = 0
or 2 x + 3 y − 3z − 5 = 0 .

Problem 8: Show that the four points (0, − 1, − 1), (4, 5, 1), (3, 9, 4) and ( − 4, 4, 4) are
coplanar. (Purvanchal 2011; Avadh 12)
Solution: We shall first find the equation of the plane passing through the first three
given points and then show that the fourth point also satisfies this equation.
The equation of any plane through (0, − 1, − 1) is
a ( x − 0 ) + b ( y + 1) + c ( z + 1) = 0. …(1)
If the plane (1) passes through the points (4, 5, 1) and (3, 9, 4) also, then
G-55

a (4 − 0 ) + b (5 + 1) + c (1 + 1) = 0
i. e., 4a + 6b + 2c = 0 or 2a + 3b + c = 0 …(2)
and a (3 − 0 ) + b (9 + 1) + c (4 + 1) = 0 i. e., 3a + 10 b + 5c = 0. …(3)
Solving (2) and (3) by cross-multiplication, we have
a b c a b c
= = or = = ⋅
15 − 10 3 − 10 20 − 9 5 − 7 11
Putting the proportionate values of a, b, c in (1), the equation of the plane passing
through the points (0, − 1, − 1), (4, 5, 1) and (3, 9, 4,) is
5 ( x − 0 ) − 7 ( y + 1) + 11 ( z + 1) = 0
or 5 x − 7 y + 11z + 4 = 0. …(4)
Putting x = − 4, y = 4, z = 4, the L.H.S. of (4)
= 5 . ( − 4) − 7 . 4 + 11 . 4 + 4 = 0 = the R.H.S. of (4).
Hence the equation (4) is satisfied by the point ( − 4, 4, 4) i. e., the point ( − 4, 4, 4) also lies
on the plane (4). Therefore the given four points are coplanar. The equation (4) is the
equation of the plane containing the given four points.
Remark: If the given four points are A, B, C and D, then their coplanarity can also be
→ → →
established by showing that the scalar triple product [ AB, AC, AD] is zero.

Problem 9: Show that the four points (0 , − 1, 0 ), (2, 1, − 1 ), (1, 1, 1 ) and (3, 3, 0 ) are
co-planar and hence show that the equation of the plane passing through these points is
4 x − 3 y + 2z = 3 . (Kanpur 2011)
Solution: We shall first find the equation of the plane passing through the first three
given points and then show that the fourth point also satisfies this equation.
Now the eq. of plane through (0, − 1, 0 ) is
a( x − 0 ) + b ( y + 1) + c ( z − 0 ) = 0. …(1)
This plane (1) passes through the points (2, 1, − 1 ) and (1, 1, 1 ) also, then
a(2 − 0 ) + b (1 + 1) + c( −1 − 0 ) = 0
or 2a + 2b − c = 0 …(2)
and a(1 − 0 ) + b (1 + 1) + c(1 − 0 ) = 0
or a + 2b + c = 0. …(3)
By cross-multiplication in eqs. (2) and (3) we get
a b c a b c
= = or = = ⋅
2 + 2 −1 − 2 4 − 2 4 −3 2
The R.H.S. part of eq. (1) is 0, so put a = 4, b = − 3, c = 2 in eq. (1), we get
4( x − 0 ) − 3( y + 1) + 2( z − 0 ) = 0
or 4 x − 3 y + 2z = 3. …(4)
The fourth point (3, 3, 0 ) satisfies the eq. (4)
4×3−3×3+ 2×0 =3 or 3=3
so the given four points are coplanar.
G-56

Problem 10: Find the equation of the plane through (1, 0, − 2) and perpendicular to each of the
planes 2 x + y − z − 2 = 0 and x − y − z − 3 = 0. (Rohilkhand 2009B)
Solution: The equation of any plane through the point (1, 0, − 2) is
a ( x − 1) + b ( y − 0 ) + c ( z + 2) = 0. …(1)
If the plane (1) is perpendicular to the planes 2 x + y − z − 2 = 0 and x − y − z − 3 = 0,
we have [See article 12]
a (2) + b (1) + c ( − 1) = 0 i. e., 2 a + b − c = 0, …(2)
and a (1) + b ( −1) + c ( −1) = 0
i. e., a − b − c = 0. …(3)
3
Adding the equations (2) and (3), we have c = a.
2
1
Subtracting (3) from (2), we have b = − a.
2
Putting the values of b and c in (1), the equation of the required plane is given by
1 3
a ( x − 1) − ay + a ( z + 2) = 0
2 2
or 2 x − 2 − y + 3z + 6 = 0 or 2 x − y + 3z + 4 = 0.

Problem 11(i): Find the equation of the plane through the points (1, − 2 , 2), ( −3, 1, − 2) and
x + 2 y − 3z = 5.
perpendicular to the plane (Meerut 2009; Rohilkhand 13)
Solution: The equation of any plane passing through the point (1, − 2 , 2) is
a ( x − 1) + b ( y + 2) + c ( z − 2) = 0. …(1)
If the plane (1) passes through the point ( −3, 1, − 2), we have
a ( −3 − 1) + b (1 + 2) + c ( −2 − 2) = 0
or −4a + 3b − 4c = 0 or 4a − 3b + 4c = 0. …(2)
If the plane (1) is perpendicular to the plane x + 2 y − 3z = 5, we have
a . 1 + b . 2 + c . ( −3) = 0 or a + 2b − 3c = 0. …(3)
Solving the equations (2) and (3) for a, b, c , we have
a b c
= =
( −3)( −3) − 2 . (4) 1 × 4 − 4 . ( −3) 4 × 2 − 1 . ( −3)
a b c
or = = = λ, (say).
1 16 11
∴ a = λ, b = 16 λ , c = 11 λ .
Putting the values a, b, c in (1), the equation of the required plane is
λ [( x − 1) + 16 ( y + 2) + 11 ( z − 2)] = 0
or x + 16 y + 11z + 9 = 0.

Problem 11(ii): Find the equation to the plane through the points ( −1, 1, 1) and (1, − 1, 1) and
perpendicular to the plane x + 2 y + 2z = 5. (Kumaun 2009)
Solution: The equation of any plane passing through the point (–1, 1, 1) is
a( x + 1) + b ( y − 1) + c ( z − 1) = 0. …(1)
G-57

If plane (1) passes through the point (1, –1, 1) we have


a(1 + 1) + b ( −1 − 1) + c (1 − 1) = 0
or 2a − 2b + 0 c = 0. …(2)
If the plane (1) is perpendicular to the plane x + 2 y + 2z = 5, we have
a ⋅1 + b ⋅ 2 + c ⋅ 2 = 0
or a + 2b + 2c = 0. …(3)
Solving the equation (2) and (3) for a, b, c we have
a b c
= =
(2)( −2) − 0 ⋅ 2 a ⋅ 0 − 2 ⋅ 2 2 ⋅ 2 − 1⋅ ( −2)
a b c
or = = = λ (say)
−4 −4 6
∴ a = − 4λ, b = − 4λ, c = 6λ .
Putting the values of a, b, c in (1), the equation of the required plane is
λ[ − 4( x + 1) − 4( y − 1) + 6( z − 1) = 0 or 4 x + 4 y − 6z + 6 = 0.

Problem 11(iii): Find the equation of the plane through the points (1, − 2, 4 ) and (3, − 4, 5 )

and parallel to the x-axis ( i. e., perpendicular to the yz-plane). (Kumaun 2007, 13)

Solution: The equation of any plane through the point (1, − 2 , 4) is


a ( x − 1) + b ( y + 2) + c ( z − 4) = 0. …(1)
If the plane (1) also passes through the point (3, − 4, 5), we have
a (3 − 1) + b ( −4 + 2) + c (5 − 4) = 0 or 2 a − 2 b + c = 0. …(2)
Now the plane (1) is to be parallel to the x-axis i. e., perpendicular to the y z-plane whose
equation is
x = 0 i. e., 1 . x + 0 . y + 0 . z = 0.
Hence we have
a . 1 + b . 0 + c . 0 = 0 or a = 0. …(3)
Putting the value of a from (3) in (2), we get c = 2 b.
Substituting the values of a and c in the equation (1), the equation of the required plane
is
0 + b ( y + 2) + 2 b ( z − 4) = 0 or y + 2 z − 6 = 0.

Problem 11(iv): Find the equation of the plane passing through (2, 3, − 4 ) and (1, − 1, 3 ) and
parallel to the x-axis. (Kumaun 2014)
Solution: The equation of any plane through the point (2, 3, − 4) is
a ( x − 2) + b ( y − 3) + c ( z + 4) = 0. …(1)
If the plane (1) also passes through the point (1, –1, 3), we have
a(1 − 2) + b ( −1 − 3) + c (3 + 4) = 0
or − a − 4b + 7c = 0. …(2)
G-58

Now the plane (1) is to be parallel to the x-axis, i. e., perpendicular to the yz-plane whose
equation is
x = 0, i. e.,1⋅ x + 0 ⋅ y + 0 ⋅ z = 0.
Hence, we have
a ⋅1 + b ⋅ 0 + c ⋅ 0 = 0 or a = 0. …(3)
4b
Putting the value of a from (3) in (2), we get c = ⋅
7
Substituting the values of a and c in the equation (1), the equation of the required plane
is
4
0 + b ( y − 3) + b ( z + 4) = 0 or 7 y + 4z − 5 = 0.
7
Problem 11(v): Find the equation to the plane through the points ( 2, 3, 1) and ( 4, − 5, 3 ) ,
parallel to the x -axis. (Kanpur 2015)
Solution: Proceed as in part (iv). Ans. y + 4z − 7 = 0.

Problem 12(i): Find the angle between the planes 2 x − y + z = 11 and x + y + 2z = 3.


(Rohilkhand 2012)
Solution: The angle θ between the given planes in the angle between their normals
whose d.r.’s are 2, − 1, 1 and 1, 1, 2.
(2)(1) + ( −1)(1) + (1)( 2 ) 3 1
∴ cos θ = = =
2 2 2 2 2
(2) + ( −1) + (1) (1) + (1) + (2) 2 6 6 2

π
or θ= .
3
π
Hence the angle between the given planes is .
3

Problem 12(ii): Find the angle between the planes 3 x + 4 y − 5z = 3 and 2 x + 6 y + 6z = 7.


(Kumaun 2007)
Solution: Proceed as in part (i). Ans. π /2.

Problem 13: Find the distance of the point P(2, 1, − 1 ) from the plane x − 2 y + 4z = 9.
(Rohilkhand 2010)
Solution: The distance of the point P(2, 1, − 1 ) from the plane x − 2 y + 4z − 9 = 0
is
(2) − 2(1) + 4( −1) − 9 −13 13
= = = .
1 + 4 + 16 21 21

Problem 14(i): Find the distance between the parallel planes


2x − 2 y + z + 3 = 0 and 4 x − 4 y + 2z + 7 = 0. (Rohilkhand 2010)
G-59

Solution: The given planes are


2x − 2 y + z + 3 = 0 and 4 x − 4 y + 2z + 7 = 0.
These can be written in the normal form as
2x − 2 y + z + 3 4 x − 4 y + 2z + 7
=0 and =0
2 2 2
(2) + ( −2) + (1) (4)2 + ( −4)2 + (2)2
2 2 1 4 4 2 7
or x − y + z +1=0 x − y + z + = 0.
and
3 3 3 6 6 6 6
7
∴ The length of perpendiculars from origin to these planes are 1 and .
6
So these planes are on the same side of the origin.
7 −1 1
∴ The required distance = 1 − = = .
6 6 6

Problem 14(ii): Find the distance between the parallel planes


2 x − 2 y + z + 1 = 0 and 4 x − 4 y + 2z + 3 = 0. (Kumaun 2014)
Solution: Let P ( x1, y1, z1) be any point on the plane 2 x − 2 y + z + 1 = 0. Then
2 x1 − 2 y1 + z1 + 1 = 0
or 2 x1 − 2 y1 + z1 = −1. …(1)
The distance between the given parallel planes
= length of the perpendicular from P to the plane 4 x − 4 y + 2z + 3 = 0
|4 x1 − 4 y1 + 2z1 + 3| |2(2 x1 − 2 y1 + z1 ) + 3|
= =
√ {(4)2 + ( −4)2 + (2)2 } √ (16 + 16 + 4)
|2( −1) + 3|
= [ Using (1)]
√ (36)
1
= ⋅
6
Problem 15(i): Find the equations of the planes parallel to the plane x + 2 y − 2z + 8 = 0
which are at a distance of 2 units from the point (2, 1, 1 ).
Solution: The equation of plane parallel to plane x + 2 y − 2z + 8 = 0 is
x + 2 y − 2z + K = 0. …(1)
Given the distance from the point (2, 1, 1 ) to the plane (1) is 2 units
 2 + 2(1) − 2(1) + K  2 + K
∴ 2=±   or 2 = ±  
 1+ 4 + 4   3 

or 6 = ± (2 + K )
or K = −2 ± 6
or K = − 8, 4.
Putting the values of K in eq. (1), we get the equation of required planes given by
x + 2 y − 2z − 8 = 0 and x + 2 y − 2z + 4 = 0.
G-60

Problem 15(ii): Find the equations of the planes parallel to the plane x − 2 y + 2z − 3 = 0
which are at a unit distance from the point (1, 2, 3).
Solution: The equation of any plane parallel to the plane x − 2 y + 2z − 3 = 0 is
x − 2 y + 2z + λ = 0. …(1)
If the distance of the point (1, 2, 3) from the plane (1) is 1, then
|1 − 2 (2) + 2 (3) + λ | |3 + λ |
= 1 or = 1 or |λ + 3| = 3.
2 2
1 + ( − 2) + 2 2 3

∴ λ + 3 = ± 3 i. e., λ = 0 or λ = − 6.
Putting λ = 0 and λ = − 6 successively in (1), the required planes are
x − 2 y + 2z = 0 and x − 2 y + 2z − 6 = 0.

Problem 15(iii): Find the equation of the plane that passes through the point (2, 3, 4) and is
parallel to the plane 5 x − 6 y + 7z = 3. (Kumaun 2015)

Solution: The equation of any plane parallel to the plane 5 x − 6 y + 7z − 3 = 0 is


5 x − 6 y + 7z + λ = 0. …(1)
If the plane (1) passes through the point (2, 3, 4), we have
5(2) − 6(3) + 7(4) + λ = 0 or λ = −20.
Substituting this value of k in the equation (1), the equation of the required plane
is
5 x − 6 y + 7z − 20 = 0.

Problem 15(iv): Find the equation of the plane through (0, 1, − 2) and parallel to the plane
2 x − 3 y + 4z = 0. (Kumaun 2008)
Solution: Proceed as in part (iii). We have
2 x − 3 y + 4z + λ = 0
or 2(0 ) − 3(1) + 4( −2) + λ = 0 or λ = 11.
The equation of the plane is
2 x − 3 y + 4z + 11 = 0.

Problem 16: A variable plane is at a constant distance p from the origin and meets the axes in
A, B and C. Show that the locus of the centroid of the triangle ABC is x −2 + y −2 + z −2 = 9 p−2 .
(Meerut 2005B, 07B, 13; Kanpur 09,10)
Solution: Proceed as in Example 8.

Problem 17: A variable plane is at a constant distance p from the origin and meets the
coordinate axes in A, B, C. Show that the locus of the centroid of the tetrahedron OABC is
x − 2 + y − 2 + z − 2 = 16 p− 2 .
(Kanpur 2005; Agra 06; Meerut 06, 07; Avadh 13)
G-61

Solution: Let the equation of the variable plane be


x y z
+ + = 1, …(1)
a b c
where a, b, c are parameters i. e., a, b, c are variables.
The plane (1) is at a constant distance p from the origin O, whatever a, b, c may be.
1
∴ = p2
(1 / a)2 + (1 / b )2 + (1 / c )2
1 1 1 1
or + + = ⋅ …(2)
2 2 2
a b c p2
The plane (1) meets the coordinate axes at the points A ( a, 0, 0 ), B (0, b, 0 ), C (0, 0, c ).
Let ( x1, y1, z1) be the coordinates of the centroid of the tetrahedron OABC, where
O (0, 0, 0 ) is the origin. Then
0 + a+0 +0 a 0+0+ b+0 b c
x1 = = , y1 = = , z1 = ⋅
4 4 4 4 4
∴ a = 4 x1, b = 4 y1, c = 4z1 .
Putting these values of a, b, c in (2), we get
1 1 1 1
+ + = or x1− 2 + y1− 2 + z1− 2 = 16 p− 2 .
16 x12 16 y12 16 z12 p2
Hence the locus of the point ( x1, y1, z1 ) is x − 2 + y − 2 + z − 2 = 16 p− 2 .

Comprehensive Problems 2
Problem 1: Find the equation of the plane passing through the line of intersection of the planes
2 x − 7 y + 4z = 3, 3 x − 5 y + 4 z + 11 = 0, and the point ( −2 , 1, 3).
(Bundelkhand 2005)
Solution: The equation of any plane through the line of intersection of the given
planes is [See article 16]
(2 x − 7 y + 4z − 3) + λ (3 x − 5 y + 4z + 11) = 0. …(1)
If the plane (1) passes through the point ( −2 , 1, 3), then substituting the co-ordinates of
this point in the equation (1), we have
{2 ( −2) − 7 (1) + 4 (3) − 3} + λ {3 ( −2) − 5 (1) + 4 (3) + 1} = 0
or ( −2) + λ (12) = 0 or λ = 1 / 6.
Putting this value of λ in (1), the equation of the required plane is
(2 x − 7 y + 4z − 3) + (1 / 6) (3 x − 5 y + 4z + 11) = 0
or 15 x − 47 y + 28z = 7.
Problem 2: Find the equation of the plane through the line of intersection of the planes
ax + by + c z + d = 0 and αx + βy + γz + δ = 0 and parallel to x-axis.
(Bundelkhand 2006, 14)
Solution: The equation of any plane through the line of intersection of the given
planes is
ax + by + cz + d + λ (αx + βy + γz + δ) = 0 …(1)
or x ( a + α λ ) + y ( b + βλ ) + z ( c + γ λ ) + ( d + δλ ) = 0. …(2)
G-62

Now the d.c.’s of the x-axis are 1, 0, 0 and the d.r.’s of the normal to the plane (2) are
a + α λ , b + β λ , c + γ λ . The plane (2) is parallel to the x-axis if the normal to the plane
(2) is perpendicular to the x-axis, the condition for which is
1 . ( a + αλ ) + 0 . ( b + βλ ) + 0 . ( c + γ λ ) = 0, giving λ = − a / α .
Putting this value of λ in the equation (1), the required equation of the plane is given by
α ( ax + by + cz + d ) − a (αx + βy + γz + δ) = 0
or ( bα − aβ ) y + ( cα − aγ ) z + ( dα − aδ) = 0.

Problem 3: Find the equation of the plane which is perpendicular to the plane
5 x + 3 y + 6z + 8 = 0
and which contains the line of intersection of the planes x + 2 y + 3z − 4 = 0, 2 x + y − z + 5 = 0.
(Rohilkhand 2009)
Solution: Proceed as in Example 10. Ans. 33 x + 45 y + 50 z − 41 = 0.

Problem 4: A variable plane at a constant distance p from the origin meets the axes in A, B and
C. Through A, B, C planes are drawn parallel to the co-ordinate planes. Show that the locus of
their point of intersection is x −2 + y −2 + z −2 = p−2 . (Rohilkhand 2008B)
Solution: Let the equation of the variable plane be
x y z
+ + = 1, …(1)
a b c
where a, b, c are variables.
The plane (1) meets the co-ordinate axes in the points A, B and C whose co-ordinates
are ( a, 0, 0 ) , (0, b, 0 ) and (0, 0, c ) respectively.
It is given that the length of the perpendicular from (0, 0, 0 ) to the plane (1) is p.
1 1 1 1 1
∴ p= or = + + ⋅ …(2)
√ {(1 / a)2 + (1 / b )2 + (1 / c )2 } p 2 a2 b2 c2
Now we shall find the equation of the plane through the point A ( a, 0, 0 ), and parallel to
the yz-plane.
The equation of the yz-plane is x = 0.
Any plane parallel to the plane x = 0 is given by x = λ .
If it passes through the point A ( a, 0, 0 ), we have a = λ .
Hence the equation of the plane through A and parallel to the yz-plane is
x = a. …(3)
Similarly the equations of the planes through the points B and C and parallel
respectively to the co-ordinate planes y = 0 and z = 0 are
y=b …(4)
and z =c. …(5)
The locus of the point of intersection of the planes (3), (4) and (5) is obtained by
eliminating a, b, c between the equations (2), (3), (4) and (5).
Putting the values of a, b, c from (3), (4) and (5) in (2), the required locus is given by
1 1 1 1
= + + or p−2 = x −2 + y −2 + z −2 .
p2 x2 y2 z2
G-63

Problem 5: A variable plane passes through a fixed point (α, β, γ ) and meets the axes of
reference in A, B, C. Show that the locus of the point of intersection of the planes through A, B, C
parallel to the co-ordinate planes is
α x −1 + β y −1 + γ z −1 = 1. (Meerut 2009B; Kumaun 12)
Solution: Let the equation of the variable plane be
x y z
+ + = 1, …(1)
a b c
where a, b, c are parameters i. e., variables.
The plane (1) passes through the point (α, β, γ ).
α β γ
∴ + + = 1. …(2)
a b c
The plane (1) meets the co-ordinate axes in the points A, B and C whose co-ordinates
are respectively given by ( a, 0, 0 ), (0, b, 0 ) and (0, 0, c ). The equations of the planes
through A, B and C and parallel to the co-ordinate planes are
x = a, y = b, z = c respectively. …(3)
[See Problem 4 above]
The locus of the point of intersection of these planes [given by the equations (3)] is
obtained by eliminating the parameters a, b, c between the equations (2) and (3).
Putting the values of a, b, c from (3) in (2), the required locus is given by
α β γ
+ + = 1 or αx −1 + βy −1 + γz −1 = 1.
x y z
x y z
Problem 6: A point P moves on the plane + + = 1 which is fixed. The plane through P
a b c
perpendicular to OP meets the co-ordinate axes in A, B and C. The planes through A, B and C
parallel to the yz, zx and xy-planes intersect in Q. Prove that if the axes be rectangular, the locus of
Q is
1 1 1 1 1 1
+ + = + + ⋅
x2 y2 z 2 ax by cz
Solution: The equation of the plane is
x y z
+ + = 1. …(1)
a b c
Let the co-ordinates of the point P be (α , β, γ ). Since the point P (α , β, γ ) lies on the
plane (1), we have
α β γ
+ + = 1. …(2)
a b c
The direction ratios of OP are α − 0, β − 0, γ − 0 i. e., α , β, γ. Hence the equation of the
plane passing through the point P (α , β, γ ) and perpendicular to OP is
α ( x − α ) + β ( y − β ) + γ ( z − γ ) = 0,
or α x + βy + γz = α2 + β2 + γ 2 . …(3)
The plane (3) meets the axes in the points A, B and C whose co-ordinates are
respectively given by
G-64

({α2 + β2 + γ 2 } / α , 0, 0 ), (0, {α2 + β2 + γ 2 } / β, 0 )


and (0, 0, {α2 + β2 + γ 2 } / γ ).
Again the equation of the plane through A and parallel to the yz-plane i. e., the plane
x = 0 is
x = (α2 + β2 + γ 2 ) / α . …(4)
Similarly the equations of the other two planes are
y = (α2 + β2 + γ 2 ) / β …(5)
2 2 2
and z = (α + β + γ ) / γ. …(6)
Now Q is the point of intersection of the planes (4), (5) and (6). The locus of the point Q
is obtained by eliminating α , β, γ between the equations (2), (4), (5) and (6).
From (4), (5) and (6), we have
1 1 1 α2 β2
+ + = +
2 2 2 2 2 2 2
x y z (α + β + γ ) (α + β2 + γ 2 )2
2

γ2
+
(α + β2 + γ 2 )2
2

α2 + β2 + γ 2
1
= = …(7)
(α2 + β2 + γ 2 )2 α2 + β2 + γ 2
1 1 1 α /a + β /b + γ /c 1
and + + = = ⋅ [ Using (2)]
2 2 2
ax by cz α +β + γ α + β2 + γ 2
2
…(8)

From (7) and (8), the required locus of Q is given by


1 1 1 1 1 1
+ + = + + ⋅
x2 y2 z 2 ax by cz
Problem 7: Find the equations of the bisector planes of the angles between the planes
3 x − 2 y + 6z + 8 = 0 and 2 x − y + 2z + 3 = 0. (Purvanchal 2010)
Solution: The given equations of planes are
3 x − 2 y + 6z + 8 = 0 …(1)
and 2 x − y + 2z + 3 = 0. …(2)
The equations of the bisector planes of the angle between the planes (1) and (2) are
3 x − 2 y + 6z + 8 2 x − y + 2z + 3

9 + 4 + 36 4 +1+ 4
3 x − 2 y + 6z + 8 2 x − y + 2z + 3
or =± ⋅ …(3)
7 3
We take (+) ive
3(3 x − 2 y + 6z + 8) = 7(2 x − y + 2z + 3)
or 5 x − y − 4z − 3 = 0. …(4)
We take (–) ive
3(3 x − 2 y + 6z + 8) = − 7(2 x − y + 2z + 3)
or 23 x − 13 y + 32z + 45 = 0. …(5)
Here plane (4) and plane (5) are the required equations of planes.
G-65

Problem 8: Find the equation of the plane that bisects the angle between the planes
3 x − 6 y + 2z + 5 = 0 and 4 x − 12 y + 3z = 3 which contains the origin. Is this the plane that
bisects the obtuse angle ? (Avadh 2011)
Solution: First making the constant term in both planes positive, we get
3 x − 6 y + 2z + 5 = 0 and −4 x + 12 y − 3z + 3 = 0.
Therefore the equation of the plane that bisects the angle between the given planes
which contains origin is given by
3 x − 6 y + 2z + 5 −4 x + 12 y − 3z + 3
=+
9 + 36 + 4 16 + 144 + 9
1 1
or (3 x − 6 y + 2z + 5) = ( −4 x + 12 y − 7z + 3)
7 13
or 13(3 x − 6 y + 2z + 5) = 7( −4 x + 12 y − 7z + 3)
or 67 x − 162 y + 47z + 44 = 0.

Problem 9: Find the equations of the bisectors of the angles between the planes
2 x − y − 2z − 6 = 0 and 3 x + 2 y − 6z − 12 = 0 and distinguish them.
(Purvanchal 2010)
Solution: Writing the given equations in such a way that the constant terms are both
positive, the equations of the given planes are
−2 x + y + 2z + 6 = 0, …(1)
and −3 x − 2 y + 6z + 12 = 0. …(2)
The equation of the bisector plane of the angle between the planes (1) and (2) which
contains the origin is given by
−2 x + y + 2z + 6 −3 x − 2 y + 6z + 12
=
√ (4 + 1 + 4) √ (9 + 4 + 36)
or 7 ( −2 x + y + 2z + 6) = 3 ( −3 x − 2 y + 6z + 12)
or 5 x − 13 y + 4z − 6 = 0. …(3)
The equation of the other bisector plane is
−2 x + y + 2z + 6 −3 x − 2 y + 6z + 12
=−
√ (4 + 1 + 4) √ (9 + 4 + 36)
or 7 ( −2 x + y + 2z + 6) = − 3 ( −3 x − 2 y + 6z + 12)
or 23 x − y − 32z − 78 = 0. …(4)
Now let θ be the acute angle between the plane (4) and the bisecting plane (3). Then
 −2 . 5 + 1 . ( −13) + 2 . 4  15 5
cos θ =  = = ⋅
 √ (4 + 1 + 4 ) √ (25 + 169 + 16 
) 3 √ (210 ) 42

∴ tan θ = √ (sec2 θ − 1) = √ [(42 / 5) − 1] = √ (37 / 5) > 1, so that θ > 45° .


Hence the plane 5 x − 13 y + 4z − 6 = 0 bisects the obtuse angle between the given
planes (1) and (2) so that the other plane 23 x − y − 32 z − 78 = 0 bisects the acute
angle.
G-66

We also note from the given planes (1) and (2), that
a1a2 + b1b2 + c1c2 = ( −2) ( −3) + 1 ( −2) + 2 (6)
= 6 − 2 + 12 = 16 = positive.
Hence the origin lies in the obtuse angle between the given planes.
[See remark 2 of article 19]
This confirms that the plane (3) is the bisector of the obtuse angle. Hence the equation
(4) is the bisector of the acute angle.

Problem 10: Prove that the equation x2 + 4 y2 − z 2 + 4 xy = 0 represents a pair of planes


and find the angle between them. (Rohilkhand 2008)
Solution: Comparing the given equation with the homogeneous equation of second
degree in x, y, z [See equation (1), article 20], we have
a = 1, b = 4, c = − 1, f = 0, g = 0, h = 2 .
∴ abc + 2 fgh − af 2 − bg2 − ch2

= 1 . 4 . ( −1) + 2 . 0 . 0 . 2 − 0 − 0 − ( −1) (2)2 = − 4 + 4 = 0.


Hence the given equation represents a pair of planes.
If θ is the angle between the planes, then
2 √ ( f 2 + g2 + h2 − bc − ca − ab )
tan θ =
a+ b+ c
[See equation (4), article 20]
2 √ (0 + 0 + 4 + 4 + 1 − 4)
= , putting for a, b, c etc.
1+ 4 −1
1
= √ 5.
2
∴ θ = tan  √ 5 ⋅
−1  1
2 
Alternative method: The given equation may be written as
( x2 + 4 xy + 4 y2 ) − z 2 = 0 or ( x + 2 y )2 − z 2 = 0
or ( x + 2 y + z ) ( x + 2 y − z ) = 0.
∴ x + 2 y + z = 0, x + 2 y − z = 0.
These being linear equations in x, y, z represent two planes. If θ is the angle between
these planes, then
1 . 1 + 2 . 2 + 1 . ( −1) 4 2
cos θ = = = ⋅
√ (1 + 4 + 1) √ (1 + 4 + 1) 6 3
1
∴ tan θ = √ 5.
2

a b c
Problem 11: Show that the equation + + = 0 represents a pair of planes.
y−z z− x x− y
(Gorakhpur 2006; Kanpur 06; Rohilkhand 10)
G-67

Solution: Multiplying the given equation by ( y − z ) ( z − x ) ( x − y ), we have


a (z − x) ( x − y ) + b ( y − z ) ( x − y ) + c ( y − z ) (z − x) = 0
or a ( zx − yz − x2 + xy ) + b ( xy − y2 − z x + yz )
+ c ( yz − xy − z 2 + zx ) = 0
or ax2 + by2 + cz 2 − ( b + c − a) yz − ( c + a − b ) zx − ( a + b − c ) xy = 0.
…(1)
Comparing the equation (1) with the general homogeneous equation of second degree
in x, y, z i. e., the equation
Ax2 + By2 + Cz 2 + 2 Fyz + 2 Gzx + 2 Hxy = 0,
[Refer equation (1) of article 20]
1
we have A = a, B = b, C = c , F = − ( b + c − a),
2
1 1
G = − ( c + a − b ), H = − ( a + b − c ).
2 2
[Note that we have used capital letters A, B, C etc. because small letters a, b, c etc. are
used in the question.]
The equation (1) will represent a pair of planes if
 A H G
H B F  = 0. [Refer article 20]
 
 G F C
Putting for A, B, C etc. we have
 A H G
H B F
 
G F C
 1 − (c + a − b) 
1
 a − (a + b − c ) 2 
 2 
1 1
= − (a + b − c ) b − ( b + c − a) 
2 2
 1 
− 1 − ( b + c − a) c 
(c + a − b) 2
 2 

 
 0 0 0 
 1 1 
= − (a + b − c ) b − ( b + c − a) ,
2 2
 1 
− 1 − ( b + c − a) c 
(c + a − b) 2
 2 
adding the second and third rows to the first row
= 0.
Hence the given equation represents a pair of planes.
G-68

Problem 12: Find the area of the triangle whose vertices are A (1, 2 , 3), B (2 , − 1, 1) and
C (1, 2 , − 4). (Meerut 2013)
Solution: Let ∆ x , ∆ y , ∆ z be the areas of the projections of the area ∆ of triangle ABC
on the yz , zx and xy-planes respectively.
We have
 y1 z1 1  2 3 1
1 1 21
∆ x =  y2 z2 1 = −1 1 1 = ,
2  2  2
 y3 z3 1  2 −4 1
 1 1 
x z 1 1 3 1 
1 1 7
∆ y =  x2 z2 1 = 2 1 1 = (numerically),
2  2  2
 x3 z3 1 1 − 4 1 
 1
x y 1 1  1 2 1 
1 1
and ∆ z =  x2 y2 1 = 2 −1 1 = 0.
2  2 
 x3 y3 1 1 2 1 
∴ The required area ∆ = √ [ ∆ x2 + ∆ y2 + ∆ z2 ]

=  441 + 49 + 0  = 1 √ (490 ) square units.


 
 4 4  2

Problem 13: Find the area of the triangle included between the plane 3 x − 4 y + z = 12 and
the co-ordinate planes. (Meerut 2000; Rohilkhand 08)
Solution: The equation of the given plane is
3 x − 4 y + z = 12
or x / 4 − y / 3 + z / 12 = 1. …(1)
The plane (1) meets the co-ordinate axes in the points
A (4, 0, 0 ), B (0, − 3, 0 ), C (0, 0, 12).
Using the notations of Example 13, we have
1 1
∆ x = ⋅ OB . OC = ⋅ ( −3) . 12 = 18 (numerically),
2 2
1 1
∆ y = ⋅ OC . OA = ⋅ 12 . 4 = 24,
2 2
1 1
∆ z = ⋅ OA . OB = ⋅ 4 . ( −3) = 6 (numerically).
2 2
∴ Area ∆ of the triangle ABC = √ ( ∆ x2 + ∆ y2 + ∆ z2 )
1
= √ (182 + 242 + 62 ) = 3 √ (26) square units.
2

Problem 14: From a point P ( x ′, y ′, z ′ ) a plane is drawn at right angles to OP to meet the
co-ordinate axes at A, B and C. Prove that the area of the triangle ABC is r 5 / (2 x ′ y ′ z ′ ),
where r is the measure of OP.
(Meerut 2011)
G-69

Solution: The d.r.’s of the line joining O (0, 0, 0 ) and P ( x ′, y ′, z ′ ) are x ′ − 0,


y ′ − 0, z ′ − 0 i. e., x ′, y ′, z ′.
∴ The equation of the plane through P ( x ′, y ′, z ′ )and perpendicular to OP is given by
x ′ ( x − x ′ ) + y ′ ( y − y ′ ) + z ′ ( z − z ′ ) = 0,
or x x ′ + yy ′ + zz ′ = x ′ 2 + y ′ 2 + z ′ 2
or x x ′ + yy ′ + zz ′ = r 2 [∵ r = OP = √ ( x ′ 2 + y ′ 2 + z ′ 2 )]
x y z
or + + = 1. …(1)
r2 / x′ r2 / y′ r2 /z ′
The plane (1) meets the co-ordinate axes in the points
A ( r 2 / x ′, 0, 0 ), B (0, r 2 / y ′, 0 ) and C (0, 0, r 2 / z ′ ).
Let ∆ be the area of the triangle ABC. Also let ∆ x be the area of projection OBC on the
yz-plane of the area of triangle ABC. We have
1 1 r2 r2 1 r4
∆ x = OB . OC = ⋅ ⋅ = ⋅ ⋅ …(2)
2 2 y ′ z ′ 2 y ′z ′
Now d.r.’s of the normal to the plane of ∆ ABC i. e., d.r.’s of the line OP are x ′, y ′, z ′, so
that the d.c.’s of this normal are
x ′ / r , y ′ / r , z ′ / r. [∵ r = √ ( x ′ 2 + y ′ 2 + z ′ 2 )]
Also d.c.’s of the normal to the plane of triangle OBC i. e., d.c.’s of x-axis are 1, 0, 0.
If α be the angle between the planes of triangles ABC and OBC , we have
cos α = 1 . ( x ′ / r ) + 0 . ( y ′ / r ) + 0 . ( z ′ / r ) = x ′ / r.
Now ∆ x = ∆ cos α.
r4 x′ 1 r5
∴ =∆⋅ or ∆= ⋅
2 y′ z ′ r 2 x ′ y ′z ′

H ints to O bjective T ype Q uestions

Multiple Choice Questions


1. The direction ratios of OP are α − 0, β − 0, γ − 0 i. e., α, β, γ.
∴ The equation of the plane through the point P (α, β, γ ) and perpendicular
to OP is
α ( x − α ) + β ( y − β ) + γ (z − γ ) = 0
or αx + βy + γz = α2 + β2 + γ 2.
2. Let θ be the angle between the planes 3 x − 4 y + 5z = 0 and 2 x − y − 2 z = 5.
Then
3 . 2 + ( −4) . ( −1) + 5 . ( −2)
cos θ = = 0.
√ [32 + ( −4)2 + 52 ] √ [22 + ( −1)2 + ( −2)2 ]
π
∴ θ= ⋅
2
G-70

3. The equation of the xy-plane is z = 0. Suppose the xy-plane divides the line
joining the points ( −3, 4, − 8) and (5, − 6, 4) in the ratio λ : 1. The the z-coordinate
of the point of division is zero.
4λ −8
∴ = 0 ⇒ 4λ − 8 = 0 ⇒ λ = 2 .
λ +1
∴ The xy-plane meets the line joining the points ( −3, 4, − 8) and (5, − 6, 4)
in the ratio 2 : 1.
x y z
4. Let the equation of the plane ABC be + + = 1.
p q r
Then A, B and C are the points ( p, 0, 0 ), (0, q, 0 ) and (0, 0, r ) respectively.
Since the centroid of the triangle ABC is the point ( a, b, c ), therefore
p+ 0 + 0 p 0+ q+0 q 0+0+ r r
a= = , b= = and c = = ⋅
3 3 3 3 3 3
∴ p = 3 a, q = 3 b, r = 3 c .
∴ The equation of the plane ABC is
x y z x y z
+ + = 1 or + + = 3.
3a 3 b 3 c a b c
5. If the origin lies in the acute angle between the given planes, then
a1a2 + b1b2 + c1c2 is negative. See Remark 2 of article 19.
6. See article 5.
7. See article 11.
8. Let θ be the angle between the planes z = 0 and x + y = 0. Then
0 ⋅1 + 0 ⋅1 + 1 ⋅ 0 0
cos θ = = = 0.
(02 + 02 + 12 ) ⋅ (12 + 12 + 02 ) 2

∴ θ = π 2.
9. Same as question 2 above.
10. See article 8.
11. Putting the equation of the plane 4 x − y − 8z + 7 = 0 in the normal form
lx + my + nz = p, where p is positive, we get
−4 x + y + 8z 7
=
2 2 2
( −4) + 1 + 8 ( −4) + 12 + 82
2

4 1 8 7
or − x+ y+ z= ⋅
9 9 9 9
Hence, the direction cosines of the normal to the plane directed from the origin
4 1 8
to the plane are − , , ⋅
9 9 9
12. Obviously, the point ( l, m, n) satisfies the equation of the given plane.
G-71

x y z
13. Putting the equation of the plane in intercepts form + + = 1, we get
a b c
3 x 4 y 8z x y z
+ + =1 or + + = 1.
2 2 2 2 /3 2 /4 2 /8
2 1
Hence, the intercept made by the given plane on y-axis = = ⋅
4 2
14. The equation of the zx-plane is y = 0. Hence, the equation of the plane parallel to
the zx-plane and at a distance ‘ b ’ from it is y = b.
15. The length of the perpendicular drawn from the origin (0, 0, 0 ) to the plane
x + 4 y − 8z + 18 = 0 is
|0 + 4 . 0 − 8 . 0 + 18 | 18
= = =2.
√ [12 + 42 + ( −8)2 ] 9
16. The perpendicular distance of the point (2 , − 1, − 4) from the plane
3 x − 4 y + 12z − 9 = 0 is
|3 . 2 − 4 . ( −1) + 12 . ( −4) − 9 | |−47| 47
= = = ⋅
√ [32 + ( −4)2 + 122 ] 13 13

17. Let ( x1, y1, z1) be any point on the plane x + y − z + 4 = 0.


Then x1 + y1 − z1 + 4 = 0 or x1 + y1 − z1 = − 4. …(1)
Now the perpendicular distance between the parallel planes x + y − z + 4 = 0
and x + y − z + 5=0
= the perpendicular distance of the point ( x1, y1, z1) from the plane
x+ y − z + 5=0
| x1 + y1 − z1 + 5 | | − 4 + 5 |
= = [From (1)]
√ [12 + 12 + ( −1)2 ] 3
1
= ⋅
3
18. The direction ratios of the normals to the planes 2 x − y + z = 13 and
x + y + 2 z = 9 are 2 , − 1, 1 and 1, 1, 2 respectively.
We know that the angle between two planes is the angle between their normals.
Hence, if θ is the acute angle between the given planes, then
|2 . 1 + ( −1) . 1 + 1 . 2 | 3 3 1
cos θ = = = = ⋅
√ [22 + ( −1)2 + 12 ] √ [12 + 12 + 22 ] √ 6 √ 6 6 2
π
∴ θ= ⋅
3
19. The equation of the plane through the point (1, 4, − 2)and parallel to the plane
−2 x + y − 3z = 7 is
−2 ( x − 1) + ( y − 4) − 3 { z − ( −2)} = 0
or −2 x + 2 + y − 4 − 3z − 6 = 0
or 2 x − y + 3z + 8 = 0.
G-72

20. We have 22 + 32 + ( −6)2 = 7.


∴ Direction cosines of the line whose direction ratios are
2 3 −6
2 , 3, − 6 are , , ⋅
7 7 7
These are the direction cosines of the normal to the plane.
The length of the perpendicular drawn from the origin to the plane is 5.
Hence, using the formula lx + my + nz = p of the equation of a plane in normal
form, the required equation of the plane is
2 3 6
x+ y − z = 5 or 2 x + 3 y − 6z = 35.
7 7 7
21. The equation of the plane passing through the origin and parallel to the plane
4 x − 9 y + 7z + 3 = 0 is 4 x − 9 y + 7 z = 0.
22. The direction ratios of the normal to the plane are the direction ratios of the line
joining the origin (0, 0, 0 ) to the point (12 , − 4, 3) and so are
12 − 0, − 4 − 0, 3 − 0 i. e.,12 , − 4, 3.
Hence, the equation of the required plane passing through the point (12 , − 4, 3)
and having 12 , − 4, 3 as the direction ratios of its normal is
12 ( x − 12) − 4 { y − ( −4)} + 3 ( z − 3) = 0 or 12 x − 4 y + 3z = 169.
23. We have 2 . 3 + ( −1) . ( −2) + 2 . 6 = 20 which is positive.
∴ The angle between the two given planes in which the origin lies is obtuse.
[See Remark 2 of article 19]
∴ The angle between the two given planes in which the origin does not lie is
acute.
Hence, the bisector of the acute angle between the planes 2 x − y + 2 z + 3 = 0
and 3 x − 2 y + 6 z + 8 = 0 is the bisector of the angle between the given planes
in which the origin does not lie and so its equation is
2 x − y + 2z + 3 3 x − 2 y + 6z + 8
=−
√ [22 + ( −1)2 + 22 ] √ [32 + ( −2)2 + 62 ]
2 x − y + 2z + 3 3 x − 2 y + 6z + 8
or =−
3 7
or 14 x − 7 y + 14z + 21 = − 9 x + 6 y − 18z − 24
or 23 x − 13 y + 32z + 45 = 0.

Fill in the Blank(s)


1. We know that in three dimensional geometry every equation of first degree in
x, y, z always represents a plane.
2. Obviously, the origin (0, 0, 0) satisfies the equation of the given plane.
3. See article 6.
4. See article 5.
G-73

5. Putting the equation of the plane x − 2 y + 2z − 9 = 0 in the normal form


lx + my + nz = p,
x − 2 y + 2z 9 1 2 2
we get = or x− y + z = 3.
2 2
1 + ( −2) + 2 2 2 2
1 + ( −2) + 2 2 3 3 3

6. The direction ratios of the normals to the two given planes are a1, b1, c1 and
a2 , b2 , c2 respectively. We know that two planes are perpendicular if and only if
their normals are perpendicular. Hence, the given planes are perpendicular if and
only if
a1a2 + b1b2 + c1c2 = 0.
7. See Remark of article 14.
8. See article 13.
9. See article 20.
10. See article 21, Theorem 1.

True or False
1. The general equation of a plane is
a b c
ax + by + cz + d = 0 or x+ y + z +1=0
d d d
a b c
in which there are three arbitrary constants , and ⋅
d d d
2. We have 1. (3) + ( −1) . 2 + 1 . ( −1) = 0.
So the given planes are perpendicular to each other.
3 −4
3. We have ≠ ⋅
6 8
So, in the equations of the given planes the coefficients of x, y, z are not
proportional to each other. Thus the given planes are not parallel to each other.
4. 3 4 9 1
We have = = , each ratio being equal to ⋅ So, the given planes are parallel
6 8 18 2
to each other.
5. We have 2 . 3 + ( −1) . 2 + ( −2) . ( −6) = 16, which is positive. So, the angle
between the given planes in which the origin lies is obtuse.
❍❍❍
G-74

Chapter-5
The Straight Line

Comprehensive Problems 1
x−2 y +1 z −2
Problem 1(i): Find the point in which the line = = meets the plane
3 4 12
x − 2 y + z = 20.
(Rohilkhand 2013)
Solution: The equations of the line are
x−2 y +1 z −2
= = = r (say). …(1)
3 4 12
The equation of the plane is x − 2 y + z = 20. …(2)
The co-ordinates of any point Q on the line (1) are
(2 + 3r, − 1 + 4r, 2 + 12r ). …(3)
Suppose the line (1) meets the plane (2) in this point Q, hence we have
(2 + 3r ) − 2 ( −1 + 4r ) + (2 + 12r ) = 20, or r = 2 .
Putting this value of r in the co-ordinates of Q given by (3) the co-ordinates of the
required point are given by
(2 + 3 . 2 , − 1 + 4 . 2 , 2 + 12 . 2) or (8, 7, 26).

Problem 1(ii): Find the distance from the point (3, 4, 5) to the point where the line
x−3 y −4 z −5
= = meets the plane x + y + z = 2.
1 2 2 (Kumaun 2009)
Solution: Any point on the line is (3 + r, 4 + 2r, 5 + 2r ).
If it also lies on the plane x + y + z = 2, then
3 + r + 4 + 2r + 5 + 2r = 2
or 5r = −10
or r = −2.
Putting the value of r, we get the required co-ordinates of the point as (1, 0, 1).
∴ The required distance
= the distance between (3, 4, 5) and (1, 0, 1)
= (3 − 1)2 + (4 − 0 )2 + (5 − 1)2

= 4 + 16 + 16
= 36 = 6.
G-75

x −3 y −4 z −5
Problem 1(iii): Find the point where the line = = meets the plane
1 2 3
2 x + 3 y − 4z + 10 = 0. (Kanpur 2015)
Solution: Proceed as in part (i). Ans. (5, 8, 11).

Problem 2: Find the coordinates of the point where the line joining the points (2, − 3, 1) and
(3, − 4, − 5) meets the plane 2 x + y + z = 7. (Kumaun 2007; Rohilkhand 08)
Solution: The direction ratios of the line joining the points (2, − 3, 1) and (3, − 4, − 5)
are
3 − 2, − 4 − ( − 3), − 5 − 1 i. e., 1, − 1, − 6.
So the equations of the line joining these points i. e., the equations of the line passing
through the point (2, − 3, 1) and having direction ratios 1, − 1, − 6 are
x−2 y + 3 z −1
= = = λ (say). …(1)
1 −1 −6
∴ x = 2 + λ , y = − 3 − λ , z = 1 − 6λ . …(2)
Thus the coordinates of any point on the line (1) are
(2 + λ , − 3 − λ ,1 − 6λ).
If this point lies on the plane 2 x + y + z = 7, then
2 (2 + λ ) + ( − 3 − λ ) + (1 − 6λ ) = 7
or − 5λ = 5 or λ = − 1.
Putting λ = − 1 in (2), we get the required point as (1, − 2, 7).

Problem 3: Show that the distance of the point of intersection of the line
x−3 y −4 z −5
= = and the plane x + y + z = 17 from the point (3, 4, 5)is 3.
1 2 2
Solution: The equations of the given line are
x−3 y −4 z −5
= = = r (say). … (1)
1 2 2
The co-ordinates of any point on the line (1) are ( r + 3, 2r + 4, 2r + 5). If this point lies
on the plane x + y + z = 17, we have
( r + 3) + (2r + 4) + (2r + 5) = 17
or 5r = 5 or r = 1.
Putting the value of r in the co-ordinates of the point of intersection of the line (1) and
the given plane are (4, 6, 7).
∴ The required distance = The distance between the points (4, 6, 7) and (3, 4, 5)
= (4 − 3)2 + (6 − 4)2 + (7 − 5)2

= 1+ 4 + 4
= 9
= 3.
G-76

x+1 y − 12 z − 7
Problem 4: Find the points in which the line = = cuts the surface
−1 5 2
11x2 − 5 y2 + z 2 = 0. (Meerut 2004, 05, 07, 09B; Kanpur 2006)
Solution: The equations of the given line are
x+1 y − 12 z − 7
= = = r (say). …(1)
−1 5 2
The co-ordinates of any point on the line (1) are
( − r − 1, 5r + 12 , 2r + 7). …(2)
2 2 2
If this point lies on the given surface 11 x − 5 y + z = 0, we have
2 2 2
11 ( − r − 1) − 5 (5 r + 12) + (2 r + 7) = 0

or 11 ( r 2 + 2 r + 1) − 5 (25 r 2 + 120 r + 144) + (4 r 2 + 28 r + 49) = 0

or −110 r 2 − 550 r − 660 = 0, or r2 + 5r + 6=0


or ( r + 2) ( r + 3) = 0 or r = − 2 , − 3.
Putting these values of r in (2), the required points of intersection are (1, 2 , 3) and
(2 , − 3, 1).

Problem 5: Find the distance of the point (1, − 2, 3) from the plane x − y + z = 5 measured
x y z
along a line parallel to = = ⋅
2 3 −6 (Kanpur 2010)
Solution: The given plane is
x − y + z = 5. …(1)
The equations of the line passing through the point P (1, − 2, 3) and parallel to the line
x y z
= = are
2 3 −6
x −1 y + 2 z −3
= = = λ. …(2)
2 3 −6
Suppose the line (2) meets the plane (1) at the point Q whose coordinates are
(2λ + 1, 3λ − 2, − 6λ + 3) . …(3)
Since the point Q lies on the plane (1), therefore
(2λ + 1) − (3λ − 2) + ( − 6λ + 3) = 5 or − 7λ = − 1 or λ = 1 / 7.
Putting λ = 1 / 7 in (3), we get the coordinates of Q .
Now the required distance = PQ

= (2λ + 1 − 1)2 + (3λ − 2 + 2)2 + ( − 6λ + 3 − 3)2

= λ2(4 + 9 + 36)
1
= 7 λ2 = 7 | λ |2 = 7 | λ | = 7 × = 1.
7
G-77

Problem 6: Find the image of the point (1, 3, 4) in the plane 2 x − y + z + 3 = 0.


(Meerut 2004, 05, 06B, 07, 10; Rohilkhand 08)
Solution: The given plane is
2 x − y + z + 3 = 0. …(1)
Direction ratios of a line perpendicular to the plane (1) are 2, − 1, 1.
Let Q be the image of the given point P (1, 3, 4) in the plane (1). Then the line PQ is
perpendicular to the plane (1) and the middle point N of PQ lies on the plane (1).
The equations of the line PQ passing through P (1, 3, 4) and perpendicular to the plane
(1) are
x −1 y − 3 z −4
= = = λ. …(2)
2 −1 1
Let the coordinates of the point Q which is on the line (2) be
(2λ + 1, − λ + 3, λ + 4) . …(3)
Then the coordinates of the middle point N of PQ are
 1 (2λ + 1 + 1), 1 ( − λ + 3 + 3), 1 ( λ + 4 + 4)
 
2 2 2 
 λ + 1, − 1 λ + 3, 1 λ + 4 ⋅
i. e.,  
 2 2 
But the point N lies on the plane (1).
1 1
∴ 2 ( λ + 1) + λ − 3 + λ + 4 + 3 = 0
2 2
or 3λ + 6 = 0 or λ = − 2.
Putting λ = − 2 in (3), the coordinates of Q i. e., the image of P (1, 3, 4) in the given plane
are ( − 3, 5, 2).
Problem 7: A variable plane makes intercepts on the co-ordinate axes the sum of whose squares
is constant and equal to k 2 . Show that the locus of the foot of the perpendicular from the origin to
the plane is
( x −2 + y −2 + z −2 ) ( x 2 + y 2 + z 2 )2 = k 2. (Garhwal 2001)
Solution: Let the equation of the variable plane be
x / a + y / b + z / c = 1, …(1)
where a, b, c are its intercepts on the co-ordinate axes, so that
a2 + b 2 + c 2 = k 2 . …(2)
The d.r.’s of the normal to the plane (1) are 1 / a, 1 / b, 1 / c . Hence the equations of the
line perpendicular to the plane (1) [i. e., parallel to the normal to the plane] and passing
through the origin are
x−0 y −0 z −0
= = = (say). …(3)
1/ a 1/ b 1/ c
Any point on the line (3) is ( r / a, r / b, r / c ). …(4)
If this point lies on the plane (1), we have
1
r / a2 + r / b2 + r / c2 = 1 or r= ⋅
a −2 + b −2 + c −2
G-78

Let ( x , y , z ) be the co-ordinates of the foot of the perpendicular, then putting the value
of r in (4), we get
1 a −1
x= or x = ⋅
−2 −2 −2
a (a + b + c ) a + b −2 + c −2
−2

b −1 c −1
Similarly, y= and z= ⋅
−2 −2 −2 −2
a + b + c a + b −2 + c −2
a −2 + b −2 + c −21
Now x2 + y2 + z 2 = = ⋅
( a−2 + b −2 + c −2 )2 a−2 + b −2 + c −2
1
∴ ( x 2 + y 2 + z 2 )2 = ⋅ …(5)
( a + b + c −2 )2
−2 −2

Again, x −2 + y −2 + z −2 = ( a−2 + b −2 + c −2 )2 ( a2 + b 2 + c 2 )
= ( a−2 + b −2 + c −2 )2 . k 2. …(6)
[Using (2)]
Multiplying (5) and (6), we have ( x 2 + y 2 + z 2 )2 ( x −2 + y −2 + z −2 ) = k 2 .
This is the equation of the required locus.
Problem 8: Find the equations of the line through the points ( a, b, c ) and ( a′, b ′, c ′ ) and prove
that it passes through the origin if aa′ + bb ′ + cc ′ = rr ′, where r and r ′ are the distances of the
point from the origin. (Kanpur 2015, 16)
Solution: The equations of the line through the points ( a, b, c ) and ( a′, b ′, c ′ ) are
x−a y−b z−c
= = ⋅ …(1)
a′ − a b ′ − b c ′ − c
The line (1) passes through the origin, hence we have
0−a 0−b 0−c a′ − a b ′ − b c ′ − c
= = or = =
a′ − a b ′ − b c ′ − c −a −b −c
a′ b′ c′ a′ b ′ c ′
or − +1= − +1= − + 1 or = = ⋅
a b c a b c
From these relations, we immediately get
ab ′ − a′ b = 0, bc ′ − b ′ c = 0, ca′ − c ′ a = 0. …(2)
By Lagrange’s identity, we have
( a2 + b2 + c 2 ) ( a′2 + b ′2 + c ′2 ) − ( aa′ + bb ′ + cc ′ )2
= ( ab ′ − a′ b )2 + ( bc ′ − b ′ c )2 + ( ca′ − c ′ a)2 = 0 + 0 + 0 = 0
or ( a2 + b2 + c 2 ) ( a′2 + b ′2 + c ′2 ) = ( aa′ + bb ′ + cc ′ )2 . …(3)
Now r = the distance of ( a, b, c ) from the origin
= √ [( a − 0 )2 + ( b − 0 )2 + ( c − 0 )2 ] = √ ( a2 + b2 + c 2 ).
Similarly r ′ = √ ( a′2 + b ′2 + c ′2 ).
Putting these values in (3), we have
( aa′ + bb ′ + cc ′ )2 = ( rr ′ )2 or aa′ + bb ′ + cc ′ = rr ′.
This is the required result.
G-79

Comprehensive Problems 2
Problem 1: Find in symmetrical form the equations of the line
x + y + z + 1 = 0 = 4 x + y − 2 z + 2 and find its direction cosines.
Solution: The equation of the given line in general form are
x + y + z + 1 = 0, 4 x + y − 2z + 2 = 0. …(1)
Let l , m, n be the d.c.’s of the line (1). Since the line in common to both the planes, it is
perpendicular to the normals to both the planes.
Hence, we have
l + m + n = 0, 4l + m − 2n = 0.
Solving these, we get
l m n l m n l m n
= = or = = or = = ⋅
−2 − 1 4 + 2 1 − 4 −3 6 −3 1 −2 1
∴ D.r.’s of the line (1) are –1, +2, –1.
We have ( −1)2 + ( +2)2 + ( −1)2 = 6
∴ The d.c.’s l, m, n of the line (1) are given by
−1 2 −1
l= ,m= ,n= .
√6 √6 √6
Now to find the co-ordinates of the point on the line given by (1), let us find the point
where it meets the plane z = 0. Putting z = 0 in the equation given by (1), we have
x + y + 1 = 0 ; 4 x + y + 2 = 0.
Solving these, we get
−1 −2
x= , y= ⋅
3 3
−1 −2 
∴ The line meets the plane z = 0 in the point  , , 0  and has direction ratios as
 3 3 
( −1, 2, − 1 ).
Therefore the equation of the given line in symmetric form are
1 2
x+ y+
3 = 3 = z+0.
−1 2 −1
Problem 2: Find the angle between the lines x + 2 y − 2z = 11, x − 2 y + z = 9 and
x−3 y+5 z −1
= = ⋅
1 −3 2
Solution: The given equation of line in general form are
x + 2 y − 2z = 11, x − 2 y + z = 9. …(1)
Let a, b , c be the d.r.’s of the line (1). Since the line is common to both the planes, it is
perpendicular to the normals to both the planes.
Hence, we have
a + 2b − 2c = 0 , a − 2b + c = 0.
G-80

Solving these, we get


a b c a b c a b c
= = or = = or = = ⋅
2 − 4 −2 − 1 −2 − 2 −2 −3 −4 2 3 4
∴ D.r.’s of the line (1) are 2, 3, 4 .
The other line is
x−3 y + 5 z −1
= = , …(2)
1 −3 2
whose d.r.’s are 1, − 3, 2.
Now the angle between both lines (1) and (2) is
( 1 )(2) + (3) ( −3) + (4)(2)
cos θ =
(1 )2 + ( −3)2 + (2)2 (2)2 + (3)2 + (4)2
2−9+ 8 1
or cosθ = = ⋅
√14 √ 29 √ 409

θ = cos −1 
1 
∴ .
 √ 406 
Problem 3: Prove that the lines x = ay + b, z = cy + d and x = a ′ y + b ′, z = c ′ y + d′
are perpendicular if aa ′ + cc ′ + 1 = 0. (Kanpur 2006, 09; Rohilkhand 07;
Bundelkhand 09; Kumaun 09)
Solution: The equations of the first line are
x = ay + b, z = cy + d or x − b = ay, z − d = cy
x−b z−d x−b y z−d
or = y, = y or = = ,
a c a 1 c
which are in symmetrical form.
So direction ratios of the first line are a, 1, c.
Similarly the direction ratios of the second line are a ′, 1, c ′.
∴ The two lines are perpendicular if
aa ′ + (1) (1) + cc ′ = 0 i. e., aa ′ + cc ′ + 1 = 0.
Problem 4: Find the equations to the line through the point (1, 2 , 3) parallel to the line
x − y + 2z − 5 = 0 ; 3 x + y + z − 6 = 0. (Avadh 2013)
Solution: The equations of the given line in general form are
x − y + 2z − 5 = 0, 3 x + y + z − 6 = 0. …(1)
Let l, m, n be the d.c.’s of this line. Then we have
l − m + 2n = 0, 3l + m + n = 0.
Solving these, we have
l m n l m n
= = or = = ⋅
−1 − 1 2 . 3 − 1 . 1 1 . 1 + 1 . 3 −3 5 4
Since the required line is parallel to the line (1), the d.c.’s of the required line are
proportional to l, m, n i. e., −3, 5, 4. Hence the equations of required line are given by
x −1 y −2 z −3
= = ⋅
−3 5 4
G-81

Problem 5: (i) Show that the lines 2 x + 3 y − 4z = 0, 3 x − 4 y + z = 7


and 5 x − y − 3z + 12 = 0, x − 7 y + 5z − 6 = 0 are parallel.
(ii) Show that the lines x + y − z = 5, 9 x − 5 y + z = 4 and 6 x − 8 y + 4z = 3,
x + 8 y − 6z + 7 = 0 are parallel. (Kumaun 2008, Rohilkhand 12)
Solution: (i) The given lines are
 2 x + 3 y − 4z = 0
 …(1)
 3x − 4 y + z = 7
 5 x − y − 3z + 12 = 0
and  …(2)
 x − 7 y + 5z − 6 = 0.
Let a1 , b1 , c1 be the d.r.’s of the line (1) of intersection of the planes.
Then this lies on the plane so it is perpendicular to the normals of both planes.
∴ 2a1 + 3b1 − 4c1 = 0
and 3a1 − 4b1 + c1 = 0.
Solving these, we get
a1 b1 c1 a1 b c
= = or = 1 = 1
3 − 16 −12 − 2 −8 − 9 −13 −14 −17
a1 b1 c
or = = 1⋅
13 14 17
∴ D.r.’s of the line (1) are 13, 14, 17.
Again let a2, b2, c2 be the d.r.’s of the line (2) of intersection of the planes intersection of
the planes.
∴ 5a2 − b2 − 3c2 = 0
and a2 − 7b2 + 5c2 = 0.
Solving these, we get
a2 b2 c2 a2 b c
= = or = 2 = 2
−5 − 21 −3 − 25 −35 + 1 −26 −28 −34
a2 b2 c
or = = 2⋅
13 14 17
∴ D.r.’s of the line (2) are 13, 14, 17.
We see that the d.r.’s of line (1) and line (2) are proportional because we have
13 14 17
= = each ratio being equal to 1.
13 14 17
∴ Line (1) and line (2) are parallel lines.
(ii) Proceed as in part (i).

Problem 6: Show that the lines 3 x + 2 y + z = 5, x + y − 2 z = 3 and


2 x − y − z = 0, 7 x + 10 y − 8z = 15 are mutually perpendicular.
Solution: The given lines are
3 x + 2 y + z = 5 , x + y − 2z = 3 …(1)
and 2 x − y − z = 0 , 7 x + 10 y − 8z = 15. …(2)
G-82

Let a1 , b1 , c1 be the d.r.’s of line (1) of intersection of the planes then this line lies on the
plane, so it is perpendicular to the normals of both planes.
∴ 3a1 + 2b1 + c1 = 0 and a1 + b1 − 2c1 = 0.
Solving these, we get
a1 b c a1 b1 c1
= 1 = 1 or = = ⋅
−4 − 1 1 + 6 3 − 2 −5 7 1
∴ D.r.’s of the line (1) are −5, 7, 1.
Again let a2 , b2 , c2 be the d.r.’s of the line (2) of intersection of the planes.
∴ 2a2 − b2 − c2 = 0 and 7a2 + 10 b2 − 8c2 = 0.
Solving these, we get
a2 b2 c2 a b c
= = or 2 = 2 = 2
8 + 10 −7 + 16 20 + 7 18 9 27
a2 b2 c2
or = = .
2 1 3
∴ D.r.’s of line (2) are 2,1, 3.
These two lines are perpendicular to each other if
a1a2 + b1b2 + c1c2 = 0.
∴ ( −5)(2) + (7)(1) + (1)(3) = 0
or 0 = 0.
Hence, the given two lines are mutually perpendicular.

Comprehensive Problems 3
Problem 1: Find the equation of the plane which passes through the line of intersection of the
planes u1 ≡ a1 x + b1 y + c1 z + d1 = 0 and u2 ≡ a2 x + b2 y + c2 z + d2 = 0 and is
parallel to x-axis.
Solution: Equation of a plane through the given line is
u1 + λ u2 = 0 …(1)
or ( a1 x + b1 y + c1z + d1) + λ ( a2 + b2 y + c2 z + d2 ) = 0
or ( a1 + λ a2 ) x + ( b1 + λ b2 ) y + ( c1 + λ c2 ) z + d1 + λ d2 = 0. …(2)
∴ D.r.’s of the normal to this plane are
a1 + λ a2 , b1 + λ b2 , c1 + λ c2 .
Since the plane (2) is parallel to x-axis.
So d.r.’s of x-axis are 1, 0, 0.
Apply the condition of parallelism between line and plane, we get
( a1 + λ a2 )1 + ( b1 + λ b2 )0 + ( c1 + λ c2 )0 = 0
a
or λ = − 1.
a2
Putting the value of λ in eq. (2) or (1), we get
 a 
u1 +  − 1  u2 = 0 or u1a2 = u2 a1.
 a2 
G-83

Problem 2: Find the equation of the plane through the line


3 x − 4 y + 5z = 10, 2 x + 2 y − 3z = 4
and parallel to the line x = 2 y = 3z.
Solution: The equations of the given line are
3 x − 4 y + 5z = 10, 2 x + 2 y − 3 z = 4. …(1)
The equation of any plane through the line (1) is
(3 x − 4 y + 5z − 10 ) + λ (2 x + 2 y − 3z − 4) = 0
or (3 + 2λ ) x + ( −4 + 2λ ) y + (5 − 3λ ) z − 10 − 4λ = 0. …(2)
The plane (1) will be parallel to the line
x y z
x = 2 y = 3z i. e., = = if
6 3 2
(3 + 2λ ) . 6 + ( −4 + 2λ ) . 3 + (5 − 3λ ) . 2 = 0
4
or λ (12 + 6 − 6) + 18 − 12 + 10 = 0, or λ = − ⋅
3
Putting this value of λ in (2), the required equation of the plane is given by
 3 − 8  x +  − 4 − 8  y + (5 + 4) z − 10 + 16 = 0
   
 3  3 3
or x − 20 y + 27 z = 14.
Problem 3: Find the direction cosines of the line whose equations are x + y = 3 and
x + y + z = 0 and show that it makes an angle of 30 ° with the plane y − z + 2 = 0.
Solution: The equations of the line are
x + y = 3, x + y + z = 0. …(1)
Let l, m, n be the d.c.’s of the line (1), then we have
l + m + 0 . n = 0, l + m + n = 0.
Solving, we get
l m n √ ( l2 + m2 + n2 ) 1
= = = = ⋅
1 −1 0 √ {(1)2 + ( −1)2 + 02 } √ 2
1 1
∴ l= ,m= − , n = 0.
√2 √2
The equation of the plane is y − z + 2 = 0. …(2)
The d.r.’s of the normal to the plane (2) are 0, 1, − 1 i. e., direction cosines are
1 1
0, ,− ⋅
√2 √2
Now suppose θ is the acute angle between the line (1) and the normal to the plane (2).
Then using the formula cos θ = l1 l2 + m1 m2 + n1 n2 , we have

⋅ 0 +  −
1  1
+ 0  −
1 1  1
cos θ = ⋅  = .
√2  √2  √2  √2  2
∴ θ = 60 ° .
Now the angle between the line (1) and the plane (2) is the complement of the angle θ. So
the required angle between the line (1) and the plane (2) is 90 ° − θ i. e.,90 ° − 60 ° i. e.,30 °.
G-84

Problem 4: Find the equation of the plane through the points (2 , − 1, 0 ), (3, − 4, 5) and
parallel to the line 3 x = 2 y = z.
Solution: The equation of any plane through the point (2 , − 1, 0 ) is
a ( x − 2) + b ( y + 1) + c ( z − 0 ) = 0. …(1)
If the plane (1) passes through the point (3, − 4, 5), we get
a (3 − 2) + b ( −4 + 1) + c (5) = 0 or a − 3b + 5c = 0. …(2)
The equations of the line are
x y z
3 x = 2 y = z , or = = ⋅ …(3)
2 3 6
The plane (1) will be parallel to the line (3), if
2 a + 3 b + 6 c = 0. …(4)
Solving (2) and (4), we get
a b c
= = ⋅
−33 4 9
Putting these proportionate values of a, b, c in (1), the required equation of the plane is
given by
−33 ( x − 2) + 4 ( y + 1) + 9 ( z − 0 ) = 0
or 33 x − 4 y − 9z − 70 = 0.
Problem 5: Find the equation of the plane through (2 , 1, 4) perpendicular to the line of
intersection of the planes 3 x + 4 y + 7z + 4 = 0 and x − y + 2z + 3 = 0.
Solution: Let l, m, n be the d.c.’s of the line of intersection of the two planes
3 x + 4 y + 7z + 4 = 0, x − y + 2z + 3 = 0.
Then we have 3l + 4m + 7n = 0, l − m + 2 n = 0.
l m n l m n
Solving, = = or = = ⋅
8 + 7 7 − 6 −3 − 4 15 1 −7
Thus d.r.’s of the normal to the required plane are15, 1, − 7. Also the required plane is to
pass through the point (2 , 1, 4). Hence its equation is
15 ( x − 2) + 1 ( y − 1) − 7 ( z − 4) = 0 or 15 x + y − 7z − 3 = 0.

Comprehensive Problems 4
x−2 y −3 z −4
Problem 1: Find the equations of the planes through the line = = and
2 3 5
parallel to the co-ordinate axes.
Solution: The equations of the given line are
x−2 y −3 z −4
= = ⋅ …(1)
2 3 5
The equation of any plane through the line (1) [see article 6] is
a ( x − 2) + b ( y − 3) + c ( z − 4) = 0 …(2)
where 2 a + 3b + 5c = 0. …(3)
We are required to find the equations of the planes parallel to x-axis, y-axis and z-axis
respectively.
G-85

(i) The d.c.’s of x-axis are1, 0, 0. If the plane (2) is parallel to the x-axis, then the normal
to the plane (2) is perpendicular to the x-axis i. e., we have
a . 1 + b . 0 + c . 0 = 0. …(4)
Solving (3) and (4), we have
a b c a b c
= = or = = ⋅
0 5−0 0 −3 0 5 −3
Putting these proportionate values of a, b, c in (2), the required equation of the plane is
given by
0 . ( x − 2) + 5 ( y − 3) − 3 ( z − 4) = 0 or 5 y − 3z − 3 = 0.
(ii) The d.c.’s of the y-axis are 0, 1, 0. If the plane (2) is parallel to the y-axis, we have
a . 0 + b . 1 + c . 0 = 0. …(5)
Solving (3) and (5), we get
a b c a b c
= = or = = ⋅
0 −5 0 2−0 −5 0 2
Putting these proportionate values in (2), the required equation of the plane is given by
− 5 ( x − 2) + 0 . ( y − 3) + 2 ( z − 4) = 0 or 5 x − 2z − 2 = 0.
(iii) The d.c.’s of the z-axis are 0, 0, 1. If the plane (2) is parallel to the z-axis, we have
a . 0 + b . 0 + c . 1 = 0. …(6)
Solving (3) and (6), we get
a b c
= = ⋅
3 −2 0
Putting these proportionate values of a, b, c in (2), the required equation of the plane is
given by
3 ( x − 2) − 2 ( y − 3) = 0 or 3 x − 2 y = 0.
x −1 y + 6 z +1
Problem 2: Prove that the equation of the plane through the line = = and
3 4 2
x−2 y −1 z + 4
parallel to = = is 26 x − 11 y − 17z − 109 = 0 and show that the point
2 −3 5
(2 , 1, − 4) lies on it.
Solution: The equations of the given line are
x −1 y + 6 z +1
= = ⋅ …(1)
3 4 2
The equation of any plane through the line (1) is
a ( x − 1) + b ( y + 6) + c ( z + 1) = 0 …(2)
where 3 a + 4b + 2 c = 0. …(3)
The plane (2) is to be parallel to the line
x−2 y −1 z + 4
= = ⋅ …(4)
2 −3 5
Hence the normal to the plane (2) is perpendicular to the line (4), so that we have
2 a − 3 b + 5c = 0. …(5)
Solving (3) and (5), we get
a b c a b c
= = or = = ⋅
20 + 6 4 − 15 −9 − 8 26 −11 −17
G-86

Putting these proportionate values of a, b, c in the equation (2), the required equation of
the plane is
26 ( x − 1) − 11 ( y + 6) − 17 ( z + 1) = 0
or 26 x − 11 y − 17 z − 109 = 0. …(6)
Substituting the point (2 , 1, − 4) in the equation (6) of the plane, we get
26 × 2 − 11 × 1 − 17 × − 4 − 109 = 0 or 0 = 0
i. e., the point (2 , 1, − 4) satisfies the equation (6) of the plane.
Remark: In the above Problem 2, the point (2 , 1, − 4)lies on the line (4) and also on the
plane (6). Hence the line (4) will wholly lie on the plane (6).
Therefore both the lines (1) and (4) are coplanar and the equation (6) gives the plane
containing both of them.
Problem 3: Find the equation of the plane determined by the parallel lines
x−4 y −3 z −2 x−3 y+2 z
= = and = = ⋅
1 −4 5 1 −4 5
Solution: The equation of any plane through
x−4 y −3 z −2
= = is
1 −4 5
A ( x − 4) + B ( y − 3) + C ( z − 2) = 0 …(1)
where A ⋅ 1 + B ( −4) + C ⋅ 5 = 0. …(2)
If the plane (1) contains the parallel line
x−3 y+2 z
= =
1 4 5
also, then the point (3, − 2, 0 ) on this line must lie on the plane (1) which gives
A (3 − 4) + B( −2 − 3) + C (0 − 2) = 0
or A + 5B + 2C = 0, …(3)
Eliminating A , B and C from the eq.’s (1), (2) and (3), we get
x −4 y −3 z −2
1 −4 5 =0
1 5 2
or ( x − 4)( −8 − 25) − ( y − 3)(2 − 5) + ( z − 2) (5 + 4) = 0
or −33( x − 4) + 3( y − 3) + 9( z − 2) = 0
or 11( x − y ) − ( y − 3) − 3( z − 2) = 0
or 11x − y − 3z − 35 = 0,
which is the required eq. of the plane.
Problem 4: Find the equation of the plane which contains the line x = ( y − 3)/2 = ( z − 5)/3
and which is perpendicular to the plane 2 x + 7 y − 3z = 1.
Solution: The equation of any plane through
x y −3 z −5
= = is
1 2 3
A ( x ) + B ( y − 3) + C ( z − 5) = 0 …(1)
where A (1 ) + B(2) + C (3) = 0. …(2)
G-87

Also if the plane (1) is perpendicular to the plane 2 x + 7 y − 3z = 1,


we have A (2) + B(7) + C ( −3) = 0. …(3)
Eliminating A , B and C from above eq.’s, we get
x y −3 z −5
1 2 3 =0
2 7 −3
or x ( −6 − 21) − ( y − 3)( − z − 6) + ( z − 5)(7 − 4) = 0
or − 27 x + 9( y − 3) + 3( z − 5) = 0
or 9 x − 3 ( y − 3) − ( z − 5) = 0
or 9 x − 3 y − z + 14 = 0,
which is the required equation of the plane.
x+1 y −3 z + 2
Problem 5: Show that the equation to the plane containing the line = =
−3 2 1
x y −7 z + 7
and the point (0, 7, − 7) is x + y + z = 0. Hence show that the line = = also lies
1 2 −3
in the same plane. (Garhwal 2001)
Solution: The equations of the given line are
x+1 y −3 z + 2
= = ⋅ …(1)
−3 2 1
The equation of any plane through the line (1) is
a ( x + 1) + b ( y − 3) + c ( z + 2) = 0 …(2)
where −3a + 2 b + 1 . c = 0. …(3)
The plane (2) is also to pass through the point (0, 7, − 7).
∴ a + 4b − 5c = 0. …(4)
Solving (3) and (4), we get
a b c a b c
= = or = = ⋅
−10 − 4 1 − 15 −12 − 2 1 1 1
Putting these proportionate values of a, b, c in (2), the required equation of the plane is
1 . ( x + 1) + 1 . ( y − 3) + 1 . ( z + 2) = 0 or x + y + z = 0. …(5)
The equations of the second line are given to be
x y −7 z + 7
= = ⋅ …(6)
1 2 3
The line (6) passes through the point (0, 7, − 7) and this point also lies on the plane (5).
Now the line (6) will lie in the plane (5) if the normal to the plane (5) [whose d.r.’s are
1, 1, 1] is perpendicular to the line (6), the condition for which is
a1 a2 + b1 b2 + c1 c2 = 0 i. e., 1 . 1 + 1 . 2 + 1 . ( −3) = 0,
which holds good. Hence the line (6) also lies in the plane (5). This proves the required
statement.
y z
Problem 6: Find the equation of the plane containing the line + = 1, x = 0
b c
x z
and parallel to the line − = 1, y = 0.
a c
G-88

Solution: The equation of a plane containing the line


y z
+ = 1, x = 0
b c
 y z 
is  + − 1 + λ x = 0
b c 
y z
or λx+ + − 1 = 0. …(1)
b c
Now d.r.’s of normal to this plane are
1 1
λ, , ⋅
b c
Since this plane parallel to the line
x z x−0 y z −0,
− = 1, y = 0 or = =
a c a 0 c
whose d.r.’s are a, 0 , c .
1 1
∴ λ( a) + (0 ) + ( c ) = 0
b c
−1
or λa + 1= 0 ⇒ λ = ⋅
a
Putting this value of λ in (1), the equation of required plane is
x y z x y z
− + + −1= 0 or − − + 1 = 0.
a b c a b c
Problem 7: The plane lx + my = 0 is rotated about its line of intersection with the plane z = 0,
through an angle α. Prove that the equation of the plane in its new position is
lx + my ± z √ ( l2 + m2 ) tan α = 0. (Meerut 2000, 06B)
Solution: The equations of the given planes are
lx + my = 0 …(1)
and z = 0. …(2)
The equation of any plane through the line of intersection of the planes (1) and (2) is
lx + my + λz = 0. …(3)
Suppose the plane (1) when rotated through an angle α about its line of intersection
with the plane z = 0 has the equation (3). Thus the angle between the planes (1) and (3)
is α.
l . l + m. m + 0 . λ
∴ cos α =
√ ( l2 + m2 ) √ ( l2 + m2 + λ2 )
√ ( l2 + m2 )
or cos α =
√ ( l2 + m2 + λ2 )
l2 + m2
or cos2 α = or ( l2 + m2 ) (1 − cos2 α ) = λ2 cos2 α
l + m2 + λ2
2

or λ2 = ( l2 + m2 ) tan2 α , λ = ± √ ( l2 + m2 ) tan α.
Putting this value of λ in (3), the required equation of the plane in its new position is
given by
lx + my ± √ ( l2 + m2 ) . z tan α = 0.
G-89

Comprehensive Problems 5
Problem 1: Find the equations of the perpendicular from the point (1, 6, 3) to the line
x y −1 z −2
= = ⋅ Find also the co-ordinates of the foot of the perpendicular.
1 2 3
(Kanpur 2011)
Solution: The given point is P (1, 6, 3) and the equations of the given line are
x −0 y −1 z − 2
= = = r (say). …(1)
1 2 3
The co-ordinates of any point N on the line (1) are ( r, 2r + 1, 3r + 2). …(2)
Let this point N be the foot of the perpendicular from the point P (1, 6, 3) to the line (1).
Then the d.r.’s of the perpendicular PN are
r − 1,(2r + 1) − 6, (3r + 2) − 3 or r − 1, 2r − 5, 3r − 1. …(3)
The d.r.’s of the given line (1) are 1, 2, 3.
Now PN is perpendicular to the line (1). The condition of perpendicularity gives
( r − 1) ⋅1 + (2r − 5) ⋅ 2 + (3r − 1) ⋅ 3 = 0
or 14r − 14 = 0 or r = 1.
Putting the value of r in (2), the foot N of the perpendicular is the point (1, 3, 5).
Putting the value of r in (3), the d.r.’s of PN are 0, − 3,2.
Hence the equations of the perpendicular PN from the point (1, 6, 3) to the line (1)
are
x −1 y −6 z −3
= = ⋅
0 −3 2
or x − 1 = 0, 2 y + 3z = 21.

Problem 2: Find the equations of the perpendicular from the origin to the line
ax + by + cz + d = 0, a ′ x + b ′ y + c ′ z + d ′ = 0.
Solution: First read article 7 (B) again and then solve this problem.
The equations of the given line are
ax + by + cz + d = 0, a ′ x + b ′ y + c ′ z + d ′ = 0. …(1)
We know that the perpendicular from a given point P to a given line is the intersection of
the two planes, namely (i) the plane through the given point P (0, 0, 0 ) and also through
the given line and (ii) the plane through the point P perpendicular to the given line.
Thus we proceed as follows :
The equation of any plane through the line (1) is
ax + by + cz + d + λ ( a ′ x + b ′ y + c ′ z + d ′ ) = 0. …(2)
If the plane (2) passes through the point P (0, 0, 0 ), then
d + λd ′ = 0 or λ = − d / d ′.
Putting this value of λ in (2), the equation of the plane through the origin and the given
line is
( ax + by + cz + d ) − ( d / d ′ ) ( a ′ x + b ′ y + c ′ z + d ′ ) = 0
or ( ad ′ − a ′ d ) x + ( bd ′ − b ′ d ) y + ( cd ′ − c ′ d ) z = 0. …(3)
G-90

Now let l, m, n be the d.r.’s of the given line (1), then


al + bm + cn = 0, and a ′ l + b ′ m + c ′ n = 0.
l m n
Solving , = = ⋅ …(4)
bc ′ − b ′ c ca ′ − c ′ a ab ′ − a ′ b
Thus the equation of the plane through P (0, 0, 0 )and perpendicular to the line (1) is
l ( x − 0 ) + m ( y − 0 ) + n (z − 0 ) = 0
or ( bc ′ − b ′ c ) x + ( ca ′ − c ′ a) y + ( ab ′ − a ′ b ) z = 0. …(5)
Hence (3) and (5) together are the equations of the perpendicular from the origin
to the given line (1).
x y z
Problem 3: The equations to AB referred to rectangular axes are = = ⋅ Through a
2 −3 6
point P (1, 2 , 5), PN is drawn perpendicular to AB and PQ is drawn parallel to the plane
3 x + 4 y + 5z = 0 to meet AB in Q. Find the equations to PN and PQ and the co-ordinates of N
and Q.
Solution: The equations of the line AB are given as
x y z
= = = r (say). …(1)
2 −3 6
Any point on (1) is (2r, − 3r, 6r ). Let this point be N. Also P is (1, 2 , 5).
∴ The d.r.’s of PN are 2 r − 1, − 3r − 2 , 6 r − 5.
Now PN will be perpendicular to the line AB given by the equations (1) if
26
2 (2r − 1) + ( −3) ( −3r − 2) + 6 (6r − 5) = 0 or r = ⋅
49
52 −78 156 
Putting the value of r , we have N  , , 
 49 49 49 
52 −78 156
and d.r.’s of PN are − 1, −2, − 5 or 3, − 176, − 89.
49 49 49
Thus the equations to PN are
x −1 y −2 z −5
= = ⋅ …(2)
3 −176 −89
Again the co-ordinates of any point Q on the line (1) are (2r , − 3r , 6r ) and the point P is
(1, 2 , 5).
∴ The d.r.’s of PQ are 2 r − 1, − 3r − 2 , 6r − 5.
Now PQ is drawn parallel to the plane
3 x + 4 y + 5z = 0. …(3)
∴ The line PQ is perpendicular to the normal to the plane (3), whose d.r.’s are 3, 4, 5.
Therefore we have
3
3 (2r − 1) + 4 ( −3r − 2) + 5 (6 r − 5) = 0 or r = ⋅
2
Putting this value of r , we have Q (3, − 9 / 2 , 9) and d.r.’s of PQ are 3 − 1, ( −9 / 2) − 2 ,
9 − 5 i. e., 4 , − 13, 8.
∴ The equations of PQ are given by
x −1 y −2 z −5
= = ⋅
4 −13 8
G-91

Comprehensive Problems 6
Problem 1: (i) Show that the lines
x+3 y+5 z −7 x+1 y +1 z +1
= = and = =
2 3 −3 4 5 −1
are coplanar. Find the equation of the plane containing them. (Purvanchal 2007)
(ii) Show that the lines 7 x − 4 y + 7z + 16 = 0 = 4 x + 3 y − 2z + 3 and
x − 3 y + 4z + 6 = 0 = x − y + z + 1 are coplanar.
Solution: (i) The equations of the given lines are
x+3 y + 5 z −7
= = = r1 (say), …(1)
2 3 −3
x+1 y +1 z +1
and = = = r2 (say). …(2)
4 5 −1
The co-ordinates of any point P on the line (1) are
(2r1 − 3, 3r1 − 5, − 3r1 + 7)
and those of any point Q on the line (2) are (4 r2 − 1, 5 r2 − 1, − r2 − 1).
If the lines (1) and (2) are coplanar, then they intersect and hence for some values of r1
and r2 the points P and Q coincide. Thus we have
2r1 − 3 = 4r2 − 1, 3r1 − 5 = 5 r2 − 1, − 3 r1 + 7 = − r2 − 1
or r1 − 2r2 = 1, 3r1 − 5r2 = 4, 3 r1 − r2 = 8.
Solving the first two equations, we get r1 = 3, r2 = 1.
These values of r1 and r2 also satisfy the third equation, and hence the lines (1) and (2)
are coplanar (i. e., intersect).
Putting the value of r1 (or the value of r2 ) the co-ordinates of the common point of
intersection i. e., P (or Q ) are (3, 4, − 2).
Now the equation of the plane containing the lines (1) and (2) [i. e., the plane containing
the line (1) and parallel to the line (2)] is
x+3 y + 5 z −7
2 3 −3 = 0 [See article 9]
4 5 −1

or ( x + 3) {3 ( −1) − ( −3) 5} − ( y + 5) {2 ( −1) − ( −3) 4}


+ ( z − 7) {2 . 5 − 3 . 4} = 0
or ( x + 3) . 12 − ( y + 5) . 10 + ( z − 7) . ( −2) = 0
or 6 x − 5 y − z = 0.
(ii) The given lines will be coplanar if we have
7 −4 7 16
4 3 −2 3
  = 0.
1 −3 4 6
 
1 −1 1 1
G-92

Applying R1 − 7R4 , R2 − 4R4 , R2 − R4 , the determinant on the left hand side


0 3 0 9
0  3 0 9
7 −6 −1 
=   = −

7 −6 −1,

0 −2 3 5
   −2 3 5
 1 −1 1 1
expanding the determinant along the first column
 3 0 0
= −  7 −6 −22, by C3 − 3C1
 
− 2 3 11
= − 3 { − 66 − ( −66)} = − 3 ( −66 + 66) = 0.
Hence the given lines are coplanar.
x −1 y −2 z −3 x−2 y −3 z −4
Problem 2: Prove that the lines = = and = =
2 3 4 3 4 5
are coplanar; find their point of intersection. Also find the equation of the plane in which they lie.
(Meerut 2003, 13; Rohilkhand 08; Kanpur 10; Kashi 13; Kumaun 15)
Solution: The equations of the given lines are
x −1 y −2 z −3
= = = r1(say), …(1)
2 3 4
x−2 y −3 z −4
and = = = r2 (say). …(2)
3 4 5
The co-ordinates of any point P on the line (1) are (2r1 + 1, 3r1 + 2 , 4r1 + 3) and those
of any point Q on the line (2) are (3 r2 + 2 , 4 r2 + 3, 5 r2 + 4).
If the lines (1) and (2) are coplanar then they intersect and hence for some values of r1
and r2 the points P and Q coincide. Thus we have
2r 1 + 1 = 3r 2 + 2 or 2r 1 − 3r 2 = 1 …(3)
3r 1 + 2 = 4r 2 + 3 or 3r 1 − 4r 2 = 1 …(4)
and 4r 1 + 3 = 5r 2 + 4 or 4r1 − 5r2 = 1. …(5)
Solving (3) and (5), we get r1 = − 1, r2 = − 1.
These values of r1 and r2 also satisfy the equation (4), and hence the given lines (1) and
(2) are coplanar. Putting the value of r1 (or of r2 ), the point of intersection P (or Q) is
( −1, − 1, − 1).
Now the equation of the plane in which the lines (1) and (2) lie is given by
x − 1 y −2 z −3
 2 3 4 = 0 [See article 9]
 
 3 4 5 
or x − 2 y + z = 0.

Problem 3(i): Show that the two given lines are coplanar :
1 1 1 1 1
x= ( y − 2) = ( z + 3) ; ( x − 2) = ( y − 6) = ( z − 3).
2 3 2 3 4
Also find the point of intersection and the equation of the plane in which they lie.
G-93

Solution: The lines are


x y −2 z + 3
= = …(1)
1 2 3
x−2 y −6 z −3
and = = ⋅ …(2)
2 3 4
Now any plane containing line (1) and parallel to line (2) is
x y −2 z + 3
1 2 3 =0
2 3 4
or x ( −1 ) − ( y − 2)( −2 ) + ( z + 3)( −1 ) = 0
x − 2 y + z + 7 = 0. …(3)
The two lines will be, coplanar if a point on line (2) say (2, 6, 3) satisfies (3), which is
evidently true. Hence the two lines are coplanar.
The plane containing them is x − 2 y + z + 7 = 0.
Now any point on line (1) is ( r1 , 2r1 + 2, 3r1 − 3).
If these points coincide, then
r1 = 2r 2 + 2, 2r1 + 2 = 3r2 + 6, 3r1 − 3 = 3r2 + 3.
Solving any two of these, we get r1 = 2 , r2 = 0.
Hence the point of intersection is (2, 6, 3).
x −1 y −1 z −1 x −4 y −6 z −8
Problem 3(ii): Show that the lines = = and = = are
1 2 3 2 3 4
coplanar. (Kumaun 2009, 14)
Solution: Proceed as in part (i).
x−a y−b z−c x−a′ y−b′ z−c′
Problem 4: Prove that the lines = = and = =
a′ b′ c′ a b c
intersect and find the co-ordinates of the point of intersection and the equation of the plane in
which they lie. (Meerut 2000)
Solution: Any point on the first line is P ( a ′ r1 + a, b ′ r1 + b, c ′ r1 + c )and any point on
the second line is Q ( ar2 + a ′, br2 + b ′, cr2 + c ′ ). The given lines will intersect if for
some values of r1 and r2 the points P and Q coincide. Thus, we have
a ′ r1 + a = ar2 + a ′, 

b ′ r1 + b = br2 + b ′,  ⋅
and c ′ r1 + c = cr2 + c ′ 

These equations are clearly satisfied by r1 = r2 = 1.
Hence the given lines intersect. Putting the value of r1 (or of r2 ), the point of
intersection P (or Q) is ( a + a ′, b + b ′, c + c ′ ).
Now the equation of the plane in which the given lines lie is given by
x − a y−b z − c x y z
 a′ b′ c ′ = 0 or  a b c  = 0.
   
 a b c   a ′ b ′ c ′
G-94

x−4 y+3 z +1 x −1 y +1 z + 10
Problem 5: Prove that the lines = = and = =
1 −4 7 2 −3 8
intersect, and find the co-ordinates of their point of intersection. (Kanpur 2007)

Solution: The lines are


x−4 y + 3 z +1
= = …(1)
1 −4 7
x −1 y + 1 z + 10
= = ⋅
2 −3 8
…(2)
Any point on line (1) is (r1 + 4, − 4r1 + 3, 7r1 − 1)
and any point on line (2) is (4r2 + 1, − 3r2 − 1, 8r2 − 10 ).
The lines will have common point if three equations
three equations
r1 + 4 = 2r2 + 1 …(3)
− 4r1 − 3 = − 3r2 − 1 …(4)
7r1 − 1 = 8r2 − 10 …(5)
are simultaneously satisfied. From equations (3) and (4), we get
r1 = 1 , r2 = 2
which clearly satisfy (5).
Hence the two lines intersect at the point (5 , −7, 6).
x y z x y z x y z
Problem 6: Prove that the lines = = ; = = and = =
l1 m1 n1 l2 m2 n2 l3 m3 n3
l1 m1 n1
will be co-planar if l2 m2 n2 = 0.
l3 m3 n3
Solution: The lines are
x y z
= = …(1)
l1 m1 n1
x y z
= = …(2)
l2 m2 n2
x y z
and = = ⋅ …(3)
l3 m3 n3
We clearly see that the three given lines pass through the origin O and therefore they
will be coplanar if they are perpendicular to a line through the origin O.
Let l , m, n be the d.c.’s of of this line through the origin O.
Hence if this line is perpendicular to the given lines, we have
ll 1 + mm1 + nn1 = 0 …(4)
ll2 + mm2 + nn2 = 0 …(5)
ll3 + mm3 + nn3 = 0. …(6)
Eliminating l , m, n between the equations (4), (5) and (6), we get the condition for the
coplanarity of the given lines as
G-95

l1 m1 n1
l2 m2 n2 = 0 .
l3 m3 n3

Problem 7: Prove that the lines x = ay + b = cz + d and x = αy + β = γz + δ are coplanar if


( aβ − bα ) ( γ − c ) − ( cδ − dγ ) (α − a) = 0.
Solution: The equations of the given lines are
x−0 y + b /a z + d/c
x = ay + b = c z + d or = = …(1)
1 1/ a 1/ c
x−0 y + β /α z + δ / γ
and x = α y + β = γz + δ or = = ⋅ …(2)
1 1 /α 1/ γ
The lines (1) and (2) will be coplanar if
0 − 0 − (β /α ) − ( b / a) − (δ / γ ) − ( − d / c )
1 1/ a 1/ c = 0, [See article 9]
1 1 /α 1/ γ
0 β /α − b / a δ / γ − d / c
or 0 1 / a − 1 /α 1 / c − 1 / γ = 0,
1 1 /α 1/ γ
subtracting the third row from the second row
 β − b   1 − 1  −  δ − d   1 − 1  = 0,
   
 α a   c γ   γ c   a α 
or

( aβ − bα ) ( γ − c ) ( cδ − d γ ) (α − a)
or = = 0,
aαc γ c γaα
or ( aβ − bα ) ( γ − c ) − ( cδ − d γ ) (α − a) = 0.
This is the required condition.

Problem 8: Show that the following pairs of lines are coplanar :


1 1 1
(i) ( x + 4) = ( y + 6) = − ( z − 1) and
3 5 2
3 x − 2 y + z + 5 = 0 = 2 x + 3 y + 4 z − 4.
Also find their point of intersection and the equation of the plane in which they lie.
1
(ii) x − 4 = − ( y + 1) = z and 4 x − y + 5z − 7 = 0 = 2 x − 5 y − z − 3.
2
Also find the equation of the plane containing them. (Kumaun 2008)
1 1
(iii) ( x − 3) = − ( y − 2) = z + 1 and x + 2 y + 3z = 0 = 2 x + 4 y + 3z + 3.
3 4
Also find the point of intersection.
Solution: (i) The equations of the given lines are
x+4 y + 6 z −1
= = = r (say) …(1)
3 5 −2
G-96

and 3 x − 2 y + z + 5 = 0, 2 x + 3 y + 4 z − 4 = 0. …(2)
The equation of any plane through the line (2) is
(3 x − 2 y + z + 5) + λ (2 x + 3 y + 4z − 4) = 0
or (3 + 2 λ ) x + ( −2 + 3 λ ) y + (1 + 4 λ ) z + (5 − 4 λ ) = 0. …(3)
Now if the plane (3) is parallel to the line (1), then we have
3
(3 + 2 λ ) (3) + ( −2 + 3 λ ) (5) + (1 + 4 λ ) ( −2) = 0 or λ=

13
Putting this value of λ in the equation (3), the equation of the plane through the line (2)
and parallel to the line (1) is given by
 3 + 6  x +  −2 + 9  y + 1 + 12  z +  5 − 12  = 0
       
 13   13   13   13 
or 45 x − 17 y + 25 z + 53 = 0. …(4)
Clearly the line (1) passes through the point ( −4, − 6, 1) and it satisfies the equation (4)
as
45 ( −4) − 17 ( −6) + 25 (1) + 53 = 0.
Hence the lines (1) and (2) intersect and they lie in the plane (4).
To find the point of intersection:
The co-ordinates of any point on the line (1) are
(3r − 4, 5 r − 6, − 2r + 1). …(5)
The lines (1) and (2) intersect, if this point also lies on the line (2) i. e., if it satisfies both
the equations of the line (2).
Hence we have
3 (3r − 4) − 2 ( 5r − 6) + ( −2r + 1) + 5 = 0 or r = 2
and 2 (3r − 4) + 3 (5r − 6) + 4 ( −2r + 1) − 4 = 0 or r = 2 .
Since both the equations give the same value of r , the two given lines intersect.
Putting the value of r in (5), the point of intersection is (2 , 4, − 3).
(ii) The lines are
x−4 y +1 z
= = …(1)
1 −2 1
and 4 x − y + 5z − 7 = 0 , 2 x − 5 y − z − 3 = 0. …(2)
Any plane containing (2) is
(4 x − y + 5z − 7) + λ (2 x − 5 y − z − 3) = 0
or (4 + 2λ ) x + ( −1 − 5λ ) y + (5 − λ )z − 7 − 3λ = 0. …(3)
If it is parallel to (1), then
1(4 + 2λ ) − 2( −1 − 5λ ) + 1(5 − λ ) = 0
or 4 + 2λ + 2 + 10 λ + 5 − λ = 0
or 11λ = − 11 or λ = −1.
Hence a plane containing (2) and parallel to (1) is
{4 + 2( −1 )} x + {− 1 − 5( − 1)} y + {5 − ( −11 )}z − 7 − 3( −1 ) = 0
or 2 x + 4 y + 6z = 4
or x + 2 y + 3z = 2. …(4)
G-97

The point (4, − 1, 0) on (1) clearly satisfies this equation.


Hence the two lines are coplanar and they lie in the plane (4).
(iii) The lines are
x−3 y −2 z +1
= = …(1)
3 −4 1
and x + 2 y + 3z = 0, 2 x + 4 y + 3z + 3 = 0. …(2)
The co-ordinates of any point on the line (1) are
(3r + 3, − 4r + 2, r − 1 ). …(3)
The lines (1) and (2) will intersect (i.e., will be coplanar) if the point (3) also lies on the
line (2).
This point satisfies both the equations of the line (2), if we have
(3r + 3) + 2( −4r + 2) + 3( r − 1) = 0 or r = + 2
and 2(3r + 3) + 4( −4r + 2) + 3( r − 1) + 3 = 0 or r = + 2.
As both the values of r are the same, the two given lines intersect (or coplanar).
Putting this value of r in (3), the point of intersection is (9, − 6, 1).
x y z
Problem 9: Find the equation of the plane through the line = =
l m n
x y z x y z
and perpendicular to the plane containing the lines = = and = = ⋅
m n l n l m
Solution: The equation of any plane through the line
x y z
= = is
l m n
Ax + By + Cz = 0, …(1)
where Al + Bm + Cn = 0. …(2)
Also the equation of the plane through the lines
x y z x y z
= = and = =
m n l n l m
(note that both of these lines are passing through the origin) is
x y z
m n l  = 0, [See article 9]
 
 n l m 
or ( mn − l2 ) x + ( ln − m2 ) y + ( ml − n2 ) z = 0. …(3)
According to the question, the planes (1) and (3) should be perpendicular the condition
for which is
A ( mn − l2 ) + B ( ln − m2 ) + C ( ml − n2 ) = 0. …(4)
Solving the relations (2) and (4), we get
A B
=
m ( ml − n2 ) − n ( ln − m2 ) n ( mn − l2 ) − l ( ml − n2 )
C
=
l ( ln − m ) − m ( mn − l2 )
2
G-98

A B
or =
m2 l − mn2 − n2 l + nm2 mn2 − nl2 − ml2 + ln2
C
=
l n − lm − m2 n + ml2
2 2

A B
or =
( m2 − n2 ) l + mn ( m − n) ( n2 − l2 ) m + nl ( n − l )
C
=
2 2
( l − m ) n + lm ( l − m)
A B
or =
( m − n) ( ml + nl + mn) ( n − l ) ( nm + lm + nl )
C
=
( l − m) ( ln + mn + lm)
A B C
or = = ⋅
m− n n− l l − m
Putting these proportionate values of A, B, C in (1), the equation of the required plane
is given by
( m − n) x + ( n − l ) y + ( l − m) z = 0.
Problem 10: Find the foot and hence the length of the perpendicular from the point (5, 7, 3) to
x − 15 y − 29 z − 5
the line = = ⋅ Find the equations of the perpendicular. Also find the
3 8 −5
equation of the plane in which the perpendicular and the given straight line lie.
(Purvanchal 2011)
Solution: Let the given point (5, 7, 3) be P.
The equations of the given line are
x − 15 y − 29 z − 5
= = = r (say). …(1)
3 8 −5
Let N be the foot of the perpendicular from the point P to the line (1). The co-ordinates
of N may be taken as
(3r + 15, 8r + 29, − 5r + 5). …(2)
∴ The direction ratios of the perpendicular PN are
3r + 15 − 5, 8r + 29 − 7, − 5r + 5 − 3,
i. e., are 3r + 10, 8r + 22 , − 5r + 2 . …(3)
Since the line (1) and the line PN are perpendicular to each other, therefore
3 (3r + 10 ) + 8 (8r + 22) − 5 ( −5r + 2) = 0
or 98r + 196 = 0 or r = − 2 .
Putting this value of r in (2) and (3), the foot of the perpendicular N is (9, 13, 15) and the
direction ratios of the perpendicular PN are 4, 6, 12 or 2 , 3, 6.
∴ The equations of the perpendicular PN are
x−5 y −7 z −3
= = ⋅ …(4)
2 3 6
G-99

Length of the perpendicular PN


= the distance between P (5, 7, 3) and N (9, 13, 15)
= √ {(9 − 5)2 + (13 − 7)2 + (15 − 3)2 } = 14.
Lastly the equation of the plane containing the given line (1) and the perpendicular (4)
is given by
x − 15 y − 29 z − 5
3 8 −5 = 0 [See article 9]
2 3 6
or ( x − 15) (48 + 15) − ( y − 29) (18 + 10 ) + ( z − 5) (9 − 16) = 0
or 9 x − 4 y − z − 14 = 0.

Comprehensive Problems 7
Problem 1: Find the equations to the planes through the point (1, 0, − 1) and the lines
4 x − y − 13 = 0 = 3 y − 4z − 1 and y − 2z + 2 = 0 = x − 5 and show that the equations to
the line through the given point which intersects the two given lines can be written as
x = y +1= z + 2.
Solution: The equations of the given lines are
4 x − y − 13 = 0, 3 y − 4z − 1 = 0 …(1)
and y − 2z + 2 = 0, x − 5 = 0. …(2)
The equations of any planes through the given lines (1) and (2) are respectively given by
4 x − y − 13 + µ 1 (3 y − 4z − 1) = 0,
and ( y − 2z + 2) + µ 2 ( x − 5) = 0.
If these planes pass through the point (1, 0, − 1), we get
4 − 0 − 13 + µ 1 (0 + 4 − 1) = 0
giving µ 1 = 3, and 0 + 2 + 2 + µ 2 (1 − 5) = 0 giving µ 2 = 1.
Putting these values of µ 1 and µ 2 , the required equations of the planes are
4 x − y − 13 + 3 (3 y − 4z − 1) = 0,
and ( y − 2z + 2) + 1 . ( x − 5) = 0
or x + 2 y − 3z − 4 = 0 and x + y − 2z − 3 = 0. …(3)
The equations (3) are the general equations of a line through the given point (1, 0, − 1)
and intersecting the given lines (1) and (2).
Transforming the equations (3) to the symmetrical form, we get
x−0 y +1 z + 2
= = or x = y + 1 = z + 2 .
1 1 1
Problem 2: Find the equations to the straight line drawn from the origin to intersect the lines
2 x + 5 y + 3z − 4 = 0 = x − y − 5z − 6
and 3 x − y + 2z − 1 = 0 = x + 2 y − z − 2 .
G-100

Solution: The equations of any line intersecting the given lines are
(2 x + 5 y + 3z − 4) + µ 1 ( x − y − 5z − 6) = 0,

and (3 x − y + 2z − 1) + µ 2 ( x + 2 y − z − 2) = 0.
[See article 12, case II]
If this line passes through (0, 0, 0 ), then we have
−4 − 6µ 1 = 0 or µ 1 = − 2 / 3 ;
and −1 − 2µ 2 = 0 or µ 2 = − 1 / 2 .
Putting these values of µ 1 and µ 2 , the equations of the required line are
(2 x + 5 y + 3z − 4) − (2 /3) ( x − y − 5z − 6) = 0
1
and (3 x − y + 2z − 1) − ( x + 2 y − z − 2) = 0
2
or 4 x + 17 y + 19z = 0 and 5 x − 4 y + 5z = 0.
1
Problem 3: Find the equations to the line drawn parallel to x = y = z , so as to meet the lines
4
5 x − 6 = 4 y + 3 = z and 2 x − 4 = 3 y + 5 = z.
Solution: The general equations of the line 5 x − 6 = 4 y + 3 = z may be written as
5 x − 6 − (4 y + 3) = 0
and 5x − 6 − z = 0
or 5 x − 4 y − 9 = 0 = 5 x − z − 6. …(1)
Similarly the general equations of the line 2 x − 4 = 3 y + 5 = z may be written as
2 x − 3 y − 9 = 0 = 2 x − z − 4. …(2)
The equations of any line intersecting the given lines [i. e., the lines (1) and (2)] are
(5 x − 4 y − 9) + µ 1 (5 x − z − 6) = 0
and (2 x − 3 y − 9) + µ 2 (2 x − z − 4) = 0
or 5 (1 + µ 1 ) x − 4 y − µ 1 z − (9 + 6µ 1 ) = 0 
 …(3)
and 2 (1 + µ 2 ) x − 3 y − µ 2 z − (9 + 4µ 2 ) = 0.
x y z
=
If the line (3) is parallel to the line= , then this latter line is perpendicular to the
4 1 1
normals of each of the two planes given by (3), so that we have
4 . 5 (1 + µ 1 ) + 1 . ( −4) + 1 . ( − µ 1) = 0 giving µ 1 = − 16 / 19
and 4 . 2 (1 + µ 2 ) + 1 . ( −3) + 1 . ( − µ 2 ) = 0 giving µ 2 = − 5 / 7.
Putting these values of µ 1 and µ 2 in (3) and simplifying, the required equations of the
line are
15 x − 76 y + 16z − 75 = 0 and 4 x − 21 y + 5z − 43 = 0.
Problem 4: A line with direction cosines proportional to 2 , 1, 2 meets each of the lines given by
the equations x = y + a = z , x + a = 2 y = 2z .
Find the co-ordinates of each of the points of intersection.
Solution: The equations of the given lines are
x y+ a z
= = = r1 (say), …(1)
1 1 1
G-101

x+ a y z
and = = = r2 (say). …(2)
2 1 1
Any point P on (1) is ( r1, r1 − a, r1 ),
and any point Q on (2) is (2r2 − a, r2 , r2 ).
The d.r.’s of PQ are r1 − 2 r2 + a, r2 − r1 − a, r1 − r2 .
If the d.r.’s of PQ are proportional to 2 , 1, 2 , then
r1 − 2 r2 + a r1 − r2 − a r1 − r2
= = ⋅ …(3)
2 1 2
From the first two of (3), we get r1 − 3a = 0 or r1 = 3a and from the last two of (3),
we get
r1 − r2 − 2a = 0 or r2 = a.
Putting these values of r1 and r2 , the co-ordinates of the points of intersection are
P (3a, 2a, 3a) and Q ( a, a, a).
Problem 5: Find the equations to the line intersecting the lines
x − 1 = y = z − 1, 2 x + 2 = 2 y = z + 1
1 1
and parallel to the line ( x − 1) = ( y − 1) = ( z − 2).
2 3
Solution: Here the required line being parallel to the given line
1 1
( x − 1) = ( y − 1) = ( z − 2),
2 3
will have its d.r.’s 2 , 1, 3.
Proceeding as in Example 19, the equations of the required line PQ are
1 1
( x − 1) = y = ( z − 1).
2 3
Problem 6: Find the equations to the straight line drawn through the origin which will
intersect both the lines
x −1 y + 3 z −5 x−4 y + 3 z − 14
= = and = = ⋅
1 4 3 2 3 4
Solution: The equation of any plane through the first line is
A ( x − 1) + B ( y + 3) + C ( z − 5) = 0, …(1)
where 1 . A + 4 . B + 3 . C = 0. …(2)
If the plane (1) passes through (0, 0, 0 ), we have
− A + 3B − 5C = 0. …(3)
A B C
Solving (2) and (3), we get = = ⋅
29 −2 −7
Substituting these proportionate values of A, B, C in (1), the equation of the plane
through the origin and the first line is
29 x − 2 y − 7z = 0. …(4)
Similarly the equation of the plane through the origin and the second line is
9 x − 2 y − 3 z = 0. …(5)
The planes (4) and (5) together give the required line.
G-102

Comprehensive Problems 8
Problem 1: Find the distance of ( −2 , 1, 5) from the line through (2 , 3, 5) whose direction
cosines are proportional to 2 , − 3, 6.
Solution: The equation of line through (2, 3, 5) where d.r.’s 2, − 3, 6 is
x−2 y −3 z−5
= = ⋅ …(1)
2 −3 6
The co-ordinates of any point N on the line (1) are
(2r + 2, − 3r + 3, 6r + 5).
Let P be ( −2, 1, 5) be any point then the d.r.’s of PN are 2r + 4, − 3r + 2, 6r but PN is
perpendicular to given line (1).
∴ 2(2r + 4) − 3(3r + 2) + 6(6r ) = 0
−2
or 49r = − 2 or r= .
49
∴ Co-ordinates of N are  ,
94 153 237 
, .
 49 49 49 
2 2 2
So the distance PN is  + 2 +  − 1 +  − 5
94 153 237
 49   49   49 
2 2 2
 192  +  104  +  −12 
or      
 49   49   49 
47824 976 4 61
or or or .
49 × 49 49 7
Problem 2: Prove that the equations of the perpendicular from the point (1, 6, 3) to the line
y −1 z −2 x −1 y −6 z −3
x= = are = = and the co-ordinates of the foot of the
2 3 0 −3 2
perpendicular are (1, 3, 5).
x y −1 z − 2
Solution: Here the point P is (1, 6, 3) and the equation of line AB is = = .
1 2 3
The d.r.’s of line AB are 1, 2, 3.
Let N be the foot of perpendicular from P to AB be
( r , 1 + 2r , 2 + 3r ). …(1)
The d.r.’s of the line PN are
r − 1, 1 + 2r − 6, 2 + 3r − 3
or r − 1, 2r − 5, 3r − 1. …(2)
As AN is perpendicular to AB,
so 1( r − 1) + 2(2r − 5) + 3(3r − 1) = 0 or r = 1.
∴ From (1) the co-ordinates of N are (1, 3, 5)and from (2) d.r.’s of line PN are 0 , − 3, 2.
∴ Equation of the line perpendicular to PN are
x −1 y −6 z −3
= = ⋅
0 −3 2
G-103

Problem 3: How far is the point (4, 1, 1) from the line of intersection of
x + y + z −4=0 = x −2y − z −4?
Solution: Let the point (4, 1, 1) be taken as P.
The equations of the given line AB (say) are
( x − 4) + y + z = 0, ( x − 4) − 2 y − z = 0.
Solving for x − 4, y, z we get
x−4 y z x−4 y z
= = or = = ⋅ …(1)
−1 + 2 1 + 1 −2 − 1 1 2 −3
Equations (1) are the symmetrical form of the given line AB. The line (1) has d.r.’s
1, 2 , − 3 and passes through the point A (4, 0, 0 ).
Draw PN perpendicular to the line AB given by (1), so that N, the foot of perpendicular,
lies on (1). Hence the co-ordinates of N may be written as
( r + 4, 2r, − 3r ). …(2)
The d.r.’s of PN are
r + 4 − 4, 2r − 1, − 3r − 1 i. e., r, 2r − 1, − 3r − 1.
Since PN is perpendicular to AB, we have
r . 1 + (2r − 1) . 2 + ( −3r − 1) ( −3) = 0 or r = − 1 / 14.
Putting the value of r in (2), we get
N ≡ (55 / 14, − 1 / 7, 3 / 14).
∴ The distance of P (4, 1, 1) from the given line (1)
= the distance between the points P and N
 55 2 2 2  √ (378) 3 √ (42)
− 4 +  − − 1 +  − 1
1 3
=  = = ⋅
 14   7   14   14 14

Problem 4: Find the length of the perpendicular drawn from origin to the line
x + 2 y + 3z + 4 = 0 = 2 x + 3 y + 4z + 5. Also find the equations of this perpendicular and
the co-ordinates of the foot of the perpendicular.
Solution: The equations of the given line AB (say) are
x + 2 y + 3z + 4 = 0 = 2 x + 3 y + 4z + 5.
The symmetrical form of the above given line is
x−2 y + 3 z −0
= = ⋅ …(1)
1 −2 1
The line AB given by (1) is passing through A (2 , − 3, 0 ) and has d.r.’s 1, − 2 , 1. Here
the point P is (0, 0, 0 ).
Draw PN perpendicular to AB, so that N, the foot of the perpendicular lies on (1). Hence
the co-ordinates of N may be written as
( r + 2 , − 2r − 3, r ). …(2)
The d.r.’s of PN are r + 2 − 0, − 2 r − 3 − 0, r − 0
i. e., r + 2 , − 2r − 3, r. …(3)
Since PN is perpendicular to AB, we have
( r + 2) . 1 + ( −2 r − 3) ( −2) + r . 1 = 0 or r = − 4 / 3.
G-104

Putting the value of r in (2), we get


N ≡ (2 / 3, − 1 / 3, − 4 / 3).
Again putting the value of r in (3), the d.r.’s of PN are
2 / 3, − 1 / 3, − 4 / 3 i. e., 2 , − 1, − 4.
∴ The equations to PN i. e., the equations of a line passing through P (0, 0, 0 ) and
having d.r.’s 2 , − 1, − 4 are
x−0 y −0 z −0 x y z
= = or = = ⋅
2 −1 −4 2 −1 −4
The length of the perpendicular PN = the distance between the points P and N

 2 
2
 1 
2
 4   √ (21) ⋅
2
=  − 0  +  − − 0  +  − − 0   =
 3 3 3  3

Comprehensive Problems 9
Problem 1: Examine the nature of the intersection of the sets of the planes :
(i) x − y + z = 3, 2 x + 5 y + 3z = 0, 3 x − 2 y − 6z + 1 = 0 ;
(ii) 3 x + 2 y + z = 6, 5 x + 4 y + 3z = 4, 3 x + 4 y + 5z + 12 = 0 ;
(iii) x − y + z − 4 = 0, 2 x − y − z + 4 = 0, x + y − 5z + 14 = 0.
(iv) 2 x − 3 y − z = − 3, x + 2 y + 3z = 2 and − x + 2 y + z = 2. (Kumaun 2010, 15)
Solution: (i) The equation of the given planes are
x− y + z =3 …(1)
2 x + 5 y + 3z = 0 …(2)
3 x − 2 y − 6z + 1 = 0. …(3)
The rectangular array of coefficients is
1 −1 1 −3
D= 2 5 3 0 . …(4)
3 −2 −6 1
Omitting the fourth column from (4), we have
1 −1 1
∆4 = 2 5 3 .
3 −2 6
Expanding this determinant along the first row, we have
∆4 = 1(30 + 6) + 1(12 − 9) + 1( −4 − 15)
= 36 + 3 − 19 = 20 ≠ 0.
Hence the given planes intersect in a point.
(ii) The equation of the given planes are
3x + 2 y + z = 6 …(1)
5 x + 4 y + 3z = 4 …(2)
3 x + 4 y + 5z + 12 = 0. …(3)
G-105

The rectangular array of coefficients is


3 2 1 −6
D = 5 4 3 −4 . …(4)
3 4 5 12
Omitting the fourth column from (4), we have
3 2 1
∆4 = 5 4 3 .
3 4 5
Expanding by means of the first row, we have
∆4 = 3(20 − 12) − 2(25 − 9) + 1(20 − 12)
= 24 − 32 + 8 = 0.
Now omitting the third column from (4), we have
3 2 −6
∆3 = 5 4 − 4
3 4 13
= 3(52 + 16) − 2(65 + 12) − 6(20 − 12)
= 0.
Similarly, we find that
3 1 −6 2 1 −6
∆2 = 5 3 − 4 and ∆1 = 4 3 − 4
3 5 + 12 4 5 12
Hence the given three planes intersect in a line.
(iii) The equation of the given planes are
x− y + z −4=0 …(1)
2x − y − z + 4 = 0 …(2)
x + y − 5z + 14 = 0. …(3)
The rectangular array of coefficients is
1 −1 1 −4
D = 2 −1 −1 4 . …(4)
1 1 −5 14
Omitting the fourth column from (4), we have
1 −1 1
∆4 = 2 − 1 − 1 .
1 1 −5
Expanding by means of the first row, we have
∆4 = 1(5 + 1) + 1( −10 + 1) + 1(2 + 1)
= 6 − 9 + 3 = 0.
Now omitting the third column from (4), we have
G-106

1 −1 −4
∆3 = 2 − 1 4
1 1 14
= 1( −14 − 4) + 1(28 − 4) − 4(2 + 1)
= − 18 + 24 − 12 = − 6 ≠ 0.
Hence the given three planes form a triangular prism.
(iv) Proceed as in part (ii).
Ans. The planes have a common line of intersection.

Problem 2: Examine the nature of the intersection of the sets of planes :


(i) x + 2 y − 5z = 1, 4 x + y + z = 2 , 6 x + y + 3z = 3 ;
(ii) x + 4 y + 6z = 5, 2 x + 5 y + 9z = 10, x + 3 y + 5z = 5 ;
(iii) x − y + z = 2 , 2 x − 3 y + 4z = 8, x + y + z = 2 ;
(iv) x + 3 y − z = 6, x + 2 y + 4z + 5 = 0, 2 x + 6 y − 2 z + 7 = 0.
Solution: (i) The equation's of the given planes are
x + 2 y − 5z = 1 …(1)
4x + y + z = 2 …(2)
6 x + y + 3z = 3. …(3)
The rectangular array of coefficients is
1 2 −5 −1
D = 4 1 1 −2 .
6 1 3 −3
1 2 −5
We have ∆4 = 4 1 1
6 1 3
= 1(3 − 1) − 2(12 − 6) − 5(4 − 6)
= 2 − 12 + 10 = 0 .
Since ∆4 = 0, therefore, the three planes either intersect in a line or form a triangular
prism.
1 2 −1
Now ∆3 = 4 1 − 2
6 1 −3
= 1( −3 + 2) − 2( −12 + 12) − 1(4 − 6)
= − 1 + 0 + 2 = 1 ≠ 0.
Hence the three planes form a triangular prism.
(ii) The equation of the given planes are
x + 4 y + 6z − 5 = 0 …(1)
2 x + 5 y + 9z − 10 = 0 …(2)
x + 3 y + 5z − 5 = 0. …(3)
The rectangular array of coefficients is
G-107

1 4 6 −5
D = 2 5 9 −10 .
1 2 5 −5
1 4 6
We have ∆4 = 2 5 9
1 3 5
= 1(25 − 27) − 4(10 − 9) + 6(6 − 5)
= − 2 − 4 + 6 = 0.
Since ∆4 = 0, therefore, the three planes either intersect in a line or form a triangular
prism.
1 4 −5
Now ∆3 = 2 5 −10 = 0 .
1 3 −5
Similarly ∆2 = 0 and ∆1 = 0 .
Hence the three planes intersect in a common line.
(iii) The equation of the given planes are
x− y + z −2=0 …(1)
2 x − 3 y + 4z − 8 = 0 …(2)
x + y + z − 2 = 0. …(3)
The rectangular array of coefficients is
1 −1 1 −2
D = 2 −3 4 −8 .
1 1 1 −2
1 −1 1
We have ∆4 = 2 − 3 4
1 1 1
∆4 = 1( −3 − 4) + 1(2 − 4) + 1 (2 + 3)
= − 7 − 2 + 5 = − 4 ≠ 0.
Hence the given planes intersect at a point.
(iv) The rectangular array of coefficients is
1 3 −1 −6
D= 1 2 4 5 .
2 6 −2 7
1 3 −1
We have ∆4 = 1 2 4
2 6 −2
= 1( −4 − 24) − 3( −2 − 8) − 1(6 − 4)
= − 28 − 30 − 2 = − 60 ≠ 0.
Hence the given planes intersect at a point.
Here given planes (1) and (3) are parallel to each other.
G-108

Problem 3: Show that the planes


2 x − 3 y − 7z = 0, 3 x − 14 y − 13z = 0, 8 x − 31 y − 33z = 0
pass through one line and find its equations. (Meerut 2011)
Solution: The rectangular array of coefficients is
2 −3 −7 0
3 −14 −13 0 ⋅
8 −31 −33 0
2 −3 −7  2 −1 −1
We have, ∆4 = 3 −14 −13  = 3 −11 −4 , by C2 + C1, C2 + 3C1
   
8 −31 −33  8 −23 −9 
 0 0 −1
=  −5 −7 −4 , by C1 + 2 C3 , C2 − C3
 
−10 −14 −9 
= − 1 (70 − 70 ) = 0.
Since ∆4 = 0, therefore, the three planes either intersect in a line or form a triangular
prism.
2 −3 0 
Now ∆3 = 3 −14 0  = 0.
 
8 −31 0 
Similarly ∆2 = 0 and ∆1 = 0.
Hence the three planes intersect in a common line.
Clearly the three planes pass through (0, 0, 0 ) and hence the common line of
intersection will pass through (0, 0, 0 ). The equations of the common line are given by
any of the two given planes. Therefore the equations of the common line are given by
2 x − 3 y − 7z = 0 and 3 x − 14 y − 13z = 0.
∴ The symmetrical form of the line is given by
x y z x y z
= = or = = ⋅
39 − 98 −21 + 26 −28 + 9 −59 5 −19
Problem 4: Prove that the planes
x + ay + ( b + c ) z + d = 0, x + by + ( c + a) z + d = 0, x + cy + ( a + b ) z + d = 0,
pass through one line. (Rohilkhand 2008)
Solution: The rectangular array (or matrix) is
1 a b+ c d
1 b c+a d ⋅
1 c a+ b d
1 a b+ c  1 a+ b+ c b+ c
We have, ∆4 = 1 b c+ a  = 1 a+ b+ c c+ a ,
   
1 c a+ b  1 a+ b+ c a+ b
adding 3rd column to 2nd
G-109

1 1 b+ c
= ( a + b + c )1 1 c+ a  = 0.
 
1 1 a+ b

1 a d 1 a 1
Also ∆3 = 1 b d= d1 b 1 = 0.
   
1 c d 1 c 1
Similarly ∆2 = 0 and ∆1 = 0.
Since ∆4 = 0, ∆3 = 0, ∆2 = 0 and ∆1 = 0, therefore the given planes intersect in a line.

Problem 5: Prove that the planes x = y sin ψ + z sin φ, y = z sin θ + x sin ψ , and
x y z
z = x sin φ + y sin θ will intersect in the line = = if θ + φ + ψ = π /2 .
cos θ cos φ cos ψ
Solution: The equation of the planes may be written as
x − y sin ψ − z sin φ = 0 , …(1)
x sin ψ − y + z sinθ = 0, …(2)
x sin φ + y sinθ − z = 0. …(3)
Let us find the line of intersection of the planes (1) and (2). Let l, m, n be the direction
cosines of the line of intersection of the planes (1) and (2). It being perpendicular to the
normals of both the planes, we have
l − msin ψ − nsin φ = 0, l sin ψ − m + nsinθ = 0.
Solving, we get
l m n
= =
− sin ψsinθ − sin φ − sin φ sin ψ − sinθ −1 + sin2 ψ
l m n
or = = ⋅ …(4)
sin ψsinθ + sin φ sin φ sin ψ + sinθ 1 − sin2 ψ
1
If θ + φ + ψ = π, then we have
2
sinθ = sin  π − ( φ + ψ) = cos ( φ + ψ) = cos φ cos ψ − sin φ sin ψ
1
2 
or sinθ + sin φ sin ψ = cos φ cos ψ. …(5)
Similarly sin φ + sin ψ sinθ = cos ψ cos θ. …(6)
Using the relations (5) and (6), (4) becomes
l m n l m n
= = or = = ⋅ …(7)
cos ψ cos θ cos ψ cos φ cos2 ψ cos θ cos φ cos ψ
Clearly the planes (1) and (2) pass through (0, 0, 0) and so their line of intersection will
pass through (0, 0, 0) so that its equations are given by
x y z
= = ⋅ …(8)
cos θ cos φ cos ψ
Now it remains to prove that the line (8) must lie on the plane (3).
G-110

The point (0, 0, 0) through which the line (8) passes also lies on the plane (3). Also the
normal to the plane (3) whose d.r.’s are sin φ, sinθ, −1 must be perpendicular to (8), the
condition for which is
cos θsin φ + cos φ sinθ + cos ψ ( −1) = 0

sin(θ + φ) − cos ψ = 0 or sin  π − ψ − cos ψ = 0


1
or
2 
or cos ψ − cos ψ = 0 or 0 = 0 , which is true.
Hence the line (8) also lies on the plane (3). Thus the equations (8) are the
equations of the required line.

Comprehensive Problems 10
Problem 1: Find the equations of the shortest distance and its length between the lines
x−3 y −5 z − 2 x −1 y +1 z +1
= = ; = = ⋅
1 −2 1 7 −6 1
(Meerut 2000, 06B; Rohilkhand 05, 09, 09B; Kanpur 06)
Solution: The given lines are
x −3 y −5 z −2
= = = r1 (say) …(1)
1 −2 1
x −1 y +1 z +1
and = = = r2 (say). …(2)
7 −6 1
Let l , m, n be the d.c.’s of the line S.D.
Since it is perpendicular to both the given lines (1) and (2), therefore we have
l − 2m + n = 0
7l − 6m + n = 0 .
Solving these, we get
l m n l m n
= = or = =
−2 + 6 7 − 1 −6 + 14 4 6 8

l m n l2 + m2 + n2 1
or = = = = ⋅
2 3 4 2 2
(2) + (3) + (4)2 √ 29

2 3 4
∴ The d.c.’s of S.D. are , , .
√ 29 √ 29 √ 29
Now A(3, 5, 2) is a point on line (1) and B(1, − 1, − 1) is a point on the line (2).
The length of S.D.= the projection of join of A and B on the line whose d.c.’s
2 3 4
are , ,
√ 29 √ 29 √ 29
2 3 4
= (3 − 1) + (5 + 1) + (2 + 1)
√ 29 √ 29 √ 29
1 34
= (4 + 18 + 12) = .
√ 29 √ 29
G-111

The equation of S.D.:


The equation of the plane through the line (1) and S.D. is
x −3 y − 5 z −2
1 −2 1 = 0 or −11x − 2 y + 7z + 9 = 0. …(3)
2 3 4
And the equation of the plane through the line (2) and the S.D. is
x −1 y + 1 z + 1
7 −6 1 = 0 or 27 x + 26 y − 33z − 34 = 0. …(4)
2 3 4
∴ From equations (3) and (4) the equations of the S.D. are
−11x − 2 y + 7z + 9 = 0, 27 x + 26 y − 33z − 34 = 0.

Problem 2: Find the equations of the shortest distance and its length between the lines
x −3 y −5 z −7 x +1 y +1 z +1
= = ; = = ⋅
1 −2 1 7 −6 1
(Rohilkhand 2005; Kanpur 08; Kumaun 09, 15; Lucknow 11; Meerut 12)
x −1 y −2 z −3
Show also that its equations are given by = = ⋅
2 3 4
Solution: The equations of the given lines are
x−3 y −5 z −7
= = = r1 (say) ; …(1)
1 −2 1
x+1 y +1 z +1
and = = = r2 (say). …(2)
7 −6 1
Any point on the line (1) is
( r1 + 3, − 2 r1 + 5, r1 + 7), say P. …(3)
Any point on the line (2) is
(7r2 − 1, − 6 r2 − 1, r2 − 1), say Q. …(4)
The d.r.’s of the line PQ are
(7r2 − 1) − ( r1 + 3), ( −6 r2 − 1) − ( −2 r1 + 5), ( r2 − 1) − ( r1 + 7)
or 7r2 − r1 − 4, − 6r2 + 2 r1 − 6, r2 − r1 − 8. …(5)
Let the line PQ be the line of S.D., so that PQ is perpendicular to both the given lines (1)
and (2), and so we have
1 (7r2 − r1 − 4) − 2 ( −6 r2 + 2 r1 − 6) + 1 ( r2 − r1 − 8) = 0
and 7 (7r2 − r1 − 4) − 6 ( −6 r2 + 2 r1 − 6) + 1 . ( r2 − r1 − 8) = 0
or 20 r2 − 6 r1 = 0 and 86 r2 − 20 r1 = 0.
Solving these equations, we get r1 = r2 = 0.
Substituting the values of r1 and r2 in (3), (4) and (5), we have
P (3, 5, 7), Q ( −1, − 1, − 1)
and the d.r.’s of PQ (the line of S.D.) are −4, − 6, − 8 or 2 , 3, 4.
The length of S.D. = PQ = √ {( −1 − 3)2 + ( −1 − 5)2 + ( −1 − 7)2 }
= √ {(4)2 + (6)2 + (8)2 } = 2 √ (29).
G-112

Now the line PQ of shortest distance is a line passing through P (3, 5, 7) and having d.r.’s
2 , 3, 4 and hence its equations are given by
x−3 y −5 z −7 x−3 y −5 z −7
= = or +1= +1= +1
2 3 4 2 3 4
x −1 y −2 z −3
or = = ⋅
2 3 4
Problem 3: Find the length of the shortest distance between the lines
x −3 y −5 z −7 x +1 y +1 z +1
= = ; = = . Find also its equations.
1 −2 1 7 −6 −1
(Meerut 2006B; Kanpur 06, 10)
Solution: Proceed as in Problem 2. Ans. 6 √ 3.

Problem 4: Find the shortest distance between the lines


x −1 y −2 z −3 x−2 y −3 z −4
= = ; = = ⋅
2 3 4 3 4 5
Hence show that the lines are coplanar. (Bundelkhand 2005; Meerut 07B)
Solution: We are solving this problem by projection method. The equations of the
given lines are
x −1 y −2 z −3
= = ; …(1)
2 3 4
x−2 y −3 z −4
and = = ⋅ …(2)
3 4 5
Let l, m, n be the d.c.’s of the line of S.D. The line of S.D. being perpendicular to both the
lines (1) and (2), we have
2 l + 3m + 4n = 0 and 3l + 4m + 5n = 0.
Solving these equations, we have
l m n
= =
15 − 16 12 − 10 8 − 9
l m n √ ( l2 + m2 + n2 ) 1
or = = = = ⋅ …(3)
−1 2 −1 √ {( −1)2 + (2)2 + ( −1)2 } √ 6
Clearly the line (1) passes through the point A (1, 2 , 3) and the line (2) passes through
B (2 , 3, 4).
∴ The length of S.D. = the projection of join of A and B on
the line of S.D. whose d.c.’s are l, m, n
= l (2 − 1) + m (3 − 2) + n (4 − 3)
−1 2 1
= (1) + (1) − (1) = 0.
√6 √6 √6
Since the length of S.D. = 0, hence the given lines are coplanar i. e., intersecting.
Problem 5: Find the points on the lines
x−6 −x y + 9 z −2
= − ( y − 7) = z − 4 and = =
3 3 2 4
which are nearest to each other. Hence find the shortest distance between the lines and also its
equations.
G-113

Solution: The equations of the given lines are


x−6 y −7 z −4
= = = r1 (say) …(1)
3 −1 1
x y + 9 z −2
and = = = r2 (say). …(2)
−3 2 4
The points on the lines (1) and (2) which are nearest to each other are the points
where the line of S.D. meets the lines (1) and (2).
Any point P on (1) is (3r1 + 6, − r1 + 7, r1 + 4) …(3)
and any point Q on (2) is ( −3r2 , 2 r2 − 9, 4r2 + 2). …(4)
The d.r.’s of PQ are
−3r2 − 3r1 − 6, 2 r2 + r1 − 16, 4r2 − r1 − 2 . …(5)
Let the required points be P and Q, so that PQ is the line of S.D. Hence PQ is
perpendicular to both the given lines (1) and (2) and so we have
3 ( −3r2 − 3r1 − 6) − 1 (2 r2 + r1 − 16) + 1 . (4 r2 − r1 − 2) = 0
and −3 ( −3r2 − 3r1 − 6) + 2 (2 r2 + r1 − 16) + 4 . (4 r2 − r1 − 2) = 0
or −7r2 − 11r1 − 4 = 0 and 29 r2 + 7r1 − 22 = 0.
Solving these equations, we get r1 = − 1, r2 = 1.
Substituting these values in (3), (4) and (5), we have
P (3, 8, 3), Q ( −3, − 7, 6)
and d.r.’s of PQ (the line of S.D.) are
−6, − 15, 3 or 2 , 5, − 1.
The length of S.D. = PQ = √ {( −3 − 3)2 + ( −7 − 8)2 + (6 − 3)2 }
= √ {(6)2 + (15)2 + (3)2 } = 3 √ (30 ).
The line PQ of shortest distance is a line passing through P (3, 8, 3) and having d.r.’s
2 , 5, − 1 and hence its equations are
x−3 y −8 z −3
= = ⋅
2 5 −1
Problem 6: Find the equations of the straight line perpendicular to both the lines
x −1 y −1 z+2 x+2 y −5 z+3
= = ; = = ⋅
1 2 2 2 −1 2
Solution: The given lines are
x −1 y −1 z + 2
= = = r1 (say) …(1)
1 2 2
x+2 y −5 z+ 3
and = = = r2 (say). …(2)
2 −1 2
Any point P on the line (1) is
( r1 + 1, 2r1 + 1, 2r1 − 2) …(3)
and any point Q on the line (2) is
(2r2 − 2, − r2 + 5, 2r2 − 3). …(4)
The d.r.’s of the line PQ are
r1 + 1 − 2r2 + 2, 2r1 + 1 + r2 − 5, 2r1 − 2 − 2r2 + 3
or r1 − 2r2 + 3, 2r1 + r2 − 4, 2r1 − 2r2 + 1. …(5)
G-114

If PQ be S.D. line, then PQ is perpendicular to both the given lines (1) and (2) and so we
have
1( r1 − 2r2 + 3) + 2(2r1 + r2 − 4) + 2(2r1 − 2r2 + 1) = 0
and 2 ( r1 − 2r2 + 3) − (2r1 + r2 − 4) + 2(2r1 − 2r2 + 1) = 0
or 9r1 − 4r2 − 3 = 0
and 4r1 − 9r2 + 12 = 0.
x−2 y − 3 z −1
Now proceeding as in Example 25; the required equations are = = .
7 4 −5
Problem 7: Find the length and equations of the common perpendicular to the two lines
x+3 y −6 z x+2 y z −7
= = ; = = ⋅
−4 3 2 −4 1 1 (Meerut 2010B)
Solution: Let l , m, n be the d.c.’s of the line of the common perpendicular (or S.D.) to
the two given lines. Then we have
− 4 ⋅ l + 3m + 2n = 0 , …(1)
− 4l + m + n = 0. …(2)
l m n
Solving these, we get = =
3 − 2 −8 + 4 −4 + 12

l m n ( l2 + m2 + n2 ) 1
or = = = = .
1 −4 8 [1 + ( −4) + 8 ] 9
2 2 2

1 4 8
∴ The d.c.’s of the S.D. are ,− , …(3)
9 9 9
and the d.r.’s of the S.D. are 1, − 4, 8 .
Also A( −3, 6, 0 ) is a point on the line (1) and B( −2, 0 , 7) is a point on the line (2).
∴ The length of S.D. = projection of join of A and B on the line whose
d.c.’s are given by (3)
1 4 8
= [( −3) − ( −2)] − [(6) − (0 )] + [(0 ) − (7)]
9 9 9
= − (1 /9) − (24 /9) − (56 /9) = 9 (numerically).
Equation of S.D.:
The equations of the plane through the line (2) and S.D. is
x+3 y −6 z
−4 3 2 = 0 or 32 x + 34 y + 13z = 108. …(4)
1 −4 8
And the equation of the plane through the line (2) and S.D. is
x+2 y z −7
−4 1 1 = 0 or 4 x + 11 y + 5z − 27 = 0. …(5)
1 −4 8
∴ From (4) and (5) the equations of S.D. are
32 x + 34 y + 13z = 108, 4 x + 11 y + 5z = 27.
G-115

Substituting the values of r1 and r2 in (3), (4) and (5), we have

P  ,
28 43 4 
,  , Q  , ,
23 41 35 

 13 13 13   13 13 13 
6 2 −5
and the d.r.’s of PQ (the line of S.D.) are , , or 6, 2, –5.
13 13 13
Now the line PQ of shortest distance is a line passing through P  ,
28 43 4 
,  and having
 13 13 13 
d.r.’s 6, 2, − 5
28 43 4
x− y− z−
13 = 13 = 13 .
6 2 −5

Problem 8: Show that the shortest distance between any two opposite edges of the tetrahedron
formed by the planes y + z = 0, z + x = 0, x + y = 0, x + y + z = a is 2 a / √ 6 and that the
three lines of shortest distance intersect at the point x = y = z = − a. ( Garhwal 2002)
Solution: The equations of the edge of the tetrahedron determined by the planes
y + z = 0, z + x = 0 are
x y z
= = ⋅ …(1)
1 1 −1
The equations of the edge opposite to that given by (1) are determined by the planes
x + y = 0, x + y + z = a
and hence are given by
x y z−a
= = ⋅ …(2)
1 −1 0
Let l, m, n be the d.c.’s of the line of S.D. between (1) and (2). This line of S.D. being
perpendicular to both the lines (1) and (2), we have
l . 1 + m . 1 + n . ( −1) = 0
and l . 1 + m . ( −1) + n . 0 = 0.
l m n √ ( l2 + m2 + n2 ) 1
Solving, = = = = ⋅
1 1 2 √ {(1)2 + (1)2 + (2)2 } √ 6
…(3)
Clearly the line (1) passes through the point A (0, 0, 0 ) and the line (2) passes through
B (0, 0, a).
∴ The length of S.D. = The projection of the join of A and B on the line of
S.D. whose d.c.’s l, m, n are given by (3)
= l (0 − 0 ) + m (0 − 0 ) + n ( a − 0 ) = na = 2 a / √ 6.
The equations of S.D.: (See article 17).
The equation of the plane through the line (1) and S.D. is
x y z
1 1 −1 = 0
 
1 1 2
or x (2 + 1) − y (2 + 1) + z (1 − 1) = 0 or x − y = 0. …(4)
G-116

The equation of the plane through the line (2) and S.D. is
x y z − a
1 −1 0 = 0
 
1 1 2 
or x ( −2 − 0 ) − y (2 − 0 ) + ( z − a) (1 + 1) = 0
or x + y − z + a = 0. …(5)
The equations (4) and (5) together are the equations of the S.D. These equations are
clearly satisfied by the point
x = y = z = − a.
By the symmetry of the co-ordinates of the point
x = y = z = − a,
it follows that the other lines of S.D. between other two pairs of opposite edges will pass
through the point
x = y = z = − a.
Hence the three lines of shortest distance intersect at the point
x = y = z = − a.

Problem 9: Show that the shortest distance between the lines


x + a = 2 y = − 12z and x = y + 2a = 6z − 6a is 2a.
(Meerut 2011; Avadh 11; Rohilkhand 11)
Solution: The equations of the given lines are
x+ a y z
= = , …(1)
12 6 −1
x y+2a z −a
and = = ⋅ …(2)
6 6 1
Let l, m, n be the d.c.’s of the line of S.D. The line of S.D. being perpendicular to both the
lines (1) and (2), we have
12l + 6m − n = 0 and 6l + 6m + n = 0.
Solving these relations, we have
l m n
= =
6 + 6 −6 − 12 72 − 36

l m n √ ( l2 + m2 + n2 ) 1
or = = = = ⋅
2 −3 6 √ {(2)2 + ( −3)2 + (6)2 } 7
Clearly the line (1) passes through the point A ( − a, 0, 0 ) and the line (2) passes through
B (0, − 2 a, a).
∴ The length of S.D. = The projection of join of A and B on the line of S.D.
whose d.c.’s are l, m, n
= l (0 + a) + m ( −2 a − 0 ) + n ( a − 0 )
= (2 / 7) a − (3 / 7) ( −2 a) + (6 / 7) a = 2 a.
G-117

Problem 10: Find the length of the shortest distance between the z-axis and the line
x + y + 2 z − 3 = 0 = 2 x + 3 y + 4 z − 4. (Kumaun 2008)
Solution: Here we shall use third method of article 17.
The equations of z-axis are x / 0 = y / 0 = z /1. …(1)
The equations of the other line are
x + y + 2 z − 3 = 0, 2 x + 3 y + 4z − 4 = 0. …(2)
The equation of any plane through the line (2) is
( x + y + 2 z − 3) + λ (2 x + 3 y + 4z − 4) = 0
or x (1 + 2 λ ) + y (1 + 3 λ ) + z (2 + 4 λ ) − 3 − 4 λ = 0. …(3)
If the plane (3) is parallel to the line (1) [i. e., z-axis] then the normal to the plane (3) will
be perpendicular to the line (1) and hence we have
0 . (1 + 2 λ ) + 0 . (1 + 3λ ) + 1 . (2 + 4 λ ) = 0 or λ = −1/2⋅
Putting this value of λ in (3), the equation of the plane through the line (2) and parallel
to z-axis [ i. e., (1)] is given by
x . 0 + y . (1 − 3 / 2) + z . 0 − 3 + 2 = 0
or y + 2 = 0. …(4)
Clearly z-axis i. e., the line (1) passes through the point (0, 0, 0 ).
∴ Length of S.D. = the length of perpendicular from the point (0, 0, 0 ) to the plane (4)
0+2 2
= = =2.
√ {(0 )2 + (0 )2 + (1)2 } 1
Problem 11: Find the shortest distance between the z-axis and the line
ax + by + c z + d = 0 = a ′ x + b ′ y + c ′ z + d ′. (Kumaun 2008)
Show also that it meets the z-axis at a point whose distance from the origin is
( ab ′ − d ′ b ) ( bc ′ − b ′ c ) + ( c a ′ − c ′ a) ( ad ′ − a ′ d )

( bc ′ − b ′ c )2 + ( c a ′ − c ′ a)2
Solution: The equations of the z-axis are
x y z
= = = r1 (say). …(1)
0 0 1
The equations of the other line are
ax + by + c z + d = 0 = a ′ x + b ′ y + c ′ z + d ′. …(2)
The length of S.D.: We shall find it by Method III of article 17.
The equation of any plane through the line (2) is
( ax + by + c z + d ) + λ ( a ′ x + b ′ y + c ′ z + d ′ ) = 0
or x ( a + λa ′ ) + y ( b + λb ′ ) + z ( c + λc ′ ) + ( d + λd ′ ) = 0. …(3)
If the plane (3) is parallel to the line (1) [i. e., z-axis], then
0 . ( a + λa′ ) + 0 . ( b + λb ′ ) + 1 . ( c + λc ′ ) = 0 or λ = − c / c ′.
Putting this value of λ in (3), the equation of the plane through the line (2) and parallel
to z-axis is given by
x ( a − ca ′ / c ′ ) + y ( b − cb ′ / c ′ ) + z ( c − c ) + ( d − cd ′ / c ′ ) = 0
or x ( ac ′ − ca ′ ) + y ( bc ′ − cb ′ ) + ( dc ′ − cd ′ ) = 0. …(4)
G-118

Clearly z-axis i. e., the line (1) passes through the point (0, 0, 0 ).
∴ Length of S.D. = the length of perpendicular from the point (0, 0, 0 )to the plane (4)
0 + 0 + ( dc ′ − cd ′ )
=
√ {( ac ′ − ca ′ )2 + ( bc ′ − b ′ c )2 }
dc ′ − d ′ c
= ⋅
√ {( ac ′ − a ′ c )2 + ( bc ′ − b ′ c )2 }
The distance from origin of the point where the line of S.D. meets the z-axis:
It will be convenient to use method II of article 17.
The equations of line (2) in symmetrical form are
bd ′ − b ′ d a′ d − ad ′
x− y−
ab ′ − a′ b ab ′ − a′ b z −0
= = ⋅ …(5)
bc ′ − b ′ c ca′ − c ′ a ab ′ − a′ b
[See article 4]
For convenience, let (5) be written as
x − x2 y − y2 z − 0
= = = r2 (say). …(6)
l2 m2 n2
Since 0, 0, 1 are the actual d.c.’s of the z-axis [i. e., the line (1)], therefore r1 is the actual
distance of any point on the line (1) from (0, 0, 0 ). Let P (0, 0, r1)be any point on the line
(1).
Also any point on the line (6) is
( l2 r2 + x2 , m2 r2 + y2 , n2 r2 ),
say the point Q.
The d.r.’s of PQ are l2 r2 + x2 , m2 r2 + y2 , n2 r2 − r1.
Let the line PQ be the line of S.D., so that PQ is perpendicular to both the lines (1) and
(2), and therefore, we have
0.( l2 r2 + x2 ) + 0.( m2 r2 + y2 ) + 1. ( n2 r2 − r1) = 0 or r1 = n2 r2
and l2 ( l2 r2 + x2 ) + m2 ( m2 r2 + y2 ) + n2 ( n2 r2 − r1 ) = 0
or l22 r2 + l2 x2 + m22 r2 + m2 y2 + 0 = 0 [ ∵ r1 = n2 r2 ]

or r2 ( l22 + m22 ) = − ( l2 x2 + m2 y2 )

or r2 = − ( l2 x2 + m2 y2 ) / ( l22 + m22 ).

∴ r1 = n2 r2 = − n2 ( l2 x2 + m2 y2 ) / ( l22 + m22 ).
Substituting the corresponding values of l2 , m2 , n2 , x2 and y2 from (5) and (6), we
have
− ( ab ′ − a ′ b )  ( bd ′ − b ′ d )
r1 =  ( bc ′ − b ′ c ) ⋅
2
( bc ′ − b ′ c ) + ( ca ′ − c ′ a) 2
 ab ′ − a′ b

a′ d − ad ′ 
+ ( ca′ − c ′ a) ⋅ 
( ab ′ − a′ b ) 
( db ′ − d ′ b ) ( bc ′ − b ′ c ) + ( ca′ − c ′ a) ( ad ′ − a′ d )
= ⋅
( bc ′ − b ′ c )2 + ( ca′ − c ′ a)2
G-119

Problem 12: Show that the equation of the plane containing the line y / b + z / c = 1, x = 0
and parallel to the line x / a − z / c = 1, y = 0 is x / a − y / b − z / c + 1 = 0 and if 2d is the
shortest distance, then show that d −2 = a−2 + b −2 + c −2 .
(Meerut 2008; Rohilkhand 06; Avadh 09)
Solution: The equations of the given lines are
y / b + z / c = 1, x = 0 …(1)
x z x−a y z
and − = 1, y = 0 or = = , …(2)
a c a 0 c
the equations of the second line being put in symmetrical form.
The equation of any plane through the line (1) is
( y / b + z / c − 1) + λx = 0 or λx + (1 / b ) y + (1 / c ) z − 1 = 0. …(3)
If the plane (3) is parallel to the line (2), then the normal to the plane (3) whose d.r.’s are
λ, 1 / b, 1 / c will be perpendicular to the line (2), and so we have
a . λ + 0 . (1 / b ) + c . (1 / c ) = 0 or λ = − (1 / a).
Putting this value of λ in (3), the equation of the plane containing the line (1) and
parallel to the line (2) is
x y z x y z
− + + − 1 = 0 or − − + 1 = 0. …(4)
a b c a b c
Clearly ( a, 0, 0 ) is a point on the line (2).
Hence the length 2d of the S.D. = the perpendicular distance of ( a, 0, 0 ) from the plane
(4).
a (1 / a) − 0 − 0 + 1 2
∴ 2d = =
√ {(1 / a) + ( −1 / b ) + ( −1 / c ) } √ ( a + b −2 + c −2 )
2 2 2 −2

1
or d2 =
( a −2 + b −2 + c −2 )
or d −2 = a −2 + b −2 + c −2 .
Problem 13: Show that the shortest distance between the diagonals of a rectangular
parallelopiped and the edges not meeting it are
bc ca ab
, ,
√ ( b + c ) √ ( c + a ) √ ( a + b2 )
2 2 2 2 2

where a, b, c are the lengths of the edges.


Solution: Consider a rectangular
parallelopiped whose three coterminous
edges OA, OB, OC are taken along the
axes of x, y and z respectively. Also
OA = a, OB = b and OC = c . The
co-ordinates of the different vertices are as
follows :
O (0, 0, 0 ), A ( a, 0, 0 ), B (0, b, 0 ),
C (0, 0, c ), D (0, b, c ), E ( a, 0, c ),
F ( a, b, 0 ), P ( a, b, c ).
G-120

Consider a diagonal AD and the edge OB not meeting this diagonal. Now we shall find
the length of S.D. between AD and OB.
The equations of AD are
x−a y −0 z −0 x−a y z
= = or = = ⋅
0 − a b−0 c −0 −a b c …(1)
The equations of OB are
x y−b z
= = ⋅ …(2)
0 1 0
Let l, m, n be the d.c.’s of the line of S.D. between the lines (1) and (2). Since the line of
S.D. is perpendicular to both the lines (1) and (2), we have
l ( − a) + m . b + n . c = 0 and l . 0 + m . 1 + n . 0 = 0.
2 2 2
l m n √ (l + m + n ) 1
Solving, = = = = ⋅ …(3)
c 0 a √ ( c 2 + 0 + a2 ) √ ( c 2 + a2 )
∴ The length of S.D. between AD and OB
= the projection of the join of ( a, 0, 0 ) {(a point on (1)}
and (0, b, 0 ) {a point on (2)} on the line of S.D. whose d.c.’s l, m, n are given by (3)
= l ( a − 0 ) + m (0 − b ) + n (0 − 0 ) = la
= ac / √ ( c 2 + a2 ). …(4)
Similarly the shortest distance between other pairs of lines can be found.

Comprehensive Problems 11
Problem 1: Find the surface generated by the lines which intersect the lines
y = mx, z = c ; y = − mx, z = − c and x-axis.
Solution: As in Example 28 a line intersecting the first two lines namely
y = mx, z = c ; y = − mx, z = − c
is given by the planes
( y − mx ) + λ ( z − c ) = 0, …(1)
and ( y + mx ) + µ ( z + c ) = 0. …(2)
The planes (1) and (2) intersect in a line and if this line meets the x-axis i. e., y = 0, z = 0,
then putting y = 0 = z in (1) and (2), we have
(0 − mx ) + λ (0 − c ) = 0
and (0 + mx ) + µ (0 + c ) = 0
or λ = − mx / c and µ = − mx / c .
From these two relations, we get λ = µ . …(3)
The required locus is given by eliminating λ and µ between (1), (2) and (3) and is given
by
y − mx y + mx
− =−
z−c z+ c
or ( y + mx ) ( z − c ) = ( y − mx ) ( z + c ) or cy = mz x.
G-121

Problem 2: Show that the straight lines which intersect the three lines y − z = 1, x = 0 ;
z − x = 1, y = 0 and x − y = 1, z = 0 lie on the surface whose equation is
x2 + y2 + z 2 − 2 yz − 2zx − 2 xy − 1 = 0.
Solution: The equations of the given lines are
y − z − 1 = 0, x = 0 …(1)
z − x − 1 = 0, y = 0 …(2)
and x − y − 1 = 0, z = 0. …(3)
The equations of any planes through the lines (1) and (2) are
( y − z − 1) + λx = 0 …(4)
and ( z − x − 1) + µy = 0. …(5)
The planes (4) and (5) intersect in a line and if this line is to meet the line (3), then we
shall eliminate x, y, z between the equations (3), (4) and (5). From (3), we have
z = 0, x = y + 1.
Putting these values in the equations (4) and (5), we have
( y − 0 − 1) + λ ( y + 1) = 0 and (0 − y − 1 − 1) + µy = 0
or y − 1 − λy + λ = 0 and − y − 2 + µy = 0
or y (1 + λ ) = 1 − λ and y ( −1 + µ ) = 2
1− λ 2
or y= and y= ⋅
1+ λ µ −1
Equating the two values of y, we get
1− λ 2
= or µ − 1 − µλ + λ = 2 + 2λ
1+ λ µ −1
or µλ + λ − µ + 3 = 0. …(6)
The required locus is obtained by eliminating λ and µ between (4), (5) and (6); and
hence is given by
z − x −1 y − z −1 y − z −1 z − x −1
− ×− − + + 3=0
y x x y
or ( z − x − 1) ( y − z − 1) − y ( y − z − 1) + x ( z − x − 1) + 3 xy = 0
or yz − z 2 − z − xy + xz + x − y + z + 1 − y2 + yz + y
+ xz − x2 − x + 3 xy = 0
or x2 + y2 + z 2 − 2 yz − 2zx − 2 xy − 1 = 0.
Problem 3: Prove that the locus of a line which meets two lines y = ± mx, z = ± c and the circle
x2 + y2 = a2 , z = 0 is c2 m2 ( cy − mzx )2 + c2 ( yz − cmx )2 = a2 m2 ( z 2 − c2 )2 .
Solution: The equations of the given lines are
y − mx = 0, z − c = 0 …(1)
and y + mx = 0, z + c = 0. …(2)
Also the equations of the given circle are
x2 + y2 = a2, z = 0. …(3)
G-122

The equations of any planes through the lines (1) and (2) are
( y − mx ) + λ ( z − c ) = 0 …(4)
and ( y + mx ) + µ ( z + c ) = 0. …(5)
The planes (4) and (5) intersect in a line and if this line meets the circle (3), then we are
to eliminate x, y, z between the equations (3), (4) and (5).
From (3), we have z = 0. Putting z = 0 in (4) and (5), we get
( y − mx ) − c λ = 0, ( y + mx ) + µc = 0.
Solving for x and y, we have
c (λ + µ ) c (λ − µ )
x=− , y= ⋅
2m 2
Putting these values of x and y in x2 + y2 = a2 , the x, y, z eliminate of (3), (4) and (5) is
given by
c 2 ( λ + µ )2 c 2 ( λ − µ )2
+ = a2
2 4
4m
2
or c ( λ + µ )2 + c 2 m2 ( λ − µ )2 = 4a2 m2 . …(6)
The required locus is obtained by eliminating λ and µ between (4), (5) and (6) and hence
is given by
2 2
 y − mx y + mx  2 2  y − mx y + mx 
c 2 − −  + c m − + 2 2
 = 4a m
 z−c z+ c   z−c z+ c 
or c 2 {( y − mx ) ( z + c ) + ( z − c ) ( y + mx )}2
+ c 2 m2 {( y + mx ) ( z − c ) − ( y − mx ) ( z + c )}2 = 4a2 m2 ( z 2 − c 2 )2
or c 2 ( yz − cmx )2 + c 2 m2 ( cy − mzx )2 = a2 m2 ( z 2 − c 2 )2 .
Problem 4: Find the surface generated by a straight line which meets two lines
y = mx, z = c ; y = − mx, z = − c at the same angle.
Solution: The equations of the given lines may be written as
x /1 = y / m = ( z − c ) / 0 = r1 (say) …(1)
and x /1 = y /( − m) = ( z + c ) / 0 = r2 (say). …(2)
Let the moving line meet the given lines (1) and (2) in the points C ( r1, mr1, c ) and
D ( r2 , − mr2 , − c ) respectively. The d.r.’s of CD are r1 − r2 , m ( r1 + r2 ), c + c. The
equations to CD are
x − r1 y − mr1 z−c
= = ⋅ …(3)
r1 − r2 m ( r1 + r2 ) 2c
Let the line (3) make angles α and β with the lines (1) and (2) respectively, so that we
have
1 . ( r1 − r2 ) + m . m ( r1 + r2 ) + 0 . 2 c
cos α = …(4)
√ (1 + m2 + 0 ) √ {( r1 − r2 )2 + m2 ( r1 + r2 )2 + (2 c )2 }
1 . ( r1 − r2 ) − m . m ( r1 + r2 ) + 0 . 2 c
and cos β = ⋅ …(5)
√ (1 + m + 0 ) √ {( r1 − r2 )2 + m2 ( r1 + r2 )2 + (2 c )2 }
2
G-123

But according to the given problem, cos α = ± cos β. Hence from (4) and (5), we get
r1 − r2 + m2 ( r1 + r2 ) = ± [( r1 − r2 ) − m2 ( r1 + r2 )],
the denominators being same have been cancelled.
Taking +ve sign, we get
2m2 ( r1 + r2 ) = 0 or r2 = − r1
and taking −ve sign, we get
2 ( r1 − r2 ) = 0 or r1 = r2 .
Now when r2 = − r1, the line (3) becomes
x − r1 y − mr1 z − c
= =
2 r1 0 2c
x − r1 y−c
i. e., = and y − mr1 = 0
2r1 2c
x z y
i. e., −1= −1 and r1 =
r1 c m
x z y
i. e., = and r1 = ⋅
r1 c m
Eliminating r1 , we get
xm z
= or mc x = yz. …(6)
y c
Again, when r2 = r1 , the line (3) becomes
x − r1 y − mr1 z − c
= =
0 2 mr1 2c
i. e., x − r1 = 0 and y / mr1 = z / c ; i. e., r1 = x and c y = mz r1.
Eliminating r1, we get c y = mz x. …(7)
Multiplying (6) and (7) the required surface is given by
( mcx − yz ) ( cy − mzx ) = 0.
Problem 5: Show that the locus of lines which meet the lines
x+ a y z x−a y z
= = ; = =
0 sin α − cos α 0 sin α cos α
at the same angle is ( xy cos α − az sin α ) ( z x sin α − ay cos α ) = 0.
Solution: The equations of the given lines are
x+ a y z
= = = r1 …(1)
0 sin α − cos α
x−a y z
and = = = r2 . …(2)
0 sin α cos α
Let the moving line meet the given lines (1) and (2) in the points
C ( − a, r1 sin α, − r1 cos α )
and D ( a, r2 sin α , r2 cos α )
respectively. The d.r.’s of the line CD are a + a, ( r2 − r1) sin α, ( r2 + r1 ) cos α .
G-124

The equations to CD are


x+ a y − r1 sin α z + r1 cos α
= = ⋅ …(3)
2a ( r2 − r1 ) sin α ( r2 + r1 ) cos α
Let the line (3) make angles θ and φ with the lines (1) and (2) respectively, so that we
have
2 a . 0 + ( r2 − r1 ) sin α . sin α + ( r2 + r1 ) cos α . ( − cos α )
cos θ =
√ {(2 a)2 + ( r2 − r1 )2 sin2 α + ( r2 + r1 )2 cos2 α} √ {0 + sin2 α + cos2 α}
…(4)

2 a . 0 + ( r2 − r1) sin α . sin α + ( r2 + r1) cos α . cos α


cos φ = ⋅
2
√ {(2 a) + ( r2 − r1)2 sin2 α + ( r2 + r1)2 cos2 α} √ {0 + sin2 α + cos2 α}
…(5)

But according to the given problem, cos θ = ± cos φ. Hence from (4) and (5), we get
( r2 − r1) sin2 α − ( r2 − r1) cos2 α = ± {( r2 − r1 ) sin2 α + ( r2 + r1 ) cos2 α}.
Taking +ve sign, we get
2 ( r2 + r1 ) cos2 α = 0 or r2 + r1 = 0 or r2 = − r1
and taking −ve sign, we get
2 ( r2 − r1 ) sin2 α = 0 or r2 = r1.

Now when r2 = − r1 , the line (3) becomes


x+ a y − r1 sin α z + r1 cos α
= =
2a −2 r1 sin α 0
x y
i. e., +1= − + 1 and z + r1 cos α = 0
a r1 sin α
i. e., r1 x sin α = − ay and r1 = − z / cos α.
Eliminating r1 , we get
z x sin α − ay cos α = 0. …(6)
Again when r2 = r1 , the line (3) becomes
x+ a y − r1 sin α z + r1 cos α
= =
2a 0 2 r1 cos α
x z
or +1= + 1 . y = r1 sin α
a r1 cos α
or r1 x cos α = az, r1 = y /sin α.
Eliminating r1 , we get
xy cos α − az sin α = 0. …(7)
Multiplying (6) and (7), the required locus is given by
( xy cos α − az sin α ) ( z x sin α − ay cos α ) = 0.
G-125

Problem 6: P and P ′ are two variable points on two given non-intersecting lines and PP ′ is of
constant length 2 λ. Find the surface generated by PP ′.
Solution: Let the equations of the given lines be
x y z−c
+ = = r1 …(1)
1 m 0
x y z+ c
and + = = r2. …(2)
1 −m 0
The co-ordinates of the variable points P and P ′ on (1) and (2) are
( r1 , mr1 , c ) and ( r2 , − mr2 , − c ) respectively.
It is given that PP ′ = constant, say 2λ .
∴ ( PP ′ )2 = 4λ2
or ( r1 − r2 )2 + m2 ( r1 + r2 )2 + 4c2 = 4λ2 . …(3)
Also d.r.’s of PP ′ are r1 − r2 , m( r1 + r2 ), 2c.
∴ The equations to PP ′ are
x − r1 y − mr1 z−c
= = . …(4)
r1 − r2 m( r1 + r2 ) 2c
The locus of PP ′ is obtained by eliminating r1 and r2 between (3) and (4).
From (4), we have
x − r1 z−c y − mr1 z−c
= and =
r1 − r2 2c m( r1 + r2 ) 2c
or 2 c x − 2 c r1 = r1 ( z − c ) − r2 ( z − c );
and 2 c y − 2 mc r1 = mr1 ( z − c ) + mr2 ( z − c )
or r1 ( z + c ) − r2 ( z − c ) = 2 c x …(5)
and mr1 ( z + c ) + mr2 ( z − c ) = 2 c y. …(6)
Solving (5) and (6) for r1 and r2 , we get
r1 = c ( y + mx )/ {m( z + c )}, r2 = c ( y − mx )/ {m( z − c )}. …(7)
c  y + mx y − mx  2c ( yz − cmx )
∴ r1 + r2 =  + =
m z + c z−c  m( x2 − c2 )
c  y + mx y − mx  2c ( mzx − cy )
and r1 − r2 =  − = .
m z+ c z−c  m( z 2 − c2 )
Substituting the values of r1 + r2 and r1 − r2 in (3), the required locus is given by
4c2 ( mzx − cy )2 4c2 ( yz − mcx )2
+ m2 ⋅ + 4c2 = 4λ2
2 2 2 2 2 2 2 2
m (z − c ) m (z − c )
or c2( mzx − cy )2 + c2 m2 ( yz − mcx )2 = m2 ( λ2 − c2 ) ( z 2 − c2 )2.
Problem 7: Find the equation to the surface generated by a straight line which is parallel to the
x2 y2
line y = mx, z = nx and intersects the ellipse + = 1, z = 0.
2
a b2
G-126

Solution: Line y = mx , z = nx can be written as


x y z
= = .
1 m n
x−α y −β z − γ
Any line parallel to it is = = . …(1)
1 m n
x−α y − β −γ
It meets z = 0 where = = ,
1 m n
i.e., x = α − γ / n, y = β − mγ / n, z = 0.
So line (1) will cut x2 / a2 + y2 / b2 = 1, z = 0 ,
2 2
1  γ 1  my 
if α −  + 2 β −  = 1,
a2  n b  n 
1 1
i.e., (α n − γ )2 + (βn − mγ )2 = n2 .
2 2
a b
Since (α , β, γ ) is an arbitrary point on the moving line, hence generalizing (α, β, γ ) the
required locus is
1 1
( xn − z )2 + ( yn − mz )2 = n2.
a2 b2

Problem 8: How many lines can be drawn from a point to intersect two non-coplanar lines
neither of which passes through the point? Find the equations of the lines or line which can be
drawn from the point (2 , − 1, 3) to intersect the lines
x −1 y −2 z −3 x−4 y z+3
= = ; = = ⋅
2 3 4 4 5 3 (Bundelkhand 2005)
Solution: Let (α, β, γ ) be the co-ordinates of a given point. Let the equations of the two
non-coplanar lines be
y − mx = 0, z − c = 0 …(1)
and y + mx = 0, z + c = 0. …(2)
The equations of any planes through the lines (1) and (2) are
( y − mx ) − λ ( z − c ) = 0 …(3)
and ( y + mx ) − µ ( z + c ) = 0. …(4)
Since the planes (3) and (4) pass through the point (α , β , γ ), so we have
( β − mα ) − λ ( γ − c ) = 0 and ( β + mα ) − µ ( γ + c ) = 0
or λ = ( β − mα ) /( γ − c ) and µ = ( β + mα ) / ( γ + c ).
Putting the values of λ and µ in (3) and (4), the equations of the line through the given
point (α , β , γ ) and intersecting the given lines (1) and (2) are given by
( y − mx ) ( γ − c ) − ( z − c ) ( β − mα ) = 0, 
⋅
and ( y + mx ) ( γ + c ) − ( z + c ) ( β + mα) = 0. 
Thus only one line can be drawn from a point to intersect two non-coplanar lines neither
of which passes through the point.
G-127

Second part: The equations of the given lines are


x −1 y −2 z −3 x − 4 y z+3
= = ; = = ⋅
2 3 4 4 5 3
These lines can be reduced to general form as
3x − 2 y + 1 = 0 = 2 x − z + 1 …(1)
and 5 x − 4 y − 20 = 0 = 3 x − 4z − 24. …(2)
The equations of any planes through the lines (1) and (2) are
(3 x − 2 y + 1) + λ (2 x − z + 1) = 0 …(3)
and (5 x − 4 y − 20 ) + µ (3 x − 4z − 24) = 0. …(4)
The planes (3) and (4) pass through the point (2 , − 1, 3), so we have
(6 + 2 + 1) + λ (4 − 3 + 1) = 0 or λ = − 9 / 2 ;
(10 + 4 − 20 ) + µ (6 − 12 − 24) = 0 or µ = − 1 / 5.
Putting the values of λ and µ in (3) and (4), the equations of the line through the point
(2 , − 1, 3) and intersecting the given lines (1) and (2) are given by
12 x + 4 y − 9z + 7 = 0, 11x − 10 y + 2 z − 28 = 0.

H ints to O bjective T ype Q uestions

Multiple Choice Questions


1. The direction cosines of z-axis are 0, 0, 1. The direction cosines of any straight line
perpendicular to z-axis are cos α, sin α, 0 because
cos α . 0 + sin α . 0 + 0 . 1 = 0 and cos2 α + sin2 α + 02 = 1.
2. The d.c.’s of y-axis are 0, 1, 0 and these are also the d.c.’s of any line parallel to
y-axis.
The equations of the straight line passing through the point (2 , 5, 7) and parallel
x−2 y −5 z −7
to y-axis are = = ⋅
0 1 0
3. The coordinates of the foot of the perpendicular from the point ( x1, y1, z1) to
z-axis are (0, 0, z1 ). The distance between the points ( x1, y1, z1) and (0, 0, z1 ) is

( x1 − 0 )2 + ( y1 − 0 )2 + ( z1 − z1 )2 i. e., x12 + y12 .


4. The coordinates of the foot of the perpendicular drawn from the point ( −5, 3, 2)
to the yz-plane are (0, 3, 2). The image of the point ( −5, 3, 2) in the yz-plane is the
point (α, β, γ ) such that the middle point of (α, β, γ ) and ( −5, 3, 2) is the point
(0, 3, 2).
α −5 β+3 γ+2
∴ = 0, = 3, = 2 ⇒ α = 5, β = 3, γ = 2 .
2 2 2
5. See Problem 9 of Comprehensive Problems 10.
G-128

6. Let l, m, n be the d.c.’s of the line of intersection of the planes


3 x + 4 y + 7z + 4 = 0 and x − y + 2 z + 3 = 0. Then
3 l + 4 m+ 7 n=0
l − m + 2 n = 0.
Solving these equations by the method of cross-multiplication, we have
l m n l m n
= = or = = ⋅
8 + 7 7 − 6 −3 − 4 15 1 −7
∴ The direction ratios of the line of intersection of the two given planes are
15, 1, − 7.
∴ The equation of the plane through (2 , 1, 4) perpendicular to the line whose
d.r.’s are 15, 1, − 7 is
15 ( x − 2) + 1 ( y − 1) − 7 ( z − 4) = 0 or 15 x + y − 7z − 3 = 0.
7. The equations of the given line in general form are
x + 2 y + 3z + 4 = 0, 2 x + 3 y + 4z + 5 = 0. …(1)
Let l, m, n be the d.c.’s of the line (1). We have
l + 2m + 3n = 0 and 2l + 3m + 4n = 0.
Solving these, we have
l m n l m n
= = or = = ⋅
8−9 6−4 3−4 −1 2 −1
Therefore, the d.r.'s of the line (1) are −1, 2, − 1.
Now to find the co-ordinates of the point on the line given by (1), let us find the
point where (1) meets the plane z = 0. Putting z = 0 in the equations (1), we have
x + 2 y + 4 = 0 and 2 x + 3 y + 5 = 0.
x y 1 x y 1
Solving these = = or = = ,
10 − 12 8 − 5 3 − 4 −2 3 −1
giving x = 2 , y = −3 .
Therefore, the line meets the plane z = 0 in the point (2, − 3, 0 ) and has direction
ratios as −1, 2, − 1. Thus the equations of the given line in symmetrical form are
x −2 y + 3 z −0
= = ⋅
−1 2 −1
8. See article 2.
9. See article 2. The equations of the given line are
x−2 y − ( −3) z − 5 x−2 y + 3 z −5
= = i. e., = = ⋅
2 −1 3 2 −1 3
10. Direction ratios of the given line are 2 , 1, − 2 .
We have 22 + 12 + ( −2)2 = 3.
2 1 2
Hence, the direction cosines of the given line are , ,− ⋅
3 3 3
x−3 y −4 z −5
11. Let = = = r (say).
1 2 −2
Then x = r + 3, y = 2r + 4, z = − 2r + 5.
Hence, the coordinates of any point on the given line are
( r + 3, 2r + 4, − 2r + 5), where r is a parameter.
G-129

x−2 y + 3 z +1
12. The coordinates of any point on the line = = = r (say) are
6 2 3
(6r + 2 , 2r − 3, 3 r − 1). …(1)
If the distance of this point from the point (2 , − 3, − 1) is 14, then
[(6 r + 2) − 2]2 + [(2r − 3) − ( −3)]2 + [(3r − 1) − ( −1)]2 = 142
or 36r 2 + 4r 2 + 9r 2 = 196 or 49r 2 = 196
or r 2 = 4 or r = ± 2 .
Putting r = 2 and −2 successively in (1), the coordinates of the required points are
(14, 1, 5) and ( −10, − 7, − 7).
13. We know that the direction ratios of two parallel lines are proportional to each
other.
We have 8 = 4 × 2 .
Hence, the equations of the straight line parallel to
x −1 y −2 z x−2 y −2 z
= = are = = ⋅
2 3 4 4 6 8
14. The direction ratios of the given line are −1, 3, 5. The direction ratios of any line
parallel to this line are also −1, 3, 5.
Hence, the equations of the straight line passing through the point (2 , − 5, − 3)
and parallel to the given line are
x−2 y+5 z+3
= = ⋅
−1 3 5
15. The straight line y = 0, z = 0 is the axis of x. Hence, its direction cosines are1, 0, 0.
16. The equations of the straight line x − y = 0, z = 1 can be written as
x = y, z − 1 = 0.
In symmetrical form these equations can be written as
x−0 y − 0 z −1
= = ⋅
1 1 0
17. The equation of the xy-plane is z = 0.
So, the straight line 3 x + 2 y − z − 4 = 0, 4 x + y − 2 z + 3 = 0 meets the
xy-plane at the point whose x and y coordinates are given by the simultaneous
equations
4 x + y + 3 = 0, 3 x + 2 y − 4 = 0.
Solving these equations, we get x = − 2 , y = 5.
Hence, the required point is ( −2 , 5, 0 ).
18. x−3 y+4 z
The direction ratios of the straight line = = are1, − 1, 0 and its d.c.’s
1 −1 0
1 1
are ,− , 0.
2 2
The direction ratios of the normal to the plane 0 x + y − z + 2 = 0 are 0, 1, − 1and
1 1
its d.c.’s are 0, ,− ⋅
2 2
G-130

If θ is the acute angle between the given straight line and the normal to the given
plane, then
⋅ 0 +  −
1 1   1   1  = 1⋅
cos θ = ⋅  + 0 ⋅ − 
2  2  2  2 2
π
∴ θ= ⋅
3
Now the angle between a straight line and a plane is defined as the complement of
the angle between the line and the normal to the plane.
π π π
Hence, the angle between the given line and the given plane = − = ⋅
2 3 6
19. The direction ratios of a line perpendicular to the plane x + y − z + 4 = 0 are
1, 1, − 1.
∴ Equations of the straight line drawn from the point (2 , 3, 4) perpendicular
to the plane
x−2 y −3 z −4
x + y − z + 4 = 0 are = = = r (say).
1 1 −1
The coordinates of any point on this line are
( r + 2 , r + 3, − r + 4). …(1)
If this point lies on the plane x + y − z + 4 = 0, then
5
r + 2 + r + 3 − ( − r + 4) + 4 = 0 ⇒ 3 r = − 5 ⇒ r = − ⋅
3
5
Putting r = − in (1), the required coordinates of the foot of the perpendicular
3
are
 − 5 + 2 , − 5 + 3, 5 + 4 i. e.,  1 , 4 , 17  ⋅
   
 3 3 3  3 3 3 
20. The shortest distance is at the point of intersection and is zero.
21. If the shortest distance between two straight lines is zero, then the lines are
intersecting lines and hence they are coplanar.
22. See article 15, definition of shortest distance between two skew lines.
23. x+1 y −2 z + 4
The straight line = = whose d.r.’s are 1, 3, − 5 is parallel to the
1 3 −5
plane 6 x + 8 y + 6 z = 7 if it is perpendicular to the normal to this plane whose
d.r.’s are 6, 8, 6 and the point ( −1, 2 , − 4) on this line does not lie on the given
plane.
We have 1 . 6 + 3 . 8 + ( −5) . 6 = 0 and 6 . ( −1) + 8 . 2 + 6 . ( −4) = − 14 ≠ 7.
∴ The given straight line is parallel to the given plane.
 2 − ( −1) 4 − ( −3) 6 − ( −5)   3 7 11
24. We have  3 5 7  = 3 5 7
   
 1 4 7   1 4 7
= 3 (35 − 28) − 7 (21 − 7) + 11 (12 − 5) = 21 − 98 + 77 = 0.
∴ The given lines are coplanar.
G-131

25. We know that two parallel straight lines always lie in a plane. Thus two parallel
straight lines are always coplanar.
26. If two straight lines intersect, then they lie in the same plane and so they are
coplanar.

Fill in the Blank(s)


1. The intersection of two planes which are not parallel is always a straight line.
2. See article 2.
3. See article 3.
4. The direction ratios of normal to the plane 3 x + 2 y − 5z = 0 are 3, 2 , − 5 and the
direction ratios of normal to the plane 4 x + 2 y − 3 z + 3 = 0 are 4, 2 , − 3.
The line of intersection of these two planes is perpendicular to the normal of each
of these two planes.
Hence, the direction cosines l, m, n of the straight line
3 x + 2 y − 5z = 0 = 4 x + 2 y − 3z + 3
satisfy the equations 3 l + 2 m − 5 n = 0, 4 l + 2 m − 3 n = 0.
5. The direction ratios of a line perpendicular to the plane
2 x − 3 y + 4z = 7 are 2 , − 3, 4.
Hence, the equations of the straight line passing through the point (2 , 3, − 7) and
perpendicular to the plane 2 x − 3 y + 4 z = 7 are
x−2 y −3 z + 7
= = ⋅
2 −3 4
6. x y z
The line = = whose direction ratios are l, m, n is perpendicular to the plane
l m n
ax + by + cz + d = 0 if it is parallel to the normal to this plane whose direction
ratios are a, b, c. The direction ratios of two parallel lines are proportional to each
other.
x y z
Hence, the line = = is parallel to the plane ax + by + cz + d = 0 if
l m n
a b c
= = ⋅
l m n
7. x − x1 y − y1 z − z1
If the line = = is to be parallel to the plane
l m n
ax + by + cz + d = 0 but should not lie in it, then this line must be perpendicular
to the normal to the plane ax + by + cz + d = 0 whose direction ratios are a, b, c
and the point ( x1, y1, z1 ) on this line should not lie on this plane. Hence, the
required conditions are
al + bm + cn = 0, ax1 + by1 + cz1 + d ≠ 0.
8. See Corollary to article 6.
9. See article 9.
10. See article 17.
G-132

True or False
1. x−2 y + 9 z −6
The straight line = = whose d.r.’s are 3, 5, 1 is perpendicular to
3 5 1
the plane 6 x + 10 y − 2 z = 9 if it is parallel to the normal to this plane whose
d.r.’s are 6, 10, − 2 .
Now the two lines whose d.r.’s are 3, 5, 1 and 6, 10, − 2 are not parallel because
these two d.r.’sare not proportional to each other.
3 5 1
Obviously, = ≠ ⋅
6 10 −2
x−2 y −1 z + 1
2. The straight line = = lies in the plane 2 x + 3 y − 7 z = 5 if it
5 6 4
is perpendicular to the normal to this plane whose d.r.’s are 2 , 3, − 7 and if the
point (2 , 1, − 1) on this line also lies on the plane 2 x + 3 y − 7 z = 5.
We have 5 . 2 + 6 . 3 + 4 . ( −7) = 0 and 2. 2 + 3 . 1 + ( −7) . ( −1) = 14 ≠ 5.
Thus the given straight line does not lie in the given plane.
3. We know that two non-coplanar lines never intersect.
4. The equations of the line x = ay + b, z = cy + d may be written as
x−b y z−d y
x − b = ay, z − d = cy or = , =
a 1 c 1
x−b y z−d
or = = which are in symmetrical form.
a 1 c
∴ D.r.’s of this line are a, 1, c.
Similarly, the d.r.’s of the line x = a′ y + b ′, z = c ′ y + d ′ are a′, 1, c ′.
Now the two given lines are perpendicular if
aa′ + 1 . 1 + cc ′ = 0 i. e., aa′+ cc ′ + 1 = 0.
x − x1 y − y1 z − z1
5. The conditions that the line = = is parallel to the plane
l m n
ax + by + cz + d = 0 but does not lie in it are
al + bm + cn = 0 but ax1 + by1 + cz1 + d ≠ 0.
6. See article 15, definition of shortest distance between two skew lines.
7. See article 2.
❍❍❍
G-133

Chapter-6
The Sphere

Comprehensive Problems 1
Problem 1: Find the equation of the sphere whose centre is ( −3, 4, 5) and radius 7.
(Bundelkhand 2007)
Solution: The equation of the sphere with centre ( −3, 4, 5) and radius 7 is given by
{ x − ( −3)}2 + ( y − 4)2 + ( z − 5)2 = (7)2
or x2 + 6 x + 9 + y2 − 8 y + 16 + z 2 − 10 z + 25 = 49
or x2 + y2 + z 2 + 6 x − 8 y − 10 z + 1 = 0.

Problem 2: Find the centre and radius of the sphere


2 x2 + 2 y2 + 2z 2 − 2 x + 4 y − 6z = 15.
Solution: We know that the centre of the sphere
x2 + y2 + z 2 + 24 x + 2vy + 2wz + d = 0 …(1)
2 2 2
is the point ( − u, − v , − w ) and its radius u + v + w − d .
The given sphere in
2 x2 + 2 y2 + 2z 2 − 2 x + 4 y − 6z − 15 = 0
15
or x2 + y2 + z 2 − x + 2 y − 3z − = 0. …(2)
2
Comparing (2) with (1); we have
15
2u = − 1, 2v = 2, 2w = − 3, d = − ⋅
2
1 3
∴ u = − , v = 1, w = −
2 2
1 9 15 1 + 4 + 9 + 30
u2 + v2 + w2 − d = + 1 + + = = 11.
4 4 2 4
∴ The centre of the given sphere is the point ( − u, − v , − w ) i.e.,  , − 1,  and its radius
1 3
2 2
is u2 + v2 + w2 − d i.e., 11.

Problem 3: Find the equation of the sphere which passes through the points
(0, 0, 0 ), (0, 1, − 1), ( − 1, 2, 0 ) and (1, 2, 3). (Avadh 2010)
Solution: Since the required sphere passes through the origin O (0, 0, 0 ), so let its
G-134

equation be
x2 + y2 + z 2 + 2ux + 2vy + 2wz = 0. …(1)
If the sphere (1) passes through the points (0, 1, − 1), ( − 1, 2, 0 ) and (1, 2, 3), these points
must satisfy the equation (1). Therefore
0 + 1 + 1 + 0 + 2v − 2w = 0 i. e., v − w + 1 = 0 …(2)
1 + 4 + 0 − 2u + 4v + 0 = 0 i. e., − 2u + 4v + 5 = 0 …(3)
and 1 + 4 + 9 + 2u + 4v + 6w = 0 i. e., 2u + 4v + 6w + 14 = 0. …(4)
Adding (3) and (4), we have
8v + 6w + 19 = 0. …(5)
Multiplying (2) by 6 and adding to (5), we get
14v + 25 = 0 or v = − 25 / 14.
Putting the value of v in (2), we get
25 11
− − w + 1 = 0 or w = − ⋅
14 14
Putting the value of v in (3), we get
50 15
− 2u − + 5 = 0 or u = − ⋅
7 14
Putting the values of u, v, w in (1), the required equation of the sphere is
15 25 11
x2 + y2 + z 2 − x− y− z =0
7 7 7
or 7 x2 + 7 y2 + 7z 2 − 15 x − 25 y − 11z = 0.

Problem 4: Find the equation of the sphere circumscribing the tetrahedron whose faces are
x = 0, y = 0, z = 0, x / a + y / b + z / c = 1.
Solution: The equations of the planes [i. e., faces of tetrahedron] are
x=0 …(1)
y =0 …(2)
z =0 …(3)
x / a + y / b + z / c = 1. …(4)
Any three faces (i. e., planes) meet at a vertex of the tetrahedron.
Solving (1), (2) and (3), we have the vertex O (0, 0, 0 ).
Solving (2), (3) and (4), we have the vertex A ( a, 0, 0 ).
Similarly solving (1), (3), (4) and (1), (2), (4) we have the vertices B (0, b, 0 ) and
C (0, 0, c ) respectively.
Therefore the sphere circumscribing the tetrahedron OABC is the sphere passing
through the four points O (0, 0, 0 ), A ( a, 0, 0 ), B (0, b, 0 ) and C (0, 0, c ). Now proceed as
in Example 3. Ans. x2 + y2 + z 2 − ax − by − cz = 0.
Problem 5: Find the equation of the sphere circumscribing the tetrahedron whose faces are
y z z x x y x y z
+ = 0, + = 0, + = 0, + + = 1.
b c c a a b a b c
Solution: The equations of the planes (i. e., faces of the tetrahedron) are
G-135

y z
+ = 0, …(1)
b c
z x
+ = 0, …(2)
c a
x y
+ = 0, …(3)
a b
x y z
+ + = 1. …(4)
a b c
Any three faces (i. e., planes) meet at a vertex of the tetrahedron. Hence, solving (1), (2),
(3), we get the vertex O (0, 0, 0 ) and solving (1), (2), (4), we get the vertex ( a, b, − c ).
Similarly solving other two triplets of the equations of the planes the remaining two
vertices are ( a, − b, c ) and ( − a, b, c ).
Hence the required sphere passes through the four points (0, 0, 0 ), ( a, b, − c ), ( a, − b, c )
and ( − a, b, c ).
Let the equation of the sphere be
x2 + y2 + z 2 + 2ux + 2vy + 2wz + d = 0. …(5)
If (5) passes through (0, 0, 0 ), we have d = 0. Hence (5) becomes
x2 + y2 + z 2 + 2ux + 2vy + 2wz = 0. …(6)
If (6) passes through ( a, b, − c ), ( a, − b, c ) and ( −a, b, c ), we have
a2 + b2 + c 2 + 2ua + 2vb − 2wc = 0, …(7)
2 2 2
a + b + c + 2ua − 2vb + 2wc = 0, …(8)
2 2 2
and a + b + c − 2ua + 2vb + 2wc = 0. …(9)
Adding (7) and (8),
2 ( a2 + b2 + c 2 ) + 4ua = 0 or 2u = − ( a2 + b2 + c 2 ) / a.
Similarly, we have
2v = − ( a2 + b2 + c2 ) / b, 2w = − ( a2 + b2 + c 2 ) / c.
Substituting the values of u, v, w and d in (5), the equation of the required sphere is given
by
x2 + y2 + z 2 − (1 / a) ( a2 + b2 + c 2 ) x − (1 / b ) ( a2 + b2 + c 2 ) y
− (1 / c ) ( a2 + b2 + c 2 ) z = 0
x2 + y2 + z 2 x y z
or − − − = 0.
a2 + b2 + c 2 a b c

Problem 6: Find the equation of the sphere which passes through the points (1, − 3, 4),
(1, − 5, 2), (1, − 3, 0 ) and whose centre lies on the plane x + y + z = 0. (Meerut 2012)
Solution: Let the equation of the sphere be given by
x2 + y2 + z 2 + 2ux + 2vy + 2wz + d = 0. …(1)
If (1) passes through (1, − 3, 4), then we have
1 + 9 + 16 + 2 u − 6v + 8w + d = 0. …(2)
If (1) passes through (1, − 5, 2), then we have
G-136

1 + 25 + 4 + 2 u − 10 v + 4 w + d = 0. …(3)
If (1) passes through (1, − 3, 0 ), then we have
1 + 9 + 0 + 2 u − 6v + 0 + d = 0. …(4)
The co-ordinates of the centre of the sphere (1) are ( − u, − v, − w )and if it lies on the plane
x + y + z = 0, then we have
− u − v − w = 0. …(5)
Subtracting (4) from (2),
16 + 8w = 0 or w = − 2 .
Subtracting (3) from (2),
−4 + 4 v + 4w = 0 or − 1 + v + w = 0. …(6)
Putting the value of w in (6),
−1 + v − 2 = 0 or v = 0.
Adding (5) and (6),
−1 − u = 0 or u = − 1.
Putting the values of u and v in (4), d = 10.
Substituting the values of u, v, w and d in (1), the equation of the required sphere is given
by
x2 + y2 + z 2 − 2 x + 6 y − 4z + 10 = 0.

Problem 7: Find the equation of the sphere having its centre on the line
2 x − 3 y = 0 = 5 y + 2z and passing through the points (0, − 2, − 4) and (2, − 1, − 1).
Solution: Let the equation of the sphere be
x2 + y2 + z 2 + 2ux + 2vy + 2wz + d = 0. …(1)
If (1) passes through the points (0, − 2, − 4) and (2, − 1, − 1), then
0 + 4 + 16 + 0 − 4v − 8w + d = 0 i. e., 20 − 4v − 8w + d = 0 …(2)
and 4 + 1 + 1 + 4u − 2v − 2w + d = 0
i. e., 6 + 4u − 2v − 2w + d = 0. …(3)
The coordinates of the centre of the sphere (1) are ( − u, − v, − w ) and if it lies on the line
2 x − 3 y = 0 = 5 y + 2z, then we have
− 2u + 3v = 0 …(4)
and − 5v − 2w = 0. …(5)
Subtracting (2) from (3), we get
4u + 2v + 6w − 14 = 0 or 2u + v + 3w = 7. …(6)
Putting the values of u and w from (4) and (5) in (6), we get
4 .  v  + 2v + 6 .  − v  − 14 = 0
3 5
2   2 
or 7v = − 14 or v = − 2.
Putting v = − 2 in (4) and (5), we get u = − 3, w = 5.
Putting the values of v and w in (2), we get d = 12.
Substituting the values of u, v, w and d in (1), the required equation of the sphere is
x2 + y2 + z 2 − 6 x − 4 y + 10 z + 12 = 0.
G-137

Problem 8: A plane passes through a fixed point ( p, q, r ) and cuts the axes in A, B, C. Show
that the locus of the centre of the sphere OABC is p / x + q / y + r / z = 2.
(Gorakhpur 2005; Rohilkhand 05; Avadh 09;
Kanpur 10; Purvanchal 10; Bundelkhand 12)
Solution: Let the equation of the plane be
x / a + y / b + z / c = 1, …(1)
where a, b, c are variables.
The plane (1) meets the coordinate axes at the points A ( a, 0, 0 ), B (0, b, 0 ) and
C (0, 0, c ). The equation of the sphere OABC is
x2 + y2 + z 2 − ax − by − cz = 0. …(2)
If the centre of the sphere (2) be the point (α, β, γ ), then
1 1 1
α = a, β = b, γ = c.
2 2 2
∴ a = 2α, b = 2β, c = 2γ. …(3)
It is given that the plane (1) passes through the fixed point ( p, q, r ). Hence we have
p / a + q / b + r / c = 1. …(4)
Putting the values of a, b and c from (3) in (4), we get
p q r p q r
+ + = 1 or + + = 2.
2α 2β 2γ α β γ
p q r
∴ The locus of the centre (α, β, γ ) is + + = 2.
x y z

Problem 9: A sphere of constant radius 2 k passes through origin and meets the axes in A, B
and C. Prove that the locus of the centroid of the tetrahedron OABC is x2 + y2 + z 2 = k2 .
(Bundelkhand 2012)
Solution: The equation of the sphere OABC is [ See Example 3]
x2 + y2 + z 2 − ax − by − cz = 0. …(1)
The radius of (1)
1 1 1
= √ {( a)2 + ( b )2 + ( c )2 } = 2k (given).
2 2 2
Squaring, we get
a2 + b2 + c 2 = 16 k2 . …(2)
Let (α , β, γ ) be the centroid of the tetrahedron OABC.
Then
1
α = (0 + a + 0 + 0 ) or a = 4 α.
4
Similarly b = 4 β and c = 4 γ.
Substituting the values of a, b, c in (2), we have
(4α )2 + (4 β )2 + (4γ )2 = 16 k2 or α2 + β2 + γ 2 = k2 .
∴ The required locus of the centroid (α , β, γ ) is
x2 + y2 + z 2 = k2 .
G-138

Problem 10: Find the equation of the sphere which passes through the points (1, 0, 0 ), (0, 1, 0 )
and (0, 0, 1) and has its radius as small as possible. (Agra 2008, 14)
Solution: Let the equation of the sphere be
x2 + y2 + z 2 + 2ux + 2vy + 2wz + d = 0. …(1)
If (1) passes through the points (1, 0, 0 ), (0, 1, 0 ) and (0, 0, 1), we have
1 + 2u + d = 0, 1 + 2v + d = 0, 1 + 2w + d = 0.
1
Solving these, we have u = v = w = − ( d + 1).
2
Let r be the radius of the sphere (1), so that we have
r2 = u2 + v2 + w2 − d = ( d + 1)2 − d =  d2 + 2d + 1 − d
3 3 4
4 4  3 

 d2 + 2 d + 1 = 3   d + 1  + 1 − 1  = 3   d + 1  + 8  ⋅
2 2
3
=        

4 3  4  3 9  4   3 9 

Obviously r2 is minimum when
1 1
d + = 0 i. e., d = − ⋅
3 3
∴ Radius of the sphere is as small as possible when d = − 1 / 3.
1 1
When d = − , we have u = v = w = − ⋅
3 3
Putting the values of u, v, w and d in (1), the required equation of the sphere is
2 2 2 1
x2 + y2 + z 2 − x − y − z − =0
3 3 3 3
or 3 ( x2 + y2 + z 2 ) − 2 x − 2 y − 2z − 1 = 0.

Problem 11: A point moves so that the ratio of its distances from two fixed points is constant.
Show that its locus is a sphere.
Solution: Let the moving point be P ( x , y , z ) and the fixed points A and B be ( a, 0 , 0 )
and ( −a, 0 , 0 ) respectively.
PA
Given = k or PA2 = k2 ⋅ PB2
PB
or ( x − a)2 + ( y + 0 )2 + ( z − 0 )2 = k2 [( x + a)2 + ( y − 0 )2 + ( z − 0 )2 ]
or x2(1 − k2 ) + y2(1 − k2 ) + z 2(1 − k2 ) − 2ax (1 + k2 ) + a2 (1 − k2 ) = 0
2a(1 + k2 )
or x2 + y2 + z 2 − x + a2 = 0, which evidently represents
(1 − k3 )
a sphere as it in second degree equation in x , y , z , coefficients of x2 , y2 , z 2 are equal
and terms containing product terms xy , yz , zx are absent.

Problem 12: A point moves so that the sum of the squares of its distances from the six faces of a
cube is constant. Prove that its locus is a sphere.
Solution: Consider a cube with the vertex O as origin and edges OA, OB and OC as
co-ordinate axes.
G-139

Let the length of each edge be a.


The six faces of the cube OBDC , OAEC , OBFA,
AFHE, BFHD, DHEC and their equations are
x = 0, y = 0, z = 0, x = a,
y = a, z = a respectively.
Let P (α , β, γ ) be the moving point. It is given
that the sum of the squares of the distances of
P (α , β, γ ) from the six faces of the cube is
constant, say k, so that we have
2 2
 α   β 
 2  +  2 
 √ (1 )   √ (1 )
2 2 2 2
 γ  α − a  β − a  γ − a
+  +  +  +  =k
2  2  2  2 
 √ (1 )   √ (1 )   √ (1 )   √ (1 )
or 2 (α2 + β2 + γ 2 ) − 2 a (α + β + γ ) + (3a2 − k ) = 0.
∴ The locus of P (α , β, γ ) is
2 ( x2 + y2 + z 2 ) − 2a ( x + y + z ) + (3a2 − k ) = 0
which is clearly the equation of a sphere.

Problem 13: A plane passes through a fixed point ( a, b, c ). Show that the locus of the foot of
perpendicular to it from the origin is the sphere x2 + y2 + z 2 − ax − by − cz = 0.
(Meerut 2008)
Solution: Let P ( x1, y1, z1 ) be the foot of the perpendicular drawn from the origin
O (0, 0, 0 ) to any plane passing through the fixed point ( a, b, c ). Direction ratios of the
line OP are x1 − 0, y1 − 0, z1 − 0, i. e., x1, y1, z1.
The equation of the plane passing through P ( x1, y1, z1) and perpendicular to OP is
x1 ( x − x1 ) + y1 ( y − y1 ) + z1 ( z − z1 ) = 0. …(1)
Since the plane (1) passes through the given point ( a, b, c ), therefore
x1 ( a − x1 ) + y1 ( b − y1 ) + z1 ( c − z1) = 0
or x1 ( x1 − a) + y1 ( y1 − b ) + z1 ( z1 − c ) = 0.
Hence the required locus of the foot of the perpendicular P ( x1, y1, z1 ) is
x ( x − a) + y ( y − b ) + z ( z − c ) = 0 .
which is the sphere x2 + y2 + z 2 − ax − by − cz = 0.

Problem 14: If O is the centre of a sphere of radius unity and A and B be two points in a line
with O such that OA . OB = 1 and if P be any variable point on the sphere, show that
PA : PB = constant.
Solution: Let us take O as the origin and OAB as the axis of x.

A is the point ( a, 0 , 0 ), B is  , 0 , 0  since OA = a, OB = and OA ⋅ OB = 1.


1 1
a  a
Also equation of the sphere of unit radius and centre as origin is
G-140

x2 + y2 + z 2 = 1. …(1)
Let P ( f , g , h) be the moving point on the sphere; then
f 2 + g2 + h2 = 1.

PA [( f + a)2 + g2 + h2 ]
We now have =
PB [( f − 1 / a)2 + g + h2 ]

(1 + a2 − 2af ) + a2 − 2af
= =a = a (constant).
(1 + 1 / a2 − 2 f / a) ( a + 1 − 2af )

Problem 15: OA, OB, OC are three mutually perpendicular lines through the origin and
their direction cosines are l1 , m1 , n1 ; l2 , m2 , n2 ; l3 , m3 , n3 . If OA = a, OB = b, OC = c ,
prove that the equation of the sphere OABC is
x2 + y2 + z 2 − x ( al1 + bl2 + cl3 ) − y ( am1 + bm2 + cm3 ) − z ( an1 + bn2 + cn3 ) = 0.
Solution: The co-ordinates of the points O, A, B and C are
(0, 0, 0 ), ( al1 , am1 , an1 ), ( bl2 , bm2 , bn2 ) and ( cl3 , cm3 , cn3 ) respectively.
Since the sphere passes through the origin O, its equation can be put as
x2 + y2 + z 2 + 2 ux + 2 vy + 2 wz = 0. …(1)
Substituting the co-ordinates of A in (1), we get
a2 ( l12 + m12 + n12 ) + 2 a ( ul1 + vm1 + wn1) = 0
or a + 2 ul1 + 2 vm1 + 2 wn1 = 0. …(2)
[∵ l12 + m12 + n12 = 1]
Similarly substituting the co-ordinates of B and C in (1), we get
b + 2ul2 + 2 vm2 + 2 wn2 = 0, …(3)
and c + 2 ul3 + 2 vm3 + 2 wn3 = 0. …(4)
Now l1 , l2 , l3 ; m1 , m2 , m3 ; n1 , n2 , n3 are the d.c’s of OX, OY , OZ respectively with
reference to the three mutually perpendicular given lines.
∴ l12 + l22 + l32 = m12 + m22 + m32 = n12 + n22 + n32 = 1,
and l 1 m1 + l 2 m2 + l 3 m3 = m1 n1 + m2 n2 + m3 n3
= l 1 n1 + l 2 n2 + l 3 n3 = 0.
Multiplying (2), (3) and (4) by l1, l2 and l3 respectively, adding and using the above
relations, we get
al1 + bl2 + cl3 + 2u = 0
or 2 u = − ( al1 + bl2 + cl3 ).
Similarly multiplying (2), (3) and (4) by m1, m2 and m3 , and then by n1 , n2 and n3 , and
proceeding as above, we get
2 v = − ( am1 + bm2 + cm3 ), 2 w = − ( an1 + bn2 + cn3 ).
Putting the values of 2 u, 2 v and 2 w in (1), we get the equation of the sphere as given.
G-141

Problem 16: A sphere of constant radius r passes through the origin O and cuts the axes in A,
B, C. Find the locus of the foot of the perpendicular from O to the plane ABC.
(Kumaun 2000)
Solution: Let ( a, 0, 0 ), (0, b, 0 ) and (0, 0, c ) be the co-ordinates of the points A , B and
C respectively.
The equation of the sphere OABC is
x 2 + y 2 + z 2 − ax − by − c z = 0. …(1)
[See Example 3]
As the intercepts made by the plane ABC on the co-ordinate axes are a, b, c , therefore
the equation of the plane ABC is
x / a + y / b + z / c = 1. …(2)
The radius r of the sphere (1) is given by
1 1 1
√ {( a)2 + ( b )2 + ( c )2 } = r or a2 + b 2 + c 2 = 4r 2 . …(3)
2 2 2
Now the equations of the line through the origin (0,0, 0) and perpendicular to the plane
(2) are
x−0 y −0 z −0
= = = λ , (say). …(4)
1/ a 1/ b 1/ c
The coordinates of any point on the line (4) are ( λ / a, λ / b , λ / c ).
If this is the foot, say (α, β, γ ), of the perpendicular from the origin to the plane (2), then
we have
λ / a = α, λ / b = β, λ / c = γ or a = λ / α, b = λ / β, c = λ / γ. …(5)
Since this foot of the perpendicular lies on the plane (2), we have
α / a + β / b + γ / c = 1. …(6)
Now putting the values of a, b, c from (5) in (3) and (6), we get
 1 1 1
λ2  + +  = 4r
2
…(7)
2 2
α β γ2 
1 2
and (α + β2 + γ 2 ) = 1. …(8)
λ
Eliminating λ between (7) and (8), we have
(α2 + β2 + γ 2 )2 (α −2 + β −2 + γ −2 ) = 4r 2 .
∴ The required locus of the foot of the perpendicular (α, β, γ ) is
( x2 + y2 + z 2 )2 ( x −2 + y −2 + z −2 ) = 4r2 .

Comprehensive Problems 2
Problem 1: Find the radius of the circle given by the equations
3 x2 + 3 y2 + 3z 2 + x − 5 y − 2 = 0, x + y = 2 .
Solution: The equations of the given circle are
1 5 2
x2 + y2 + z 2 + x − y − =0 …(1)
3 3 3
and x + y =2. …(2)
G-142

The centre of the sphere (1) is the point O  − , , 0 


1 5
 6 6 
1 5 2
and radius OP = √ {( − )2 + ( )2 + (0 )2 − ( − )} [See figure of article 6]
6 6 3
or OP = 5 /(3 √ 2).
Now OC = the length of the perpendicular from the centre O  − , , 0  to the plane
1 5
 6 6 
(2)
−1 / 6 + 5 / 6 − 2 4
= = (numerically).
√ {(1)2 + (1)2 } 3 √ 2
∴ The radius of the circle
= CP = √ ( OP2 − OC2 ) =  25 − 16  = 9 = 1 ⋅
   
 18 18   18  √ 2

Problem 2: Find the radius and centre of the circle of intersection of the sphere
x2 + y2 + z 2 − 2 y − 4z − 11 = 0 and the plane x + 2 y + 2z − 15 = 0.
(Bundelkhand 2006; Meerut 08, 09, 13; Kanpur 09; Rohilkhand 13)
Solution: The equations of the given circle are
x2 + y2 + z 2 − 2 y − 4z − 11 = 0 …(1)
and x + 2 y + 2z − 15 = 0. …(2)
The centre of the sphere (1) is the point C (0, 1, 2) and its radius
r = (0 )2 + (1)2 + (2)2 − ( − 11) = 16 = 4.
Let CN be the perpendicular from the point C to the plane (2) and let p = CN. The point
N is the centre of the circle in which the plane (2) cuts the sphere (1).
We have p = CN = the length of the perpendicular from the point C (0, 1, 2)to the plane
(2)
|0 + 2 (1) + 2 (2) − 15| 9
= = = 3.
2
1 +2 +22 2 3

∴ The radius of the circle in which the plane (2) cuts the sphere (1)
= r2 − p2 = 42 − 32 = 7.
Let us find the coordinates of the centre N of this circle. Obviously N is the point of
intersection of the line CN and the plane (2).
The line CN is perpendicular to the plane (2) and so direction ratios of CN are 1, 2, 2.
Also the line CN passes through the point C (0, 1, 2). So the equations of the line CN are
x−0 y −1 z − 2
= = = λ. …(3)
1 2 2
Let the coordinates of the point N which is on the line CN be
( λ + 0, 2 λ + 1, 2 λ + 2). …(4)
But N lies on the plane (2).
∴ λ + 2 (2 λ + 1) + 2 (2 λ + 2) − 15 = 0
G-143

or 9λ − 9 = 0 or λ = 1.
Putting λ = 1 in (4), the coordinates of N are (1, 3, 4).
Hence the centre of the circle in which the plane (2) cuts the sphere (1) is the point
N (1, 3, 4) and its radius is 7.

Problem 3: If r is the radius of the circle


x2 + y2 + z 2 + 2ux + 2vy + 2wz + d = 0, lx + my + nz = 0,

prove that ( r 2 + d ) ( l2 + m2 + n2 ) = ( mw − nv )2 + ( nu − lw )2 + ( lv − mu)2 .


(Purvanchal 2009; Rohilkhand 10; Avadh 11)
Solution: The centre of the given sphere is the point O ( − u, − v, − w ) and its radius
OP = √ ( u2 + v2 + w2 − d ). [See figure of article 6]
Now OC = the length of the perpendicular from the centre O ( − u, − v, − w )
to the plane lx + my + nz = 0
l ( − u) + m ( − v ) + n ( − w ) lu + mv + nw
= = (numerically).
√ ( l2 + m2 + n2 ) √ ( l2 + m2 + n2 )
∴ The radius r of the circle is given by
( lu + mv + nw )2
r 2 = OP2 − OC2 = ( u2 + v2 + w2 − d ) −
l2 + m2 + n2
or ( r 2 + d ) ( l2 + m2 + n2 ) = ( u2 + v2 + w2 ) ( l2 + m2 + n2 )
− ( lu + mv + nw )2
or ( r 2 + d ) ( l2 + m2 + n2 ) = ( mw − nv )2 + ( nu − lw )2 + ( lv − mu)2 ,
using Lagrange’s identity.

Problem 4(i): Find the equation of the sphere through the circle
x2 + y2 + z 2 = 9, x + y − 2z + 4 = 0 and the origin.
Solution: The equation of any sphere through the given circle is
( x2 + y2 + z 2 − 9) + λ ( x + y − 2 z + 4) = 0. …(1)
If the sphere (1) also passes through the origin (0, 0, 0 ), then we have
(0 + 0 + 0 − 9) + λ (0 + 0 − 0 + 4) = 0 or λ = 9 / 4.
Putting the value of λ in (1), the equation of the required sphere is given by
x2 + y2 + z 2 − 9 + (9 / 4) ( x + y − 2 z + 4) = 0
or 4 ( x2 + y2 + z 2 ) + 9 x + 9 y − 18 z = 0.
Problem 4(ii): Find the equation to the sphere through the circle x2 + y2 + z 2 = 9,
2 x + 3 y + 4z = 5 and the origin. (Kanpur 2015)
Solution: Proceed as in part (i). Ans. 5 ( x2 + y2 + z 2 ) − 18 x − 27 y − 36z = 0.

Problem 5: A circle, centre (2 , 3, 0 ) and radius 1, is drawn in the plane z = 0. Find the
equation of the sphere which passes through this circle and the point (1, 1, 1).
(Bundelkhand 2005)
G-144

Solution: The equations of the circle of radius 1, in the z = 0 plane and with centre
(2 , 3, 0 ) are given by ( x − 2)2 + ( y − 3)2 = 12 , z = 0.
These on introducing the term of z 2 can be rewritten as
( x − 2)2 + ( y − 3)2 + z 2 = 1, z = 0. …(1)
The equation of any sphere through the circle (1) is
( x − 2)2 + ( y − 3)2 + z 2 − 1 + λz = 0. …(2)
If (2) passes through (1, 1, 1), we have λ = − 5.
Putting the value of λ in (2), the equation of the required sphere is
( x − 2)2 + ( y − 3)2 + z 2 − 1 − 5z = 0
or x2 + y2 + z 2 − 4 x − 6 y − 5z + 12 = 0.

Problem 6: Show that the two circles x 2 + y 2 + z 2 − y + 2 z = 0, x − y + z = 2 ;

x 2 + y 2 + z 2 + x − 3 y + z − 5 = 0, 2 x − y + 4z = 1 lie on the same sphere. Find its


equation. (Kanpur 2005)
Solution: Any sphere through circle (1) is
x2 + y2 + z 2 − y + 2z + λ( x − y + z − 2) = 0 …(3)
and any sphere through the circle (2) is
x2 + y2 + z 2 + 3 y + z − 5 + µ (2 x − y + 4z − 1) = 0 . …(4)
The given circles will lie on the same sphere if λ and µ can be so chosen that (3) and
(4)represent the same sphere.
Equating coefficients of like terms from (3) and (4), we have
λ = 2µ + 1, …(5) −1 − λ = − µ − 3 , …(6)
2 + λ = 4µ + 1 …(7) −2λ = − µ − 5 . …(8)
Equations (5) and (6) give λ = 3, µ = 1and these values of λ and µ satisfy (7) and (8).
Thus the two circles lie on the same sphere.
Putting λ = 3 in (3), or µ = 1 in (4), the equation of the sphere on which both the given
circles lie is
x2 + y2 + z 2 + 3 x − 4 y + 5z − 6 = 0 .

Problem 7: Prove that the sphere


S1 ≡ x2 + y2 + z 2 + 2u1 x + 2v1 y + 2w1z + d1 = 0 cuts the sphere
S2 ≡ x2 + y2 + z 2 + 2u2 x + 2v2 y + 2w2 z + d2 = 0 in a great circle if
2 ( u22 + v22 + w22 ) − d2 = 2 ( u1u2 + v1v2 + w1w2 ) − d1 or if
2 ( u1u2 + v1v2 + w1w2 ) = 2 r22 + d1 + d2
where r2 is the radius of the second sphere S2 = 0. (Bundelkhand 2006)
Solution: The equation of the plane through the circle of intersection of the given
spheres S1 = 0 and S2 = 0 is S1 − S2 = 0,
i. e., 2 ( u1 − u2 ) x + 2 ( v1 − v2 ) y + 2 ( w1 − w2 ) z + d1 − d2 = 0. …(1)
If the sphere S1 = 0 cuts the sphere S2 = 0 in a great circle, then the plane (1) will pass
G-145

through the centre ( − u2 , − v2 , − w2 ) of the sphere S2 = 0, so that we have


2 ( u1 − u2 ) ( − u2 ) + 2 ( v1 − v2 ) ( − v2 ) + 2 ( w1 − w2 ) ( − w2 ) + d1 − d2 = 0
or 2 ( u22 + v22 + w22 ) − d2 = 2 ( u1u2 + v1v2 + w1w2 ) − d1 . …(2)
Again r2 is the radius of the sphere S2 = 0, hence
r22 = u22 + v22 + w22 − d2
1
or r22 = {2 ( u1u2 + v1v2 + w1w2 ) − d1 − d2} [Using (2)]
2
or 2 ( u1u2 + v1v2 + w1w2 ) = 2 r22 + d1 + d2 . …(3)
The relations (2) and (3) are the required conditions.

Problem 8: A variable plane is parallel to the given plane x / a + y / b + z / c = 0 and meets


the axes in A, B, C respectively. Prove that the circle ABC lies on the cone
y z ( b / c + c / b ) + zx ( c / a + a / c ) + xy ( a / b + b / a) = 0.
(Meerut 2003, 06B, 09, 09B)
Solution: The equation of any plane parallel to the given plane
x / a + y / b + z / c = 0 is
x / a + y / b + z / c = λ. …(1)
The plane (1) meets the co-ordinate axes in the points A, B and C whose co-ordinates
are ( aλ , 0, 0 ), (0, b λ , 0 )and (0, 0, c λ )respectively. The equation of the sphere OABC is
x2 + y2 + z 2 − aλx − b λy − c λz = 0
or x2 + y2 + z 2 − λ ( ax + by + cz ) = 0. …(2)
The equations (1) and (2) together represent the circle ABC.
Eliminating the variable λ between (1) and (2), the required locus is given by
x2 + y2 + z 2 − ( x / a + y / b + z / c ) ( ax + by + cz ) = 0
or yz ( b / c + c / b ) + zx ( c / a + a / c ) + xy ( a / b + b / a) = 0.
Problem 9: Find the equation of the sphere whose centre is the point (1, 2 , 3) and which touches
the plane 3 x + 2 y + z + 4 = 0. Show also that the radius of the circle in which the sphere is cut
by the plane x + y + z = 0 is 2 .
Solution: The centre of sphere is the point (1, 2, 3) and sphere touches the plane
3x + 2 y + z + 4 = 0
We have, radius of sphere = perpendicular distance from centre to touches plane of
sphere
3(1 ) + 2(2) + (3) + 4 14
= = = 14 = C1. P (say).
(3)2 + (2)2 + (1)2 14
∴ Equation of sphere with centre G(1, 2, 3) and radius 14 is
( x − 1 )2 + ( y − 2)2 + ( z − 3)2 = ( 14 )2
or x2 + y2 + z 2 − 2 x − 4 y − 6z = 0. …(1)
Again, this sphere is cut by the plane
x + y + z = 0. …(2)
Now C1C2 = perpendicular distance from C1 to plane (2)
G-146

1+ 2 + 3 6
= = = 2 3.
1+1+1 3 C1
In ∆ C1 C2 P, we have
C1C22 + C2 P2 = C1P2
(2 3 )2 + ( C2 P )2 = ( 14 )2
P
12 + ( C2 P )2 = 14 C2
or C2 P = 2 ,
which is the radius of circle.

Problem 10: A is a point on OX and B on OY , so that the angle OAB is constant and equal to
α. On AB as diameter a circle is drawn whose plane is parallel to OZ. Prove that as AB varies, the
circle generates the cone 2 xy − z 2 sin 2 α = 0. (Garhwal 2001)
Solution: Since A is a point on OX, we may take A as ( a, 0, 0 ).
Similarly B being on OY may be taken as (0, b, 0 ). Given ∠OAB = α , therefore from
right angled triangle AOB, we have
OB b
tan α = = ⋅ …(1)
OA a
The equation of the sphere on AB as diameter is
( x − a)( x − 0 ) + ( y − 0 ) ( y − b ) + ( z − 0 )( z − 0 ) = 0 [See article 4]
or x2 + y2 + z 2 = ax + by. …(2)
Also the equation of the plane passing through the points A and B and parallel to OZ is
x / a + y / b = 1. …(3)
Thus the equations (2) and (3) are the equations of the circle on AB as diameter and
having its plane parallel to OZ.
The locus of this circle is obtained by eliminating the variables a and b between (1),(2)
and (3).
From (2) and (3), we get x2 + y2 + z 2 = ( ax + by ) ( x / a + y / b )
or x2 + y2 + z 2 = x2 + y2 + xy ( a / b + b / a)
or x2 + y2 + z 2 = x2 + y2 + xy (cot α + tan α ), using (1)
or z 2 = xy (cos α / sin α + sin α / cos α )
or z 2 = xy / (sin α cos α )
or sin α cos α z 2 = xy or 2 xy − z 2 sin 2α = 0,
which is the required locus.

Problem 11: If A , B , C are the points ( a, 0, 0 ), (0, b, 0 ), (0, 0, c ) and the axes are
rectangular, prove that the diameter of the circle ABC is
( b2 + c2 )( c2 + a2 ) ( a2 + b2 )
 .
 b2 c2 + c2 a2 + a2 b2 
G-147

Solution: The point A , B, C are ( a, 0 , 0 ), (0 , b , 0 ) and (0 , 0 , c ) respectively.


Circum circle of ∆ ABC in the section of any sphere passing through through A , B and C
by the plane ABC.
∴ Equation of plane is
x y z
+ + = 1. …(1)
a b c
To find the equation of a sphere through A , B and C a forth point is necessary, which for
the sake of convenience in being taken as the origin 0.
Now the equation of the sphere through ( a, 0 , 0 ), (0 , b , 0 ), (0 , 0 , c ) and (0 , 0 , 0 ) is
x2 + y2 + z 2 − ax − by − cz = 0. …(2)
Hence the equation of the circum circle of ∆ ABC are
x y z
+ + = 1, x2 + y2 + z 2 − ax − by − cz = 0.
a b c
∴ Centre of the sphere (2) is  , ,  ⋅
a b c
 2 2 2
Let r be the radius of the circle and R the radius of the sphere. If p be the length of the
perpendicular from the centre of the sphere to the plane (1) then
r2 = R2 − p2 . …(3)
1 2
Now R= a + b2 + c2
2
1 1 1
+ + −1
2 2 2 1
and p= = ⋅
2 2
 1  +  1  +  1
2
2 a −2 + b −2 + c −2
     
 a  b  c
Putting the value of R and p in (3), we get
a2 + b2 + c2 1
r2 = −
4 4 ( a + b −2 + c −2 )
−2

a2 b2 c2
or 4r2 = a2 + b2 + c2 −
b2 c2 + c2 a2 + a2 b2
( b2 + c2 )( c2 + a2 )( a2 + b2 )
or 4r2 = ⋅
b2 c2 + c2 a2 + a2 b2
( b2 + c2 )( c2 + a2 )( a2 + b2 )
∴ The required diameter = 2r = .
b2 c2 + c2 a2 + a2 b2
x y z
Problem 12: The plane ABC , whose equation is + + = 1, meets the axes in A, B , C .
a b c
Find equations to determine the circumcircle of the triangle ABC, and obtain the co-ordinates of
its centre.Also find the diameter of this circle. (Kanpur 2007)
Solution: The given plane ABC is
x y z
+ + = 1. …(1)
a b c
G-148

Its points of intersection with the axes are A , B , C . Hence the points A , B , C are
respectively ( a, 0, 0 ), (0, b, 0 ), (0, 0, c ).
Circumcircle of ∆ ABC is the section of any sphere passing through A , B and C by the
plane ABC .
To find the equation of a sphere through A, B and C a fourth point is necessary, which
for the sake of convenience is being taken as the origin O (0, 0, 0).
Now the equation of the sphere through ( a, 0, 0 ), (0, b, 0 ), (0, 0, c ) and (0, 0, 0 ) is
x 2 + y 2 + z 2 − ax − by − c z = 0. …(2)
Hence the equations of the circumcircle of the ∆ ABC are
x y z
+ + = 1, x 2 + y 2 + z 2 − ax − by − c z = 0.
a b c
Centre of the sphere (2) is  , ,  ⋅
a b c
 2 2 2
A line perpendicular to the plane (1) and passing through the centre of the sphere (2)
will meet the plane (1) at the centre of the circle ABC .
Equations of the perpendicular from the centre of the sphere (2) upon the plane (1) are
x − a /2 y − b /2 z − c /2
= = = r (say). …(3)
1/ a 1/ b 1/ c
[Perpendicular is || to the normal to the plane.]
Any point on the line (3) is  + , + , +  ⋅
a r b r c r
2 a 2 b 2 c 
If it lies on the plane (1), then
1 a r 1 b r 1  c r
 +  +  +  +  +  =1
a  2 a b  2 b  c  2 c
1 1 1  1 1
or  2 + 2 + 2 r = − or r =−
−2

a b c  2 2 (a + b −2 + c −2 )
a r a 1 a2 ( a−2 + b −2 + c −2 ) − 1
∴ + = − =
− − −
2 2
2 a 2 2 a (a + b + c ) 2
2 a ( a −2 + b −2 + c −2 )
a ( b −2 + c −2 )
= ⋅
2 ( a −2 + b −2 + c −2 )
b r c r
Similarly we find + and + ⋅
2 b 2 c
Hence the centre of the circle ABC is
 a ( b −2 + c −2 ) b ( c −2 + a −2 ) c ( a −2 + b −2 ) 
 , ,  ⋅
 2 ( a −2 + b −2 + c −2 ) 2 ( a −2 + b −2 + c −2 ) 2 ( a −2 + b −2 + c −2 ) 
To find the diameter of the circle: Let r be the radius of the circle and R the radius of
the sphere. If p be the length of the perpendicular from the centre of the sphere to the
plane (1), then
r 2 = R2 − p2 . …(4)
G-149

1
Now R = radius of the sphere (2) = √ ( a2 + b2 + c 2 )
2
p = the length of the perpendicular from  a, b, c 
1 1 1
and
2 2 2 
i. e., the centre of the sphere (2) to the plane (1)
1 1 1
+ + −1
2 2 2 1
= = ⋅
√ {(1 / a)2 + (1 / b )2 + (1 / c )2 } 2 √ ( a−2 + b −2 + c −2 )
Putting the values of R and p in (4), we get
a2 + b2 + c 2 1
r2 = −
4 4 ( a −2 + b −2 + c −2 )
a2 b2 c 2
or 4 r 2 = a2 + b2 + c 2 −
2 2
b c + c 2 a2 + a2 b2
( b2 + c 2 ) ( c 2 + a2 ) ( a2 + b2 )
= ⋅
b2 c 2 + c 2 a2 + a2 b2
( b2 + c 2 ) ( c 2 + a2 ) ( a2 + b2 )
∴ The required diameter = 2 r =  ⋅
 b2 c 2 + c 2 a2 + a2 b2 

Comprehensive Problems 3
Problem 1: Find the equation of the sphere described on the line joining the points (3, 4, 1) and
( −1, 0, 5) as diameter and find also the equation of the tangent plane at ( −1, 0, 5).
Solution: The equation of the sphere described on the line joining the points A(3, 4, 1)
and B ( −1, 0, 5) as diameter is giving by
( x − 3){x − ( −1)} + ( y − 4)( y − 0 ) + ( z − 1)( z − 5) = 0
or x2 + y2 + z 2 − 2 x − 4 y − 6z + 2 = 0 .
The equation of the tangent plane at ( −1, 0, 5) of the above sphere is given by
x .( −1) + y .( 0 ) + z .(5) − 1( x − 1) − 2( y + 0 ) − 3( z + 5) + 2 = 0
x + y − z + 6 = 0.
Problem 2: Find the equations of the tangent planes of the sphere
x2 + y2 + z 2 + 2 x − 4 y + 6 z − 7 = 0
which intersect in the line 6 x − 3 y − 23 = 0 = 3z + 2 .
Solution: The equation of the given sphere is
x2 + y2 + z 2 + 2 x − 4 y + 6z − 7 = 0. …(1)
The equation of any plane through the given line
6 x − 3 y − 23 = 0 = 3z + 2
is 6 x − 3 y − 23 + λ (3z + 2) = 0
G-150

or 6 x − 3 y + 3λz + (2 λ − 23) = 0. …(2)


If the plane (2) is the tangent plane to the sphere (1), then
the length of the perpendicular from the centre ( −1, 2 , − 3) of the
sphere (1) to the plane (2) = the radius of the sphere (1)
6 . ( −1) − 3 . 2 + 3λ ( −3) + 2 λ − 23
i. e., = √ {(1)2 + ( −2)2 + (3)2 − ( −7)}
2 2 2
√ {(6) + (3) + (3λ ) }
or −7λ − 35 = √ (21) √ (45 + 9λ2 ).
Squaring and simplifying, we get 2 λ2 − 7λ − 4 = 0
1
or 2 λ2 − 8λ + λ − 4 = 0 or λ = 4, −

2
Putting the values of λ in (2), the required equations of the tangent planes are given by
2 x − y + 4 z − 5 = 0 and 4 x − 2 y − z − 16 = 0.

Problem 3: Find the equations of the tangent planes to the sphere


x2 + y2 + z 2 + 6 x − 2 z + 1 = 0
which pass through the line 3 (16 − x ) = 3z = 2 y + 30.
Solution: The equations of the given line may be re-written as
3 (16 − x ) − 3z = 0 = 2 y − 3z + 30.
Now proceed as in Problem 2, the equations of the tangent planes are given by
x + 2 y − 2 z + 14 = 0 and 2 x + 2 y − z = 2 .

Problem 4: Find the equations of the tangent planes to the sphere x2 + y2 + z 2 = 24 which
are parallel to the plane 2 x + y − z = 0.
Solution: The equation of the given sphere is
x2 + y2 + z 2 = 24. …(1)
Let the equation of the tangent plane parallel to the plane
2 x + y − z = 0 be 2 x + y − z + λ = 0. …(2)
If the plane (2) touches the sphere (1), then the length of the perpendicular from the
centre (0, 0, 0 ) of the sphere (1) to the plane (2) = ± the radius of the sphere (2)
2.0 + 0 − 0 + λ
i. e., = ± √ (24)
√ {(2)2 + (1)2 + ( −1)2 }
or λ = ± √ (24) . √ 6 = ± 12 .
Putting the values of λ in (2), the equations of the required tangent planes are given by
2 x + y − z ± 12 = 0.

Problem 5: Show that the plane 2 x − 2 y + z + 12 = 0 touches the sphere

x 2 + y 2 + z 2 − 2 x − 4 y + 2 z − 3 = 0 and find the point of contact.


Solution: The equation of the given sphere is
x 2 + y 2 + z 2 − 2 x − 4 y + 2 z − 3 = 0. …(1)
2 2 2
Its centre C is (1, 2 , − 1) and its radius = √ {( −1) + ( −2) + (1) − ( −3)} = 3.
G-151

The equation of the given plane is


2 x − 2 y + z + 12 = 0. …(2)
The length of the perpendicular from the centre C (1, 2 , − 1) to the plane (2)
2 . 1 − 2 . 2 − 1 + 12 9
= = =3
√ {(2)2 + ( −2)2 + (1)2 } 3
= the radius of the sphere (1).
Since the length of the perpendicular from the centre C (1, 2 , − 1) of the sphere (1) to the
plane (2) is equal to the radius of the sphere, therefore, the plane (2) touches the given
sphere (1).
To find the point of contact: The d.r.’s of the normal to the tangent plane (2) are
2 , − 2 , 1 and therefore, the d.r.’s of the radius CP are 2 , − 2 , 1.
[ ∵ radius CP is perpendicular to the tangent plane at P ]
Thus the equations of the line (radius) CP are
( x − 1) / 2 = ( y − 2) / ( −2) = ( z + 1) / 1 = r (say). …(3)
The co-ordinates of any point on (3) are (2 r + 1, − 2 r + 2 , r − 1). If this is the point of
contact P, then it will lie on the tangent plane (2) and so we have
2 (2 r + 1) − 2 ( −2 r + 2) + ( r − 1) + 12 = 0 or r = − 1.
Putting the value of r, the co-ordinates of the point of contact P are
(2 ( −1) + 1, − 2 ( −1) + 2 , − 1 − 1) or ( −1, 4, − 2).

Problem 6: (i) Find the equation of the sphere which touches the sphere
2 2 2
x + y + z + 2 x − 6 y + 1 = 0 at (1, 2 , − 2) and passes through the point (1, − 1, 0 ).
(ii) Find the equation of the sphere which has its centre as the origin and which touches the line
2 ( x + 1) = 2 − y = z + 3.
Solution: (i) The equation of the given sphere is
x2 + y2 + z 2 + 2 x − 6 y + 1 = 0. …(1)
The equation of the tangent plane to the sphere (1) at the point (1, 2 , − 2) is
x . 1 + y . 2 + z . ( −2) + ( x + 1) − 3 ( y + 2) + 1 = 0
or 2 x − y − 2 z − 4 = 0. …(2)
Therefore, the equation of the sphere which touches the sphere (1) at (1, 2 , − 2) is given
by
x2 + y2 + z 2 + 2 x − 6 y + 1 + λ (2 x − y − 2 z − 4) = 0. …(3)
If (3) passes through the point (1, − 1, 0 ), then we have
1 + 1 + 0 + 2 + 6 + 1 + λ (2 + 1 − 0 − 4) = 0 or λ = 11.
Putting the value of λ in (3), the equation of the required sphere is given by
x2 + y2 + z 2 + 24 x − 17 y − 22 z − 43 = 0.
(ii) Centre of the sphere is O (0, 0, 0 ), AN is the given line where A ( −1, 2 , − 3) is a
point on the line and N is its point of contact with the sphere.
The equations of the line AN are
2 ( x + 1) = 2 − y = z + 3
G-152

x+1 y −2 z+3
or = =
1 −2 2
x+1 y −2 z+3
or = = ⋅
1 2 2

3 3 3
1 2 2
Here , − , are actual d.c.’s of AN.
3 3 3
AN = Projection of OA on the line AN
1 2 2 11
= (0 + 1) − (0 − 2) + (0 + 3) = ⋅
3 3 3 3
AO = 1 + 4 + 9 = 14.
121 5 5
ON 2 = AO 2 − AN 2 = 14 − = i. e., (radius)2 = ⋅
9 9 9
∴ Equation of the sphere is x 2 + y 2 + z 2 = 5 / 9 or 9 ( x 2 + y 2 + z 2 ) = 5.

Problem 7: Prove that the equation of the sphere which touches


4 ( x2 + y2 + z 2 ) + 10 x − 25 y − 2 z = 0 at (1, 2 , − 2) and passes through
the point ( −1, 0, 0 ) is x2 + y2 + z 2 + 2 x − 6 y + 1 = 0.
Solution: Proceed as in Problem 6(i).

Problem 8: Find the equations of the spheres which pass through the circle
x2 + y2 + z 2 = 5, x + 2 y + 3z = 3 and touch the plane 3 x + 4 y = 15.
Solution: Proceed as in problem 2.
x2 + y2 + z 2 + 2 x + 4 y + 6z − 11 = 0 ;
5( x2 + y2 + z 2 ) − 4 x − 8 y − 12z − 13 = 0 .

Problem 9: Find the equations of the tangent line to the circle


3 x + 3 y2 + 3z 2 − 2 x − 3 y − 4z − 22 = 0, 3 x + 4 y + 5z − 26 = 0
2

at the point (1, 2 , 3).


Solution: The tangent line to a circle at a point is the line of intersection of the tangent
plane to the sphere at the point and the plane of the circle.
The equations of the given circle are given by the sphere
2 4 22
x2 + y2 + z 2 − x − y − z − =0 …(1)
3 3 3
and the plane 3 x + 4 y + 5z − 26 = 0. …(2)
The equation of the tangent plane to the sphere (1) at (1, 2 , 3) is
1 1 2 22
x . 1 + y . 2 + z . 3 − ( x + 1) − ( y + 2) − ( z + 3) − =0
3 2 3 3
or 4 x + 9 y + 14z − 64 = 0. …(3)
Thus the non-symmetrical equations of the required tangent line are given by (2) and
(3) together. To find its symmetrical form, let l, m, n be its d.r.’s so that we have
G-153

3l + 4m + 5n = 0 and 4l + 9m + 14 = 0.
Solving, we get
l m n l m n
= = or = = ⋅
−11 22 −11 1 −2 1
∴ The equations of the tangent line at (1, 2 , 3) and having d.r.’s 1, − 2 , 1 are
x −1 y −2 z −3
= = ⋅
1 −2 1

Problem 10: If three mutually perpendicular chords of lengths d1, d2 , d3 be drawn through the
point ( x1, y1, z1 ) to the sphere x2 + y2 + z 2 = a2 , prove that

d12 + d22 + d32 = 12 a2 − 8 ( x12 + y12 + z12 ).


Solution: The equation of the given sphere is
x2 + y2 + z 2 = a2 . …(1)
Let l1, m1, n1 ; l2 , m2 , n2 and l3 , m3 , n3 be the actual d.c.’s of the three mutually
perpendicular chords drawn through the same point A ( x1 , y1 , z1). The equations of
the first chord are given by
x − x1 y − y1 z − z1
= = = r (say). …(2)
l1 m1 n1
Any point on (2) is ( l1 r + x1 , m1 r + y1 , n1 r + z1).
Substituting the co-ordinates of this point in (1), the points of intersection of the chord
(2) with the sphere (1) are given by
( l1 r + x1)2 + ( m1 r + y1 )2 + ( n1 r + z1 )2 = a2
or r 2 ( l12 + m12 + n12 ) + 2 r ( l1 x1 + m1 y1 + n1 z1)
+ ( x12 + y12 + z12 − a2 ) = 0
or r 2 + 2 r ( l1 x1 + m1 y1 + n1 z1 ) + ( x12 + y12 + z12 − a2 ) = 0. …(3)
Let the chord (2) meet the sphere in the points P and Q so that AP = r1 , AQ = r2 where r1
and r2 are the roots of (3).
∴ d1 = PQ = AP − AQ = r1 − r2
or d12 = ( r1 − r2 )2 = ( r1 + r2 )2 − 4 r1 r2
or d12 = {−2 ( l1 x1 + m1 y1 + n1 z1 )}2 − 4 ( x12 + y12 + z12 − a2 )
or d12 = 4 ( l1 x1 + m1 y1 + n1 z1)2 − 4 ( x12 + y12 + z12 − a2 ). …(4)
Similarly d22 = 4 ( l2 x1 + m2 y1 + n2 z1 ) − 4 ( x12 +
2
y12 + z12 2
−a ) …(5)
and d32 = 4 ( l3 x1 + m3 y1 + n3 z1 )2 − 4 ( x12 + y12 + z12 2
−a ) …(6)
Adding (4), (5) and (6) and using l12 + l22 + l32 = 1 etc., l1 m1 + l2 m2 + l3 m3 = 0 etc.,
we get
d12 + d22 + d32 = 4 ( x12 + y12 + z12 ) − 12 ( x12 + y12 + z12 − a2 )
= 12 a2 − 8 ( x12 + y12 + z12 ).
G-154

Problem 11: (i) Show that the sum of the squares of the intercepts made by a given sphere on
any three mutually perpendicular straight lines through the fixed point is constant.
(ii) Find the equation of the sphere touching the three co-ordinate axes. How many such spheres
can be drawn ?
(iii) Find the locus of the centre of the sphere which passes through a given point and intercepts a
constant length on a given straight line.
Solution: (i) Let the equation of the given sphere be x2 + y2 + z 2 = a2 and the fixed
point be ( x1 , y1 , z1).
Now this problem is nothing but a different statement of the Problem 10.
(ii) Let the equation of the sphere be
x2 + y2 + z 2 + 2 ux + 2 vy + 2 wz + d = 0. …(1)
If the sphere (1) meets the x-axis i. e., y = 0, z = 0, then from (1), we have
x2 + 2 ux + d = 0. …(2)
Since the sphere (1) is touching y = 0 = z , the roots of (2) must be equal and so we have
(2 u)2 − 4 . 1 . d = 0 [Using B 2 − 4 AC = 0 ]
or u2 = d.
Similarly if the sphere touches y and z axes, we have
v2 = d and w2 = d.
∴ u2 = v2 = w2 = d = λ2 (say).
∴ u= v = w= ± λ.
Putting the values in (1), the equation of the sphere (1) becomes
x2 + y2 + z 2 ± 2 λ ( x + y + z ) + λ2 = 0. …(3)
Since λ may have any real value, an infinite number of spheres can be drawn satisfying
the given conditions.
In case the radius of the sphere be given, say r, we shall have only eight spheres as
shown below.
Now r = the radius of the sphere = √ ( u2 + v2 + w2 − d ).
∴ r2 = d+ d+ d− d [ ∵ u2 = v2 = w2 = d ]
or r 2 = 2 d.
1
∴ u2 = v2 = w2 = d = r 2 or u = v = w = ± r / ( 2 ).
2
Hence the equation (1) of the sphere becomes
1
x2 + y2 + z 2 ± r x 2 ± r y 2 ± r z 2 + r 2 = 0. …(4)
2
The equation (4) represents only eight spheres satisfying the given conditions.
(iii) Let the equation of the sphere be
x 2 + y 2 + z 2 + 2 u x + 2 vy + 2 w z + d = 0. …(1)
Let the given point be (0, 0, λ ) and the given line be taken as axis of x.
G-155

The sphere (1) passes through (0, 0, λ ), therefore λ2 + 2 w λ + d = 0. …(2)


The sphere (1) meets the axis of x ( y = 0, z = 0 ) where x 2 + 2 u x + d = 0. …(3)
Let x1, x2 be the roots of (3), then x1 + x2 = − 2 u, x1 . x2 = d.
Hence if k is the constant intercept made by the sphere on x-axis, then
k = x1 − x2 or k 2 = ( x1 − x2 )2 = ( x1 + x2 )2 − 4 x1 x2
1
or k 2 = 4u 2 − 4d or d = (4u 2 − k 2 ). …(4)
4
Eliminating d between (2) and (4), we get
1
λ2 + 2 w λ + u2 − k 2 = 0 or 4u2 + 8λ w + 4λ2 − k 2 = 0.
4
∴ The locus of the centre ( − u, − v, − w ) is 4 x 2 − 8λz + 4λ2 − k 2 = 0.

Problem 12: Find the equation of a sphere touching the three co-ordinate planes. How many
such spheres of given radius r can be drawn ?
Solution: Let the equation of the sphere be
x2 + y2 + z 2 + 2 ux + 2 vy + 2 wz + d = 0. …(1)
If the sphere (1) touches the yz-plane i. e., x = 0, then the length of the perpendicular
from the centre ( − u, − v, − w ) of the sphere (1) to the plane ( x = 0 ) = the radius of the
sphere (1)
i. e., − u / 1 = √ ( u2 + v2 + w2 − d ).
Squaring, u2 = u2 + v2 + w2 − d or v2 + w2 = d. …(2)
Similarly if the sphere (1) touches the planes y = 0 and z = 0, then we have
w2 + u2 = d …(3)
2 2
u + v = d. …(4)
Adding (2), (3) and (4), we get
3
2 ( u2 + v2 + w2 ) = 3d or u2 + v2 + w2 = d. …(5)
2
Subtracting (2), (3) and (4) from (5) in turn, we get
1
u2 = v2 = w2 = d = r 2 (say),
2
or u = v = w = ± r.
Also radius of the sphere = √ ( u2 + v2 + w2 − d ) = √ ( r 2 + r 2 + r 2 − 2 r 2 ) = r.
Substituting the values in (1), the equation of the required sphere is given by
x2 + y2 + z 2 ± 2 r x ± 2 r y ± 2 rz + 2 r 2 = 0. …(6)
Since r may have any real value, an infinite number of such spheres are possible. In case
the radius, r of the sphere be given then only eight such spheres are possible.
Problem 13: A sphere touches the three co-ordinate planes and passes through the point
(2 , 1, 5). Find its equation.
Solution: The given point (2 , 1, 5) lies in the positive octant OXYZ . Hence the
equation of the required sphere is of the form [See problem 12 above]
G-156

x2 + y2 + z 2 − 2 r ( x + y + z ) + 2 r 2 = 0. …(1)
[Deduce this result here]
If the sphere (1) passes through the point (2 , 1, 5), we have
4 + 1 + 25 − 2 r (2 + 1 + 5) + 2 r 2 = 0
or 2 r 2 − 16 r + 30 = 0 or r 2 − 8r + 15 = 0
or ( r − 3) ( r − 5) = 0 or r = 3, 5.
Substituting the values of r in turn in (1), the equations of the two required spheres are
given by
x2 + y2 + z 2 − 6 x − 6 y − 6z + 18 = 0,
and x2 + y2 + z 2 − 10 x − 10 y − 10 z + 50 = 0.

Problem 14: Find the equation of the sphere inscribed in the tetrahedron whose faces are
x = 0, y = 0, z = 0 and 2 x + 6 y − 3z + 9 = 0.
Solution: The equation of the fourth plane is 2 x + 6 y − 3z + 9 = 0. This plane clearly
meets the x, y and z axes in the −ve, −ve and +ve directions. Hence the sphere touching
the three co-ordinate planes and the plane 2 x + 6 y − 3z + 9 = 0 lies in the OX ′Y ′ Z
octant and so the centre of such a sphere is of the form ( − r, − r, r ) where r is the radius of
the sphere.
Since the sphere touches the plane
2 x + 6 y − 3z + 9 = 0, …(1)
therefore the length of the perpendicular from the centre ( − r, − r, r ) to the plane (1)
= the radius r of the sphere
−2 r − 6 r − 3 r + 9 1
i. e., = r or −11 r + 9 = 7r or r = ⋅
√ (4 + 36 + 9) 2
1 1 1 1
∴ The centre of the required sphere is ( − , − , ) and radius is ⋅ Therefore its
2 2 2 2
equation is given by
2 2 2 2
 x + 1 +  y + 1 +  z − 1 =  1
       
 2  2  2  2
2 2 2 1
or x + y + z + x + y − z + = 0.
2

Comprehensive Problems 4
x −1 y −2 z −3
Problem 1: Show that the polar of = = with respect to the sphere
2 3 4
x2 + y2 + z 2 = 9 is given by x + 2 y + 3z − 9 = 0, 2 x + 3 y + 4z = 0.
Solution: This problem is based on article 19. The co-ordinates of any point on the
line
x −1 y −2 z −3
= = are (2r + 1, 3r + 2 , 4r + 3).
2 3 4
G-157

Its polar plane w.r.t. the sphere x2 + y2 + z 2 = 9 is


(2r + 1) x + (3r + 2) y + (4r + 3) z = 9
or ( x + 2 y + 3z − 9) + r (2 x + 3 y + 4z ) = 0.
This plane for all values of r passes through the line
x + 2 y + 3z − 9 = 0, 2 x + 3 y + 4z = 0.
Thus these are the equations of the polar line of the given line.

Problem 2: Show that the two spheres x2 + y2 + z 2 + 6 y + 2z + 8 = 0


and x2 + y2 + z 2 + 6 x + 8 y + 4z + 20 = 0 are orthogonal. (Avadh 2009)
Solution: The equations of the two spheres are
x2 + y2 + z 2 + 6 y + 2z + 8 = 0 …(1)
2 2 2
x + y + z + 6 x + 8 y + 4z + 20 = 0. …(2)
For the sphere (1), we have u1 = 0, v1 = 3, w1 = 1, d1 = 8, and for the sphere (2), we have
u2 = 3, v2 = 4, w2 = 2 , d2 = 20.
We know that the condition of orthogonality of two spheres is [See article 21,
equation (3)]
2u1u2 + 2v1v2 + 2w1w2 = d1 + d2 . …(3)
Putting the values in (3), we have
2 . 0 . 3 + 2 . 3 . 4 + 2 . 1 . 2 = 8 + 20
or 0 + 24 + 4 = 28, which is true.
Hence the given spheres are orthogonal.

Problem 3: Two points P and Q are conjugate with respect to a sphere S ; prove that the sphere
on PQ as diameter cuts S orthogonally.
Solution: Let the equation of the sphere S be given by
S ≡ x2 + y2 + z 2 + 2ux + 2 vy + 2wz + d = 0. …(1)
Suppose the co-ordinates of the given points P and Q are ( x1 , y1 , z1 ) and ( x2 , y2 , z2 )
respectively.
The equation of the polar plane of P ( x1 , y1 , z1) w.r.t. the sphere (1) is
xx1 + yy1 + zz1 + u ( x + x1 ) + v ( y + y1 ) + w ( z + z1) + d = 0.
The point Q ( x2 , y2 , z2 ) will lie on this plane, since P and Q are conjugate points and
hence, we have
x1 x2 + y1 y2 + z1z2 + u ( x1 + x2 ) + v ( y1 + y2 ) + w ( z1 + z2 ) + d = 0.
…(2)
The equation of the sphere on PQ as diameter is
( x − x1) ( x − x2 ) + ( y − y1 ) ( y − y2 ) + ( z − z1) ( z − z2 ) = 0
or x2 + y2 + z 2 − x ( x1 + x2 ) − y ( y1 + y2 ) − z ( z1 + z2 )
+ ( x1 x2 + y1 y2 + z1z2 ) = 0. …(3)
Now the sphere (3) will cut the sphere (1) orthogonally, if
( x + x2 ) ( y1 + y2 ) ( z + z2 )
−2 1 ⋅u−2 ⋅v −2 1 ⋅w
2 2 2
= d + ( x1 x2 + y1 y2 + z1z2 )
G-158

or x1 x2 + y1 y2 + z1z2 + u ( x1 + x2 ) + v ( y1 + y2 ) + w ( z1 + z2 ) + d = 0,
which is true by virtue of (2).

Problem 4: Obtain the condition that the spheres


a ( x2 + y2 + z 2 ) + 2lx + 2my + 2nz + p = 0 and b ( x2 + y2 + z 2 ) = k2
may cut orthogonally. (Meerut 2009B, 12)
Solution: The equations of the given spheres may be re-written as
x2 + y2 + z 2 + 2 ( l / a) x + 2 ( m / a) y + 2( n / a) z + ( p / a) = 0, …(1)
2 2 2 2
and x + y + z − ( k / b ) = 0. …(2)
The spheres (1) and (2) will cut orthogonally if
2 ( l / a) . 0 + 2 ( m / a) . 0 + 2 ( n / a) . 0 = ( p / a) + { − ( k2 / b )}
[See article 21]
p k2
or = or pb = ak2
a b
which is the required condition.

Problem 5: Prove that every sphere through the circle x2 + y2 − 2ax + r 2 = 0, z = 0

cuts orthogonally every sphere through the circle x2 + z 2 = r 2 , y = 0.


Solution: The equations of any spheres through the given circles are given by
x2 + y2 + z 2 − 2ax + r 2 + 2λ1 z = 0 …(1)
2 2 2 2
and x + y + z −r + 2λ2 y = 0. …(2)
If the spheres (1) and (2) cut orthogonally, then applying the condition
2u1u2 + 2v1v2 + 2w1w2 = d1 + d2, we have
2 . ( − a) . 0 + 2 . 0 . λ2 + 2 . λ1 . 0 = r 2 + ( − r 2 )
or 0 =0
which is true for all values of λ1 and λ2 . Hence the result.

Problem 6: Find the equation of the sphere that passes through the circle
x2 + y2 + z 2 − 2 x + 3 y − 4z + 6 = 0, 3 x − 4 y + 5z − 15 = 0

and cuts the sphere x2 + y2 + z 2 + 2 x + 4 y − 6z + 11 = 0 orthogonally.


(Avadh 2007; Purvanchal 08)
Solution: The equation of any sphere through the given circle is
x2 + y2 + z 2 − 2 x + 3 y − 4z + 6 + λ (3 x − 4 y + 5z − 15) = 0
or x2 + y2 + z 2 + (3λ − 2) x + (3 − 4 λ ) y + (5λ − 4) z + (6 − 15λ ) = 0.
…(1)
The equation of the other given sphere is
x2 + y2 + z 2 + 2 x + 4 y − 6 z + 11 = 0. …(2)
If the spheres (1) and (2) cut orthogonally, then applying the condition
2 u1 u2 + 2 v1 v2 + 2 w1 w2 = d1 + d2 , we have
G-159

(3λ − 2) (3 − 4λ ) (5λ − 4)
2⋅ ⋅1 + 2 ⋅ ⋅2 + 2⋅ ⋅ ( −3) = (6 − 15λ ) + 11
2 2 2
or 3λ − 2 + 6 − 8λ + 12 − 15λ = 17 − 15λ
or 16 − 5λ = 17 or λ = − 1 / 5.
Putting the value of λ in (1), the equation of the required sphere is given by
5 ( x2 + y2 + z 2 ) − 13 x + 19 y − 25z + 45 = 0.

Problem 7: Prove that a sphere S = 0 which cuts the two spheres S1 = 0 and S2 = 0 at right
angles will also cut the sphere λ1 S1 + λ2 S2 = 0 at right angles.
Solution: Let the equations of the spheres be given by
S ≡ x2 + y2 + z 2 + 2ux + 2vy + 2wz + d = 0,
S1 ≡ x2 + y2 + z 2 + 2u1 x + 2v1 y + 2w1z + d1 = 0,
and S2 ≡ x2 + y2 + z 2 + 2u2 x + 2v2 y + 2w2 z + d2 = 0.
The sphere S = 0 cuts the sphere S1 = 0 orthogonally, so we have
2uu1 + 2vv1 + 2ww1 = d + d1. …(1)
The sphere S = 0 cuts the sphere S2 = 0 orthogonally, so we have
2uu2 + 2vv2 + 2ww2 = d + d2 . …(2)
The equation of the sphere λ1S1 + λ2 S2 = 0 is given by
λ u + λ2 u2  λ v + λ2 v2 
x2 + y2 + z 2 + 2  1 1  x+2 11  y
 λ1 + λ2   λ1 + λ2 
 λ w + λ2 w2   d λ + d2 λ2 
+2 1 1  z+  1 1  = 0. …(3)
 λ1 + λ2   λ1 + λ2 
The sphere S = 0 will cut the sphere λ1 S1 + λ2 S2 = 0 [i. e., the sphere (3)] orthogonally
if
 λ u + λ2 u2   λ v + λ2 v2   λ w + λ2 w2 
2u 11  +2v 11  +2w 1 1 
 λ1 + λ2   λ1 + λ2   λ1 + λ2 
 d λ + d2 λ2 
=d+  1 1 
 λ1 + λ2 
or 2u ( λ1u1 + λ2 u2 ) + 2v ( λ1v1 + λ2 v2 ) + 2w ( λ1w1 + λ2 w2 )
= d ( λ1 + λ2 ) + ( d1λ1 + d2 λ2 )
or λ1 (2 uu1 + 2 vv1 + 2 ww1) + λ2 (2 uu2 + 2 vv2 + 2 ww2 )
= λ1 ( d + d1 ) + λ2 ( d + d2 )
or λ1 ( d + d1 ) + λ2 ( d + d2 ) = λ1 ( d + d1 ) + λ2 ( d + d2 ) [Using (1) and (2)]
which is true for all values of λ1 and λ2 .
Hence the result.

Problem 8: Prove that all the spheres, that can be drawn through the origin and each set of
points where the plane parallel to the plane x / a + y / b + z / c = 0 cut the co-ordinates axes,
from a system of spheres which are cut orthogonally by the sphere
x 2 + y 2 + z 2 + 2ux + 2vy + 2wz = 0 , if au + bv + cw = 0.
G-160

x y z
Solution: A plane parallel to + + = 1 is
a b c
x y z
+ + = λ. …(1)
a b c
This meets co-ordinate axes in A ( aλ , 0 , 0 ), B(0 , bλ , 0 ) and C (0 , 0 , c λ ).
∴ Equation of sphere OABC is
x2 + y2 + z 2 − aλx − bλy − cλz = 0. …(2)
This sphere (2) will cut the given sphere orthogonally if
−aλ u − bλ v − c λ w = 0
or au + bv + cw = 0.

Problem 9: Find the angle of intersection of the spheres


x 2 + y 2 + z 2 − 2 x − 4 y − 6z + 10 = 0
and x 2 + y 2 + z 2 − 6 x − 2 y + 2 z + 2 = 0.
Solution: The equations of the given spheres are
x 2 + y 2 + z 2 − 2 x − 4 y − 6z + 10 = 0, …(1)
2 2 2
and x + y + z − 6 x − 2 y + 2z + 2 = 0. …(2)
The centre of (1) is C1 (1, 2 , 3), its radius r1 = √ (12 + 22 + 32 − 10 ) = 2.

The centre of (2) is C2 (3, 1, − 1), its radius r2 = √ {32 + 12 + ( −1)2 − 2} = 3.


Now C1 C2 = the distance between the centres C1 and C2
= {(3 − 1)2 + (1 − 2)2 + ( −1 − 3)2} = 21.
Let θ be the angle of intersection of the spheres (1) and (2), then
r12 + r22 − ( C1C2 )2 (2)2 + (3)2 − 21 −8 2
cos θ = = = =−
2r1 r2 2.2.3 12 3
2
= (taking the acute angle).
3
θ = cos −1   ⋅
2

 3

Comprehensive Problems 5
Problem 1: Prove that the members of the co-axial system intersect one another, touch one
another, or do not intersect one another according as d < , = or > 0.
Solution: Let the co-axial system of spheres be given by
x2 + y2 + z 2 + 2ux + d = 0 . …(1)
where d is an absolute constant and u is a parameter.
The common radical plane of the system (1) is x = 0.
The radical plane x = 0 intersects the sphere (1) in a circle given by
x2 + y2 + z 2 + 2ux + d = 0, x = 0
G-161

or given by y2 + z 2 = − d, x = 0. …(2)
The equations (2) represent a circle of radius √ ( − d ) in the yz-plane.
Now the following three cases arise :
Case I: When d < 0 i.e., d is –ve.
In this case ( −d ) is +ve and the radius √ ( − d ) is real i. e., the circle of intersection is real,
and hence the two spheres of the system intersect.
Case II: When d = 0. The radius √ ( − d ) of the circle reduces to zero i. e., the circle
reduces to a point circle and hence the two spheres of the system touch.
Case III: When d > 0 i.e., d is +ve. In this case the radius √ ( − d ) is imaginary i. e.,
the circle of intersection is imaginary, and hence the two spheres of the system do not
intersect.

Problem 2: Find the limiting points of the co-axial system of spheres determined by the spheres
x2 + y2 + z 2 + 3 x − 3 y + 6 = 0, x2 + y2 + z 2 − 6 y − 6z + 6 = 0.
Solution: The equations of the given spheres are
S ≡ x2 + y2 + z 2 + 3 x − 3 y + 6 = 0, …(1)
2 2 2
and S′ ≡ x + y + z − 6 y − 6z + 6 = 0. …(2)
The radical plane of the spheres (1) and (2) is given by
S − S ′ = 0 i. e., P ≡ 3 x + 3 y + 6z = 0
or P ≡ x + y + 2 z = 0.
∴ The equation of the co-axial system of spheres determined by the given spheres (1)
and (2) is given by S + λP = 0
or x2 + y2 + z 2 + 3 x − 3 y + 6 + λ ( x + y + 2z ) = 0
or x2 + y2 + z 2 + x (3 + λ ) − y (3 − λ ) + 2λz + 6 = 0. …(3)
−3 − λ 3 − λ
The centre of (3) is  , , − λ 
 2 2 
(3 + λ )2 ( λ − 3)2  1
and its radius =  + + ( λ )2 − 6 = √ (6λ2 − 6). …(4)
 4 4  2
We know that the limiting points are the centres of the spheres when radius = 0.
Hence equating the radius given by (4) to zero, we get
6 λ2 − 6 = 0 or λ2 = 1 or λ = ± 1.
Putting the values of λ in the co-ordinates of the centre of the sphere (3), the limiting
points are ( −2 , 1, − 1) and ( −1, 2 , 1).

Problem 3: Show that the locus of a point from which equal tangents may be drawn to the
spheres x2 + y2 + z 2 = 1, x2 + y2 + z 2 + 2 x − 2 y + 2z − 1 = 0,
x2 + y2 + z 2 − x + 4 y − 6z − 2 = 0 is the straight line
( x − 1) / 2 = ( y − 2) / 5 = ( z − 1) / 3.
G-162

Solution: Let P ( x , y, z ) be the moving point. Then the lengths of the tangents from
the point P to the given spheres are
√ ( x2 + y2 + z 2 − 1), √ ( x2 + y2 + z 2 + 2 x − 2 y + 2z − 1)
and √ ( x2 + y2 + z 2 − x + 4 y − 6z − 2).
By the given condition, these lengths are equal. Hence on squaring and equating, we get
x2 + y2 + z 2 − 1 = x2 + y2 + z 2 + 2 x − 2 y + 2z − 1
= x2 + y2 + z 2 − x + 4 y − 6z − 2 . …(1)
From the first two relations, we get
2 x − 2 y + 2 z = 0. …(2)
From the first and third, we get
− x + 4 y − 6z − 1 = 0. …(3)
The symmetrical form of (2) and (3) is
( x − 1) / 2 = ( y − 2) / 5 = ( z − 1) / 3. …(4)
Remark: By definition the line (4) is the radical line (i. e., radical axis) of the three given
spheres.

H ints to O bjective T ype Q uestions

Multiple Choice Questions


1. The centre of the sphere ( x + 3)2 + ( y − 4)2 + ( z + 5)2 = 16 is the point
( −3, 4, − 5).
2. The equation of the given sphere is
2
16  4 
x2 + y 2 + z 2 = =  [Form x2 + y 2 + z 2 = a2 ]
9  3
4
So, the radius of this sphere is ⋅
3
3. Obviously the four given points (1, 0, 0 ), (0, 2 , 0 ), (0, 0, 3) and (1, 2 , 3) are
non-coplanar. So one and only one sphere can be made to pass through them.
4. Radius of the sphere = √ [ u2 + v2 + w2 − d] .
For different values of d we get infinite spheres.
5. See Example 4.
6. See Problem 4 of Comprehensive Problems 1.
7. If the plane 2 x − 2 y + z − 9 = 0 touches the sphere x2 + y 2 + z 2 = r 2 , then
the length of the perpendicular drawn from the centre (0, 0, 0 ) of the sphere
to this plane is equal to the radius of the sphere
|2 . 0 − 2 . 0 + 0 − 9| 9
i. e., = r i. e., = r i. e., r = 3.
2 2 2
√ [2 + ( −2) + 1 ] 3
G-163

8. The plane l x + m y + n z − p = 0 touches the sphere x2 + y 2 + z 2 = r 2 if


the length of the perpendicular drawn from the centre (0, 0, 0 ) of the sphere
to the plane is equal to the radius of the sphere i. e., if
|l . 0 + m . 0 + n . 0 − p|
=r
√ [ l2 + m2 + n2 ]
p2
i. e., if = r2 i. e., if p 2 = r 2 ( l2 + m2 + n2 ).
l2 + m2 + n2
9. See Example 3.
The equation of the given sphere is
x2 + y 2 + z 2 − ax − by − cz = 0. …(1)
Comparing (1) with general equation of sphere i. e.,
x2 + y 2 + z 2 + 2ux + 2vy + 2wz + d = 0,
we have
a b c
u=− , v=− , w=− ⋅
2 2 2
Centre is ( − u, − v, − w ) i. e.,  , ,  ⋅
a b c

 2 2 2
10. The given sphere is
x2 + y 2 + z 2 − x − y − z = 0. …(1)
Comparing (1) with general equation of sphere i. e.,
x2 + y 2 + z 2 + 2ux + 2vy + 2wz + d = 0,
we have
1 1 1
u=−
, v = − , w = − , d = 0.
2 2 2
2 2 2 1 1 1 3
∴ Radius = ( u + v + w − d) = + + = ⋅
4 4 4 2
11. The given sphere is
x2 + y 2 + z 2 − x + z − 4 = 0. …(1)
Comparing (1) with general equation of sphere i. e.,
x2 + y 2 + z 2 + 2ux + 2vy + 2wz + d = 0,
we have
1 1
u=−, v = 0 , w = , d = − 4.
2 2
∴ Centre is ( − u, − v, − w ) i. e.,  , 0 , −  ⋅
1 1
2 2
12. See Problem 2 of Comprehensive Problems 1.
13. See article 3.
14. The given sphere is
x2 + y 2 + z 2 + 2 x + 4 y − 6z − 11 = 0. …(1)
Comparing (1) with general equation of sphere i. e.,
G-164

x2 + y 2 + z 2 + 2ux + 2vy + 2wz + d = 0,


we have
u = 1, v = 2, w = − 3, d = −11.
∴ Radius = ( u2 + v2 + w2 − d) = 1 + 4 + 9 + 11 = 25 = 5.
15. The given sphere is
3 x2 + 3 y 2 + 3z 2 + 2 x − 4 y + 5z + 13 = 0. …(1)
Comparing (1) with general equation of sphere i. e.,
x2 + y 2 + z 2 + 2ux + 2vy + 2wz + d = 0,
we have
1 2 5
u= , v = − , w = , d = 13.
3 3 6
∴ Centre is ( − u, − v, − w ) i. e.,  − , , −  ⋅
1 2 5
 3 3 6
16. See article 4.
17. See article 3.
18. Dividing throughout by 2, the equation of the given sphere can be written as
15
x2 + y2 + z 2 − x + 2 y − 3z − =0 …(1)
2
Comparing (1) with x2 + y 2 + z 2 + 2 ux + 2 vy + 2 wz + d = 0, we have
15
2 u = − 1, 2 v = 2 , 2 w = − 3, d = − ⋅
2
1 3 15
∴ u=− , v = 1, w = − , d = − ⋅
2 2 2
The coordinates of the centre of the given sphere are

( − u, − v, − w ) i. e.,  , − 1,  ⋅
1 3
2 2
19. See article 6.
20. See article 7.
21. See article 8.
22. The plane 2 x + 3 y − 4 z = 0 passes through the centre (0, 0, 0 ) of the sphere
x2 + y 2 + z 2 = 25.
So it cuts the given sphere in a great circle.
Hence, the radius of the given circle = the radius of the given sphere = 5.
23. See article 12, corollary 1.
24. See article 17.
25. See article 21.
26. See article 22.
27. See article 22.
G-165

28. The equation of the radical plane of the given spheres is


( x2 + y 2 + z 2 − 3 x + 4 y − 5 z − 7)
− ( x2 + y 2 + z 2 + 2 x + 7 y + 6 z − 8) = 0
i. e., −5 x − 3 y − 11 z + 1 = 0 i. e., 5 x + 3 y + 11 z − 1 = 0.
29. The length of the perpendicular drawn from the centre (0, 0, 0 ) of the sphere
25
x2 + y 2 + z 2 = to the plane 2 x − 3 y + 4z − 5 = 0 is
29
|2 . 0 − 3 . 0 + 4 . 0 − 5| 5
= =
√ [22 + ( −3)2 + 42 ] 29
5
which is equal to the radius of the sphere.
29
So the given plane touches the given sphere.
30. For the two given spheres, we have
5
u1 = − 1, v1 = 0, w1 = − , d1 = 4
2
and u2 = 0, v2 = 3, w2 = 0, d2 = − 4.
We have 2 u1 u2 + 2 v1 v2 + 2 w1 w2 = 0 + 0 + 0 = 0
and d1 + d2 = 4 − 4 = 0.
Thus 2 u1 u2 + 2 v1 v2 + 2 w1 w2 = d1 + d2 and so the given spheres cut
orthogonally.

Fill in the Blank(s)


1. The equation of a sphere whose centre is the origin (0, 0, 0 ) and radius a is
( x − 0 )2 + ( y − 0 )2 + ( z − 0 )2 = a2 i. e., x2 + y2 + z 2 = a2 .
2. The equation of the sphere whose centre is the point ( x1, y1, z1) and radius r
is ( x − x1)2 + ( y − y1)2 + ( z − z1)2 = r 2.
3. Obviously the point (0, 0, 0 ) satisfies the given equation of the sphere.
4. We know that in the equation of a sphere the coefficients of x2, y2, z 2 are all
equal.
5. See theorem of article 3.
6. See Example 3.
7. See article 6. Definition of a great circle of a sphere.
8. See article 12, corollary 3.
9. See article 24 Property I.

True or False
1. The given equation does not represent a sphere because in this equation the
coefficients of x2, y 2, z 2 are not all equal.

The coordinates of the centre of the given sphere are  , ,  ⋅


a b c
2.
 2 2 2
G-166

3. The plane 2 x − 3 y + 9z = 0 passes through the centre (0, 0, 0 ) of the sphere


x2 + y 2 + z 2 = 9.
So it cuts the sphere in a great circle whose centre is the same as is the centre
of the sphere.
Thus the centre of the given circle is the point (0, 0, 0 ).
4. The equation of any sphere passing through the circle
x2 + y 2 − a2 = 0, z = 0 is ( x2 + y 2 + z 2 − a2 ) + λ z = 0.
5. See article 18, Property IV.
6. The equation of the radical plane of the two given spheres is
( x2 + y 2 + z 2 + 2 u1 x + 2 v1 y + 2 w1 z + d1)
− ( x2 + y 2 + z 2 + 2 u2 x + 2 v2 y + 2 w2 z + d2 ) = 0
i. e., 2 ( u1 − u2 ) x + 2 ( v1 − v2 ) y + 2 ( w1 − w2 ) z + d1 − d2 = 0.
7. The equation of the sphere passing through (0, 0, 0 ), ( a, 0, 0 ), (0, b, 0 ), (0, 0, c )
is x2 + y 2 + z 2 − ax − by − cz = 0.
∴ The equation of the sphere passing through
(0, 0, 0 ), (1, 0, 0 ), (0, 1, 0 ), (0, 0, 1)
is x2 + y 2 + z 2 − x − y − z = 0.
8. See article 27, definition of coaxial system of spheres.
❍❍❍
G-167

Chapter-7
The Cone

Comprehensive Problems 1
Problem 1 : (i) Find the equation of the cone with vertex at (0, 0, 0 ) and passing through the
circle given by x + y2 + z 2 + x − 2 y + 3z − 4 = 0, x − y + z = 2.
2
(Meerut 2010)
(ii) Find the equation of the cone with vertex at (0, 0, 0 ) and passing through the circle given by
x 2 + y 2 + z 2 + 4 x − 2 y + 3z − 9 = 0, x − y + z = 3 . (Meerut 2009B)
Solution: (i) The equations of the given circle are
x2 + y2 + z 2 + x − 2 y + 3z − 4 = 0, …(1)
and x− y + z =2
or ( x − y + z ) / 2 = 1. …(2)
Making (1) homogeneous with the help of (2), the equation of the required cone with
the vertex at the origin is given by
1 1
x2 + y2 + z 2 + ( x − 2 y + 3z ) ( x − y + z ) − 4 ⋅ ( x − y + z )2 = 0
2 4
or 2 ( x2 + y2 + z 2 ) + ( x2 − 3 xy + 4zx + 2 y2 − 5 yz + 3z 2 )
− 2 ( x2 + y2 + z 2 − 2 xy + 2zx − 2 yz ) = 0
or x2 + 2 y2 + 3z 2 + xy − yz = 0.
(ii) The equation of the given circle are
x2 + y2 + z 2 + 4 x − 2 y + 3z − 9 = 0 , …(1)
and x− y + z =3
x− y+ z
or = 1. …(2)
3
Making (1) homogeneous with the help of (2), the equation of the required cone with
the vertex at the origin is given by
2
x2 + y2 + z 2 + ( x − y + z )(4 x − 2 y + 3z ) − 9  ( x − y + z ) = 0
1 1
3 3 
or 3( x2 + y2 + z 2 ) + (4 x2 − 2 xy + 3 xz − 4 xy + 2 y2 − 3 yz + 4 xz
−2zy + 3z 2 ) − ( x2 + y2 + z 2 − 2 xy + 2 xz − 2 yz ) = 0
or 6 x2 + 4 y2 + 5z 2 − 4 xy + 5 xz − 3 yz = 0.
G-168

Problem 2: Find the equation of the cone whose vertex is (0, 0, 0 ) and which passes through the
curve of intersection of the plane lx + my + nz = p and the surface ax2 + by2 + cz 2 = 1.
(Kanpur 2005; Rohilkhand 13)
Solution: The equations of the given curve are
ax2 + by2 + cz 2 = 1, …(1)
and ( lx + my + nz ) / p = 1. …(2)
Making (1) homogeneous with help of (2), the equation of the required cone with the
vertex at the origin is
( ax2 + by2 + cz 2 ) = {( lx + my + nz ) / p}2
or p2 ( ax2 + by2 + cz 2 ) = ( lx + my + nz )2 .

Problem 3: Prove that the equation of the cone whose vertex is the origin and base the curve
z = k, f ( x, y ) = 0 is f ( xk / z, yk / z ) = 0(Agra
. 2006; Purvanchal 13; Kumaun 13)
Solution: Let x /l = y /m = z /n …(1)
be a generator of the cone.
Since it meets the base curve z = k, f ( x, y ) = 0, therefore
x y k l m
= = , so that x = k, y = k.
l m n n n
Putting these values of x, y in f ( x, y ) = 0, we get
f  k, k  = 0.
l m
…(2)
n n 
x y z
Eliminating l, m, n between (1) and (2), the locus of = = is the cone
l m n
f  k, k  = 0.
x y
z z 
Alternate Solution: We know that the equation of a cone whose vertex is at the origin
is homogeneous in x , y and z. From z = k, we have k / z = 1. Therefore, making
f ( x, y ) = 0 homogeneous with the help of z = k, the equation of the required cone is
f ( xk / z, yk / z ) = 0.

Problem 4: Find the equation to the cone with the vertex at the origin and which passes through
the curve x2 + y2 + z 2 + x − 2 y + z − 4 = 0, x2 + y2 + z 2 + 2 x − 3 y + 4z − 5 = 0.
Solution: The equations of the guiding curve are
x2 + y2 + z 2 + x − 2 y + 3z − 4 = 0, …(1)
2 2 2
and x + y + z + 2 x − 3 y + 4z − 5 = 0. …(2)
Subtracting (2) from (1), we get
−x + y − z + 1 = 0
or x − y + z = 1. …(3)
Making (1) homogeneous with the help of (3), the required equation of the cone with
the vertex at the origin is
x2 + y2 + z 2 + ( x − 2 y + 3z ) ( x − y + z ) − 4 ( x − y + z )2 = 0
G-169

or 2 x2 + y2 − 5 xy − 3 yz + 4zx = 0. …(4)
Note: We can also make the equation (2) homogeneous with the help of (3) and then
also we shall get the equation (4) as the equation of the required cone.

Problem 5: (i) Find the equation of the cone with the vertex at the origin and which passes
x2 y2 z2 x2 y2
through the curve + + = 1, + = 2z .
a2 b2 c2 α2 β2
(ii) Find the equation of the cone with the vertex at the origin and which passes through the curve
ax2 + by2 + cz 2 = 1, α x2 + β y2 = 2z .
(Avadh 2006, 08; Meerut 07B; Kumaun 07; Rohilkhand 13)
Solution: (i) The equations of the guiding curve are
x2 y2 z2 x2 y2
+ + =1 and + = 2z .
a2 b2 c2 α2 β2
Making these equations homogeneous with the help of a new variable t [See working
rule given in article 2], we get
x2 y2 z2
+ + = t2 , …(1)
2 2 2
a b c
x2 y2
and + = 2z t. …(2)
2
α β2
Eliminating ‘t ’ between (1) and (2), we get
2
x2 y2 z2  x2 / α2 + y2 / β2 
+ + = 
a2 b2 c2  2z 
2
 x2 y2 z 2   x2 y2 
or 4z 2  + +  =  +  ⋅
2 2 2 2
a b c  α β2 
This is the required equation of the cone with the vertex at the origin.
(ii) Let the question of the curve be written in the homogeneous form as
ax2 + by2 + cz 2 = t2 …(1)
2 2
and α x + βy = 2zt. …(2)
Here a new variable t has been introduced to make them homogeneous.
αx2 + βy2
From (2), t= ⋅
2z
Putting this value on (1), we get the required cone as
( ax2 + by2 + cz 2 )4z 2 = (αx2 + βy2 )2 .

Problem 6: Show that a cone can be found to contain any two sets of three mutually
perpendicular concurrent lines as generators.
Solution: Take one set of mutually perpendicular lines as axes of co-ordinate and the
other three lines with direction cosine l1 , m1 , n1 ; l2 , m2 , n2 ; and l3 , m3 , n3 .
G-170

Now the equation of the cone through the axes is


fyz + gzx + hxy = 0. …(1)
If it passes through line whose d.c.’s are l1 , m1 , n1 ; l2 , m2 , n2 then
fm1m1 + gn1l1 + h1m1 = 0 …(2)
and fm2 n2 + gn2 l2 + h l2 m2 = 0. …(3)
Again adding (2) and (3), we get
f ( m1n1 + m2 n2 ) + g ( n1l1 + n2 l2 ) + h( l1 + m1 + l2 + m2 ) = 0. …(4)
When the other three mutually perpendicular lines are taken as axes, the d.c.’s of the
axes of co-ordinates become l1 , l2 , l3 ; m1 , m2 , m3 ; n1 , n2 , n3 which being also mutually
perpendicular.
We have m1n1 + m2 n2 + m3 n 3 = 0
or m1 + n1 + m2 n2 = − m3 n3 etc.
∴ (4) becomes
f ( − m3 n3 ) + g ( − n3 l3 ) + h( − l3 m3 ) = 0
i.e., f m3 n3 + g n3 l3 + h l3 m3 = 0.
This shows that the cone also passes through the line whose d.c.’s are l3 , m3 , n3 . This
proves the result.

Problem 7: Show that the lines drawn through the point (α, β, γ ) whose direction cosines
satisfy al + bm2 + cn2 = 0 generate the cone
2

a ( x − α )2 + b ( y − β )2 + c ( z − γ )2 = 0.
Solution: The equations of any line through the point (α, β, γ ) are
x−α y −β z − γ
= = ⋅ …(1)
l m n
Its direction cosines l, m, n satisfy the relation
al2 + bm2 + cn2 = 0. …(2)
Eliminating l, m, n between the equations (1) and (2), the locus of the line (1) i. e., the
equation of the required cone is given by
a ( x − α )2 + b ( y − β )2 + c ( z − γ )2 = 0.

Problem 8: Find the equation of the cone with vertex at the origin and direction cosines of its
generators satisfy the relation l2 + 2m2 − 3n2 = 0.
Solution: The equations of any line through the origin are
x y z
= = ⋅ …(1)
l m n
Its direction cosines satisfy the relation
l2 + 2m2 − 3n2 = 0. …(2)
Eliminating l, m, n between (1) and (2), the equation of the required cone is given by
x2 + 2 y2 − 3z 2 = 0.
G-171

Comprehensive Problems 2
Problem 1: (i) Find the equation of the cone, whose vertex is (α, β, γ ) and base
2 2
ax + by = 1, z = 0.
(ii) Prove that the equation of a cone whose vertex is the point (α, β, γ ) and base curve as
z 2 = 4by, x = 0 is ( γ x − α z )2 = 4b (α − x ) (α y − β x ). (Kanpur 2009,11)

(iii) Find the equation to the cone whose vertex is (α, β, γ ) and base y 2 = 4a x , z = 0.
(Meerut 2007B; Purvanchal 10)
(iv) Find the equation of the cone whose vertex is (1, 2, − 2) and guiding curve is the parabola
y 2 = 4ax, z = 0. (Kanpur 2007)
Solution: (i) We are to find the equation of a cone with the vertex at (α , β , γ ) and base
curve as
ax2 + by2 = 1, z = 0. …(1)
The equation of any line through the point (α , β , γ ) are
x−α y −β z − γ
= = ⋅ …(2)
l m n
The line (2) meets the plane z = 0 at the point given by
x−α y −β 0 − γ
= =
l m n
 lr mγ 
i.e., at the point  α − , β − , 0 .
 n n 
This point will lie on the curve (1), if
2 2
lγ mγ 
a  α −  + b  β −  . …(3)
 n  n 
1 x−α m y −β
Now from (2), = and = ⋅
n z−γ n z−γ
Putting these value in (3), the required equation of the cone is given by
2 2
 γ γ
 x − α     y − β  
a α −  
  + b β −    =1
z − γ  z−γ 
   
or a(αz − αγ − xγ + 2γ )2 + b (β z − βγ − yγ + βγ )4 = ( z − r )2
or a(αz − xγ )2 + b (βz − yγ )2 = ( z − γ )2 .

(ii) Clearly in the given problem, we are to find the equation of a cone with the vertex
at (α , β , γ ) and base are
z 2 = 4by , x = 0. …(1)
The equation of any line through the point (α , β , γ ) are
x−α y −β z − γ
= = ⋅ …(2)
l m n
G-172

The line (2) meets the plane x = 0 cut the point given
0 −α y −β z − γ
= =
l m n
 m n 
i.e., at the point  0 , β − α , γ − α  ⋅
 l l 
This point will lie on the curve (1) if,
2
 γ − nα  = 4b  β − m α  ⋅
    …(3)
 l   l 
n z−γ m y −β
Now from (2), = and = ⋅
l x−α l x−α
Putting these value in (3), the required equation of the cone is given by
2
 z − γ    y − β 
γ −   α  = 4b β −   α
  x − α    x −α 
or {α ( x − 2) − ( z − γ )α }2 = 4b {β ( x − α ) − ( y − β )α}( x − α )
or ( γx − α z )2 = 4b (αy − βx ) (α − x ).
(iii) We are to find the equation of a cone with the vertex at (α , β , γ )and base curve as
y2 = 4ax, z = 0. …(1)
The equation of any line through the point (α , β , γ ) are
x−α y −β z − γ
= = ⋅ …(2)
l m n
The line (2) meets the plane z = 0 at the point given by
x−α y−b 0−γ
= =
l m n
 lγ mγ 
i.e., at the point  α − , β − , 0 .
 n n 
This point will lie on the curve (1) is
2
 β − mγ  = 4a  α − lγ  .
    …(3)
 n   n
l x−2 m y −β
Now from (2), = and = ⋅
n z−γ n z−γ
Putting these value in (3), the required equation of the cone is given by
2
  y − β    x − α 
β −   γ  = 4a α −   γ
 z−γ   z − γ 
or {β ( z − γ ) − ( y − β ) γ }2 = 4a {α (3 − γ ) − ( x − α ) γ } ( z − γ )
or (βz − γ y )2 = 4a(αz − γ x )( z − γ ).

(iv) We are to find the equation of a cone with the vertex at (1, 2, − 2)and base curve as
y2 = 4ax, z = 0. …(1)
The eq. of any line through the point (1, 2, − 2) are
G-173

x −1 y −2 z + 2
= = ⋅ …(2)
l m n
The line (2) meets the plane z = 0 at the point given by
x −1 y −2 0 + 2
= =
l m n
i.e., at the point 1 + 2, 2 + 2, 0  .
 l m
 n n 
This point will lie on the curve (1), if
2
 2 + m 2 = 4a 1 + 2l  .
    …(3)
 n   n
m y −2 l x −1
From eq. (2), = and = ⋅
n z+2 n z+2
Putting these value in (3), the required equation of the cone is given by
2
  y − 2    x −1
2 +   2 = 4a 1 + 2  
  z + 2    x + 2 
or {2( z + 2) + 2( y − 2)}2 = 4a {( z + 2) + 2( x − 1)} ( z + 2)
or (2z + 2 y )2 = 4a ( z + 2 x )( z + 2).

Problem 2: Find the equation of a cone whose vertex is the point (α, β, γ ) and whose generating
x2 y2
lines pass through the conic + = 1, z = 0.
a2 b2 (Purvanchal 2008, 09; Kashi 14)
Solution: The equations of the given conic are
x2 y2
+ = 1, z = 0. …(1)
a2 b2
The equations of any line through the vertex (α, β, γ ) are
x−α y −β z − γ
= = ⋅ …(2)
l m n
The line (2) meets the plane z = 0 at the point given by
x−α y −β 0 − γ
= =
l m n
 lγ mγ 
i. e., at the point  α − , β − , 0 ⋅
 n n 
If this point lies on the conic (1), then
2 2
1  lγ  1  mγ 
α −  + 2 β −
2 
 = 1. …(3)
a n  b  n 
Eliminating l, m, n between the equations (2) and (3), the required equation of the cone
is given by
2 2
1   x − α  1   y − β 
α −  γ  + 2 β −   γ = 1
2   z − γ  z−γ
a   b  
or b2 (αz − γx )2 + a2 ( βz − γy )2 = a2 b2 ( z − γ )2 . …(4)
G-174

Problem 3: The section of a cone whose vertex is P and the base curve the ellipse
x2 y2
+ = 1, z = 0 by the plane x = 0 is a rectangular hyperbola. Show that the locus of P is
2
a b2
x2 y2 + z 2
+ = 1.
a2 b2 (Avadh 2007)
Solution: Let (α, β, γ ) be the co-ordinates of the vertex P of the cone. The equation of
the cone whose vertex is the point (α, β, γ ) and base curve the ellipse
x2 y2
+ = 1, z = 0 is
2
a b2
b2 (αz − γx )2 + a2 ( βz − γy )2 = a2 b2 ( z − γ )2 . …(1)
[See equation (4) of Problem 2 above. Derive it here.]
The section of the cone (1) by the plane x = 0 is obtained by putting x = 0 in the
equation (1) and is given by
b2α2 z 2 + a2 ( βz − γy )2 = a2 b2 ( z − γ )2 , x = 0. …(2)
If the equations (2) represent a rectangular hyperbola in the x = 0 plane, then in the first
of the equations (2) we should have
the coefficient of y2 + the coefficient of z 2 = 0
i. e., ( a2 γ 2 ) + ( b2α2 + a2β2 − a2 b2 ) = 0
or b2α2 + a2 ( β2 + γ 2 ) = a2 b2 .
∴ The locus of the point P (α, β, γ ) is
b2 x2 + a2 ( y2 + z 2 ) = a2 b2
or x2 / a2 + ( y2 + z 2 ) / b2 = 1.
Problem 4: Find the equation of the cone whose vertex is (1, 2, 3) and guiding curve is the circle
2 2 2
x + y + z = 4, x + y + z = 1. (Garhwal 2002)
Solution: The equations of the guiding circle are given by
x2 + y2 + z 2 = 4 …(1)
and x + y + z = 1. …(2)
The equations of any line (generator) through (1, 2, 3) are
x −1 y −2 z −3 x+ y + z −6
= = = ⋅ …(3)
l m n l + m+ n
(Note)
The line (3) meets the plane x + y + z = 1 at the point given by
x −1 y −2 z −3 1−6 −5
= = = =
l m n l + m+ n l + m+ n
 5l 5m 5n 
i. e., at the point 1 − ,2 − ,3 − 
 l + m+ n l + m+ n l + m + n
G-175

 m + n − 4l 2l + 2n − 3m 3l + 3m − 2n 
or  , , ⋅
 l + m+ n l + m+ n l + m+ n 
This point lies on the base circle given by the equations (1) and (2), hence this point will
satisfy the equation (1) and so we have
( m + n − 4l )2 + (2l + 2 n − 3m)2 + (3l + 3m − 2n)2 = 4 ( l + m + n)2 .
…(4)
Eliminating l, m, n between (3) and (4), we get the locus of the line (3) i. e., the equation of
the required cone. We observe that the equation (4) is homogeneous in l, m, n, so to
eliminate l, m, n we can put even the proportionate values of l, m, n from (3) in (4). Then
the required equation of the cone is given by
{( y − 2) + ( z − 3) − 4 ( x − 1)}2 + {2 ( x − 1) + 2 ( z − 3) − 3 ( y − 2)}2
+ {3 ( x − 1) + 3 ( y − 2) − 2 ( z − 3)}2 = 4 {( x − 1) + ( y − 2) + ( z − 3)}2
or ( y + z − 4 x − 1)2 + (2 x + 2z − 3 y − 2)2 + (3 x + 3 y − 2z − 3)2
= 4 ( x + y + z − 6)2
2 2 2
or 5 x + 3 y + z − 6 yz − 4zx − 2 xy + 6 x + 8 y + 10 z − 26 = 0.

Problem 5: Find the equation of the cone with vertex (5, 4, 3) and with
2 2
3 x + 2 y = 6, y + z = 0 as base. (Avadh 2010)
Solution: The equations of the base curve are
3 x2 + 2 y2 = 6 and y + z = 0. …(1)
The equations of any line (generator) through (5, 4, 3) are
x−5 y −4 z −3 y + z −7
= = = ⋅ …(2)
l m n m+ n
The line (2) meets the plane y + z = 0 at the point given by
x−5 y −4 z −3 0 −7 −7
= = = =
l m n m+ n m+ n
i. e., at the point
 7l 7m 7n 
5 − ,4 − ,3 − 
 m+ n m+ n m + n
 5m + 5n − 7l 4n − 3m 3m − 4n 
or  , , ⋅
 m+ n m+ n m+ n 
If this point lies on the base curve given by (1), then this point will satisfy 3 x2 + 2 y2 = 6
and hence we have
3 (5m + 5n − 7l )2 + 2 (4n − 3m)2 = 6 ( m + n)2 . …(3)
Putting proportionate values of l, m, n from (2) in (3) and thus eliminating l, m, n
between the equations (2) and (3), the required equation of the cone is given by
3 {5 ( y − 4) + 5 ( z − 3) − 7 ( x − 5)}2 + 2 {4 ( z − 3) − 3 ( y − 4)}2
= 6 {( y − 4) + ( z − 3)}2
G-176

or 3 (5 y + 5z − 7 x )2 + 2 (4z − 3 y )2 = 6 ( y + z − 7)2
or 147 x2 + 87 y2 + 101 z 2 + 90 yz − 210 zx − 210 xy + 84 y + 84z − 294 = 0.

Problem 6: Two cones with a common vertex pass through the curves
z 2 = 4ax, y = 0 and z 2 = 4by, x = 0.
The plane z = 0 meets them in two conics which intersect in four noncyclic points. Show that the
vertex lies on the surface z 2 ( x / a + y / b ) = 4 ( x2 + y2 ). (Avadh 2010)
Solution: Let (α, β, γ ) be the common vertex. Proceeding as in Example 4, the
equations of two cones are respectively given by
( γ y − βz )2 = 4a (α y − β x ) ( y − β ) …(1)
2
( γ x − αz ) = 4b ( β x − α y ) ( x − α ). …(2)
The plane z = 0 meets the cones (1) and (2) in two conics whose equations in xy-plane
are respectively given by
S1 = γ 2 y2 − 4a (αy − βx ) ( y − β ), z = 0 …(3)
2 2
S2 = γ x − 4b ( βx − αy ) ( x − α ), z = 0. …(4)
Any curve through the intersection of (3) and (4) is given by
S1 + λS2 = 0, z = 0
i. e., γ 2 y2 − 4a (αy − βx ) ( y − β ) + λ [ γ 2 x2 − 4b ( βx − αy ) ( x − α )] = 0,
z = 0.
If the last equation represents a circle, then we must have :
The coefficient of x2 = the coefficient of y2 and the coefficient of xy = 0
i. e., λ ( γ 2 − 4bβ ) = γ 2 − 4aα …(5)
and 4aβ + 4bαλ = 0 i. e., λ = − ( aβ ) / ( bα ). …(6)
Eliminating λ between (5) and (6), we have
γ 2 ( bα + aβ ) = 4ab (α2 + β2 ).
∴ The locus of (α, β, γ ) is z 2 ( x / a + y / b ) = 4 ( x2 + y2 ).

Problem 7: Prove that the line x = pz + q, y = r z + s, intersects the conic


a x 2 + by 2 = 1, z = 0 if aq 2 + bs 2 = 1.
Hence show that the co-ordinates of any point on a line which intersects the conic and passes
through the point (α, β, γ ) satisfy the equation a ( γ x − α z )2 + b ( γ y − β z )2 = ( z − γ )2 .
Solution: The given line
x = pz + q , y = rz + s …(1)
meet the plane z = 0 at x = q , y = s i.e. in the point ( q , s , 0 ).
If this point ( q , s , 0 ) lies on the given conic
ax2 + by2 = 1, z = 0 ...(2)
then we have aq2 + bs2 = 1. …(3)
which is the required condition. .
G-177

Again if the line (1) passes through (α , β , γ ), then


α = pγ + q, β = r γ + s. …(4)
From (1) and (4), we get
x γ − β z = γ (r z + s ) − z (r γ + s ) = s (γ − z )
and y γ − β z = γ ( r z + s ) − z ( r γ + s ) = s ( γ − z ).
γ x −αz yγ − βz
These give q= , s= .
γ−z γ−z
Substituting these values in (3) we obtain the required locus
2 2
 γ x − αz   γ y − βz 
as a  + b  =1
 γ−z   γ−z 
or a( γ x − az )2 + b ( γy − βz )2 = ( z − γ )2 .

Problem 8: Prove that the equation


4 x2 − y2 + 2z 2 + 2 xy − 3 yz + 12 x − 11 y + 6z + 4 = 0
represents a cone. Find the co-ordinates of its vertex. (Meerut 2005, 12; Agra 2005, 14)
Solution: Let
F ( x, y, z ) ≡ 4 x2 − y2 + 2z 2 + 2 xy − 3 yz + 12 x − 11 y + 6z + 4 = 0. …(1)
Now we introduce a new variable t and making (1) homogeneous with the help of t [See
working rule of article 6], we get
F ( x, y, z, t) ≡ 4 x2 − y2 + 2z 2 + 2 xy − 3 yz + 12 xt − 11 yt + 6zt + 4t2. …(2)
Differentiating (2) partially with respect to x, y, z and t respectively, we get
∂F / ∂x = 8 x + 2 y + 12t, ∂F / ∂y = − 2 y + 2 x − 3z − 11t 
 …(A)
∂F / ∂z = 4z − 3 y + 6t, ∂F / ∂t = 12 x − 11 y + 6z + 8t. 
Putting t = 1 in each of the relations in (A) and then equating them to zero, we get
8x + 2 y + 1 = 0 …(3)
2 x − 2 y − 3z − 11 = 0 …(4)
−3 y + 4z + 6 = 0 …(5)
12 x − 11 y + 6z + 8 = 0. …(6)
Now we shall find x, y, z by solving the equations (3), (4) and (5).
Eliminating x between (3) and (4), we get
5 y + 6z + 28 = 0. …(7)
Solving (5) and (7), we get
y = − 2, z = − 3.
Putting the value of y in (3), we get
x = −1.
Substituting these values i. e., x = − 1, y = − 2, z = − 3 in (6), we have
12 ( −1) − 11 ( −2) + 6 ( −3) + 8 = 0
or 0 =0
i. e., the equation (6) is satisfied. Hence the given equation (1) represents a cone with the
vertex at ( −1, − 2 , − 3).
G-178

Problem 9: Prove that the equation


x 2 − 2 y 2 + 3z 2 − 4 xy + 5 yz − 6zx + 8 x − 19 y − 2 z − 20 = 0
represents a cone. Show that the co-ordinates of its vertex are (1, − 2 , 3). (Meerut 2013B)
Solution: Making the given equation homogeneous by multiplying the proper power
of t, we get
f ( x , y , z , t) = x2 − 2 y2 + 3z 2 − 4 xy + 5 yz − 6zx + 8 xt − 19 y
−2zt − 20 t2 = 0. …(1)
Differentiating (1) partially w.r.t. x , y , z and t and put t = 1, we have
∂f
= 2 x − 4 y − 6z + 8t .
∂x
∂t
∴ At t = 1 = 0 ⇒ 2 x − 4 y − 6z + 8 = 0. …(2)
∂x
∂t
Similarly =0 ⇒ −4 y − 4 x + 5z − 19 = 0 …(3)
∂y
∂f
= 0 ⇒ 6z + 5 y − 6 x − 2 = 0 …(4)
∂z
∂f
and =0 ⇒ 8 x − 19 y − 2z − 40 = 0. …(5)
∂t
Solving (2), (3) and (4), we get
x = 1, y = − 2, z = 3
these values of x , y , z satisfy the equation(s).
Hence the given equation represent a cone and its vertex is (1, − 2, 3).

Comprehensive Problems 3
Problem 1: Find the equation of the cone reciprocal to the cone
f yz + gz x + hxy = 0.
Solution: The equation of the given cone is
f yz + gz x + h x y = 0. …(1)
Let the reciprocal cone of (1) be
Ax2 + By2 + Cz 2 + 2Fyz + 2Gzx + 2Hxy = 0. …(2)
Comparing the equation (1) with the equation
ax2 + by2 + cz 2 + 2 fyz + 2 gzx + 2hxy = 0, we have
1 1 1
a = 0, b = 0, c = 0, f = f , g = g and h = h.
2 2 2
2 1 2 1 2 1 2
∴ A = “bc − f ” ≡ − f ;B= − g ;C= − h ;
4 4 4
1 1 1 1 1
F = “ gh − af ” = g ⋅ h − 0 = gh ; G = hf ; H = fg.
2 2 4 4 4
Putting these values in (2), the equation of the cone reciprocal to (1) is given by
G-179

1 2 2 1 2 2 1 2 2 1 1 1
− f x − g y − h z + 2 ⋅ ghyz + 2 ⋅ hfzx + 2 ⋅ fgxy = 0
4 4 4 4 4 4
or f 2 x2 + g2 y2 + h2 z 2 − 2 ghyz − 2hfzx − 2 fgxy = 0. …(3)

Problem 2: Prove that the general equation to a cone which touches the co-ordinate planes is
a2 x2 + b2 y2 + c2 z 2 − 2bcyz − 2cazx − 2abxy = 0. (Garhwal 2002)
Solution: The co-ordinate axes are the normals to the co-ordinate planes and hence
the cone which touches the three co-ordinate planes is the reciprocal cone to the cone
which contains the three co-ordinate axes.
The equation of the cone which contains the three co-ordinate axes is given by
fyz + gzx + hxy = 0. …(1)
[See article 4]
Now rewriting the steps of Problem 1 above, the cone reciprocal to (1) is given by
f 2 x2 + g2 y2 + h2 z 2 − 2 ghyz − 2hfzx − 2 fgxy = 0, …(2)
which is of the form
a2 x2 + b2 y2 + c2 z 2 − 2bcyz − 2 cazx − 2abxy = 0.

Problem 3: Show that the tangent planes to the cone fyz + gzx + hxy = 0 are perpendicular to
the generators of the cone f 2 x2 + g2 y2 + h2 z 2 − 2 ghyz − 2hfzx − 2 fgxy = 0.
Solution: In view of the definition of the reciprocal cone, to prove the given problem it
is sufficient to prove that the two cones are reciprocal to each other.
Rewriting the steps of Problem 1 above, the cone reciprocal to
fyz + gzx + hxy = 0 is
f 2 x2 + g2 y2 + h2 z 2 − 2 ghyz − 2hfzx − 2 fgxy = 0.

Problem 4: Prove that the perpendiculars drawn from the origin to the tangent planes to the
x2 y2 z2
cone ax2 + by2 + cz 2 = 0 lie on the cone + + = 0.
a b c
(Kanpur 2006; Rohilkhand 08; Kashi 13)
Solution: By the definition of reciprocal cone (article 9), we know that the locus of the
normals (i. e., perpendiculars) drawn from the vertex (i. e., the origin in the present case)
to the tangent planes of a given cone is called the reciprocal cone. Therefore, in the
present case, we are required to find the cone reciprocal to the cone ax2 + by2 + cz 2 = 0.
Hence it is the same problem as Example 7.

Problem 5: Prove that the perpendiculars drawn from the origin to the tangent planes to the
cone 3 x2 + 4 y2 + 5z 2 + 2 yz + 4zx + 6 xy = 0 lie on the cone
19 x2 + 11 y2 + 3z 2 + 6 yz − 10 zx − 26 xy = 0.
Solution: In accordance with the definition of the reciprocal cone, in the present
problem it is required to prove that the two given cones are reciprocal to each other.
The equation of the first cone is given by
G-180

3 x2 + 4 y2 + 5z 2 + 2 yz + 4zx + 6 xy = 0. …(1)
Let the cone reciprocal to (1) be
Ax2 + By2 + Cz 2 + 2Fyz + 2Gzx + 2Hxy = 0. …(2)
For the cone (1), we have
a = 3, b = 4, c = 5, f = 1, g = 2, h = 3.
∴ A = bc − f 2 = 4 . 5 − (1 )2 = 19, B = ca − g2 = 15 − 4 = 11,
C = ab − h2 = 12 − 9 = 3, F = gh − af = 6 − 3 = 3,
G = hf − bg = 3 − 8 = − 5, H = fg − ch = 2 − 15 = − 13.
Putting these values in the equation (2), the cone reciprocal to (1) is given by
19 x2 + 11 y2 + 3z 2 + 6 yz − 10 zx − 26 xy = 0
which is the equation of the second given cone.

Comprehensive Problems 4
Problem 1: Find the angle between the lines of section of the plane 3 x + y + 5z = 0 and the
cone 6 yz − 2 zx + 5 xy = 0.
Solution: The equation of the given plane is
4 x + y + 5z = 0. …(1)
The equation of the given cone is
6 yz − 2zx + 5 xy = 0. …(2)
Let the equations of a line of section of the cone (2) by the plane (1) be given by
x / l = y / m = z / n. …(3)
Then 3l + m + 5n = 0 …(4)
and 6mn − 2nl + 5lm = 0. …(5)
Eliminating m between (4) and (5), we get
6 ( −3l − 5n) n − 2nl + 5l ( −3l − 5n) = 0
or 30 n2 + 45nl + 15l2 = 0
or 2n2 + 3nl + l2 = 0 or (2n + l ) ( n + l ) = 0.
∴ 2n + l = 0 or n + l = 0.
When 2n + l = 0 i. e., l = − 2n then from (4), m = n.
∴ l /( −2) = m /1 = n /1.
Again when n + l = 0 i. e., l = − n then from (4),
m = − 2n.
∴ l /1 = m / 2 = n /( −1).
Hence the equations of the lines of section are
x y z x y z
= = and = = ⋅
−2 1 1 1 2 −1
Let θ be the angle between these lines of section.
( −2) . 1 + (1) . (2) + (1) . ( −1) 1
Then cos θ = =− ⋅
√ {( −2)2 + (1)2 + (1)2} √ {(1)2 + (2)2 + ( −1)2} 6
G-181

∴ The acute angle θ between the lines of section is given by


θ = cos −1 (1 / 6).

Problem 2: Show that the condition that the plane ux + vy + wz = 0 may cut the cone
ax + by + cz 2 = 0 in perpendicular generators is
2 2

( b + c ) u2 + ( c + a) v2 + ( a + b ) w2 = 0.
Solution: Let the equations of a line of section of the cone ax2 + by2 + cz 2 = 0 by
the plane ux + vy + wz = 0 be given by
x / l = y / m = z / n. …(1)
Then al2 + bm2 + cn2 = 0 …(2)
and ul + vm + wn = 0. …(3)
Eliminating n between (2) and (3), we get
al2 + bm2 + c {( − ul + vm) / w }2 = 0
or ( aw2 + cu2 ) l2 + 2cuvlm + ( bw2 + cv2 ) m2 = 0
or ( aw2 + cu2 ) ( l / m)2 + 2cuv ( l / m) + ( bw2 + cv2 ) = 0. …(4)
This is a quadratic equation in l / m and hence it shows that the plane ux + vy + wz = 0
cuts the given cone in two lines (or generators). Let l1, m1, n1 and l2, m2, n2 be the d.c.’s of
these lines. Then l1 / m1 and l2 / m2 are the roots of the equation (4). The product of the
roots of the equation (4) is given by
l1 l2 bw2 + cv2
⋅ = ⋅
m1 m2 aw2 + cu2
l1l2 m1m2 n1n2
∴ = = ,
2 2 2 2
bw + cv cu + aw av + bu2
2

writing the third fraction by symmetry.


Now the generators (i. e., the lines) will be perpendicular if
l1l2 + m1m2 + n1n2 = 0
i. e., if ( bw2 + cv2 ) + ( cu2 + aw2 ) + ( av2 + bu2 ) = 0
i. e., if ( b + c ) u2 + ( c + a) v2 + ( a + b ) w2 = 0.

Problem 3: Prove that the angle between the lines given by


1
x + y + z = 0, ayz + bzx + cxy = 0 is π
2
1
if a + b + c = 0 but π if 1 / a + 1 / b + 1 / c = 0.
3
(Bundelkhand 2007, 08)
Solution: Let the equations of a line of section of the cone ayz + bzx + cxy = 0 by the
plane x + y + z = 0 be given by
x / l = y / m = z / n. …(1)
Then amn + bnl + clm = 0 …(2)
and l + m + n = 0. …(3)
G-182

Eliminating n between the equations (2) and (3), we get


am ( − l − m) + bl ( − m − l ) + clm = 0
or bl2 + ( a + b − c ) lm + am2 = 0
or b ( l / m)2 + ( a + b − c ) ( l / m) + a = 0. …(4)
This is a quadratic equation in l / m and hence it shows that the given plane cuts the
given cone in two lines (i. e., generators). Let l1, m1, n1 and l2 , m2 , n2 be the d.c.’s of these
two lines, so that we have from (4), the product of the roots
l l a
= 1 ⋅ 2 = ⋅
m1 m2 b
l1l2 m1m2 n1n2
∴ = = (by symmetry) = λ (say). …(5)
a b c
1
Case I: The angle between the lines of section is π .
2
In this case, we have
l1l2 + m1m2 + n1n2 = 0 or aλ + bλ + cλ = 0, using (5)
or a + b + c = 0. [ ∵ λ ≠ 0]
1
Case II: The angle between the lines of section is π .
3
The sum of the roots of the equation (4) is given by
l1 l a+ b−c
+ 2 =− ⋅
m1 m2 c
l1m2 + l2 m1 m1m2
∴ = = λ [from (5)]. …(6)
c−a−b b
∴ ( l1m2 − l2 m1 )2 = ( l1m2 + l2 m1 )2 − 4l1l2 m1m2
= λ2 [( c − a − b )2 − 4 . ab ], using (6) and (5)
= λ2 [ a2 + b2 + c 2 − 2bc − 2ca − 2ab ].
By symmetry, we have
( m1n2 − m2 n1 )2 = ( n1l2 − n2 l1 )2
= λ2 [ a2 + b2 + c 2 − 2bc − 2ca − 2ab ].
Σ ( l1m2 − l2 m1 )2
Now tan2 θ = ⋅
( l1l2 + m1m2 + n1n2 )2
Since the angle between the lines is π / 3, therefore we have
1 3λ2 ( a2 + b2 + c2 − 2bc − 2ca − 2 ab )
tan2 π=
3 λ2 ( a + b + c )2
or 3 ( a + b + c )2 = 3 ( a2 + b2 + c2 − 2bc − 2ca − 2ab )
or ( a2 + b2 + c2 + 2bc + 2ca + 2ab ) = a2 + b2 + c 2 − 2bc − 2ca − 2ab
or 4 ( bc + ca + ab ) = 0
or 1 / a + 1 / b + 1 / c = 0.
G-183

Problem 4: Find the angle between the lines of section of the plane 6 x − y − 2z = 0 and the
cone 108 x2 − 7 y2 − 20 z 2 = 0.
Solution: Let the equations of a line of section (i. e., generator) of the cone
108 x2 − 7 y2 − 20 z 2 = 0
by the plane 6 x − y − 2z = 0 be given by
x / l = y / m = z / n. …(1)
Then 108l2 − 7m2 − 20 n2 = 0 …(2)
and 6l − m − 2n = 0. …(3)
Eliminating m between (2) and (3), we get
108l2 − 7 (6l − 2n)2 − 20 n2 = 0 or 144l2 − 168 ln + 48n2 = 0
or 6l2 − 7ln + 2n2 = 0 or (3l − 2n) (2l − n) = 0
or 3l = 2n, 2l = n.
When 3l = 2n, from (3) we have m = 3 l.
l m n
∴ 3l = m = 2n or = = ⋅ …(4)
2 6 3
When 2l = n, from (3), we have m = 2l.
l m n
∴ 2l = m = n or = = ⋅ …(5)
1 2 2
The direction ratios of the two lines of section are given by the relations (4) and (5). If θ
be the angle between these two lines of section, then we have
l1l2 + m1m2 + n1n2
cos θ =
√ ( l12 + m12 + n12 ) √ ( l22 + m22 + n22 )
2 .1 + 6 . 2 + 3 . 2 20 20
= = = ⋅
2 2 2 2 2 2
√ {(2) + (6) + (3) } √ {(1) + (2) + (2) } 7 . 3 21
∴ θ = cos −1 (20 / 21).

Problem 5: If the plane 2 x − y + c z = 0 cuts the cone yz + zx + xy = 0 in perpendicular


lines, find the value of c.
Solution: Let the equations of a line of section of the cone yz + zx + xy = 0 by the
plane 2 x − y + cz = 0 be given by
x y z
= = ⋅ …(1)
l m n
Then mn + nl + lm = 0 …(2)
and 2l − m + cn = 0. …(3)
Eliminating m between the equations (2) and (3), we get
(2l + cn) n + nl + l (2l + cn) = 0 or 2l2 + (3 + c ) nl + cn2 = 0
or 2 ( l / n)2 + (3 + c ) ( l / n) + c = 0. …(4)
This is a quadratic equation in l / n and hence it shows that the given plane cuts the given
cone in two lines. Let l1 , m1, n1 and l2, m2 , n2 be the d.c.’s of these lines, so that we have
G-184

l l c
from (4) the product of the roots = 1 ⋅ 2 = , so that
n1 n2 2
l1l2 / c = n1 n2 / 2. …(5)
Now eliminating l between the equations (2) and (3), we get
mn + n {( m − cn) / 2} + {( m − cn) / 2} m = 0
or m2 + (3 − c ) mn − cn2 = 0
or ( m / n)2 + (3 − c ) ( m / n) − c = 0.
m m2
∴ The product of the roots = 1 ⋅ = − c, so that
n1 n2
m1 m2 /( −2c ) = n1 n2 / 2. …(6)
From the relations (5) and (6), we get
l1l2 m1 m2 n1 n2
= = ⋅ …(7)
c −2c 2
According to the given problem, if the lines of section are perpendicular, then we have
l1 l2 + m1 m2 + n1 n2 = 0
or c − 2c + 2 = 0 or c = 2.

Problem 6: Show that the planes which cut ax2 + by2 + cz 2 = 0 in perpendicular generators
x2 y2 z2
touch the cone + + = 0.
b+ c c+ a a+ b
Solution: The equation of the given cone is
ax2 + by2 + cz 2 = 0. …(1)
Let the equation of any plane through the vertex (0, 0, 0 ) of the cone (1) be
ux + vy + wz = 0. …(2)
Suppose the plane (2) cuts the cone (1) in perpendicular generators. Then proceeding as
in problem 2, we have
( b + c ) u2 + ( c + a) v2 + ( a + b ) w2 = 0. …(3)
From (3) we observe that the normal to the plane (2) through the origin i. e., the line
x / u = y / v = z / w lies on the cone
( b + c ) x2 + ( c + a) y2 + ( a + b ) z 2 = 0. …(4)
Now the plane (2) is a tangent plane to the cone which is reciprocal of the cone (4). The
equation of the cone reciprocal to the cone (4) is
x2 y2 z2
+ + = 0. …(5)
b+ c c+ a a+ b
[See Example 7]
Hence if the plane (2) cuts the cone (1) in perpendicular generators then it touches the
cone (5).
G-185

Comprehensive Problems 5
Problem 1: Show that the plane lx + my + nz = 0 cuts the cone
( b − c ) x2 + ( c − a) y2 + ( a − b ) z 2 + 2 fyz + 2 gzx + 2hxy = 0
in perpendicular lines if
( b − c ) l2 + ( c − a) m2 + ( a − b ) n2 + 2 fmn + 2 gnl + 2hlm = 0.
Solution: The equation of the given cone is
( b − c ) x2 + ( c − a) y2 + ( a − b ) z 2 + 2 fyz + 2 gzx + 2 hxy = 0. …(1)
2 2 2
Here we see that the sum of the coefficients of x , y and z in the equation of the given
cone
= ( b − c ) + ( c − a) + ( a − b ) = 0 .
Hence the given cone (1) has an infinite number of sets of three mutually perpendicular
generators. [See article 11]
Now if the plane lx + my + nz = 0 cuts the given cone (1) in two perpendicular lines (i. e.,
generators) then the third mutually perpendicular generator is the normal to the plane
lx + my + nz = 0 through the vertex (0, 0, 0 ) of the cone (1) i. e., the line x / l = y / m = z / n
is the third generator of the cone (1) and hence its d.r.’s l, m, n will satisfy the equation
(1) of the cone. Therefore, the required condition is given by
( b − c ) l2 + ( c − a) m2 + ( a − b ) n2 + 2 fmn + 2 gnl + 2hlm = 0.

Problem 2: Prove that the locus of the points from which three mutually perpendicular lines
can be drawn to intersect a given circle x2 + y2 = a2 , z = 0 is a surface of revolution.
Solution: Let the point whose locus is to be determined be P (α, β, γ ). Now first we
shall find the equation of the cone whose vertex is the point P (α, β, γ )and the base curve
is the circle
x2 + y2 = a2, z = 0. …(1)
The equations of any line through P (α, β, γ ) are
x−α y −β z − γ
= = ⋅ …(2)
l m n
lγ mγ
It meets the plane z = 0 at the point  α − , β − , 0  ⋅
 n n 
This point will lie on the circle (1) if
(α − lγ / n)2 + ( β − mγ / n)2 = a2 . …(3)
Eliminating l, m, n between (2) and (3), the equation of the cone with the vertex at
P (α, β, γ ) is
2 2
  x − α    y − β  2
α −   ⋅ γ  + β −   ⋅ γ = a
  z − γ    z−γ 
or (αz − γx )2 + ( βz − γ y )2 = a2 ( z − γ )2 . …(4)
The cone (4) will have three mutually perpendicular generators if
G-186

the coeff. of x2 + the coeff. of y2 + the coeff. of z 2 = 0


i. e., γ 2 + γ 2 + (α2 + β2 − a2 ) = 0 or α2 + β2 + 2γ 2 = a2 .
∴ The required locus of P (α, β, γ ) is x2 + y2 + 2z 2 = a2 ,
which is the equation of an ellipsoid (a surface of revolution) obtained by revolving the
ellipse
y = 0, x2 + 2z 2 = a2 or x = 0, y2 + 2z 2 = a2
about the z-axis.

Problem 3: If x = y = z / 2 be one of a set of three mutually perpendicular generators of the


cone 3 yz − 2zx − 2 xy = 0, find the equations of the other two generators. (Agra 2007)
Solution: The equation of the given cone is
3 yz − 2zx − 2 xy = 0. …(1)
In the equation of the cone (1), the sum of the coefficients of x2, y2 and z 2 is zero, hence
the cone (1) has an infinite set of three mutually perpendicular generators.
Thus if x /1 = y /1 = z / 2 is one of a set of three mutually perpendicular generators then
the other two mutually perpendicular generators will be the lines of intersection of the
given cone (1) by the plane through the vertex (0, 0, 0 ) and perpendicular to the given
generator namely x /1 = y /1 = z / 2 i. e., by the plane 1 . x + 1 . y + 2 . z = 0. Let
x / l = y / m = z / n be a line of intersection, so that we have
3mn − 2nl − 2lm = 0 …(2)
and l + m + 2n = 0. …(3)
Eliminating m between (2) and (3), we get
3 ( − l − 2n) n − 2nl − 2l ( − l − 2n) = 0
or 2l2 − ln − 6n2 = 0 or ( l − 2n) (2l + 3n) = 0
3
or l = 2n, l = − n.
2
When l = 2n, from (3) we have m + 4n = 0 or m = − 4n.
l m n
∴ 2l = − m = 4n or = = ⋅ …(4)
2 −4 1
3
When l = − n, from (3) we have
2
1 1
m + n = 0 or m = − n.
2 2
3
∴ l = 3m = − n or l / 3 = m /1 = n /( −2). …(5)
2
Therefore, from (4) and (5), the equations of the other two generators are
x / 2 = y /( −4) = z /1 and x / 3 = y /1 = z /( −2). …(6)
Clearly both these generators are perpendicular since
2 . 3 + ( −4) .(1) + 1 . ( −2) = 0.
Also each of these two generators is perpendicular to the given generator
x y z
= = since
1 1 2
G-187

2 . 1 + ( −4) . (1) + 1 . 2 = 0 and 3 . 1 + 1 . 1 + ( −2) . 2 = 0.


Hence the required two generators are given by the equations (6) above.

Problem 4: Prove that the angle between the lines in which the plane x + y + z = 0 cuts the
cone ayz + bzx + cxy = 0 will be π /2 if a + b + c = 0. (Bundelkhand 2007, 08)
Solution: The equation of the cone is
ayz + bzx + cxy = 0. ...(1)
2 2 2
Since in the equation of the cone (1) the sum of the coefficients of x , y and z is zero,
therefore the cone (1) has an infinite set of three mutually perpendicular generators. If
two perpendicular generators are the sections of the cone (1) by the plane x + y + z = 0,
then the third mutually perpendicular generator will be the normal to the plane
x + y + z = 0 passing through the vertex (0, 0, 0 )of the cone i. e., the line x /1 = y /1 = z /1
will be the third generator. Hence the d.r.’s of the third generator x /1 = y /1 = z /1 will
satisfy the equation (1) of the cone. Hence the required condition is given by
a . 1 . 1 + b . 1 . 1 + c . 1 . 1 = 0 or a + b + c = 0.

Comprehensive Problems 6
Problem 1: Find the equation to the right circular cone whose vertex is (2, − 3, 5), axis makes
equal angles with the co-ordinate axes and semi-vertical angle is 30 °.
Solution: The co-ordinates of the vertex A of the
cone are given as (2, − 3, 5). If l, m, n are the d.c.’s of the
axis of the cone then since the axis makes equal angles
with the co-ordinate axes, we have
l m n √ ( l2 + m2 + n2 ) 1
= = = = ⋅
1 1 1 √ (12 + 12 + 12 ) √3
Consider a general point R ( x, y, z ) on the cone so
that the d.r.’s of the generator AR are
x − 2, y + 3, z − 5.
Now the semi-vertical angle of the cone is 30° i. e., the
angle between the axis of the cone and the generator
AR is 30°, therefore, the required equation of the right
circular cone is given by
(1 / √ 3) . ( x − 2) + (1 / √ 3) ( y + 3) + (1 / √ 3) ( z − 5)
cos 30 ° =
√ {( x − 2)2 + ( y + 3)2 + ( z − 5)2 }
√3 x+ y + z −4
or = ⋅
2 √ 3 √ {( x − 2)2 + ( y + 3)2 + ( z − 5)2 }
Squaring and cross-multiplying, we get
9 { x2 − 4 x + 4 + y2 + 6 y + 9 + z 2 − 10 z + 25}
= 4 { x2 + y2 + z 2 + 16 + 2 xy + 2 xz + 2 yz − 8 x − 8 y − 8z}
or 5 ( x2 + y2 + z 2 ) − 8 ( yz + zx + xy ) − 4 x + 86 y − 58 z + 278 = 0.
G-188

Problem 2: A right circular cone is passing through the point (1, 1, 1) and its vertex is the point
(1, 0, 1). The axis of the cone is equally inclined to the co-ordinate axes. Find the equation of the
cone. (Meerut 2011)
Solution: The co-ordinates of the vertex A of the cone are (1, 0, 1).
Also the cone is passing through the point (1, 1, 1) and hence the d.r.’s of the generator
passing through (1, 0, 1) and (1, 1, 1) are 1 − 1, 1 − 0, 1 − 1 i. e., 0, 1, 0.
The axis of the cone is equally inclined to the co-ordinate axes and so its d.c.’s are
1 / √ 3, 1 / √ 3, 1 / √ 3. [See problem 1]. Thus if θ be the semi-vertical angle of the cone, then
0 . (1 / √ 3) + 1 . (1 / √ 3) + 0 . (1 / √ 3) 1
cos θ = = ⋅ …(1)
√ (02 + 12 + 02 ) √3
Now let R ( x, y, z )be a general point on the cone and so the d.r.’s of the generator AR are
x − 1, y − 0, z − 1. The angle between the axis of the cone and the generator AR is also θ
and so we have
( x − 1) . (1 / √ 3) + y . (1 / √ 3) + ( z − 1) . (1 / √ 3)
cos θ =
√ {( x − 1)2 + ( y )2 + ( z − 1)2 }
1 x+ y + z −2
or = , using (1).
√ 3 √ 3 √ ( x2 + y2 + z 2 − 2 x − 2z + 2)
Squaring and cross-multiplying, we get
( x2 + y2 + z 2 − 2 x − 2z + 2) = ( x + y + z − 2)2
or yz + zx + xy − x − 2 y − z + 1 = 0.
This is the required equation of the right circular cone.

Problem 3: Find the equation of the right circular cone whose vertex is the origin and whose axis
is the line x = t, y = 2t, z = 3t and which has a vertical angle of 60 °.
Solution: The vertex of the cone is O (0, 0, 0 ) and the equations of its axis are
x = t, y = 2t, z = 3t i. e., x /1 = y / 2 = z / 3 = t. …(1)
∴ The d.r.’s of the axis of the cone are 1, 2, 3.
Consider a general point R ( x, y, z ) on the cone. The d.r.’s of the generator OR are x − 0,
y − 0, z − 0 i. e., x, y, z. The vertical angle of the cone is 60° and so its semi-vertical angle
30° is given by
1. x + 2 . y + 3 . z
cos 30 ° =
√ (12 + 22 + 32 ) √ ( x2 + y2 + z 2 )
√3 x + 2 y + 3z
or =
2 √ (14) √ ( x2 + y2 + z 2 )
or 21 ( x2 + y2 + z 2 ) = 2 ( x + 2 y + 3z )2
or 19 x2 + 13 y2 + 3z 2 − 24 yz − 12zx − 8 xy = 0.
This is the required equation of the right circular cone.

Problem 4: Find the equation of the right circular cone with vertex at (1, − 2, − 1), semi-vertical
angle 60 ° and the axis ( x − 1) / 3 = − ( y + 2) / 4 = ( z + 1) / 5. (Kumaun 2012)
G-189

Solution: The vertex of the cone is A (1, − 2, − 1). The equations of the axis of the cone
are
( x − 1) / 3 = ( y + 2) /( −4) = ( z + 1) / 5.
∴ The d.r.’s of the axis of the cone are 3, − 4, 5. The semi-vertical angle of the cone is 60 °.
Consider a general point R ( x, y, z ) on the cone and so the d.r.’s of the generator AR are
x − 1, y + 2, z + 1. Hence the required equation of the right cone is given by
3 . ( x − 1) + ( −4) . ( y + 2) + 5 . ( z + 1)
cos 60 ° =
√ {(3)2 + ( −4)2 + (5)2 } √ {( x − 1)2 + ( y + 2)2 + ( z + 1)2 }
1 3 x − 4 y + 5z − 6
or = ⋅
2 √ (50 ) √ ( x2 + y2 + z 2 − 2 x + 4 y + 2z + 6)
Squaring and cross-multiplying, we get
25 ( x2 + y2 + z 2 − 2 x + 4 y + 2z + 6) = 2 (3 x − 4 y + 5z − 6)2
or 7 x2 − 7 y2 − 25z 2 + 80 yz − 60 zx + 48 xy + 22 x + 4 y + 170 z + 78 = 0.

Problem 5: Find the equation of the cone formed by rotating the line 2 x + 3 y = 6, z = 0 about
the y-axis. (Kanpur 2007)

Solution: A generator of the cone is given by


2 x + 3 y = 6, z = 0 or 2 ( x − 3) = − 3 y, z = 0
or ( x − 3) / 3 = y /( −2) = z / 0. …(1)
The axis of the cone is y-axis whose d.c.’s are 0, 1, 0. The vertex A of the cone is the point
of intersection of (1) with the y-axis (i. e., x = 0, z = 0) and is given by (0, 2, 0 ).
If θ be the semi-vertical angle of the cone, then it being the angle between the generator
(1) and the y-axis (i. e., the axis of the cone ) is given by
3 . 0 + ( −2) . 1 + 0 . 0 −2
cos θ = = ⋅ …(2)
√ {(3)2 + ( −2)2 + (0 )2 } √ {02 + 12 + 02 } √ (13)
Consider a general point R ( x, y, z ) on the cone and so the d.r.’s of the generator AR are
x − 0, y − 2, z − 0 i. e., x, y − 2, z. Also the angle between the generator AR and the axis of
the cone (i. e., y-axis) is θ. Hence we have
x . 0 + ( y − 2) . 1 + z . 0
cos θ =
√ { x2 + ( y − 2)2 + z 2 } √ (02 + 12 + 02 )
−2 y −2
or = , using (2).
√ (13) √ ( x2 + y2 + z 2 − 4 y + 4)
Squaring and cross-multiplying, we have
4 ( x2 + y2 + z 2 − 4 y + 4) = 13 ( y2 − 4 y + 4)
or 4 x2 − 9 y2 + 4z 2 + 36 y − 36 = 0.
This is the equation of the required cone.

Problem 6: Find the equation of the cone generated by rotating the line x / l = y / m = z / n
about the line x / a = y / b = z / c as axis.
Solution: The equations of the axis of the cone are
G-190

x / a = y / b = z / c. …(1)
The equations of a generator of the cone are
x / l = y / m = z / n. …(2)
Let θ be the semi-vertical angle of the cone then it is the angle between the lines (1) and
(2) and so we have
al + bm + cn
cos θ = ⋅ …(3)
√ ( a + b + c2 ) √ ( l2 + m2 + n2 )
2 2

Consider a general point R ( x, y, z ) on the cone. Now the vertex A of the cone being the
point of intersection of (1) and (2) is given by (0, 0, 0 ). Thus the d.r.’s of the generator
AR are x − 0, y − 0, z − 0 i. e., x, y, z. Also the angle between the axis (1) and the
generator AR is θ and hence we have
ax + by + cz
cos θ = ⋅ …(4)
√ ( a2 + b2 + c2 ) √ ( x2 + y2 + z 2 )
Equating the two values of cos θ given by (3) and (4), the required equation of the cone is
given by
ax + by + cz al + bm + cn
=
√ ( x2 + y2 + z 2 ) √ ( l2 + m2 + n2 )
or ( ax + by + cz )2 ( l2 + m2 + n2 ) = ( x2 + y2 + z 2 ) ( al + bm + cn)2.

Problem 7: If a right circular cone has three mutually perpendicular tangent planes , then
show that the semi-vertical angle of the cone is given by cot−1 √ 2.
Solution: Let us consider the equation of the right circular cone with the vertex at the
origin, axis the z-axis and the semi-vertical angle θ. Its equation is given by [See article
14, Particular case I]
x2 + y2 − z 2 tan2 θ = 0. …(1)
The cone (1) will have three mutually perpendicular tangent planes if
‘bc + ca + ab = f 2 + g2 + h2 ’ [See article 12]
or 1 . ( − tan2 θ) + ( − tan2 θ) . 1 + 1 . 1 = 0 + 0 + 0
or 2 tan2 θ = 1 or cot2 θ = 2 or θ = cot −1 √ 2.

Comprehensive Problems 7
Problem 1: Find the enveloping cone of the sphere x2 + y2 + z 2 − 2 y + 6z + 2 = 0 with its
vertex at (1, 1, 1).
Solution: We have S ≡ x2 + y2 + z 2 − 2 y + 6z + 2 = 0.
The vertex of the enveloping cone is (1, 1, 1).
∴ S1 = 12 + 12 + 12 − 2 . 1 + 6 . 1 + 2 = 9,
T = x . 1 + y . 1 + z . 1 − ( y + 1) + 3 ( z + 1) + 2 = x + 4z + 4.
The required equation of the enveloping cone is given by
‘SS1 = T 2 ’ or ( x2 + y2 + z 2 − 2 y + 6z + 2) . 9 = ( x + 4z + 4)2
G-191

or 9 x2 + 9 y2 + 9z 2 − 18 y + 54z + 18 = x2 + 16 z 2 + 16 + 8zx + 8 x + 32z


or 8 x2 + 9 y2 − 7z 2 − 8zx − 8 x − 18 y + 22z + 2 = 0.

Problem 2: Show that the lines drawn from the origin so as to touch the sphere
x2 + y2 + z 2 + 2ux + 2vy + 2wz + d = 0

lie on the cone d ( x2 + y2 + z 2 ) = ( ux + vy + wz )2 .


Solution: Here it is required to find the enveloping cone of the given sphere with the
vertex at the origin.
The equation of the given sphere is
S ≡ x2 + y2 + z 2 + 2ux + 2vy + 2wz + d = 0. …(1)
We have S1 = 0 + 0 + 0 + 0 + 0 + 0 + d = d,
T = x . 0 + y . 0 + z . 0 + u ( x + 0 ) + v ( y + 0 ) + w (z + 0 ) + d
= ux + vy + wz + d.
Therefore the equation of the enveloping cone of the sphere (1) with the vertex at the
origin is given by ‘SS1 = T 2 ’
or ( x2 + y2 + z 2 + 2ux + 2vy + 2wz + d) . d = ( ux + vy + wz + d)2
or ( x2 + y2 + z 2 ) d + 2 ( ux + vy + wz ) d + d2
= ( ux + vy + wz )2 + 2 ( ux + vy + wz ) . d + d2
or d ( x2 + y2 + z 2 ) = ( ux + vy + wz )2 .

Problem 3: Prove that the plane z = 0 cuts the enveloping cone of the sphere
x2 + y2 + z 2 = 11 which has its vertex at (2, 4, 1) in a rectangular hyperbola.
Solution: Here S ≡ x2 + y2 + z 2 − 11 = 0, S1 = (2)2 + (4)2 + (1)2 − 11 = 10,
T = x . 2 + y . 4 + z . 1 − 11 = 2 x + 4 y + z − 11.
Therefore by the formula SS1 = T 2, the equation of the enveloping cone of the sphere
S = 0 with the vertex at (2, 4, 1) is given by
( x2 + y2 + z 2 − 11) . 10 = (2 x + 4 y + z − 11)2. …(1)
The plane z = 0 cuts the enveloping cone (1) in the conic given by
( x2 + y2 − 11) . 10 = (2 x + 4 y − 11)2, z = 0
or 6 x2 − 6 y2 − 16 xy + 44 x − 88 y − 231 = 0, z = 0. …(2)
Now in the first of the equations (2),
the coefficient of x2 + the coefficient of y2 = 6 + ( −6) = 0.
Hence the conic (2) represents a rectangular hyperbola.

Problem 4: Find the locus of a luminous point which moves so that the sphere
x2 + y2 + z 2 − 2az = 0 casts a parabolic shadow on the plane z = 0.
Solution: Let the luminous point be P ( x1, y1, z1 ).
The equation of the given sphere is S ≡ x2 + y2 + z 2 − 2az = 0.
G-192

∴ S1 = x12 + y12 + z12 − 2az1


and T = xx1 + yy1 + zz1 − a ( z + z1).
The equation of the enveloping cone of the sphere S = 0 with the vertex at P ( x1, y1, z1 )
is given by SS1 = T 2
i. e., ( x2 + y2 + z 2 − 2az ) ( x12 + y12 + z12 − 2az1)
= { xx1 + yy1 + zz1 − a ( z + z1 )}2 . …(1)
The plane z = 0 cuts the cone (1) in the conic given by
( x2 + y2 ) ( x12 + y12 + z12 − 2az1 ) = ( xx1 + yy1 − az1 )2 , z = 0
or x2 ( y12 + z12 − 2az1 ) − 2 x1 y1 xy + y2 ( x12 + z12 − 2az1 )
+ 2ax1z1 x + 2 ay1z1 y − a2 z12 = 0, z = 0. …(2)
Now we know that the conic ax2 + 2hxy + by2 + 2 gx + 2 fy + c = 0, z = 0
represents a parabola if h2 = ab.
Applying this condition the conic (2) will represent a parabola if
( − x1 y1)2 = ( y12 + z12 − 2az1) ( x12 + z12 − 2az1)
or x12 y12 = x12 y12 + y12 z12 − 2az1 y12 + z12 x12 + z14
− 2az13 − 2az1 x12 − 2az13 + 4a2 z12
or z12 ( x12 + y12 + z12 − 4az1 + 4a2 ) − 2az1 ( x12 + y12 ) = 0
or z1 ( x12 + y12 + z12 − 4az1 + 4a2 ) − 2a ( x12 + y12 ) = 0.
∴ The locus of the point P ( x1 , y1, z1) is
z ( x2 + y2 + z 2 − 4az + 4a2 ) − 2a ( x2 + y2 ) = 0.

Problem 5: Find the locus of points from which three mutually perpendicular tangent lines can
be drawn to the paraboloid ax2 + by2 = 2cz.
Solution: Let P ( x1, y1, z1) be any point whose locus is required.
Here S = ax2 + by2 − 2cz, S1 = ax12 + by12 − 2c z1, and T = axx1 + byy1 − c ( z + z1 ).
∴ The enveloping cone of the given paraboloid with the vertex at P ( x1, y1, z1 ) is
‘SS1 = T 2 ’
i. e., ( ax2 + by2 − 2cz ) ( ax12 + by12 − 2cz1) = { axx1 + byy1 − c ( z + z1 )}2.
…(1)
Now three mutually perpendicular tangent lines can be drawn from P ( x1, y1, z1 ) to
S = 0 if the enveloping cone (1) has three mutually perpendicular generators, the
condition for which is that in the equation (1) the sum of the coefficients of x2 , y2 and
z 2 should be equal to zero
i. e., a ( by12 − 2cz1 ) + b ( ax12 − 2cz1 ) − c2 = 0
or ab ( x12 + y12 ) − 2 c ( a + b ) z1 = c2 .
∴ The locus of P ( x1, y1, z1 ) is ab ( x2 + y2 ) − 2c ( a + b ) z = c2 .
G-193

H ints to O bjective T ype Q uestions

Multiple Choice Questions


x y z
1. The d.c.’s of y-axis are 0, 1, 0. So the equations of y-axis are = = ⋅
0 1 0
If the y-axis is a generator of the cone
ax2 + by2 + cz 2 + 2 fyz + 2 gzx + 2hxy = 0,
then x = 0, y = 1, z = 0 must satisfy the equation of this cone.
∴ 0 + b + 0 + 0 + 0 + 0 = 0 or b = 0.
2. See article 4.
3. The required condition is
coefficient of x2 + coefficient of y2 + coefficient of z 2 = 0
⇒ a + 4 + 5 = 0 ⇒ a = − 9.
4. See Example 10.
5. See article 11.
6. Every homogeneous equation of second degree in x, y and z represents a cone
whose vertex is at the origin.
7. The condition that the cone
ax2 + by2 + cz 2 + 2 fyz + 2 gzx + 2hxy = 0,
may have three mutually perpendicular generators is a + b + c = 0.
8. See article 6.
9. See article 2.
10. See article 3.
11. Like Problem 8 of Comprehensive Problems 1.
12. See article 3.
13. See article 7.
14. See article 8 and article 9.
15. See article 9.
16. The plane lx + my + nz = 0 will touch the cone ax2 + by2 + cz 2 = 0 if the normal
to this plane through the vertex (0, 0, 0 ) of the cone ax2 + by2 + cz 2 = 0 lies on
x y z
the reciprocal cone of this cone i. e., if the line = = is a generator of the cone
l m n
x2 y2 z2 l2 m2 n2
+ + = 0. So, the required condition is + + = 0.
a b c a b c
17. The line whose d.c.’s are l, m, n and which passes through the origin is a generator
of the cone 4 x2 − y2 + 3z 2 = 0 whose vertex is at the origin. So x = l, y = m, z = n
must satisfy the equation 4 x2 − y2 + 3z 2 = 0.
G-194

18. See article 11.


19. See article 14. Particular Cases. Case III.
20. See Example 1.
21. See article 13.
22. In the given equation of the cone we have
coeff. of x2 + coeff. of y2 + coeff. of z 2
= a + b + c = 2 − 5 + 3 = 0.
So the given cone has three mutually perpendicular generators.
23. Comparing the equation 5 yz − 8zx − 3 xy = 0 of the given cone with the
equation
ax2 + by2 + cz 2 + 2 fyz + 2 gzx + 2hxy = 0,
we have a = 0, b = 0, c = 0 so that
a + b + c = 0.
But a + b + c = 0 is the condition for the cone
ax2 + by2 + cz 2 + 2 fyz + 2 gzx + 2hxy = 0
to have three mutually perpendicular generators. So the given cone has three
mutually perpendicular generators.

Fill in the Blank(s)


1. See article 1.
2. See article 1.
3. See article 2.
4. See article 7, Corollary 1.
5. See article 8.
6. See article 9, definition of reciprocal cone of a given cone.
7. See article 10, Corollary 1.
8. See article 12.
9. See articles 11 and 12.
10. See article 15.
11. See article 15, Case I.
12. See article 9.

True or False
1. See article 2, converse part.
2. See article 2, converse part.
3. x y z
The direction ratios of the line = = are 1, − 2, 3. Since this line passes
1 −2 3
through the origin which is the vertex of the cone x2 + 2 y2 − z 2 = 0, therefore it
will be a generator of this cone iff x = 1, y = − 2, z = 3 satisfy the equation
x2 + 2 y2 − z 2 = 0.
G-195

We have 12 + 2 . ( −2)2 − 32 = 0 i. e., x = 1, y = − 2, z = 3 satisfy the equation


x2 + 2 y2 − z 2 = 0.
4. We have 2 . (2)2 − 3 . 32 + 4 . 42 = 45 ≠ 0. Thus x = 2, y = 3, z = 4 do not satisfy
x y z
the equation 2 x2 − 3 y2 + 4z 2 = 0. So the line = = is not a generator of the
2 3 4
cone 2 x2 − 3 y2 + 4z 2 = 0.
5. Obviously the direction cosines 1, 0, 0; 0, 1, 0 ; 0, 0, 1 of the three coordinate axes
satisfy the equation 3 yz − 4zx + 5 xy = 0. So the given equation represents a cone
passing through each of the three coordinate axes.
6. Comparing the equation x2 − y2 + 2z 2 = 0 with the equation
ax2 + by2 + cz 2 + 2 fyz + 2 gzx+ 2hxy = 0, we have a = 1, b = − 1, c = 2.
We have a + b + c = 1 − 1 + 2 = 2 ≠ 0. So the cone x2 − y2 + 2z 2 = 0 has not
three mutually perpendicular generators.
7. The semi-vertical angle of a right circular cone which has three mutually
perpendicular generators is tan−1 2 which is constant.
❍❍❍
G-196

Chapter-8
The Cylinder

Comprehensive Problems 1
Problem 1: (i) Find the equation of a cylinder whose generators are parallel to the line
x = y / 2 = − z and whose guiding curve is 3 x2 + 2 y2 = 1, z = 0.
x y z
(ii) Find the equation of the cylinder whose generators are parallel to the line = = and
1 2 3
pass through the curve x 2 + y 2 = 16, z = 0. (Meerut 2006; Rohilkhand 10)
Solution: (i) The equations of the given guiding curve are
3 x2 + 2 y2 = 1, z = 0. …(1)
The equations of the given line are
x /1 = y / 2 = z /( −1). …(2)
Consider a current point P ( x1, y1, z1) on the cylinder. The equations of the generator
through the point P ( x1, y1, z1) which is a line parallel to the given line (2) are
x − x1 y − y1 z − z1
= = ⋅ …(3)
1 2 −1
The generator (3) meets the plane z = 0 in the point given by
x − x1 y − y1 0 − z1
= = i. e., ( x1 + z1, y1 + 2z1, 0 ).
1 2 −1
Since the generator (3) meets the conic (1), hence the point
( x1 + z1, y1 + 2z1, 0 )
will satisfy the equations of the conic given by (1), and so we have
3 ( x1 + z1)2 + 2 ( y1 + 2 z1)2 = 1
or 3 ( x12 + 2 x1z1 + z12 ) + 2 ( y12 + 4 y1z1 + 4z12 ) = 1
or 3 x12 + 2 y12 + 11z12 + 8 y1z1 + 6z1 x1 − 1 = 0.
∴ The locus of P ( x1, y1, z1) i. e., the required equation of the cylinder is given
by
3 x2 + 2 y2 + 11 z 2 + 8 yz + 6zx − 1 = 0.
(ii) Let (α , β , γ ) be any point on the cylinder, then the equation of the generator
through P is
x−α y −β z − γ
= = …(1)
1 2 3
This generator meets the plane z = 0 in the point given by
G-197

x−α y −β 0 − γ
= =
1 2 3
1 2
i.e. in the point (α − γ , β − γ , 0 ) .
3 3
∴ The generator intersect the given curve if
2 2
 α − 1 γ  +  β − 2 γ  = 16.
   
 3   3 
∴ The locus of P(α , β , γ ) or the required equation of the cylinder is
2 2
 x − z  +  4 − 2 z  = 16
   
 3  3 
or 9 x2 + 9 y2 + 5z 2 − 12 yz − 6zx − 144 = 0 .

Problem 2: Find the equation of the cylinder whose generators are parallel to the line
x = y /( −2) = z / 3 and passing through the curve x2 + 2 y2 = 1, z = 0.
(Meerut 2004, 05; Bundelkhand 06; Kumaun 09; Kashi 14)
Solution: Proceeding exactly as in Problem 1 the required equation of the cylinder is
given by
3 ( x2 + 2 y2 + z 2 ) − 2z x + 8 yz − 3 = 0.

Problem 3: Find the equation of the surface generated by a straight line which is parallel to the
line y = mx, z = nx and intersects the ellipse x2 / a2 + y2 / b2 = 1, z = 0. (Avadh 2013)

Solution: The equations of the guiding curve (ellipse) are


x2 / a2 + y2 / b2 = 1, z = 0. …(1)
The equations of the given line may be written as
x /1 = y / m = z / n. …(2)
Consider a point P ( x1, y1, z1) on the cylinder. The equations of the generator through
the point P ( x1, y1, z1 ) which is a line parallel to the given line (2) are
x − x1 y − y1 z − z1
= = ⋅ …(3)
1 m n
The generator (3) meets the plane z = 0 in the point given by
x − x1 y − y1 0 − z1
i. e.,  x1 − 1 , y1 −
z mz1 
= = , 0 ⋅
1 m n  n n 
Since the generator (3) meets the conic (1), hence the point
( x1 − z1 / n, y1 − mz1 / n, 0 )
will satisfy the equation of the conic given by (1), and so we have
(1 / a2 ) ( x1 − z1 / n)2 + (1 / b2 ) ( y1 − mz1 / n)2 = 1
or b2 ( nx1 − z1 )2 + a2 ( ny1 − mz1 )2 = a2 b2 n2 .
∴ The locus of P ( x1, y1, z1) i. e., the required equation of the cylinder is given by
b2 ( nx − z )2 + a2 ( ny − mz )2 = a2 b2 n2 .
G-198

Problem 4: Find the equation of the cylinder which intersects the curve
ax2 + by2 + cz 2 = 1, lx + my + nz = p
and whose generators are parallel to the axis of x. (Meerut 2008)
Solution: The equations of the guiding curve are
ax2 + by2 + cz 2 = 1 …(1)
and lx + my + nz = p. …(2)
Now the equation of the cylinder whose generators are parallel to x-axis will not contain
the terms of x. [See article2, theorem]. Hence the required equation of the cylinder is
obtained by eliminating x between the equations (1) and (2), and so is given by
2
p − my − nz   ∵ From (2), x = p − my − nz 
a  2 2
 + by + cz = 1
 l   l 
or a ( p − my − nz )2 + bl2 y2 + cl2 z 2 = l2
or a ( p2 + m2 y2 + n2 z 2 − 2 pmy − 2 pnz + 2mnyz ) + bl2 y2 + cl2 z 2 = l2
or ( am2 + bl2 ) y2 + ( an2 + cl2 ) z 2 + 2amnyz
− 2ampy − 2anpz + ( ap2 − l2 ) = 0.

Problem 5: Find the equation of the cylinder with generators parallel to the x-axis and passing
through the curve ax2 + by2 = 2cz, lx + my + nz = p. (Agra 2006, 14)
Solution: The equations of the guiding curve are
ax2 + by2 = 2cz …(1)
and lx + my + nz = p. …(2)
Since the generators are parallel to the x-axis, therefore eliminating x between the
equations (1) and (2), the equation of the required cylinder is given by
a {( p − my − nz ) / l}2 + by2 − 2cz = 0
or a ( p2 + m2 y2 + n2 z 2 − 2 pmy + 2mnyz − 2 pnz ) + bl2 y2 − 2cl2 z = 0
or ( am2 + bl2 ) y2 + an2 z 2 + 2amnyz − 2apmy − 2 ( apn + c l2 ) z = 0.

Problem 6: (i) Find the equation of the right circular cylinder of radius 2 and having as axis
the line
x −1 y −2 z −3
= = ⋅
2 1 2 (Meerut 2000, 05B, 12, 13B;
Rohilkhand 05, 11, 13B; Avadh 09, 10; Kanpur 10)
(ii) Find the equation of the right circular cylinder of radius 2 whose axis is the line
x −1 y z −3
= = ⋅
2 3 1 (Kumaun 2010)
(iii) Find the equation of the right circular cylinder of radius 4 and having axis as the line
x −1 y −3 z −5
= = ⋅
2 2 1
G-199

(iv) Find the equation to the right circular cylinder whose axis is
x −1 y −3 5−z
= = and radius is 3.
2 2 1
(Agra 2006, 14; Meerut 09, 10; Kanpur 15)
(v) Find the equation of right circular cylinder whose axis is x = 2 y = − z and radius is 4 and
find the area of the section of this cylinder by the plane z = 0 .
(Rohilkhand 06; Gorakhpur 06)
(vi) Find the equation of the right circular cylinder of radius 2 whose axis passes through
(1, 2, 3) and has direction cosines proportional to 2, − 3, 6. (Meerut 2001; Agra 05)
(vii) Find the equation of the right cylinder whose axis is x / 2 = y / 3 = z / 6 and radius 5.
(Rohilkhand 2008,09)
Solution: (i) The equations of the axis of the cylinder are
x −1 y −2 z −3
= = ⋅ …(1)
2 1 2
Consider a point P ( x, y, z ) on the cylinder. The length of the perpendicular from the
point P ( x, y, z )to the given axis (1) is equal to the radius of the cylinder i. e.,2 . Hence the
required equation of the cylinder is given by [See article 3, equation (2)]
{2 ( y − 2) − 1 . ( z − 3)}2 + {2 ( z − 3) − 2 ( x − 1)}2
+ {1 ( x − 1) − 2 ( y − 2)}2 = (2)2 . {(2)2 + (1)2 + (2)2 }
or (2 y − z − 1)2 + (2z − 2 x − 4)2 + ( x − 2 y + 3)2 = 36
or 5 x2 + 8 y2 + 5z 2 − 4 yz − 8zx − 4 xy + 22 x − 16 y − 14z − 10 = 0.

(ii) Proceeding exactly as in problem 6(i), the equation of required cylinder is given by
10 x2 + 5 y2 + 13z 2 − 12 xy − 6 yz − 4zx − 8 x + 30 y − 74z + 59 = 0.
(iii)Axis of cylinder is ,
d.c. s of the
x −1 y −3 z −5
= = …(1) axis (2/3, 2/3)
2 2 1
whose d.r.’s are (2, 2, 1)
i.e. d.c.’s of the axis are
 2, 2, 1 ⋅
  4
 3 3 3 N P (x ', y', z')
On the cylinder take any point P ( x ′, y ′, z ′ ) .
axis
∵ Radius PN = 4.
∴ AN = Projection of AP on the axis AN
2 2 1
= ( x ′ − 1) + ( y ′ − 3) + ( z ′ − 5)
3 3 3
2 x ′ + 2 y ′ + z ′ − 13
= ⋅
3 A
Now since AP2 − AN2 = NP2 .
2
2 x ′+ 2 y ′ + z ′ − 13 
Therefore ( x ′−1 )2 + ( y ′ − 3)2 + ( z ′ − 5)2 −   = 16
 3 
G-200

or 9( x ′ 2 + y ′ 2 + z ′ 2 − 2 x ′ − 6 y ′ − 10 z ′ + 1 + 9 + 25)
− (4 x ′ 2 + 4 y ′ 2 + z ′ 2 + 169 + 8 x ′ y ′ + 4 x ′ z ′
− 52 x ′ + 4 y ′ z ′ − 52 y ′ − 26z ′ ) = 144
or 5 x ′ 2 + 5 y ′ 2 + 8z ′ 2 − 8 x ′ y ′ − 4 x ′ z ′ − 4 y ′ z ′ + 34 x ′
− 2 y ′ − 64z ′ + 2 = 0.
Thus locus of point ( x ′, y ′, z ′ ) is required cylinder and therefore, its equation is
5 x2 + 5 y2 + 8z 2 − 8 x y − 4 x z − 4 y z + 34 x − 2 y − 64z + 2 = 0.
,
d.c. s of the
(iv) Axis of cylinder is
x −1 y −3 z −5 axis (2/3, 2/3, 2/3)
= = …(1)
2 2 −1
whose d .r.’s are (2, 2, − 1 )
i.e., d .c .’s of the axis are 3
N P (x ', y', z')
 2, 2, − 1 ⋅
 
 3 3 3
On the cylinder take any point P whose axis
co-ordinates are ( x ′, y ′, z ′ ).
∵ Radius PN = 3
AN = Projection of AP on the axis AN
2 2 ( z ′ − 5) A (1, 3, 5)
= ( x ′ − 1) ⋅ + ( y ′ − 3) −
3 3 3
2x ′ + 2 y ′ − z ′ − 3
= ⋅
3
Now since AP2 − AN2 = NP2 ,
2
2x ′ + 2 y ′ − z ′ − 3
therefore, ( x ′ − 1 )2 + ( y ′−3)2 + ( z ′ − 5)2 −   =9
 3 
⇒ 9( x2 + y2 + z 2 − 2 x ′ − 6 y ′ − 10 z ′ + 1 + 9 + 25) − (4 x ′2 + 4 y ′2 + z ′2
+ 9 + 8 x ′ y ′ − 4 x ′ z ′ − 12 x ′ − 4 y ′ z ′ − 12 y ′ + 6z ′ ) = 81
⇒ 5 x ′2 + 5 y ′+8z ′2 − 8 x ′ y ′ + 4 y ′ z ′ + 4z ′ x ′ − 6 x ′ − 42 y ′
− 96z ′ + 225 = 0. …(2)
Thus locus of point ( x ′, y ′, z ′ ) is required cylinder and therefore, its equation can be
obtained by replacing x ′, y ′, z ′ by x , y , z respectively in (2) i.e.,
5 x2 + 5 y2 + 8z 2 − 8 xy + 4 yz + 4zx − 6 x − 42 y − 96z + 225 = 0.
(v) Given axis of right circular cylinder is
x y z
= = …(1)
2 1 −1
and radius of cylinder = 4.
d .c .’s of axis are  , , −  .
2 1 2
 3 3 3
On the cylinder take any point P whose coordinates are ( x ′, y ′, z ′ ).
Let PN = perpendicular from P on the axis.
G-201

Axis
, (2, 1, −2)
d.r. s
4
N P (x ', y', z')

O (0, 0, 0)

Right Circular Cylinder

Therefore, ON = projection of OP on the axis.


Now since OP2 − ON2 = NP2 .
2
 2 
x ′ 2 + y ′ 2 + z ′ 2 − ( x ′ − 0 ) + ( y ′ − 0 ) + ( z ′ − 0 )  −  
2 1
Therefore = 16
 3 3  3 
⇒ 9( x ′ 2 + y ′ 2 + z ′ 2 ) − [ ax ′ + y ′ − 2z ′ ] 2 = 144
⇒ 9( x ′ 2 + y ′ 2 + z ′ 2 ) − [4 x ′ 2 + y ′ 2 + 4z ′ 2 + 4 x ′ y ′
− 8 x ′ z ′ − 4 y ′ z ′ ] = 144
⇒ 5 x ′ 2 + 8 y ′ 2 + 5z ′ 2 − 4 x ′ y ′ + 8 x ′ z ′ + 4 y ′ z ′ − 144 = 0. …(2)
The locus of ( x ′, y ′, z ′ ) is required cylinder therefore is equation can be obtained by
replacing x ′, y ′, z ′ y x , y , z respectively in (2) i.e.,
5 x2 + 8 y2 + 5z 2 − 4 xy + 4 yz + 8zx − 144 = 0. …(3)
Its intersection by z = 0 is given by
5 x2 + 8 y2 − 4 xy = 144
5 2 8 2 4
or x + y − xy = 1 …(4)
144 144 144
which is standard equation of ellipse on z = 0 plane. Length of semi axes of ellipse are
given by
 1  1 2
 A − 2  B − 2  = H .
 r   r 
5 8 2
Here A= , B= ,H = − ⋅
144 144 144
 5 1  8 1 4
Therefore  −   −  =
 144 r 2   144 r 2  1442
⇒ (5r 2 − 144) (8r 2 − 144) = 4 r 4
⇒ 36 r 4 − 144 r 2 (8 + 5) + 1442 = 0
⇒ r 4 − 52 r 2 + 576 = 0 ⇒ ( r 2 − 36)( r 2 − 16) = 0
⇒ r1 = 6, r2 = 4
Area of ellipse = π r1 r2 = 24 π .
G-202

(vi) The axis of the cylinder passes through (1, 2, 3) and has d.r.’s 2, − 3, 6 and hence its
equations are
x −1 y −2 z −3
= = ⋅ …(1)
2 −3 6
Consider a point P ( x, y, z ) on the cylinder. The length of the perpendicular from the
point P ( x, y, z )to the given axis (1) is equal to the radius of the cylinder i. e., 2. Hence the
equation of the required cylinder is given by [See article 3 equation (2). Put
l = 2, m = − 3, n = 6 ; α = 1, β = 2, γ = 3 ; and r = 2 in the equation (2) of the cylinder.]
{6 ( y − 2) − ( −3) ( z − 3)}2 + {2 ( z − 3) − 6 ( x − 1)}2
+ {( −3) ( x − 1) − 2 ( y − 2)}2 = (2)2 {(2)2 + ( −3)2 + (6)2 }
or (6 y + 3z − 21)2 + (2z − 6 x )2 + ( −3 x − 2 y + 7)2 = 4 (49)
or 45 x2 + 40 y2 + 13z 2 + 36 yz − 24zx
+ 12 xy − 42 x − 280 y − 126z + 294 = 0.
(vii) The equations of the axis of the cylinder are
x−0 y −0 z −0
= = ⋅ …(1)
2 3 6
Consider a point P ( x, y, z ) on the cylinder. The length of the perpendicular from the
point P ( x, y, z )to the given axis (1) is equal to the radius of the cylinder i. e., 5. Hence the
required equation of the cylinder is [See article 3. Note that here
α = β = γ = 0 ; l = 2, m = 3, n = 6 ; and r = 5]
(6 y − 3z )2 + (2z − 6 x )2 + (3 x − 2 y )2 = 52 {(2)2 + (3)2 + (6)2 }
or 45 x2 + 40 y2 + 13z 2 − 36 yz − 24zx − 12 xy − 1225 = 0.

Problem 7: Find the equation of the right circular cylinder whose axis is x − 2 = z, y = 0 and
which passes through the point (3, 0, 0 ).
Solution: The equations of the axis of the cylinder may be written as
x−2 y −0 z −0
= = ⋅ …(1)
1 0 1
First we shall find the radius r of the cylinder.
We know that
r = the length of the perpendicular from a point (3, 0, 0 ) on the
cylinder to the axis (1)
1
= ⋅ √ [{0 . 0 − 0 . 0 }2 + {1 . 0 − 1 . (3 − 2)}2
√ {(1)2 + (0 )2 + (1)2}
+ {0 . (3 − 2) − 1 . 0 }2 ]
= 1 / √ 2.
Consider a point P ( x, y, z ) on the cylinder. The length of the perpendicular from the
point P ( x, y, z ) to the given axis (1) is equal to the radius of the cylinder. Hence the
required equation of the cylinder is [See article 3]
{1 . y − 0 . z }2 + {1 . z − 1 . ( x − 2)}2 + {0 . ( x − 2) − 1 . y }2
= (1 / √ 2)2 (1 + 0 + 1)
G-203

or y2 + ( z − x + 2)2 + y2 = 1
or x2 + 2 y2 + z 2 − 2zx − 4 x + 4z + 3 = 0.

Problem 8: Find the equation of the right circular cylinder which passes through the circle
x2 + y2 + z 2 = 9, x − y + z = 3.
(Gorakhpur 2005; Purvanchal 08, 10; Rohilkhand 09B; Kanpur 16)
Solution: The equations of the guiding circle are
x2 + y2 + z 2 = 9 …(1)
and x − y + z = 3. …(2)
Since the cylinder is a right circular cylinder, hence the axis of the cylinder will be
perpendicular to the plane (2) of the circle and so the d.r.’s of the axis of the cylinder are
1, − 1, 1.
Let P ( x1, y1, z1) be a point on the cylinder. The equations of the generator through
P ( x1, y1, z1) having d.r.’s 1, − 1, 1 are
x − x1 y − y1 z − z1
= = = r (say). …(3)
1 −1 1
Any point on the generator (3) is ( r + x1, − r + y1, r + z1) .
Since the generator (3) meets the guiding circle, hence the point ( r + x1, − r + y1, r + z1)
will satisfy the equations (1) and (2), and so we have
( r + x1)2 + ( − r + y1)2 + ( r + z1)2 = 9
and ( r + x1) − ( − r + y1) + ( r + z1) = 3
or 3r2 + 2r ( x1 + y1 + z1 ) + ( x12 + y12 + z12 − 9) = 0 …(4)
1
and r = (3 − x1 + y1 − z1 ). …(5)
3
Eliminating r between (4) and (5), we get
1 2
(3 − x1 + y1 − z1 )2 + (3 − x1 + y1 − z1 ) ( x1 + y1 + z1 )
3 3
+ ( x12 + y12 + z12 − 9) = 0
or x12 + y12 + z12 + y1z1 − z1 x1 + x1 y1 = 9.
∴ The locus of P ( x1, y1, z1) or the required equation of the cylinder is
x2 + y2 + z 2 + yz − z x + xy = 9.

Problem 9: Find the equation of the right circular cylinder described on the circle through the
points A (1, 0, 0 ), B (0, 1, 0 ) and C (0, 0, 1) as the guiding curve.
(Garhwal 2002; Purvanchal 08,10; Rohilkhand 13)
Solution: Equation of the plane through three given points is
x + y + z = 1.
It is easily seen that the three given points are equidistant from the origin (0 , 0 , 0 ) ;
therefore a sphere, with centre at (0 , 0 , ) and of radius unity, passes through the three
given points.
Length of perpendicular from (0 , 0 , 0 ) to the plane x + y + z = 1 is
G-204

1 1
i.e.
2 2
(1 + 1 + 1 ) 2 3

and radius of the sphere is 1.


∴ Radius of the guiding circle = 1 − 1  = 2 .
 
 3 3
Also, because the axis of the cylinder is normal to the plane x + y + z = 1 and passes
through the centre (0 , 0 , 0 ) of the sphere therefore equations of the axis are
x−0 y −0 z −0
= = .
1 1 1
Hence equation of the cylinder is
4
{( x − 0 ) + ( y − 0 ) + ( z − 0 )}2  3
( x − 0 )2 + ( y − 0 )2 + ( z − 0 )2 − =  .
12 + 12 + 12  2
or 3( x2 + y2 + z 2 ) − ( x + y + z )2 = 2
or 2( x2 + y2 + z 2 ) − 2 yz − 2zx − 2 xy = 2
or x2 + y2 + z 2 − yz − zx − xy = 1.

Problem 10: Find the equation of the enveloping cylinder of the sphere
x2 + y2 + z 2 − 2 x + 4 y − 1 = 0 having its generators parallel to the line x = y = z.
Solution: The given sphere is
x2 + y2 + z 2 − 2 x + 4 y − 1 = 0. …(1)
The generators of the enveloping cylinder are parallel to the line
x = y = z. …(2)
Let P (α, β, γ ) be any point on the enveloping cylinder. Then the equations of the
generator of the enveloping cylinder through the point (α, β, γ ) which is a straight line
parallel to the line (2) are
x−α y −β z−γ
= = = r (say). …(3)
1 1 1
Any point on (3) is (α + r, β + r, γ + r ).
The points of intersection of the generator (3) with the sphere (1) are given by
(α + r )2 + ( β + r )2 + ( γ + r )2 − 2 (α + r ) + 4 ( β + r ) = 1
or 3r2 + 2r (α + β + γ + 1) + (α2 + β2 + γ 2 − 2α + 4β − 1) = 0. …(4)
Since the generator (3) is a tangent to the given sphere (1), therefore the two values of r
obtained from (4) are equal and the condition for the same is
[2 (α + β + γ + 1)]2 − 4 . 3 (α2 + β2 + γ 2 − 2α + 4β − 1) = 0
[Using B2 − 4 AC = 0]
or α2 + β2 + γ 2 − βγ − γα − αβ − 4α + 5 β − γ − 2 = 0.
∴ The locus of P (α, β, γ ) i. e., the required equation of the enveloping cylinder is
x2 + y2 + z 2 − yz − z x − xy − 4 x + 5 y − z − 2 = 0.
G-205

H ints to O bjective T ype Q uestions

Multiple Choice Questions


1. In the three dimensional geometry x2 + y 2 = r2 is the equation of a cylinder.
See corollary to article 3.
2. The equation of the cylinder with generators parallel to z-axis and passing
through the curve ax2 + by 2 = 2cz, lx + my + nz = p is obtained by eliminating
z between these two equations and so is
 p − lx − my 
ax2 + by2 = 2c .  2 2
 i. e., n ( ax + by ) = 2c ( p − lx − my ).
 n 

Fill in the Blank(s)


1. See article 1.
2. See corollary to article 3.
3. See corollary 2 to article 2.
4. See article 4.
5. See article 4, definition of an enveloping cylinder.

True or False
1. We know that all the generators of a cylinder are parallel to the axis of the
cylinder and so are parallel straight lines.
2. The given statement is false because all the generators of a cylinder are
parallel straight lines and so they cannot meet at a point.
3. See article 1.
4. The equation of the cylinder y2 + z 2 = a2 does not contain x. So the
generators of this cylinder are straight lines parallel to x-axis.
5. The equation of the cylinder y2 + z 2 = 16 does not contain x. So the
generators of this cylinder are straight lines parallel to x-axis.
❍❍❍
G-206

Chapter-9
The Central Conicoids

Comprehensive Problems 1
Problem 1: Find the equation of the tangent plane to the ellipsoid
x2 / a2 + y2 / b2 + z 2 / c2 = 1 at the point ( x1 , y1, z1 ) on it. (Meerut 2007B, 08, 10B)
x2 y2 z2
Solution: The equation of tangent plane of the given ellipsoid + + = 1 at
2 2
a b c2
point ( x 1, y1, z1 ) is
x x1 yy1 zz1
+ + = 1.
2 2
a b c2
See article 7 ( B).

Problem 2 : Find the condition that the plane lx + my + nz = p may touch the ellipsoid
x2 / a2 + y2 / b2 + z 2 / c2 = 1.
Solution: Let the plane
lx + my + nz = p …(1)
touch the ellipsoid
x2 / a2 + y2 / b2 + z 2 / c2 = 1 …(2)
at the point (α , β , γ ).
Now the equation of the tangent plane to (2) at (α , β , γ ) is
xα yβ zγ
+ + = 1. …(3)
a2 b2 c2
If (1) touches (2) at (α ,β , γ ) then (1) and (3) represent the same plane and hence
comparing (1) and (3), we get
α / a2 β γ / c2 1
= = =
l m n p
a2 l b2 m c2 n
or α= ,β = ,γ = ⋅ …(4)
p p p
Also as (α , β , γ ) lis on (2), so we have
(α2 / a2 ) + (β2 / b2 ) + ( γ2 / c2 ) = 1
or ( a4 l2 / p2 a2 ) + ( b4 m2 / p2 b2 ) + ( c4 n2 / p2 c2 ) = 1
or a2 l2 + b2 m2 + c2 n2 = p2 , …(5)
which is the required condition.
G-207

Problem 3 : Find the equation of the director sphere of the ellipsoid


x2 / a2 + y2 / b2 + z 2 / c2 = 1. (Kashi 2014)
Solution: The equation of central conicoid of ellipsoid is
x2 y2 z2
+ +
= 1. …(1)
2 2
a b c2
Let the equation of three tangent plane to (1) be
l1 x + m1 y + n1z = a2 l12 + b2 m12 + c2 n12 …(2)

l2 x + m2 y + n2 z = a2 l22 + b 2
m22 + c 2
n22 …(3)

and l3 x + m3 y + n3 z = a2 l32 + b2 m32 + c2 n32 …(4)


where l1 , m1, n1 ; l2, m2, n2 and l3, m3, n3 are the actual d.c.’s of the normals to the
above three tangent planes respectively. If these tangent planes are mutually
perpendicular, then we have
l12 + l22 + l32 = 1, m12 + m22 + m32 = 1, n12 + n22 + n32 = 1 …(5)
 l1m1 + l2 m2 + l3 m3 = 0

 m1n1 + m2 n2 + m3 n3 = 0 …(6)
 n1l1 + n2 l2 + n3 l3 = 0

The director sphere of the surface (1) being the locus of the point of intersection of the
three tangent planes (2), (3) and (4) is obtained by eliminating l1, m1, n1 ; l2, m2, n2 and
l3, m3, n3 between the equation (2), (3) and (4) with the help of the relation (5) and (6).
Squaring and adding (2), (3) and (4), we get
( l1 x + m1 y + n1z )2 + ( l2 x + m2 y + n2 z )2 + ( l3 x + m3 y + n3 z )2

= a2 Σ l12 + b2 Σ m12 + c2 Σn12


or x2 Σl12 + y2 Σ m12 + z 2 Σ n12 + 2 yz Σ m1n1
+ 2zx Σ n1l1 + 2 xy Σl1m1 = a2 + b2 + c2
[Using (5)]
or x2 + y2 + z 2 = a2 + b2 + c2 . [Using (5) and (6)]

Problem 4: Find the equations to the tangent planes to the ellipsoid


x2 / a2 + y2 / b2 + z 2 / c2 = 1 which are parallel to the plane lx + my + nz = 0.
(Kanpur 2006; Meerut 09B,12)
Solution: The equation of the given ellipsoid is
x2 / a2 + y2 / b2 + z 2 / c2 = 1. …(1)
Let the equation of a tangent plane parallel to the plane lx + my + nz = 0 be given by
lx + my + nz = p. …(2)
If the plane (2) touches the ellipsoid (1) then we have [See article 8(B) give complete
proof.]
a2 l2 + b2 m2 + c2 n2 = p2 or p = ± √ ( a2 l2 + b2 m2 + c2 n2 ). …(3)
G-208

Putting the values of pfrom (3) in (2), the required equations of the tangent planes are
lx + my + nz = ± √ ( a2 l2 + b2 m2 + c2 n2 ).
Problem 5: Show that the plane x + 2 y + 3 z = 2 touches the conicoid
x2 − 2 y2 + 3z 2 = 2. (Kumaun 2008, 15; Meerut 12, 13, 13B;
Rohilkhand 06; Bundelkhand 13)
Solution: The equation of the given conicoid is
x2 − 2 y2 + 3z 2 = 2 or (1 / 2) x2 − y2 + (3 / 2) z 2 = 1. …(1)
The equation of the given plane is
x + 2 y + 3z = 2. …(2)
If the plane (2) touches the conicoid (1), then applying the condition
l2 / a + m2 / b + n2 / c = p2 , [See article 8 (A)]
1 4 9
we have + + = 4 or 2 − 4 + 6 = 4 or 4=4
1 / 2 −1 3 / 2
which is true. Hence the given plane (2) touches the given conicoid.

Problem 6: Find the equations to the tangent planes to the hyperboloid 2 x2 − 6 y2 + 3z 2 = 5


which pass through the line 3 x − 3 y + 6z − 5 = 0 = x + 9 y − 3z.
(Kanpur 2009; Purvanchal 08; Rohilkhand 07, 09; Kumaun 12; Agra 13)
Solution: The equation of the given hyperboloid is
2 x2 − 6 y2 + 3z 2 = 5
or (2 / 5) x2 − (6 / 5) y2 + (3 / 5) z 2 = 1. …(1)
The equations of the given line are
3 x − 3 y + 6z − 5 = 0, x + 9 y − 3z = 0. …(2)
The equation of any plane through the line (2) is
3 x − 3 y + 6z − 5 + λ ( x + 9 y − 3z ) = 0
or (3 + λ ) x + ( −3 + 9λ ) y + (6 − 3λ ) z = 5. …(3)
If the plane (3) touches the hyperboloid (1), then applying the condition
l2 / a + m2 / b + n2 / c = p2 [See article 8 (A)], we have
(5 / 2) (3 + λ )2 + ( −5 / 6) ( −3 + 9λ )2 + (5 / 3) (6 − 3λ )2 = (5)2
or 15 (9 + 6λ + λ2 ) − 5 (9 − 54λ + 81λ2 ) + 10 (36 − 36 λ + 9λ2 ) = 150
or 3 (9 + 6λ + λ2 ) − (9 − 54 λ + 81λ2 ) + 2 (36 − 36λ + 9λ2 ) = 30
or −60 λ2 + 60 = 0 or λ2 = 1 or λ = ± 1.
When λ = 1, from (3) the equation of the tangent plane is
4 x + 6 y + 3z = 5.
When λ = −1, from (3) the equation of the tangent plane is
2 x − 12 y + 9z = 5.
x2 y2 z2
Problem 7: A tangent plane to the ellipsoid + +
= 1 meets the coordinate axes in
2 2
a b c2
A, B and C. Find the locus of the centroid of the tetrahedron OABC.
G-209

Solution: The equation of the given ellipsoid is


x2 y2 z2
+ = 1. + …(1)
2 2
a b c2
The equation of the tangent plane to the ellipsoid (1) at the point ( x1, y1, z1) on it is
xx1 yy1 zz1
+ + = 1. …(2)
a2 b2 c2
The plane (2) meets the coordinate axes in the points given by
 a2   b2   c2 
A  , 0, 0  , B  0, , 0  , C  0, 0,  ⋅
 x1   y1   z1 
The coordinates of the origin O are (0, 0, 0 ).
Let (α, β, γ ) be the coordinates of the centroid of the tetrahedron OABC.
1  a2  a2 b2 c2
Then α =  + 0 + 0 + 0  = ,β = ,γ = ⋅
4  x1  4 x1 4 y1 4z1
a2 b2 c2
∴ x1 = , y1 = , z1 = ⋅
4α 4β 4γ
But the point ( x1, y1, z1) lies on the ellipsoid (1).
x12 y12 z2
∴ + + 1 =1
a2 b2 c2
a4 1 b4 1 c4 1 a2 b2 c2
⇒ ⋅ + ⋅ + ⋅ =1 ⇒ + + = 16.
2 2 2 2 2 2 2 2
16α a 16β b 16γ c α β γ2
Hence the locus of the centroid (α, β, γ ) of the tetrahedron OABC is
a2 b2 c2
+ + = 16.
2 2
α β γ2

Problem 8: Find the locus of the foot of the central perpendicular on varying tangent planes to the
ellipsoid x2 / a2 + y2 / b2 + z 2 / c2 = 1. (Kumaun 2013)
Solution: The equation of the given ellipsoid is
x2 / a2 + y2 / b2 + z 2 / c2 = 1. …(1)
The equation of any tangent plane to the ellipsoid (1) is
lx + my + nz = √ ( a2 l2 + b2 m2 + c2 n2 ). …(2)
The centre of the ellipsoid (1) is the origin and hence the equations of the normal to the
plane (2) from the centre (0, 0, 0 ) of (1) are
x y z
= = ⋅ …(3)
l m n
The foot of the perpendicular from the centre on the tangent plane (2) is the point
where the line (3) meets the plane (2) and hence its locus is obtained by eliminating
l, m, n between (2) and (3) and, therefore, is given by
x . x + y . y + z . z = √ ( a2 x2 + b2 y2 + c2 z 2 ).
G-210

Squaring, the required locus is


( x2 + y2 + z 2 )2 = a2 x2 + b2 y2 + c2 z 2 .

Problem 9: Show that the tangent planes at the extremities of any diameter of an ellipsoid are
parallel.
Solution: Let the equation of an ellipsoid be
x2 / a2 + y2 / b2 + z 2 / c2 = 1. …(1)
We know that the centre of the ellipsoid (1) is the origin. Hence every line passing
through (0, 0, 0 ) and intersecting the ellipsoid (1) is a diameter of (1). Thus let the
equations of a diameter of (1) be
x / l = y / m = z / n. …(2)
Any point on (2) is ( lr, mr, nr ). If it lies on (1), then
r2 ( l2 / a2 + m2 / b2 + n2 / c2 ) = 1
or r = ± 1 / √ ( l2 / a2 + m2 / b2 + n2 / c2 ) = ± λ (say).
Now the extremities of a diameter are
( lλ, mλ, nλ ) and ( − lλ, − mλ, − nλ ).
The equations of the tangent planes to (1) at these points are
lλx mλy nλz lx my nz 1
+ + = 1 i. e., + + = …(3)
a 2
b2
c 2
a2
b2 c2 λ
lλx mλy nλz lx my nz 1
and − − − = 1 i. e., + + =− ⋅ …(4)
a2
b2
c 2
a 2
b2
c 2 λ
The equations (3) and (4) are the equations of two parallel planes.

Problem 10: Tangent planes are drawn to the ellipsoid


x2 / a2 + y2 / b2 + z 2 / c2 = 1
through the point (α, β, γ ). Prove that the perpendiculars to them from the origin generate the cone
(αx + βy + γz )2 = ( a2 x2 + b2 y2 + c2 z 2 ).

Solution: The equation of the ellipsoid is


x2 / a2 + y2 / b2 + z 2 / c2 = 1. …(1)
The equation of any plane through the point (α, β, γ ) is
l ( x − α ) + m ( y − β ) + n (z − γ ) = 0
or lx + my + nz = lα + mβ + nγ. …(2)
If the plane (2) touches the ellipsoid (1), then we know that
a2 l2 + b2 m2 + c2 n2 = ( lα + mβ + nγ )2 [See article 8 (B)]. …(3)
Also the equations of the perpendicular from the origin to the plane (2) are given by
x / l = y / m = z / n. …(4)
Eliminating l, m, n between (3) and (4), the required equation of the cone generated by
the line (4) is given by
a2 x2 + b2 y2 + c2 z 2 = ( lx + my + nz )2 .
G-211

Problem 11: Tangent planes are drawn to the central conicoid ax2 + by2 + cz 2 = 1 through
the point (α, β, γ ). Show that the perpendiculars from the centre of the conicoid to these planes
generate the cone (αx + βy + γz )2 = x2 / a + y2 / b + z 2 / c. (Purvanchal 2011)
Solution: The centre of the conicoid ax2 + by2 + cz 2 = 1 is the origin. Hence it is the
same problem as problem 10.

Problem 12: Find the locus of the perpendiculars from the origin to the tangent planes to the
surface x2 / a2 + y2 / b2 + z 2 / c2 = 1 which cut off from its axes intercepts, the sum of whose
reciprocals is equal to a constant k.
Solution: The equation of the given ellipsoid is
x2 / a2 + y2 / b2 + z 2 / c2 = 1. …(1)
The equation of any tangent plane to the ellipsoid (1) is
lx + my + nz = √ ( a2 l2 + b2 m2 + c2 n2 ). …(2)
The intercepts made by the plane (2) on the co-ordinate axes are
√ ( a2 l2 + b2 m2 + c2 n2 ) √ ( a2 l2 + b2 m2 + c2 n2 ) √ ( a2 l2 + b2 m2 + c2 n2 )
, ,
l m n
respectively.
The sum of the reciprocals of these intercepts is given to be k.
l + m+ n
∴ =k
√ ( a2 l2 + b2 m2 + c2 n2 )
or ( l + m + n)2 = k2 ( a2 l2 + b2 m2 + c2 n2 ). …(3)
Now the equations of the perpendicular from the origin to the tangent plane (2) are
x / l = y / m = z / n.
Eliminating l, m, n between (3) and (4), the locus of the perpendicular (4) is given by
( x + y + z )2 = k2 ( a2 x2 + b2 y2 + c2 z 2 ) .

Problem 13: If P be the point of contact of a tangent plane to the ellipsoid which meets the
co-ordinate axes in A, B and C and PL, PM , PN are the perpendiculars from P on the axes, prove
that
OL . OA = a2 , OM . OB = b2 , ON . OC = c2 .
Solution: Let the equation of the ellipsoid be
x2 / a2 + y2 / b2 + z 2 / c2 = 1. …(1)
Let the co-ordinates of P be (α, β, γ ), so that the equation of the tangent plane to the
ellipsoid (1) at P (α, β, γ ) is
αx / a2 + βy / b2 + γz / c2 = 1. …(2)
The plane (2) meets the co-ordinate axes in A, B and C and so OA, OB and OC are the
intercepts made by (2) on the co-ordinate axes hence we have
OA = a2 / α, OB = b2 / β, OC = c2 / γ.
Again it is given that PL is the perpendicular from the point P (α, β, γ ) on the x-axis so
that OL = α.
G-212

Similarly OM = β and ON = γ.
Thus OL . OA = α . ( a2 / α ) = a2 ; OM . OB = β . ( b2 / β ) = b2
and ON . OC = γ . ( c2 / γ ) = c2 .

Problem 14 : A tangent plane to the conicoid ax2 + by2 + cz 2 = 1meets the coordinate axes in
P, Q and R. Prove that the centroid of the triangle PQR lies on the surface
1 1 1
+ + = 9.
ax2 by2 cz 2
Solution: The equation of given conicoid is
ax2 + by2 + cz 2 = 1. …(1)
The equation of any tangent plane to (1) is
l2 m2 n2
lx + my + nz = + + . …(2)
a b c
The plane (2) meets the co-ordinate axes in the points given by
 l2 / a + m2 / b + n2 / c 
P , 0, 0
 l 
 
 l2 / a + m2 / b + n2 / c 
Q 0 ,
 m 
 
 l2 / a + m2 / b + n2 / c 
and R 0 , 0 .
 n 
 
Let (α , β , γ ) be the co-ordinate of the centroid of the triangle PQR, then
l2 / a + m2 / b + n2 / c
+0+0
l 1 2
α= l / a + m2 b + n2 / c
3 3l
1 l2 m2 n2 1 l2 m2 n2
β= m + + ,γ = + + ⋅
3 a b c 3n a b c
l2 m2 n2 l2 m2 n2
Thus 3 lα = + + or 9l2α2 = + +
a b c a b c
l2 bc + m2 ca + n2 ab 1 9l2 bc
9l2α2 = or = .
abc aα2 l2 bc + m2 ca + n2 ab
1 9m2 ac 1 9n2 ab
Similarly = and = .
bβ2 l2 bc + m2 ca + n2 ab cγ 2 l2 bc + m2 ac + n2 ab

On adding these relation


1 1 1  l2 bc + m2 ca + n2 ab 
+ + = 9   = 9.

2 2 2 2 2
aα bβ cγ  l bc + m ca + n2 ab 
G-213

∴ The locus of the centroid (α , β , γ ) is


1 1 1
+ + = 9.
ax2 by2 cz 2

Problem 15: If the line of intersection of perpendicular tangent planes to the ellipsoid whose
equation referred to rectangular axes is x2 / a2 + y2 / b2 + z 2 / c2 = 1
passes through the fixed point (0, 0, k ), show that it lies on the cone
x2 ( b2 + c2 − k2 ) + y2 ( c2 + a2 − k2 ) + ( z − k )2 ( a2 + b2 ) = 0.
Solution: The equation of the given ellipsoid is
x2 / a2 + y2 / b + z 2 / c2 = 1. …(1)
The equation of any plane through the point (0, 0, k ) is
l ( x − 0 ) + m ( y − 0 ) + n (z − k ) = 0
or lx + my + nz = nk. …(2)
If the plane (2) touches the ellipsoid (1), then we have
a2 l2 + b2 m2 + c2 n2 = n2 k2
or a2 l2 + b2 m2 + ( c2 − k2 ) n2 = 0. …(3)
Now the equations of any line through the point (0, 0, k ) are
x−0 y −0 z − k x y z−k
= = or = = ⋅ …(4)
λ µ ν λ µ ν
If the line (4) lies in the plane (2), we have
lλ + mµ + nν = 0. …(5)
The direction cosines l, m, n of the normal to the tangent plane (2) are related by the
relations (3) and (5) and since (3) is a quadratic relation in l, m, n, it shows that there are
two sets of values of l, m, n and hence for every line (4) through the fixed point (0, 0, k )
there are two tangent planes to the ellipsoid (1) which intersect in the line (4).
Eliminating n between (3) and (5), we have
2
lλ + mµ 
a2 l2 + b2 m2 + ( c2 − k2 )   =0
 −ν 
or ( a2 ν2 + λ2 c2 − λ2 k2 ) l2 + 2l mλ µ ( c2 − k2 )
+ ( b2 ν2 + µ 2 c2 − µ 2 k2 ) m2 = 0
or ( a2 ν2 + λ2 c2 − λ2 k2 ) ( l / m)2 + 2λ µ ( c2 − k2 ) ( l / m)
+ ( b2 ν2 + µ 2 c2 − µ 2 k2 ) = 0.
Let its roots be l1 / m1, l2 / m2 .
Then the product of the roots
l l b2 ν2 + µ 2 c2 − µ 2 k 2
= 1 . 2 = ⋅
m1 m2 a2 ν2 + λ2 c2 − λ2 k2
l1 l2 m1 m2 n1 n2
∴ = =
b2 ν2 + ( c2 − k2 ) µ 2 ( c2 − k2 ) λ2 + a2 ν2 a2 µ 2 + b2 λ2
[By symmetry]
G-214

Since the tangent planes are to be perpendicular, we have


l1 l2 + m1 m2 + n1 n2 = 0
or {b2 ν2 + ( c2 − k2 ) µ 2 } + {( c2 − k2 ) λ2 + a2 ν2 } + {a2 µ 2 + b2 λ2 } = 0
or ( b2 + c2 − k2 ) λ2 + ( a2 + c2 − k2 ) µ 2 + ( a2 + b2 ) ν2 = 0. …(6)
Eliminating λ, µ , ν between (4) and (6), the equation of the required cone generated by
(4) is given by
( b2 + c2 − k2 ) x2 + ( a2 + c2 − k2 ) y2 + ( a2 + b2 ) ( z − k )2 = 0.

Comprehensive Problems 2
Problem 1 : Show that the direction cosines of the normal to the ellipsoid
px p y1 pz1
x2 / a2 + y2 / b2 + z 2 / c2 = 1 at the point P ( x1, y1, z1) on it are 1 , , , where p
a2 b2 c2
is the length of the perpendicular drawn from the centre of the given ellipsoid to the tangent plane to
it at the point P ( x1, y1, z1 ).
Solution: Given equation of ellipsoid is
x2 y2 z2
+ + = 1. …(1)
a2 b2 c2
Equation of tangent plane of (1) at P ( x1, y1, z1 ) is
xx1 yy1 zz1
+ + = 1. …(2)
a2 b2 c2
Equation of a line through ( x1, y1, z1) and perpendicular to (1) is
x − x1 y − y1 z − z1
= = ,
x1 / a2 y1 / b2 z1 / c2
which is the equation of normal at ( x1, y1, z1).
If p is the perpendicular distance of (1) from centre (0, 0, 0) then
1
p= .
2 2 2
 x1   y1   z1 
 2 +  2  +  2
a  b  c 
x y z
Also d.c.’s of normals are proportional to 1 , 1 , 1 .
a2 b2 c2
x1
∴ Actual d.c.’s of normal are ,
2 2
2  x1   y1   z1 
a   +   +  
 a2   b2   c2 
y1 z1
,
2 2 2 2 2 2
 x1   y1   z1  x   y  z 
b2  2 +  2  +  2 c2  1  +  1  +  1 
a  b  c   a2   b2   c2 
x1 p y1 p z1 p
i.e. , , .
a2 b2 c2
G-215

Problem 2: If the normal at P to the ellipsoid x2 / a2 + y2 / b2 + z 2 / c2 = 1 meets the


principal planes in G1 , G2 , G3 , show that P G1 : P G2 : P G3 = a2 : b2 : c2 .

If P G12 + P G22 + P G32 = k2 , then show that the locus of the point P is the curve of

x2 y2 z2 k2
intersection of the given ellipsoid and the ellipsoid + + = .
4 4 4
a b c a + b4 + c4
4

Solution: Let the point P be ( x1 , y1 , z1)


Then normal at P is
x − x1 y − y1 z − z1
= =
x1 / a2 y1 / b2 z1 / c2
x − x1 y − y1 z − z1
i.e., = = =r …(1)
2 2 pz1 / c2
p x1 / a py1 / b
1 px py pz
where p= and 1 , 1 , 1
 x1 
2
 y1 
2
 z1 
2 a2 b2 c2
 2 +  2  +  2
a    c 
are the actual d.c.’s of normal.
Now any point on (1) at a distance r from P is
 rpx1 y1 + rpy1 rpz1 
 x1 + 2 , , z1 + .
 a b2 c2 
rpx1
This will lie on x = 0 if x1 + = 0.
a2
a2
∴ PG1 = r = − .
p
b2 c2
Similarly PG2 = − , PG3 = − .
p p
∴ PG12 : PG2 : PG3 = a2 : b2 : c2 .
Now we are given that
PG12 + PG22 + PG32 = K 2

a4 b4 c4
i.e., + + = K2
2 2 2
p p p
1 K2
or =
p2 a4 + b4 + c4
x12 y12 z2 K2
or + + 1 = (putting the value of p).
4 4 4
a b c a + b4 + c4
4

∴ Locus of P ( x1 , y1 , z1 ) is given by
x2 y2 z2 K2
+ + = ⋅
a4 b4 c4 a4 + b4 + c4
G-216

Problem 3: Prove that the lines drawn from the origin parallel to the normals to
ax2 + by2 + cz 2 = 1 at its points of intersection with the plane lx + my + nz = p generate the
cone
 x2 y2 z 2   lx my nz 
2
p2  + +  = +
 +  ⋅
 a b c   a b c 

Solution: The equation of the given conicoid is


ax2 + by2 + cz 2 = 1. …(1)
The equation of the given plane is
lx + my + nz = p. …(2)
Let (α, β, γ ) be a point of intersection of the conicoid (1) with the plane (2), so that we
have
aα2 + bβ2 + cγ 2 = 1 and lα + mβ + nγ = p. …(3)
Now the equations of the normal to the conicoid (1) at (α, β, γ ) are
x−α y −β z − γ
= = ⋅ …(4)
aα bβ cγ
The d.r.’s of this normal are aα, bβ, cγ.
∴ The equations of the line from the origin parallel to the normal (4) are
x y z
= = ⋅ …(5)
aα bβ cγ
From the relations (3), we have
2
 lα + mβ + nγ 
aα2 + bβ2 + cγ 2 =  
 p 
or p2 {aα2 + bβ2 + cγ 2 } = ( lα + mβ + n γ )2
( aα )2 ( bβ )2 ( cγ )2   l ( aα ) m ( bβ ) n ( cγ )2
or p2  + + = + +  ⋅ …(6)
 a b c   a b c 
Eliminating aα, bβ, cγ between (5) and (6), we see that the line (5) generates the cone
 x2 y2 z 2   lx my nz 
2
p2  + +  = +
 +  ⋅ Proved
 a b c  a b c 

Problem 4: Prove that the points on the ellipsoid the normals at which intersect a given straight
line lie on the curve of intersection of the ellipsoid and a conicoid.
Solution: Let the equation of the ellipsoid be
x2 y2 z2
+ + = 1. …(1)
a2 b2 c2
Let (α, β, γ ) be a point on the ellipsoid (1). The equations of the normal at (α, β, γ ) to the
ellipsoid (1) are
x−α y −β z − γ
= = ⋅ …(2)
α / a2 β / b2 γ / c2
G-217

Let the equations of the given line be


x − x1 y − y1 z − z1
= = ⋅ …(3)
l m n
If the normal (2) intersects the given line (3), then the lines (2) and (3) are coplanar so
that we have
 x1 − α y1 − β z1 − γ
 2 2 
α /a β /b γ /c 2 = 0

 l 
m n 
 
 nβ mγ   lγ nα   mα l β 
or ( x1 − α )  −  + ( y1 − β )  2 − 2  + ( z1 − γ )  2 − 2  = 0.
 b2 c2  c a   a b 
∴ The locus of (α, β, γ ) is
 ny mz   lz nx   mx ly 
( x1 − x )  − + ( y1 − y )  − + ( z1 − z )  −  = 0.
 b2 2   2 2   a2 b2 
c c a
…(4)
This is the equation of a conicoid. But the point (α , β, γ )lies on the ellipsoid (1) as well.
Hence the locus of (α, β, γ ) is the curve of intersection of the ellipsoid (1) and the
conicoid (4).

Problem 5: If a length PQ be taken on the normal at any point P of the ellipsoid


x2 y2 z2
+ + =1
a2 b2 c2
such that PQ = λ2 / p where λ is constant and p is the length of the perpendicular from the origin
to the tangent plane at P, the locus of Q is
a2 x2 b2 y2 c2 z 2
+ + = 1.
2 2 2 2 2 2
(a + λ ) (b + λ ) ( c + λ2 )2
2

Solution: The equation of the given ellipsoid is


x2 y2 z2
+ = 1. + …(1)
a2 b2 c2
Let the co-ordinates of a point P on (1) be (α, β, γ ), so that
α2 β2 γ2
+ = 1. + …(2)
2 2
a b c2
The equations of the normal to the ellipsoid (1) at P (α , β, γ ) are
x−α y −β z−γ
= = = r (say), …(3)
pα / a2 pβ / b2 pγ / c 2
where pα / a2 , pβ / b2 , p γ / c2 are the actual d.c.’s of the normal.
Let ( x1, y1, z1 ) be the co-ordinates of a point Q on the normal (3) such that
PQ = λ2 / p (as given).
Then putting x = x1, y = y1, z = z1 and r = λ2 / p in (3), we get
G-218

αλ2 βλ2 γλ2


x1 = α + , y1 = β + , z1 = γ + ⋅
a2 b2 c2
a2 x1 b2 y1 c2 z1
∴ α= ,β = ,γ = ⋅
2 2 2 2
a + λ b + λ c + λ2
2

Putting these values of (α, β, γ ) in (2), we get


a2 x12 b2 y12 c2 z12
+ + = 1.
( a2 + λ2 )2 ( b2 + λ2 )2 ( c2 + λ2 )2
∴ The locus of the point Q ( x1 , y1, z1 ) is given by
a2 x2 b2 y2 c2 z 2
+ + = 1. Proved
2 2 2 2 2 2
(a + λ ) (b + λ ) ( c + λ2 )2
2

Problem 6: The normal at a variable point P of the ellipsoid


x2 / a2 + y2 / b2 + z 2 / c2 = 1
meets the plane z = 0 (i. e., the x y-plane) in G3 and G3 Q is drawn parallel to z-axis and equal to
G3 P. Show that the locus of Q is given by
x2 /( a2 − c2 ) + y2 /( b2 − c2 ) + z 2 / c 2 = 1.
Also find the locus of R if OR is drawn from the centre equal and parallel to G3 P.
Solution: The equation of the ellipsoid is
x2 y2 z2
+ = 1.+ …(1)
2 2
a b c2
Let (α, β, γ ) be the co-ordinates of a point P on the ellipsoid (1), so that
α2 β2 γ2
+ = 1. + …(2)
a2 b2 c2
The equations of the normal to the ellipsoid (1) at P (α , β, γ ) are
x−α y −β z−γ
= = = r (say), …(3)
pα / a2 pβ / b2 pγ / c2
where pα / a2 , pβ / b2 , pγ / c2 are the actual d.c.’s of the normal so that r denotes the
actual distance of any point on the normal (3) from the point P (α, β, γ ). If the normal
(3) meets the plane z = 0 in G3 , then putting z = 0 and r = PG3 in (3), we get
0−γ c2
= PG3 , so that PG3 = − ⋅
pγ / c2 p
Putting r = PG3 = − c2 / p in (3), the co-ordinates of G3 are

(α − c2 α / a2 , β − c2 β / b2 , 0 ).
Now G3 is a point in the xy-plane and G3 Q is a line drawn parallel to z-axis such that
G3 Q = G3 P = − c2 / p. If ( x1, y1, z1 ) are the co-ordinates of Q , then

x1 = x-co-ordinate of G3 = α − ( c2 α / a2 ),

y1 = y-co-ordinate of G3 = β − ( c2 β / b2 ),
G-219

and z1 = G3 Q = − c2 / p.

a2 x1 b2 y1 z c
∴ α= ,β = , 1=− ⋅ …(4)
2 2
a −c b − c2
2 c p
From z1 / c = − c / p, we have
z12 c2 z12  α2 β2 γ2 
= or = c2  + + 
2 2 2 4 4
c p c  a b c4 
[Putting for 1 / p2 from article 10 (B)]
γ2 z2  α2 β2 
or = 1 − c2  + ⋅
 …(5)
c2 c2 a
4
b4 
Putting the values of α, β and γ from (4) and (5) in (2), we get
a2 x12 b2 y12  z 2  α2 β2  
+ +  1 − c2  +  =1

2 2 2 2 2 2 2 4
(a − c ) (b − c )  c a b4  
a2 x12 b2 y12 z2 c2 x12 c2 y12
or + + 1 − − =1
( a2 − c2 )2 ( b2 − c2 )2 c 2 ( a2 − c2 )2 ( b2 − c2 )2
[Again using (4) for α and β]
x12 (a − c ) 2 2
y12 ( b2 − c2 ) z12
or + + =1
( a2 − c2 )2 2
(b − c ) 2 2
c2

x12 y12
z2
or + + 1 = 1.
2 2 2 2
a −c b −c c2
∴ The locus of Q ( x1, y1, z1 ) is
x2 y2 z2
+ + = 1.
a2 − c2 b2 − c2 c2
Second part: The equations of the line QR passing through O (0, 0, 0 ) and drawn
parallel to the normal at P are
x y z
= = = k (say).
pα / a2 pβ / b2 pγ / c2
Taking k = OR = G3 P = − c2 / p, the co-ordinates of R are
 αc2 β c2 
− ,− ,− γ  ⋅
 2
 a a2 
Also let the co-ordinates of R be ( x2 , y2 , z2 ) , so that we have
αc2 β c2
x2 = − , y2 = − , z2 = − γ
a2 a2
a2 x2 b2 y2
or α=− ,β = −
, γ = − z2 .
2
c c2
Putting these values of α, β, γ in (2), we get
G-220

a2 x22 b2 y22 z 2
+ + 2 = 1.
c4 c4 c2
∴ The locus of R ( x2 , y2 , z2 ) is
a2 x2 + b2 y2 + c2 z 2 = c4 . Ans

Problem 7: If A, B, C ; A ′, B ′, C ′ are the feet of the six normals from a given point to the
ellipsoid x / a + y2 / b2 + z 2 / c2 = 1 and the plane ABC is given by lx + my + nz = p,
2 2

prove that the plane A ′ B ′ C ′ is given by ( x / a2 l ) + ( y / b2 m) + ( z / c2 n) + (1 / p) = 0.


(Kumaun 2011)
Solution: The equation of the given ellipsoid is
x2 y2 z2
+ = 1.+ …(1)
a2 b2 c2
Let the co-ordinates of the given point be ( x1, y1, z1). Now the co-ordinates (α, β, γ ) of
the feet of six normals from ( x1, y1, z1 ) to (1) are given by
α = a2 x1 /( a2 + λ ), β = b2 y1 /( b2 + λ ), γ = c2 z1 / ( c2 + λ ) …(2)
where λ is a parameter and its six values are given by the equation
a2 x12 b2 y12 c2 z12
+ + = 1. …(3)
( a2 + λ )2 ( b2 + λ )2 ( c2 + λ )2
Now the equation of the plane ABC is given to be
lx + my + nz = p
and three of the six feet of normals given by (2) lie on this plane, so that we have
la2 x1 mb2 y1 nc2 z1
+ + − p = 0. …(4)
a2 + λ b2 + λ c2 + λ
This being a cubic equation in λ gives us three values of λ.
Again let the equation of the plane A ′ B′ C ′ be
l ′ x + m′ y + n′ z − p′ = 0.
According to the question the remaining three of the six feet of normals given by (2) lie
on this plane and so we have
l ′ a2 x1 m′ b2 y1 n′ c2 z1
+ + − p ′ = 0. …(5)
( a2 + λ ) ( b2 + λ ) ( c2 + λ )
This equation is of third degree in λ and hence gives us the remaining three values of λ.
Clearly the product of the equations (4) and (5) must give the same equation as is
equation (3). Hence comparing the like terms, we get
ll ′ a4 x12 a2 x12 1
= or l ′ = ⋅
2 2
(a + λ ) ( a2 + λ )2 a2 l
Similarly, we have
1 1 1
m′ = , n′ = , p′ = − ⋅
2 2 p
b m c n
G-221

Putting these values in l ′ x + m′ y + n′ z − p′ = 0, the required equation of the plane


A ′ B′ C ′ is given by
x y z 1
+ + + = 0.
a l b m c2 n p
2 2

Problem 8: Find the equation of the cone through the six concurrent normals drawn from a
given point ( x1, y1, z1 ) to the conicoid ax2 + by2 + cz 2 = 1.
Solution: Let the equation of the conicoid be
ax2 + by2 + cz 2 = 1. …(1)
Let P (α, β, γ ) be the foot of a normal to the conicoid (1) passing through a given point
A ( x1, y1, z1).
The equations of the normal to the conicoid (1) at the point (α, β, γ ) on it are
x−α y −β z−γ
= = . …(2)
aα bβ cγ
Since the normal (2) passes through the point A ( x1, y1, z1), we have
x1 − α y − β z1 − γ
= 1 = = r (say).
aα bβ cγ
x1 y1 z1
∴ α= ,β = , γ= . …(3)
1 + ar 1 + br 1 + cr
If l, m, n are direction ratios of the normal AP, i. e., of the line (2), we can clearly take l, m, n
such that
l = a α, m = b β, n = c γ
ax1 by1 cz1
or l= ,m= ,n= [Using (3)]
1 + ar 1 + br 1 + cr
ax1 by cz
or = 1 + ar, 1 = 1 + br, 1 = 1 + cr.
l m n
Multiplying these equations respectively by b − c, c − a, a − b and adding , we get
ax1 by cz
( b − c ) + 1 ( c − a) + 1 ( a − b )
l m n
= (1 + ar ) ( b − c ) + (1 + br ) ( c − a) + (1 + cr ) ( a − b ) = 0.
ax1 by cz
Thus ( b − c ) + 1 ( c − a) + 1 ( a − b ) = 0 . …(4)
l m n
Also the equations to the normal AP are
x − x1 y − y1 z − z1
= = . …(5)
l m n
Eliminating l, m, n between (4) and (5), the equation of the cone on which the line (5)
lies is given by
ax1 ( b − c ) by1 ( c − a) cz1 ( a − b )
+ + = 0.
x − x1 y − y1 z − z1
This is the required equation of the quadric cone on which lie all the six normals drawn
from ( x1, y1, z1) to the conicoid (1).
G-222

Comprehensive Problems 3
Problem 1: Find the pole of the plane lx + my + nz = p with respect to the conicoid

ax2 + by2 + cz 2 = 1.
Solution: Let (α , β , γ ) be the pole of the plane
lx + my + nz = p …(1)
with the respect to the conicoid
ax2 + by2 + cz 2 = 1. …(2)
The polar plane of (α , β , γ ) with respect to (2) is
aαx + bβy + cγz = 1. …(3)
Comparing (1) and (3) which represent the same plane, we get
aα bβ cγ 1
= = = ⋅
l m n p
Hence, we get α = l / pa, β = m / pb , γ = n / pc .
1 m n
∴ The pole of the plane (1) with respect to the conicoid (2) is  , ,  .
 ap bp c p
Problem 2: Find the pole of 4 x + 8 y − 3z = 15 with respect to 3 x2 + 7 y2 + 2z 2 = 12.
Solution: Let (α , β , γ ) be the pole of the plane
4 x + 8 y − 3z = 15 …(1)
with respect to 3 x2 + 7 y2 + 2z 2 = 12. …(2)
The polar plane of (α , β , γ ) with respect to (2) is
3 xα + 7 yβ + 2zγ = 12. …(3)
Comparing (1) and (3), we get
4 8 −3 15 5
= = = = ⋅
3α 7β 2γ 12 4
16 32 12
Hence, we get α= ,β = ,γ = − ⋅
15 35 10
∴ The pole of the plane (1) with respect to (2) is  ,
16 32 6 
,−  .
 15 35 5 

x −1 y −2 z −3
Problem 3: Show that the polar of = = with respect to the conicoid
2 3 4
x+6 y −2 z −2
x2 − 2 y2 + 3z 2 − 4 = 0 is = = ⋅
3 3 1
Solution: The given line is
x −1 y −2 z −3
= = ⋅ …(1)
2 3 4
The equation of polar line of (1) with respect to the given conicoid
 1  x2 −  1  y2 +  3  z 2 = 1
     
 4  2  4
G-223

 1  (1) x + ( −1 /2)(2) y +  3  (3) z = 1


are    
 4  4
 1  (2) x + ( −1 /2)(3) y +  3  (4) z = 0
and    
 4  4
or x − 4 y + 9z = 4 and x − 3 y + 6z = 0
or ( x + 6) − 4 ( y − 2) + 9 ( z − 2) = 0
and ( x + 6) − 3 ( y − 2) + 6 ( z − 2) = 0. (Note)
Solving there simultaneously, we get
x+6 y −2 z −2
= =
−24 + 27 9 − 6 −3 + 4
1 1
or ( x + 6) = ( y − 2) = ( z − 2) .
3 3

Problem 4: Prove that the locus of the pole of the plane lx + my + nz = p with respect to the
x2 y2 z2
system of conicoids + + = 1, where k is a parameter, is a straight line
2 2 2
a + k b + k c + k
perpendicular to the given plane.
Solution: Let (α , β , γ ) be the pole of the plane
lx + my + nz = p …(1)
2 2 2
x y z
with respect to the conicoid + + = 1.
( a2 + k ) ( b2 + k ) ( c2 + k )
The polar plane of (α , β , γ ) with respect to this conicoid is
ax βy γz
+ + = 1. …(2)
2 2 2
(a + k ) (b + k ) (c + k )
Since (1) and (2) represent the same plane, therefore comparing them, we get
α /( a2 + k ) β /( b2 + k ) γ /( c2 + k ) 1
= + =
l ( b2 + k ) n p
1 m n
where α = ( a2 + k ) , β = ( b2 + k ) , γ = ( c2 + k )
p p p
α − ( a2 l / p) k β − ( b2 m / p) γ − ( c2 n / p)
or = = = ⋅
l p m n
∴ The locus of (α , β , γ ) is
x − ( a2 l / p) y − ( b2 m / p) z − ( c2 n / p)
= =
l m n
which is a straight line and its direction cosines being l , m, n is is perpendicular to the
plane (1).

Problem 5: If P ( x1, y1, z1 ) and Q ( x2 , y2 , z2 ) are any two points then find the equations of
the polar of PQ with respect to the conicoid ax2 + by2 + cz 2 = 1.
G-224

Solution: The equation of the given conicoid is


ax2 + by2 + cz 2 = 1. …(1)
Now the equations of the line joining P ( x1, y1, z1) and Q ( x2 , y2 , z2 ) are
x − x1 y − y1 z − z1
= = = r (say).
x2 − x1 y2 − y1 z2 − z1
The co-ordinates of any point on this line PQ are
( x1 + r ( x2 − x1), y1 + r ( y2 − y1), z1 + r ( z2 − z1)).
The polar plane of this point w.r.t. the conicoid (1) is
ax { x1 + r ( x2 − x1)} + by { y1 + r ( y2 − y1)} + cz {z1 + r ( z2 − z1)} = 1
or ( axx1 + byy1 + czz1 − 1) + r { ax ( x2 − x1)
+ by ( y2 − y1) + cz ( z2 − z1)} = 0.
This plane for all values of r will pass through the line given by the planes
axx1 + byy1 + czz1 = 1 …(2)
and ax ( x2 − x1 ) + by ( y2 − y1 ) + cz ( z2 − z1 ) = 0
i. e., axx2 + byy2 + czz2 = axx1 + byy1 + czz1
i. e., axx2 + byy2 + czz2 = 1. …(3)
[Using (2)]
The equations (2) and (3) are the required equations of the polar of the line PQ.

Problem 6: Find the locus of straight lines through a fixed point (α, β, γ ) whose polar lines with
respect to the conicoids ax2 + by2 + cz 2 = 1 and a′ x2 + b ′ y2 + c ′ z 2 = 1 are coplanar.
Solution: The equations of the two conicoids are
ax2 + by2 + cz 2 = 1 …(1)
2 2 2
and a′ x + b ′ y + c ′ z = 1. …(2)
The equations of any line through the point (α, β, γ ) are
x−α y −β z − γ
= = = r (say). …(3)
l m n
The equations of the polar line of the line (3) w.r.t. the conicoid (1) [See article 17] are
aαx + bβy + cγz = 1, alx + bmy + cnz = 0. …(4)
Also the equations of the polar line of the line (3) w.r.t. the conicoid (2) are
a′αx + b ′βy + c ′ γz = 1, a′ lx + b ′ my + c ′ nz = 0. …(5)
The two polar lines given by (4) and (5) are given to be coplanar and hence we have
 aα bβ cγ 1
 al bm cn 0
 = 0
a′ α b ′β c′γ 1
 
 a′ l b′ m c′n 0
( a − a′ )α ( b − b ′ ) β ( c − c ′ ) γ 0
al βµ χν 0
or =0
a′ α b ′β c′γ 1
a′ l b′ m c′n −
G-225

( a − a′ ) α (b − b′ ) β ( c − c ′ ) γ
or  al bm cn  = 0,
 
 a′ l b′ m c′n 
expanding the determinant along the fourth column
or ( a − a′ ) α ( bc ′ − b ′ c ) mn + ( b − b ′ ) β ( ca′ − c ′ a) nl
+ ( c − c ′ ) γ ( ab ′ − a′ b ) lm = 0.
Dividing by lmn throughout, we have
( a − a′ ) ( bc ′ − b ′ c ) (α / l ) + ( b − b ′ ) ( ca′ − c ′ a) ( β / m)
+ ( c − c ′ ) ( ab ′ − a′ b ) ( γ / n) = 0. …(6)
The locus of the straight line (3) is obtained by eliminating l, m, n between (3) and (6),
and therefore, it is given by
 α   β 
( a − a′ ) ( bc ′ − b ′ c )   + ( b − b ′ ) ( ca′ − c ′ a)  
 x − α  y − β
 γ 
+ ( c − c ′ ) ( ab ′ − a′ b )   = 0.
z − γ
Problem 7: Prove that the lines through (α, β, γ ) at right angles to their polars with respect to
x /( a + b ) + y2 /(2a) + z 2 /(2b ) = 1 generate the cone
2

( y − β ) (αz − γx ) + ( z − γ ) (αy − βx ) = 0.
Solution: The equation of the given conicoid is
x2 / ( a + b ) + y2 / (2a) + z 2 / (2b ) = 1.
For convenience let us put
a′ = 1 / ( a + b ), b ′ = 1 / (2a), c ′ = 1 / (2b ).
Using these values the given equation of the conicoid becomes
a′ x2 + b ′ y2 + c ′ z 2 = 1. …(1)
Now proceeding exactly as in Example 7, the equation of the required locus is
α  1 1 β 1 1 γ 1 1
 − +  − +  −  =0
x − α  b′ c ′  y − β  c ′ a′  z − γ  a′ b ′ 
α β γ
or (2a − 2b ) + (2b − a − b ) + ( a + b − 2 a) = 0
x−α y −β z−γ
[Putting the values of a′, b ′, c ′]
2α ( a − b ) β ( a − b ) γ ( a − b )
or − − =0
x−α y −β z−γ
2α β γ
or − − =0
x−α y −β z − γ
 α β   α γ 
or  −  +  −  =0
 x−α y − β  x − α z − γ 
(αy − βx ) αz − γx
or + =0
( x − α ) ( y − β ) ( x − α ) (z − γ )
or ( y − β ) (αz − γx ) + ( z − γ ) (αy − βx ) = 0.
G-226

Comprehensive Problems 4
Problem 1: Find the centre of the conic x2 / 9 + y2 /16 + z 2 / 4 = 1, 2 x + 2 y − z = 3.
Solution: The given conic is the section of the conicoid
x2 / 9 + y2 /16 + z 2 / 4 = 1 …(1)
by the plane 2 x + 2 y − z = 3. …(2)
Let (α, β, γ ) be the centre of the given conic. Then the given conic is the section of the
conicoid by the plane ‘T = S1’
αx βy γz α2 β2 γ2
i. e., + + −1= + + −1
9 16 4 9 16 4
αx βy γz α2 β2 γ2
or + + = + + ⋅ …(3)
9 16 4 9 16 4
The equations (2) and (3) must represent the same planes and hence comparing their
coefficients, we have
(α / 9) ( β /16) ( γ / 4) (α2 / 9) + ( β2 /16) + ( γ 2 / 4)
= = = = k (say).
2 2 −1 3
∴ α = 18k, β = 32k, γ = − 4k …(4)
α2 β2 γ2
and + + = 3k. …(5)
9 16 4
Putting for α, β, γ from (4) in (5), we get
36k2 + 64k2 + 4k2 = 3k or k = 3 / 104 (since k ≠ 0 ).
Putting for k in (4), we get
α = 27 / 52, β = 12 /13, γ = − 3 / 26.
∴ The required centre is (27 / 52, 12 /13, − 3 / 26). Ans.

Problem 2: Prove that the centre of the conic lx + my + nz = p, ax2 + by2 + cz 2 = 1 is the
 lp mp np 
point  , , , where
 ap 2 bp 2 cp 2 
 0 0 0 

l2 + m2 + n2 = 1 and ( l2 / a) + ( m2 / b ) + ( n2 / c ) = p0 2 .

Solution: The given conic is the section of the conicoid


ax2 + by2 + cz 2 = 1 …(1)
by the plane lx + my + nz = p. …(2)
Let (α, β, γ ) be the centre of the given conic. Then the given conic is the section of the
conicoid by the plane ‘T = S1’
i. e., aαx + bβy + cγz = aα2 + bβ2 + cγ 2 . …(3)
The equations (2) and (3) should represent the same planes and hence comparing their
coefficients, we have
aα bβ cγ aα2 + bβ2 + cγ 2
= = = = k (say).
l m n p
G-227

kl km kn
∴ α= ,β = ,γ = …(4)
a b c
and aα2 + bβ2 + cγ 2 = pk
or a . [ k2 l2 / a2 ] + b . [ k2 m2 / b2 ] + c . [ k2 n2 / c2 ] = pk,
using the relations (4)
2
or 2 2
l / a + m / b + n / c = pk 2
or p0 = pk or k = p / p0 2 .
Putting for k in (4), we get
lp mp np
α= ,β = ,γ = ⋅
p0 2 p0 2 p0 2
 lp mp np 
∴ The required centre is  , , ⋅
 p 2 p 2 p 2
 0 0 0 

Problem 3: Prove that the centre of the section of the ellipsoid


x2 / a2 + y2 / b2 + z 2 / c2 = 1
by the plane ABC whose equation is x / a + y / b + c / z = 1is the centroid of the triangle ABC.
Solution: Let (α, β, γ ) be the centre of the section of the ellipsoid
x2 / a2 + y2 / b2 + z 2 / c2 = 1 …(1)
by the plane
x / a + y / b + c / z = 1. …(2)
Now the conic section of the ellipsoid (1) having its centre at the point (α, β, γ ) is given
by the plane ‘ T = S1’
αx βy γz α2 β2 γ2
i. e., + + . = + + …(3)
a2 b2 c2 a2 b2 c2
The equations (2) and (3) represent the same plane and hence comparing their
coefficients, we have
(α / a2 ) ( β / b2 ) ( γ / c2 ) (α2 / a2 ) + ( β2 / b2 ) + ( γ 2 / c2 )
= = = = k (say).
(1 / a) (1 / b ) (1 / c ) 1
∴ α = ak, β = bk, γ = ck …(4)
2 2 2 2 2 2
and α /a + β /b + γ /c = k
a2 k2 b2 k2 c2 k 2
or + +
= k, using the relations (4)
2 2
a c2b
1
or 3k2 = k or k = . [ ∵ k ≠ 0]
3
Putting this value of k in (4), we get
1 1 1
α = a, β = b, γ = c.
3 3 3
∴ The centre of the section is  a, b, c  ⋅
1 1 1
…(5)
3 3 3 
G-228

Again the equation of the plane ABC is given by (2) and so the vertices of the triangle
ABC are A ( a, 0, 0 ), B (0, b, 0 ) and C (0, 0, c ). Hence the co-ordinates of the centroid of
the triangle ABC are  a, b, c  which are same as the co-ordinates of the centre
1 1 1
3 3 3 
given by (5). Proved

Problem 4: Prove that the centres of the sections of ax2 + by2 + cz 2 = 1 by the planes which
are at a constant distance p from the origin lie on the surface
( ax2 + by2 + cz 2 )2 = p2 ( a2 x2 + b2 y2 + c2 z 2 ).
Solution: The equation of the given conicoid is
ax2 + by2 + cz 2 = 1. …(1)
Let (α, β, γ ) be the centre of a plane section of the conicoid (1). Then the equation of the
plane giving this section of the conicoid is ‘T = S1’
i. e., aαx + bβy + cγz = aα2 + bβ2 + cγ 2 . …(2)
If the plane (2) is at a constant distance p from the origin, then
aα2 + bβ2 + cγ 2
p= ⋅
√ ( a α2 + b2 β2 + c2 γ 2 )
2

∴ ( a α2 + b β2 + c γ 2 )2 = p2 ( a2 α2 + b2 β2 + c2 γ 2 ).
Hence the locus of (α, β, γ ) is the surface
( ax2 + by2 + cz 2 )2 = p ( a2 x2 + b2 y2 + c2 z 2 ).

Problem 5: Show that the locus of the middle points of the chords of the conicoid
ax2 + by2 + cz 2 = 1 which pass through a fixed point ( x ′, y ′, z ′ ) is
ax ( x − x ′ ) + by ( y − y ′ ) + cz ( z − z ′ ) = 0.
Solution: The equation of the given conicoid is
ax2 + by2 + cz 2 = 1. …(1)
Let (α, β, γ ) be the middle point of a chord of the conicoid (1). Then this chord lies in the
plane ‘T = S1’ i. e.,
aαx + bβy + cγz − 1 = aα2 + bβ2 + cγ 2 − 1
or aαx + bβy + cγz = aα2 + bβ2 + cγ 2 . …(2)
Now if the chords of the conicoid (1) having (α, β, γ ) as their middle point pass through
the fixed point ( x ′, y ′, z ′ ), then the point ( x ′, y ′, z ′ ) should lie on the plane (2).
Therefore, we have
aαx ′ + bβy ′ + cγz ′ = aα2 + bβ2 + c γ 2
or aα (α − x ′ ) + b β ( β − y ′ ) + cγ ( γ − z ′ ) = 0.
Hence the equation of the required locus of the middle point (α, β, γ ) of the chords of the
conicoid (1) which pass through the fixed point ( x ′, y ′, z ′ ) is
ax ( x − x ′ ) + by ( y − y ′ ) + cz ( z − z ′ ) = 0.
G-229

Problem 6: Prove that the section of the ellipsoid x2 / a2 + y2 / b2 + z 2 / c2 = 1 whose centre

is at the point  a, b, c  passes through the extremities of the axes.


1 1 1
3 3 3 
Solution: The equation of the given ellipsoid is
S ≡ x2 / a2 + y2 / b2 + z 2 / c2 − 1 = 0.

The centre of the section is  a, b, c  and hence the equation of the plane giving
1 1 1
3 3 3 
this section is ‘ T = S1’
x⋅ 1a y⋅ 1b z⋅ 1c ( 1 a)2 ( 1 b )2 ( 1 c )2
i. e., 3 + 3 + 3 −1 = 3 + 3 + 3 −1
2 2 2
a b c a2 b2 c2
or x / a + y / b + z / c = 1. …(1)
The extremities of the axes of the given ellipsoid are ( a, 0, 0 ), (0, b, 0 )and (0, 0, c )and the
plane (1) clearly passes through these three points. Hence the section whose centre is at
the point  a, b, c  passes through the extremities of the axes.
1 1 1
3 3 3 

Comprehensive Problems 5
Problem 1: If OP, OQ and OR be the conjugate semi-diameters of the ellipsoid

x 2 / a2 + y 2 / b 2 + z 2 / c 2 = 1, then prove that OP 2 + OQ 2 + OR 2 = a2 + b2 + c2 .


Solution: Let OP , OQ and OR be any three conjugate semi-diameters of an ellipsoid
x2 y2 z2
+ =1 +
2 2
a b c2
where co-ordinate of P , Q and R are ( x1 , y1 , z1 ), ( x2 , y2 , z2 ) and ( x3 , y3 , z3 ).
Then we have
OP2 + OQ2 + OR2 = ( x12 + y12 + z12 ) + ( x22 + y22 + z22 )
+ ( x32 + y32 + z32 )
= ( x12 + x22 + x32 ) + ( y12 + y22 + y32 ) + ( z12 + z22 + z32 )
2 2 2
=a + b + c . [By article 22 eq. (4)]

Problem 2: Let OP, OQ, OR be the conjugate semi-diameters of 3 x2 + λ y 2 + z 2 = 1.

Find λ if OP2 + OQ2 + OR2 = 2.


Solution: The equation of the given ellipsoid is 3 x2 + λ y 2 + z 2 = 1.
1 1
∴ a2 = , b2 = , c2 = 1.
3 λ
We know that OP2 + OQ2 + OR 2 = a2 + b2 + c2 . [See property I, article 23]
1 1 3
∴ 2 = + + 1 or λ = ⋅
3 λ 2
G-230

Problem 3: Prove that if the plane lx + my + nz = p passes through the extremities P, Q, R of


the three conjugate semi-diameters of the ellipsoid x2 / a2 + y2 / b2 + z 2 / c2 = 1, then
a2 l 2 + b2 m2 + c2 n2 = 3 p2 .
Solution: Rewriting the steps of Example 11 above upto equation (4) and then
squaring (2), (3) and (4) and adding, we get
( lx1 + my1 + nz1 )2 + ( lx2 + my2 + nz2 )2 + ( lx3 + my3 + nz3 )2 = 3 p2
or l2 Σ x12 + m2 Σ y12 + n2 Σ z12 + 2 mn Σ y1z1 + 2nl Σ z1 x1
+ 2lm Σ x1 y1 = 3 p2
2 2 2 2 2 2 2
or l a + m b + n c = 3p .
[Using relations (4) and (5) of article 22]

Problem 4: If OP, OQ and OR be the conjugate semi-diameters of the ellipsoid

x2 y2 z2
+ + = 1, prove that the plane PQR touches the ellipsoid
2 2
a b c2
x2 y2 z2 1
+ + = at the centroid of the triangle PQR.
2 2
a b c2 3
Solution: Proceeding as in Example 11, the equation of the plane PQR is
( x / a2 ) ( x1 + x2 + x3 ) + ( y / b2 ) ( y1 + y2 + y3 ) + ( z / c 2 )
( z1 + z2 + z3 ) = 1 …(1)
Now clearly the co-ordinates of the centroid G of the triangle PQR are
 x1 + x2 + x3 , y1 + y2 + y3 , z1 + z2 + z3  ⋅
 
 3 3 3 
It can be easily seen that this point lies on the ellipsoid
x2 y2 z2 1
+ + = ⋅
2 2
a b c2 3
1 x2 y2 z2
The equation of the tangent plane to the ellipsoid +
at the point G is + =
3 2 2 2
a b c
x . ( x1 + x2 + x3 ) y . ( y1 + y2 + y3 ) z . ( z1 + z2 + z3 ) 1
+ + =
3a2 3b2 3 c2 3
x . ( x1 + x2 + x3 ) y . ( y1 + y2 + y3 ) z . ( z1 + z2 + z3 )
or + + = 1. …(2)
a2 b2 c2
The equation (2) comes out to be the same as the equation (1) and hence it shows that
x2 y2 z2 1
the plane PQR touches the ellipsoid + + = at the centroid of the ∆ PQR .
2 2
a b c2 3
Problem 5: If OP, OQ and OR be the conjugate semi-diameters of the ellipsoid

x / a + y / b2 + z 2 / c2 = 1, prove that the locus of the pole of the plane PQR with respect to
2 2 2

the given ellipsoid is x2 / a2 + y2 / b2 + z 2 / c2 = 3.


G-231

Solution: Let ( x1 , y1 , z1), ( x2 , y2 , z2 ) and ( x3 , y3 , z3 ) be co-ordinates of P , Q and R


respectively. The equation of the plane PQR is (see example 11).
x y z
( x1 + x2 + x3 ) + ( y1 + y2 + y3 ) + ( z1 + z2 + z3 ) = 1. …(1)
a2 b2 c2
Let (α , β , γ ) be the pole of the plane (1) with the respect to the ellipsoid.
x2 y2 z2
+ = 1.+ …(2)
2 2
a b c2
The equation of the polar plane of (α , β , γ ) with respect to the ellipsoid (2) is
αx βy γz
+ + = 1. …(3)
a2 b2 c2
Equations (1) and (3) are the same, then we have
α β γ 1
+ =
x1 + x2 + x3 y1 + y2 + y3 z1 + z2 + z3 1
α = x1 + x2 + x3 , β = y1 + y2 + y3 , γ = z1 + z2 + z3 .
α2
β2 γ 2 ( x1 + x2 + x3 )2 ( y1 + y2 + y3 )2 ( z1 + z2 + z3 )2
Now, + + = + +
2 2
a b c2 a2 2
c2
1 1 1
= [ Σ x12 + 2Σ x1 x2 ] + [ Σ y12 + 2Σ y1l2] + [ Σ z12 + 2Σ z1z2 ]
a2 b2 c2
1 2 1 2 1 2
= [a + 0 ] + [b + 0 ] + [c + 0 ]
2 2
a b c2
[∵ Σ x12 = a2 , Σ y12 = b2, Σ z12 = c2, Σ x1 x2 = 0 , Σ y1 y2 = 0 , Σ z1z2 = 0]
α2 β2 γ2
or + + = 3.
2 2
a b c2
x2 y2 z2
Hence, the locus of (α , β , γ ) is + + = 3.
a2 b2 c2
Problem 6: Prove that the locus of the foot of the perpendicular from the centre of the ellipsoid
x2 / a2 + y2 / b2 + z 2 / c2 = 1 to the plane through the extremities of three conjugate
semi-diameters is a2 x2 + b2 y2 + c2 z 2 = 3 ( x2 + y2 + z 2 )2 .
(Gorakhpur 2005; Agra 07; Avadh 07)
Solution: The equation of the given ellipsoid is
x2 y2 z2
+ = 1. + …(1)
2 2
a b c2
Proceeding exactly as in Example 11above, the equation of the plane PQR is
( x / a2 ) ( x1 + x2 + x3 ) + ( y / b2 ) ( y1 + y2 + y3 )
+ ( z / c2 ) ( z1 + z2 + z3 ) = 1. …(2)
Let N (α, β, γ ) be the foot of the perpendicular from the centre O (0, 0, 0 ) of the ellipsoid
(1) to the plane PQR.
Let p be the length of the perpendicular ON so that
G-232

p = ON = the distance between O (0, 0, 0 ) and N (α, β, γ )


= √ (α2 + β2 + γ 2 ). …(3)
The d.c.’s of ON, the normal to the plane PQR, are
α β γ
, ,
√ (α2 + β2 + γ 2 ) √ (α2 + β2 + γ 2 ) √ (α2 + β2 + γ 2 )
α β γ
i. e., , , , using (3).
p p p
Hence the equation of the plane PQR is also given by
(α / p) x + ( β / p) y + ( γ / p) z = p
or αx + βy + γz = p2. …(4)
Comparing the equations (2) and (4), we get
x1 + x2 + x3 y + y2 + y3 z1 + z2 + z3 1
= 1 = = ⋅
a2 α b2 β c2 γ p2
x1 + x2 + x3 aα y + y2 + y3 bβ z1 + z2 + z3 cγ
∴ = , 1 = , = ⋅
2 2
a p b p c p2
Squaring and adding, we get
( x1 + x2 + x3 )2 ( y1 + y2 + y3 )2 ( z1 + z2 + z3 )2
+ +
a2 b2 c2
a2α2 + b2β2 + c2 γ 2
=
p4
Σ x12 Σ y12 Σ z12  x x  x x   x x 
or + + + 2Σ  1 2  + 2Σ  3 1  + 2Σ  2 3  
  a   a2   a2  
2 2
a b c2 2

a2α2 + b2β2 + c2 γ 2
=
p4
or (1 + 1 + 1 + 0 + 0 + 0 ) p4 = a2α2 + b2β2 + c2 γ 2
[using (4) and (3) of article 22]
or 3 (α2 + β2 + γ 2 )2 = ( a2α2 + b2β2 + c2 γ 2 ), from (3).
∴ The locus of (α, β, γ ) is
3 ( x2 + y2 + z 2 )2 = a2 x2 + b2 y2 + c2 z 2 . Proved.

H ints to O bjective T ype Q uestions

Multiple Choice Questions


x2 y2 z2
1. The equation + − = 1 is the equation of a hyperboloid of one sheet
a2 b2 c2
in standard form.
G-233

x2 y2 z2
2. The equation of the director sphere of the ellipsoid + + = 1 is
2 2
a b c2
x2 + y2 + z 2 = a2 + b2 + c2.
3. See article 11, part (B).
4. See article 22, equations (4).
5. See article 9.
6. See article 8(A).
7. See article 1.
8. See article 1.
9. See article 8.
10. See article 4.
x2 y2 z2
11. The equation of an ellipsoid in standard form is + + = 1.
a2 b2 c2
12. 2 x . (1) + 3 y . (1) − z . ( −1) = 4 i. e., 2 x + 3 y + z = 4.
13. See article 7, part (B).
14. 3 x . (1) + 5 y . ( −1) + 7z . (1) = 15 i. e., 3 x − 5 y + 7z = 15.
15. See article 8, part (A).
16. The director sphere of the central conicoid 3 x2 + 4 y2 − 5z 2 = 1 is the sphere
1 1 1
x2 + y2 + z 2 = + − whose centre is the origin (0, 0, 0 ).
3 4 5
x2 y2 z2
17. The director sphere of the ellipsoid + + = 1 is the sphere
6 4 15
x2 + y2 + z 2 = 6 + 4 + 15 = 25 = 52 whose radius is 5.
18. The equations of the normal to the ellipsoid 3 x2 + 4 y2 + 5z 2 = 64 at the point
(1, 2, 3) on it are
x −1 y −2 z −3 x −1 y −2 z −3
= = i. e., = = ⋅
3 .1 4.2 5.3 3 8 15
19. See article 22, equations (4).
20. See article 22, equations (5).
21. We know that the central conicoid ax2 + by 2 + cz 2 = 1 is an ellipsoid if all
the three constants a, b, c are positive. So x2 + 2 y2 + 3z 2 = 1 represents an
ellipsoid.
x2 y2 z 2
22. The equation of the director sphere of the conicoid + − = 1 is
4 15 10
1 1 1
x2 + y2 + z 2 = + + = 4 + 15 − 10 = 9 = 32.
1 / 4 1 / 5 1 / − 10
The radius of this sphere is 3.
23. The equation of the tangent plane to the conicoid x2 − y2 + z 2 = 1 at the
point (1, 1, 1) on it is
x . 1 − y . 1 + z . 1 = 1 i. e., x − y + z = 1. …(1)
G-234

The length of the perpendicular drawn to the plane (1) from the centre
|0 − 0 + 0 − 1| 1
(0, 0, 0 ) of the given conicoid = = ⋅
√ [12 + ( −1)2 + 12 ] √ 3

Fill in the Blank(s)\


1. See article 2, part (a).
2. See article 7, part (A).
3. See article 8, part (B).
4. See article 9.
5. See article 9.
6. See article 10, part (A).
7. See article 10, part (A).
8. See article 10. Definition of normal.
9. See article 10, part (B).

True or False
1. We know that the central conicoid ax2 + by 2 + cz 2 = 1 is a hyperboloid of two
sheets if any two of the three constants a, b, c are negative and the remaining third
is positive.
So, the equation 3 x2 − 4 y2 − 5z 2 = 1 represents a hyperboloid of two sheets.
2. The equation −2 x2 + 5 y2 + 6z 2 = 4 represents a hyperboloid of one sheet.
3. The equation of the tangent plane to the conicoid 3 x2 − 4 y2 + 5z 2 = 32 at
the point (1, 2, 3) on it is
(3 . 1) x − (4 . 2) y + (5 . 3) z = 32 i. e., 3 x − 8 y + 15z = 32.
4. The equations of the normal to the ellipsoid 2 x2 + 3 y2 + 4z 2 = 18 at the
point ( −1, 2, − 1) on it are
x − ( −1) y − 2 z − ( −1) x+1 y −2 z +1
= = i. e., = = ⋅
2 . ( −1) 3.2 4 . ( −1) −2 6 −4
5. See article 23, Property I.
6. By article 22, relations (5), we have x1 y1 + x2 y2 + x3 y3 = 0.
7. The equations of the normal to the ellipsoid x2 + 2 y2 + 3z 2 = 4 at the point
(1, 0, 1) on it are
x −1 y − 0 z −1 x −1 y − 0 z −1
= = i. e., = = ⋅
1 .1 2.0 3 .1 1 0 3
The direction ratios of this normal are 1, 0, 3 and so its d.c.’s are
1 3
, 0, ⋅
10 10
8. We know that the plane lx + my + nz = p touches the conicoid
l2 m2 n2
ax2 + by2 + cz 2 = 1 if + + = p2 .
a b c
G-235

1 1 2 1
The given conicoid is x2 + 2 y2 + 3z 2 = 6 i. e., x2 + y + z 2 = 1.
6 3 2
1 1 1
So, here a = , b = , c = ⋅
6 3 2
The given plane is x + 2 y − 3z = 6 for which l = 1, m = 2, n = − 3, p = 6.
We have
l2 m2 n2 1 4 9
+ + = + + = 6 + 12 + 18 = 36 = p2 .
a b c 1/6 1/3 1/2
Thus the given plane touches the given conicoid.
9. For the conicoid x2 − y2 − z 2 = 1, we have a = 1, b = − 1, c = − 1.
For the plane x + y + z = 1, we have l = 1, m = 1, n = 1, p = 1.
l2 m2 n2 1 1 1
We have + + = + + = − 1 ≠ p2.
a b c 1 −1 −1
So the given plane does not touch the given conicoid.
❍❍❍
G-236

Chapter-10
The Paraboloids

Comprehensive Problems 1
Problem 1: Show that the plane 2 x − 4 y − z + 3 = 0 touches the paraboloid
2 2
x − 2 y = 3z at the point (3, 3, − 3) . (Gorakhpur 2005; Agra 06; Avadh 06;
Kanpur 07; Kumaun 08, 15; Purvanchal 08)
Solution: Proceed as in Example 1.

Problem 2: By completing the squares find the vertex of the paraboloid


3 x2 + 2 y2 − 12z − 6 x + 8 y − 13 = 0.
Solution: The given equation can be written as
3 ( x2 − 2 x + 1) + 2 ( y2 + 4 y + 4) = 12 ( z + 2)
or 3 ( x − 1)2 + 2 ( y + 2)2 = 12 ( z + 2).
Hence the given equation represents an elliptic paraboloid with its vertex at (1, − 2 , − 2).

Problem 3: Show that the equation to the two tangent planes to the surface
ax2 + by2 = 2z which pass through the line
u1 ≡ l1 x + m1 y + n1z − p1 = 0, u2 ≡ l2 x + m2 y + n2 z − p2 = 0
 l 2 m22   l1 l 2 m1 m2 
u12  + 2n2 p2  − 2u1u2 
2
is + + + n1 p2 + n2 p1 
 a b   a b 
 
l2 m2 
+ u22  1 + 1 + 2 n1 p1  = 0.
 a b 
 
Solution: The equation of the given surface is
ax2 + by2 = 2z. …(1)
The equation of any plane through the given line u1 = 0, u2 = 0 is given by
u1 + λu 2 = 0 …(2)
or l1 x + m1 y + n1z − p1 + λ ( l2 x + m2 y + n2 z − p2 ) = 0
or ( l1 + λl2 ) x + ( m1 + λm2 ) y + ( n1 + λn2 ) z = p1 + λ p2 . …(3)
If the plane (3) touches the surface (1), then applying the condition for tangency
‘l2 / a + m2 / b + 2np / c = 0’, [See equation (4) of article 3]
2
2
( l1 + λl2 ) ( m1 + λm2 ) 2 ( n1 + λn2 ) ( p1 + λ p2 )
we have + + =0
a b 1
G-237

l2 m2   l l m m 
or  1 + 1 + 2 n1 p1 + 2 λ  1 2 + 1 2 + n1 p2 + n2 p1
 a b   a b 
 
l 2 m22 
+ λ2  2 + + 2 n2 p2  = 0. …(4)
 a b 
 
Now putting λ = − u1 / u 2 from (2) in (4), the required equation of the two tangent
planes is given by
l 2 m22   l1 l 2 m1 m2 
u12  + 2 n2 p2  − 2 u1 u 2 
2
+ + + n1 p2 + n2 p1 
 a b   a b 
 
l2 m2 
+ u 22  1 + 1 + 2 n1 p1 = 0.
 a b 
 
Problem 4: Find the equation of the plane which cuts the paraboloid x2 − 2 y2 = 3 z in a conic
whose centre is (1, 2, 3).
Solution: The equation of the given paraboloid is
x2 − 2 y2 = 3z. …(1)
The equation of the plane which cuts the paraboloid (1) in a conic with centre (1, 2, 3) is
given by T = S1,
3
i. e., 1 . x − 2 . 2 y − ( z + 3) = (1)2 − 2 (2)2 − 3 (3)
2
or 2 x − 8 y − 3z = − 23.

Problem 5: The plane 10 x − 24 y = 21 is a diametral plane of the paraboloid


2 2
5 x − 6 y = 7z. Find the equations to the chord through (1, 2, 3) which it bisects.
Solution: The equation of the paraboloid is
5 x2 − 6 y2 = 7z. …(1)
Let l, m, n be the d.c.’s of a system of parallel chords, so that the equation of its diametral
plane with respect to the paraboloid (1) is
alx + bmy − cn = 0 [See article 3, part (vii)]
1
i. e., 5lx − 6 my − ⋅ 7n = 0
2
or 10 lx − 12 my − 7n = 0. …(2)
The equation (2) and the given equation 10 x − 24 y = 21 should be the same.
Comparing them, we have
10 l 12 m 7n l m n
= = or = = ⋅
10 24 21 1 2 3
Hence the d.c.’s of the required chord are proportional to 1, 2, 3. Since it passes through
the point (1, 2, 3), therefore its equations are
x −1 y −2 z −3
= = ⋅
1 2 3
G-238

Problem 6: Prove that the diametral planes x + 3 y = 3, 2 x − y = 1

are conjugate diametral planes for the paraboloid 2 x2 + 3 y2 = 4z.


Solution: The equation of the paraboloid is
2 x2 + 3 y2 = 4z. …(1)
The equations of the diametral planes are given as
x+ 3y =3 …(2)
and 2 x − y = 1. …(3)
We know [See article 3, part (viii)] that two diametral planes are called conjugate if each
bisects the chords parallel to the other.
Let the plane (3) bisect chords parallel to a line whose d.c.’s are l, m, n. Then the plane (3)
should be the same as the diametral plane
alx + bmy − cn = 0
i. e., 2 lx + 3my − 2 n = 0. …(4)
Comparing (3) and (4), we get
2l 3m 2n l m n
= = or = = ⋅
2 −1 1 6 −2 3
∴ l, m, n are proportional to 6, − 2, 3.
Hence the plane (2) should be parallel to the line whose d.c.’s are proportional to
6, − 2 , 3.
The direction ratios of the normal to the plane (2) are 1, 3, 0.
We have 1 . 6 + 3 . ( − 2) + 0 . 3 = 6 − 6 = 0,
showing that the normal to the plane (2) is perpendicular to the line whose d.c.’s are
proportional to 6, − 2, 3.
∴ The plane (2) is parallel to the line whose d.c.’s are proportional to
6, − 2, 3.
Hence the given diametral planes (2) and (3) are conjugate.

Problem 7: Two perpendicular tangent planes to the paraboloid x2 / a2 + y2 / b2 = 2 z


intersect in a line lying on the plane x = 0. Show that the line touches the parabola
y2 = ( a + b ) (2 z + a), x = 0.
Solution: The equation of the paraboloid is
x2 / a2 + y2 / b2 = 2 z. …(1)
Let the two tangent planes intersect in a line lying on the plane x = 0 given by
my + nz = p, x = 0. …(2)
The equation of any plane through the line (2) is given by
my + nz − p + λx = 0
or λx + my + nz = p. …(3)
The plane (3) must be a tangent plane to the paraboloid (1) and hence we have [By
equation (4) of article 3 (iii)]
aλ2 + bm2 + 2 np = 0. …(4)
G-239

This being a quadratic equation in λ gives two values of λ, say, λ1 and λ2.
∴ λ1 λ2 = ( bm2 + 2np) / a.
Hence the equation (3) gives two tangent planes for two values of λ. The d.r.’s of the
normals of these two tangent planes are λ1, m, n and λ2 , m, n. If these tangent planes be
perpendicular, then
λ1 λ2 + m . m + n . n = 0
or ( bm2 + 2np) / a + m2 + n2 = 0
or ( a + b ) m2 + an2 + 2 np = 0. …(5)
Now in order to prove that the line (2) touches a parabola, we should show that the
envelope of (2) under the condition (5) is the parabola in question.
Putting the value of p from (2) in (5), the equations of the line (2) are
( a + b ) m2 + an2 + 2 n ( my + nz ) = 0, x = 0
or ( a + b ) ( m / n)2 + 2 y ( m / n) + ( a + 2 z ) = 0, x = 0. …(6)
The first of these two equations being a quadratic in the parameter m / n, the equation of
the envelope is given by the discriminant of (6) equated to zero
i. e., by 4 y2 − 4 . ( a + b ) ( a + 2 z ) = 0, x = 0
i. e., by y2 = ( a + b ) ( a + 2 z ), x = 0.
This proves the required result.

Problem 8: Tangent planes at two points P and Q of a paraboloid meet on the line CD. Prove
that the plane through CD and middle of PQ is parallel to the axis of the paraboloid.
Solution: Let the equation of the paraboloid be
ax2 + by2 = 2 z. …(1)
Let the co-ordinates of the points P and Q on (1) be ( x1, y1, z1 ) and ( x2 , y2 , z2 )
respectively, so that, we have
ax12 + by12 − 2 z1 = 0 , ax22 + by22 − 2 z2 = 0. …(2)
The equations of the tangent planes to (1) at the points P and Q are
ax1 x + by1 y − ( z + z1) = 0 …(3)
ax2 x + by2 y − ( z + z2 ) = 0. …(4)
The tangent planes (3) and (4) intersect in the line CD. The equation of any plane
through the line CD i. e., through the line of intersection of the planes (3) and (4) is given
by
{ax1 x + by1 y − ( z + z1 )} + λ{ax2 x + by2 y − ( z + z2 )} = 0. …(5)
If the plane (5) passes through the middle point of PQ i. e., through the point
 x1 + x2 , y1 + y2 , z1 + z2  , we have
 
 2 2 2 
 1 ax ( x + x ) + 1 by ( y + y ) − 1
( z1 + z2 ) − z1 
 2 1 1 2
2
1 1 2
2 

+ λ  ax2 ( x1 + x2 ) + by2 ( y1 + y2 ) − ( z1 + z2 ) − z2  = 0
1 1 1
 2 2 2 
G-240

or [( ax12 + by12 − 2 z1 ) + ax1 x2 + by1 y2 − ( z1 + z2 )]


+ λ [( ax22 + by22 − 2 z2 ) + ax1 x2 + by1 y2 − ( z1 + z2 )] = 0.
Using the relations (2), the above equation in λ becomes
[ ax1 x2 + by1 y2 − ( z1 + z2 )] + λ [ ax1 x2 + by1 y2 − ( z1 + z2 )] = 0
giving, λ = − 1.
Putting the value of λ in (5), we get
ax1 x + by1 y − ( z + z1) − ax2 x − by2 y + ( z + z2 ) = 0
or a ( x1 − x2 ) x + b ( y1 − y2 ) y − ( z1 − z2 ) = 0.
The plane given by this equation is clearly parallel to the z-axis i. e., the axis of the
paraboloid (1).

Problem 9: Show that the locus of the tangents from a point (α, β, γ ) to the paraboloid
ax2 + by2 = 2 z is given by ( ax2 + by2 − 2 z ) ( aα2 + bβ2 − 2γ ) = ( aαx + bβy − z − γ )2
and hence deduce the locus of the points from which three mutually perpendicular tangents can be
drawn to the paraboloid.
Solution: The equation of the given paraboloid is
S ≡ ax2 + by2 − 2 z = 0. …(1)
Clearly we are required to find the equation of the enveloping cone of the paraboloid (1)
with it's vertex at (α, β, γ )and so the locus of the tangents i. e., the enveloping cone is given
by
SS1 = T 2
i. e., ( ax2 + by2 − 2 z ) ( aα2 + bβ2 − 2γ ) = ( aαx + bβy − z − γ )2 . …(2)
Now if the cone (2) has three mutually perpendicular generators, then we have
the coefficient of x2 + the coeffi. of y2 + the coeffi. of z 2 = 0

or {a ( aα2 + bβ2 − 2γ ) − a2α2} + {b ( aα2 + bβ2 − 2γ ) − b2 β2 } − 1 = 0


or abβ2 − 2aγ + abα2 − 2bγ − 1 = 0
or ab (α2 + β2 ) − 2 ( a + b ) γ − 1 = 0.
Hence, the required locus of (α, β, γ ) is
ab ( x2 + y2 ) − 2 ( a + b ) z − 1 = 0.

Problem 10: Show that the normals from ( x ′, y ′, z ′ ) to the paraboloid ax2 + by2 = 2cz lie
on the cone
x′ y′ (1 / a − 1 / b )
− + c⋅ = 0.
x − x′ y − y′ z − z′ (Gorakhpur 2006)
OR
Find the equation of the cone through the five feet of the normals drawn from ( x ′, y ′, z ′ ) to the
paraboloid ax2 + by2 = 2cz.
G-241

Solution: The equation of the paraboloid is


ax2 + by2 = 2cz. …(1)
If (α, β, γ ) be a point on (1), then the equations of the normal at (α, β, γ ) are
x−α y −β z − γ
= = ⋅
aα bβ −c
If it passes through ( x ′, y ′, z ′ ), we get
x′−α y ′ − β z′ − γ
= = = λ (say).
aα bβ −c
∴ α = x ′ /(1 + aλ ), β = y ′ /(1 + bλ ), γ = z ′ + cλ . …(2)
There are five points like (α, β, γ ) the normals at which pass through ( x ′, y ′, z ′ ).
[See article 4 (B)]
Now the equations of any line through ( x ′, y ′, z ′ ) are
x− x′ y − y ′ z − z′
= = ⋅ …(3)
l m n
If (3) is a normal to the paraboloid (1) at the point (α, β, γ ), then we clearly have
l = aα, m = bβ, n = − c
ax ′ by ′
or l= , m= , n=−c [Using (2)]
1 + aλ 1 + bλ
ax ′ by ′ c
or = 1 + aλ , = 1 + bλ , = − 1.
l m n
To eliminate λ , multiplying these relations by b, − a and ( b − a)respectively and adding,
we have
abx ′ aby ′ c ( b − a)
− + = b (1 + aλ ) − a (1 + bλ ) − ( b − a)
l m n
abx ′ aby ′ c ( b − a)
or − + = 0.
l m n
Dividing by ab, we get
x′ y ′ c (1 / a − 1 / b )
− + = 0. …(4)
l m n
Eliminating l, m, n between (3) and (4), we get the locus of the normal (3). Hence the
normal (3) lies on the cone
x′ y′ (1 / a − 1 / b )
− + c⋅ = 0. …(5)
x− x′ y− y′ z − z′
Problem 11: Prove that the normals from ( x ′, y ′, z ′ )to the paraboloid x2 / a2 + y2 / b2 = 2z

x′ y′ a2 − b2
lie on the cone − + = 0.
x− x′ y− y′ z − z′ (Kumaun 2008)
2 2
Solution: Replacing a by1 / a , b by1 / b and c by 1 in Problem 10, the required cone is
obtained.

Problem 12: Show that the cubic curve through the five feet of the normals drawn to a
paraboloid from any point ( x ′, y ′, z ′ ) lies on the cone through the five concurrent normals.
G-242

Solution: The equation of the cone through the five feet of normals is given by [as
found in Problem 10]
x′ y′ (1 / a − 1 / b )
− + c⋅ = 0. …(1)
x − x′ y − y′ z − z′
Also the equations of the cubic curve through the five feet of the normals are given by
x′ y′
x= , y= , z = z ′ + cλ . [See equations (2), article 5]
1 + aλ 1 + bλ
Putting these values of x, y, z in (1), we see that the equation (1) is satisfied. Hence the
required result is proved.

H ints to O bjective T ype Q uestions

Multiple Choice Questions


x2 y2
1. The equation of the tangent plane to the paraboloid − = z at the point
2 3
x .8 y .9 1
(8, 9, 5) on it is − = ( z + 5) i. e., 8 x − 6 y − z = 5.
2 3 2
2. See article 4, part (A).
3. In general five normals can be drawn from a given point (α, β, γ ) to the
paraboloid ax2 + by2 = 2 c z. See article 4, part (B).
4. The condition that the plane l x + my + nz = p touches the paraboloid
l2 m2 2 np
ax2 + by2 = 2 c z is + + = 0.
a b c
See article 3, part (iii).

Fill in the Blank(s)


1. The equation ax2 + by2 = 2 c z represents an elliptic paraboloid if a and b are
of the same sign. See article 3.
2. The equation ax2 + by2 = 2 c z represents a hyperbolic paraboloid if a and b
are of opposite signs. See article 3.
3. The equation of the tangent plane at the point (α, β, γ ) on the paraboloid
ax2 + by2 = 2 c z is aα x + bβy = c ( γ + z ).
See article 3, part (ii).
4. The locus of the chords of the paraboloid ax2 + by2 = 2 c z which are bisected
at the point (α, β, γ ) is aα x + bβy − c ( γ + z ) = aα2 + bβ2 − 2 c γ.
See article 3, part (vi).
5. Two diametral planes of the paraboloid ax2 + by2 = 2 c z are called conjugate
if each bisects chords parallel to the other. See article 3, part (viii).
G-243

True or False
1. See article 3, part (vii).
2. The equation of the plane which cuts the paraboloid ax2 + by2 = 2 c z in a
conic whose centre is the point (α, β, γ ) is
a α x + b βy − c ( z + γ ) = aα2 + bβ2 − 2 c γ.
See article 3, part (vi).
3. The locus of the point of intersection of three mutually perpendicular
tangent planes to the paraboloid ax2 + by2 = 2 c z is the plane

2 z + c  +  = 0.
1 1
 a b
See article 3, part (iv).
x2 y2 x2 y2 2z
4. The equation + = − 3 z is of the form + = ⋅
2
4 6 a b2 c
Hence, it represents an elliptic paraboloid.
( x − 2)2 ( y − 5)2
5. The vertex of the elliptic paraboloid + = 3 ( z + 4) is the
4 6
point (2 , 5, − 4).
❍❍❍
G-244

Chapter-11
Generating Lines

Comprehensive Problems 1
Problem 1: Find the equations to the generating lines of the hyperboloid
x2 / 4 + y2 / 9 − z 2 /16 = 1 which pass through the points (2, 3, − 4) and (2, − 1, 4 / 3).
(Avadh 2006, 08)
Solution: The equation of the hyperboloid is
x2 / 4 + y2 / 9 − z 2 /16 = 1. …(1)
Let l, m, n be the d.c.’s of the generator. Hence the equations of the generator through
the point (2, 3, − 4) are given by
x−2 y −3 z + 4
= = = r (say). …(2)
l m n
Any point on (2) is ( lr + 2, mr + 3, nr − 4). If it lies on (1), we get
1 1 1
( lr + 2)2 + ( mr + 3)2 − ( nr − 4)2 = 1
4 9 16
 l2 m2 n2 
+ 2  +
l m n
or r2  + − +  = 0. …(3)
4 9 16   2 3 4
 
If the line (2) is a generator then (3) is an identity in r, the conditions for which are
l2 m2 n2
+ − =0 …(4)
4 9 16
l m n
and + + = 0. …(5)
2 3 4
Eliminating n between (4) and (5), we get
2
l2 m2  l
−  +  = 0
m
+ or lm = 0.
4 9 2 3 
Hence either l = 0 or m = 0.
If l = 0, then from (5)
m / 3 + n / 4 = 0 or m / 3 = n / − 4.
Hence the d.c.’s of one generator are given by
l / 0 = m / 3 = n / − 4. …(6)
If m = 0, then from (5)
l n l n
+ = 0 or = ⋅
2 4 1 −2
Hence the d.c.’s of the other generator are given by
G-245

l /1 = m / 0 = n / − 2. …(7)
Thus the equations of the two generators through the point (2 , 3, − 4) are obtained by
putting the values of l, m, n from (6) and (7) in (2) and are given by
x−2 y −3 z + 4 x−2 y −3 z + 4
= = and = = ⋅
0 3 −4 1 0 −2
Proceeding similarly as above the equations of the two generators through the point
(2, − 1, 4 / 3) are given by
4 4
z− z−
x−2 y +1 3 and x − 2 y + 1 3⋅
= = = =
3 6 10 0 3 −4

Problem 2: Find the equations of the generating lines of the hyperboloid


yz + 2zx + 3 xy + 6 = 0
which pass through the point ( −1, 0, 3). (Meerut 2013B)
Solution: Proceeding exactly as in Problem 1, the two pairs of values of l, m, n are given
by
l m n l m n
= = and = = ⋅
0 1 0 1 −1 3
Hence the equations of the required generators are
x+1 y z −3
x + 1 = 0, z − 3 = 0 and = = ⋅
1 −1 3

Problem 3: Prove that if two generators of the hyperboloid


x2 / a2 + y2 / b2 − z 2 / c2 = 1
through the points P ( a cos α, b sin α, 0 ) and Q ( a cos β, b sin β, 0 ) intersect at right angles,
their projections on the plane z = 0 intersect at an angle θ, where tan θ = (ab / c 2) sin (α − β).
Solution: The equation of the hyperboloid is
x2 / a2 + y2 / b2 − z 2 / c2 = 1. …(1)
The equations of the generator of (1) through the point P ( a cos α, b sin α, 0 ) belonging
to the one system of generators [See Example 1] are
x − a cos α y − b sin α z
= = ⋅ …(2)
a sin α − b cos α c
Also the equations of the generator of (1) through the point Q ( a cos β, b sin β, 0 )
belonging to the other system of generators are
x − a cos β y − b sin β z
= = ⋅ …(3)
a sin β − b cos β −c
If the generators (2) and (3) are at right angles, then using the condition
l 1l 2 + m1m2 + n1n2 = 0, we have
a2 sin α sin β + b2 cos α cos β − c2 = 0. …(4)
Now we know that the projections of the generators (2) and (3) on the plane z = 0 are
tangents to the ellipse x2 / a2 + y2 / b2 = 1, z = 0 [i. e., to the section of (1) by the plane
G-246

z = 0 ] at the points P and Q. Therefore the projections of (2) and (3) on the plane z = 0
are respectively given by
( x / a) cos α + ( y / b ) sin α = 1, z = 0 …(5)
and ( x / a) cos β + ( y / b ) sin β = 1, z = 0. …(6)
The slopes m1 and m2 of the lines (5) and (6) lying in the plane z = 0 are given by
m1 = − ( b cos α ) / ( a sin α ) and m2 = − ( b cos β ) / ( a sin β ).
The angle θ between (5) and (6) is given by
b  cos α cos β 
−  − 
m1 − m2 a  sin α sin β 
tan θ = or tan θ =
1 + m1 m2 b2 cos α cos β
1+ ⋅ ⋅
a2 sin α sin β
− ab [sin β cos α − cos β sin α ] ab sin (α − β )
or tan θ = = ⋅ [Using (4)]
a2 sin α sin β + b2 cos α cos β c2

Problem 4: The generators through P of x2 / a2 + y2 / b2 − z 2 / c2 = 1 meet the principal


elliptic section at A and B. If the median of the triangle APB through P is parallel to the fixed
plane lx + my + nz = 0, show that P lies on the surface z ( lx + my ) + n ( c2 + z 2 ) = 0.
Solution: Let the co-ordinates of one of the two points of intersection of the
generators, say of the point P be ( x1, y1, z1). The equation of the tangent plane to the
hyperboloid
x2 / a2 + y2 / b2 − z 2 / c2 = 1
at the point ( x1, y1, z1 ) on it is
xx1 / a2 + yy1 / b2 − zz1 / c2 = 1. …(1)
The tangent plane (1) meets the plane z = 0 in the line which is given by
xx1 / a2 + yy1 / b2 = 1, z = 0. …(2)
The equations of the line joining the points A ( a cos α, b sin α, 0 ) and
B ( a cos β, b sin β, 0 ) are
1 1 1
( x / a) cos (α + β ) + ( y / b ) sin (α + β ) = cos (α − β ), z = 0. …(3)
2 2 2
The lines given by the equations (2) and (3) are the same.
Hence comparing these equations, we have
x1 / a y1 / b 1
= = ⋅ …(4)
1 1 1
cos (α + β ) sin (α + β ) cos (α − β )
2 2 2
Since the point P ( x1, y1, z1) lies on the given hyperboloid, we have
x12 / a2 + y12 / b2 − z12 / c2 = 1. …(5)
Putting the values of x1 / a and y1 / b from (4) in (5), we get
1 1
cos2 (α + β ) sin2 (α + β )
2 2 z2
+ − 1 =1
1 1 2
cos2 (α − β ) cos2 (α − β ) c
2 2
G-247

1 z2
or − 1 =1
1 2
cos2 (α − β ) c
2
1 1
or z12 / c2 = sec2 (α − β ) − 1 or z12 / c2 = tan2 (α − β )
2 2
1 1
or z1 / c = ± sin (α − β ) / cos (α − β ). …(6)
2 2
Now if M is the mid-point of AB, then
M ≡  a (cos α + cos β ), b (sin α + sin β ), 0 
1 1
2 2 

M ≡  a cos (α + β ) cos (α − β ), b sin (α + β ) cos (α − β ), 0  ⋅


1 1 1 1
i. e.,
 2 2 2 2 
∴ The direction ratios of the median PM are
1 1
x1 − a cos (α + β ) cos (α − β ),
2 2
1 1
y1 − b sin (α + β ) cos (α − β ), z1 − 0
2 2
1 1
or x1 − x1 cos2 (α − β ), y1 − y1 cos2 (α − β ), z1 [Using (4)]
2 2
1 1
or x1 sin2 (α − β ), y1 sin2 (α − β ), z1
2 2
2 1 1
or x1, y1, z1 cosec (α − β ) or x1, y1, z1 1 + cot2 (α − β )
2  2 
or x1, y1, z1 {1 + c2 / z12}. [Using (6)]
It is given that the median PM is parallel to the plane lx + my + nz = 0. Hence we have
lx1 + my1 + nz1 (1 + c2 / z12 ) = 0.
∴ The locus of P ( x1, y1, z1 ) is the surface
lx + my + nz (1 + c2 / z 2 ) = 0 or z ( lx + my ) + n( c2 + z 2 ) = 0 .

Problem 5: If the generators through a point P of the hyperboloid


x2 / a2 + y2 / b2 − z 2 / c2 = 1 meet the principal elliptic section in two points such that the
eccentric angle of one is three times that of the other, prove that P lies on the curve of intersection of
the hyperboloid with the cylinder y2 ( c2 + z 2 ) = 4b2 z 2.
Also if the difference of the eccentric angles is 2θ, then show that the locus of P is the curve of
intersection of the hyperboloid with the cone
x2 / a2 + y2 / b2 = ( z 2 / c2 ) cosec2 θ.
Solution: Here we are given that β = 3α. Proceeding as in Problem 4, we have [See
equations (4) and (6)]
1 1 1
cos (α + β ) sin (α + β ) sin (α − β )
x1 2 y1 2 z1 2
∴ = , = , =± ⋅
a 1 b 1 1
cos (α − β ) cos (α − β ) c cos (α − β )
2 2 2
G-248

x1 cos 2α
= , …(1)
a cos α
y1 sin 2α 2 sin α cos α
= = = 2 sin α or y1 = 2b sin α …(2)
b cos α cos α
and z1 = ± c tan α = ± c sin α / cos α
or z1 = ± ( cy1 ) / (2b cos α ), using (2). …(3)
∴ From (2) and (3), we get
sin α = y1 / (2b ) and cos α = cy1 / (2bz1).
Eliminating α, we have
y12 c2 y12
+ = 1 or y12 ( c2 + z12 ) = 4b2 z12 .
4b2 4b2 z12
∴ The point P lies on the curve of intersection of the given hyperboloid with the
cylinder y2 ( c2 + z 2 ) = b2 z 2 .
1
Second part: If α − β = 2θ i. e., (α − β ) = θ, then we have
2
1 1
cos (α + β ) sin (α + β )
x1 2 y1 2 z
= , = , 1 = ± tan θ.
a cos θ b cos θ c
2 1 2 1
cos (α + β ) + sin (α + β )
x12 y2 2 2 1
∴ + 1 = =
2 2 2
a b cos θ cos2 θ
2
x12 y12 1  z1 
or + = ⋅ ± 
a2 b2 cos2 θ  c tan θ 
x12 y12
z 2 cosec2 θ
or + = 1 ⋅
2 2
a b c2
Hence P lies on the curve of intersection of the given hyperboloid with the cone
x2 / a2 + y2 / b2 = ( z 2 / c2 ) cosec2 θ.

Problem 6: If the generators through P a point on the hyperboloid


x2 / a2 + y2 / b2 − z 2 / c2 = 1
whose centre is O, meet the plane z = 0 in A and B and the volume of the tetrahedron OAPB is
constant and equal to abc / 6, show that P lies on one of the planes z = ± c.
Solution: The equation of the hyperboloid is
x2 / a2 + y2 / b2 − z 2 / c2 = 1.
The co-ordinates of its centre O are (0, 0, 0 ). The co-ordinates of the points A, B and P
[See Problem 4] are
A ( a cos α, b sin α, 0 ), B ( a cos β, b sin β, 0 )
 a cos 1 (α + β ) b sin 1 (α + β ) c sin (α − β )
1

and P 2 , 2 ,± 2 ⋅
 cos 1 (α − β ) 1
cos (α − β )
1
cos (α − β ) 
 2 2 2 
G-249

Hence the volume V of the tetrahedron OAPB is given by

a cos α b sin α 0
a cos β b sin β 0
1 1 1
1 a cos (α + β ) b sin (α + β ) c sin (α − β )
V =± 2 2 2
6 ±
1 1 1
cos (α − β ) cos (α − β ) cos (α − β )
2 2 2

1 1
or V = abc tan (α − β ) {cos α sin β − sin α cos β}.
6 2
1 1 1 1
∴ abc = abc tan (α − β ) sin (α − β ) [ ∵ As given , V = abc ]
6 6 2 6
1 1
or 1 = 2 sin2 (α − β ) or cosec2 (α − β ) = 2
2 2
1 1
or 1 + cot2 (α − β ) = 2 or cot2 (α − β ) = 1
2 2
1
or tan2 (α − β ) = 1
2
1
or z 2 / c2 = 1 [ ∵ z = ± c tan (α − β ) for the point P ]
2
or z = ± c.

Problem 7: Show that the equations to the generating lines through the point (θ, φ) on the
hyperboloid x / a2 + y2 / b2 − z 2 / c2 = 1 are
2

x − a cos θ sec φ y − b sin θ sec φ z − c tan φ


= = ⋅
a sin (θ ± φ) − b cos (θ ± φ) ± c (Avadh 2009)
Solution: The equation of the hyperboloid is
x2 / a2 + y2 / b2 − z 2 / c2 = 1. …(1)
The co-ordinates of any point P (θ, φ) on (1) are
( a cos θ sec φ, b sin θ sec φ, c tan φ).
The equation of the tangent plane at P to (1) is
( x / a) cos θ sec φ + ( y / b ) sin θ sec φ − ( z / c ) tan φ = 1. …(2)
Its intersection with the plane z = 0 is the line given by
( x / a) cos θ sec φ + ( y / b ) sin θ sec φ = 1, z = 0
or ( x / a) cos θ + ( y / b ) sin θ = cos φ, z = 0. …(3)
We know that the generating lines through the point P are the lines of intersection of
the hyperboloid (1) with the tangent plane (2).
Now the principal elliptic section of (1) by the plane z = 0 is
x2 / a2 + y2 / b2 = 1, z = 0. …(4)
G-250

Let the line (3) meet the elliptic section (4) in the points A ( a cos α, b sin α, 0 ) and
B ( a cos β, b sin β, 0 ). The equations to the line AB are
1 1 1
( x / a) cos (α + β ) + ( y / b ) sin (α + β ) = cos (α − β ), z = 0.
2 2 2
…(5)
The equations (3) and (5) should represent the same line and hence comparing them,
we have
1 1
θ = (α + β ) and φ = (α − β ). …(6)
2 2
Solving the equations (6), we get
α = θ + φ, β = θ − φ.
Therefore, PA and PB are the two generators through the point P and to find their
equations, we are to determine their direction ratios.
The d.r.’s of PA are
a (cos α − cos θ sec φ), b (sin α − sin θ sec φ), c (0 − tan φ)
or a {cos (θ + φ) − cos θ / cos φ}, b {sin (θ + φ) − sin θ / cos φ},
− c sin φ / cos φ.
Now multiplying throughout by cos φ / sin φ, the d.r.’s of PA are
 cos (θ + φ) cos φ − cos θ  sin (θ + φ) cos φ − sin θ 
a , b  , − c
 sin φ   sin φ 
 cos θ cos2 φ − sin θ sin φ cos φ − cos θ 
or a ,
 sin φ 
sin θ cos φ + cos θ sin φ cos φ − sin θ 
2
b , − c
 sin φ 
 − sin θ sin φ cos φ − cos θ (1 − cos φ)
2
or a ,
 sin φ 
sin φ cos θ cos φ − sin θ (1 − cos φ)
2
b , − c
 sin φ 
or − a {sin θ cos φ + cos θ sin φ}, b {cos θ cos φ − sin θ sin φ}, − c
or a sin (θ + φ), − b cos (θ + φ), c.
∴ The equations to the generator PA are
x − a cos θ sec φ y − b sin θ sec φ z − c tan φ
= = ⋅
a sin (θ + φ) − b cos (θ + φ) c
Proceeding in a similar way for the d.r.’s of PB, we see that d.r.’s of PB are
a sin (θ − φ), − b cos (θ − φ), − c.
Hence the equations to the generator PB are
x − a cos θ sec φ y − b sin θ sec φ z − c tan φ
= = ⋅
a sin (θ − φ) − b cos (θ − φ) −c
G-251

Problem 8: Prove that the angle α between the generators through any point P on the
hyperboloid x2 / a2 + y2 / b2 − z 2 / c2 = 1 is given by
2abc
tan α = ,
p ( a2 + b2 − c2 − OP2 )
where p is the length of perpendicular from the centre on the tangent plane at P. Hence or
otherwise, find the locus of the point of intersection of perpendicular generators.
Solution: The co-ordinates of the centre O are (0, 0, 0 ) and those of the point P are
( a cos θ sec φ, b sin θ sec φ, c tan φ).
∴ OP2 = ( a2 cos2 θ + b2 sin2 θ + c2 sin2 φ) / cos2 φ. …(1)
The equation of the tangent plane at the point P is
( x / a) cos θ sec φ + ( y / b ) sin θ sec φ − ( z / c ) sin φ sec φ = 1. …(2)
∴ p = the length of perpendicular from O (0, 0, 0 ) to (2)
= 1 / [cos2 θ sec2 φ / a2 + sin2 θ sec2 φ / b2 + sin2 φ sec2 φ / c2 ]1 /2
or ( abc / p) cos φ = [ b2 c2 cos2 θ + c2 a2 sin2 θ + a2 b2 sin2 φ]1 /2 . …(3)
The d.r.’s of the two generators through P are [See problem 7]
a sin (θ + φ), − b cos (θ + φ), c ; and a sin (θ − φ), − b cos (θ − φ), − c.
∴ The angle α between these generators is given by
√ [ Σ ( l1 m2 − m1l 2 )2 ]
tan α = ⋅ …(4)
l1l 2 + m1m2 + n1n2
For the sake of convenience let us calculate Nr. and Dr. separately. We have
(Nr. )2 = [ − ab {sin (θ + φ) cos (θ − φ) − cos (θ + φ) sin (θ − φ)}]2
+ [ bc {cos (θ + φ) + cos (θ − φ)}]2 + [ ca {sin (θ + φ) + sin (θ − φ)}]2
= a2 b2 sin2 2φ + b2 c2 (2 cos θ cos φ)2 + c2 a2 (2 sin θ cos φ)2 .
∴ Nr. = √ (4 cos2 φ) {a2 b2 sin2 φ + b2 c2 cos2 θ + c2 a2 sin2 θ}1 /2
= 2 cos φ . ( abc / p) cos φ, [Using (3)]
= (2abc / p) cos2 φ.
Also Dr.= a2 sin (θ + φ) sin (θ − φ) + b2 cos (θ + φ) cos (θ − φ) − c2
= a2 (sin2 θ − sin2 φ) + b2 (cos2 θ − sin2 φ) − c2
= a2 (cos2 φ − cos2 θ) + b2 (cos2 φ − sin2 θ) − c2
= ( a2 + b2 ) cos2 φ − c2 cos2 φ . sec2 φ − ( a2 cos2 θ + b2 sin2 θ)
= cos2 φ [ a2 + b2 − c2 (1 + tan2 φ) − ( a2 cos2 θ + b2 sin2 θ) / cos2 φ]
= cos2 φ [ a2 + b2 − c2 − {a2 cos2 θ + b2 sin2 θ + c2 sin2 φ} / cos2 φ]
= cos2 φ [ a2 + b2 − c2 − OP2 ], [Using (1)]
2abc
∴ tan α = ,
p ( a2 + b2 − c2 − OP2 )
on putting the values of Nr. and Dr. in (4).
G-252

If α = 90 ° i. e., the generators intersect at right angles, then


a2 + b2 − c2 − OP2 = 0 or OP2 = a2 + b2 − c2
or x2 + y2 + z 2 = a2 + b2 − c2 , which is the required locus.

Problem 9: If A and A′ are the extremities of the major axis of the principal elliptic section and
any generator meets two generators of the same system through A and A′ respectively in P and P′,
prove that AP . A ′ P ′ = b2 + c2 .
Solution: Let the equation of the hyperboloid be
x2 / a2 + y2 / b2 − z 2 / c2 = 1.
The point of intersection of a generator of the λ-system with a generator of the µ-system
is
( a (1 + λ µ ) / ( λ + µ ), b ( λ − µ ) / ( λ + µ ), c (1 − λ µ ) / ( λ + µ )). …(1)
[See article 3, part III]
The extremities A and A′ of the major axis of the principal elliptic section are A ( a, 0, 0 )
and A ′ ( − a, 0, 0 ). Thus the y and z co-ordinates of both these extremities are zero. This
shows that for these extremities λ − µ = 0 and 1 − λ µ = 0. Hence for these extremities
λ = µ and 1 − λ2 = 0 i. e., λ = ± 1.
The value λ = 1 gives the extremity A and the value λ = − 1 gives the extremity A′.
First consider the generator through the extremity A ( a, 0, 0 ) given by λ = 1. Putting
λ = 1 in (1), the co-ordinates of the point of intersection P of this generator with a
generator of the µ-system are given by
( a, b (1 − µ ) / (1 + µ ), c (1 − µ ) / (1 + µ ))
or ( a, bt, ct), where t = (1 − µ ) / (1 + µ ).
∴ AP2 = ( a − a)2 + ( bt − 0 )2 + ( ct − 0 )2 = ( b2 + c2 ) t2 . …(2)
Again consider the generator through the extremity A ′ ( − a, 0, 0 ) given by λ = − 1.
Putting λ = −1 in (1), the co-ordinates of the point of intersection P′ of this generator
with a generator of the µ-system are given by
( − a, b (1 + µ ) / (1 − µ ), − c (1 + µ ) / (1 − µ )) or ( −a, b / t, c / t).
∴ A ′ P ′2 = ( b2 + c2 ) / t2 . …(3)
Multiplying (2) and (3), we get
AP2 . A ′ P ′2 = ( b2 + c2 )2 i. e., AP . A ′ P ′ = b2 + c2.
Note: In problem 9, if we take the extremities B and B′ of the minor axis instead of the
extremities A and A′ of the major axis, we have
BP . B′ P ′ = b2 + c2 .
Problem 10: Show that the equations y − λz + λ + 1 = 0, ( λ + 1) x + y + λ = 0
represent for different values of λ generators of one system of the hyperboloid
yz + zx + xy + 1 = 0 and find the equations of the generators of the other system.
Solution: The given equations may be written as
y + 1 = λ ( z − 1), and x + 1 = ( −1 / λ ) ( x + y ). …(1)
Eliminating λ between the equations (1), we have
G-253

( y + 1) ( x + 1) = − ( z − 1) ( x + y )
or xy + yz + zx + 1 = 0,
which is the equation of the given hyperboloid. Hence the equations (1) represent the
generators of the λ-system.
Now the generators of the other system (i. e., µ-system) are given by
y + 1 = µ ( x + y ), x + 1 = − (1 / µ ) ( z − 1)
or µx + (µ − 1) y − 1 = 0, µx + z − 1 + µ = 0.
Thus the projections of the generators of a hyperboloid on a principal plane are tangents
to the section of the surface by the principal plane.
In a similar way the projections on the principal planes y = 0 and z = 0 can be
considered.

Comprehensive Problems 2
Problem 1: Show that the point of intersection of the generators of xy = az which are inclined
at a constant angle α, lies on the curve of intersection of the paraboloid and the hyperboloid
x2 + y2 − z 2 tan2 α + a2 = 0.
Solution: The equation of the given paraboloid is
xy = az. …(1)
The two generators of the λ- and µ-systems can be taken as
x = aλ, y = z / λ …(2)
and x = z / µ , y = aµ . …(3)
The symmetrical forms of (2) and (3) are respectively given by
x − aλ y z x y − aµ z
= = and = = ⋅
0 1 λ 1 0 µ
The angle α between these generators is given by
0 .1 + 1 . 0 + λ µ (1 + λ2 ) (1 + µ 2 )
cos α = or sec2 α =
2 2
(1 + λ ) (1 + µ ) λ2µ 2
1 + λ2 + µ 2 + λ2µ 2 1 + λ2 + µ 2
or 1 + tan2 α = or tan2 α =
λ2µ 2 λ2µ 2
2 2 2 2 2
or 1 + λ + µ = λ µ tan α. …(4)
The point of intersection of the generators (2) and (3) is
x = aλ, y = aµ , z = aλ µ .
∴ λ = x / a, µ = y / a, λ µ = z / a. …(5)
Eliminating λ, µ and λµ between (4) and (5), we have
1 + x2 / a2 + y2 / a2 = ( z 2 / a2 ) tan2 α
or x2 + y2 − z 2 tan2 α + a2 = 0. …(6)
The point of intersection also lies on (1). Hence the required locus is the curve of
intersection of the surfaces (1) and (6).
G-254

Problem 2: Show that the polar lines with respect to the sphere x2 + y2 + z 2 = a2 of the
generators of the quadric x2 − y2 = 2az all lie on the quadric x2 − y2 = − 2az.
Solution: Any generator of x2 − y2 = 2az belonging to the λ-system is
x − y = λz, x + y = 2a / λ.
Its symmetrical form is
x − a/λ y − a/λ z
= = = r (say). …(1)
1 −1 2/λ
The polar plane of any point ( r + a / λ, − r + a / λ, 2r / λ ) on the generator (1) with
respect to the given sphere x2 + y2 + z 2 = a2 is
x ( r + a / λ ) + y ( − r + a / λ ) + z (2r / λ ) = a2
or r ( x − y + 2 z / λ ) + a ( x / λ + y / λ − a) = 0 .
∴ The polar line of the λ-generator is
x − y + 2z / λ = 0, x / λ + y / λ − a = 0
or x − y = − 2z / λ, ( x + y ) / λ = a.
Eliminating λ between these equations, we get
( x − y ) / a = − 2z /( x + y ) or x2 − y2 = − 2az,
which is the required quadric on which the polar lines lie.

Problem 3: Show that the equations to the generators through the point “r, θ” on the hyperbolic
paraboloid x / a2 − y2 / b2 = 2z are
2

1
z − r2 cos 2θ
x − ar cos θ y − br sin θ 2
= = ⋅
a ± b r ( cos θ + sin θ)
Solution: Any point “r, θ” on the given hyperbolic paraboloid is given by
1
x = ar cos θ, y = br sin θ, z = r2 cos 2θ. …(1)
2
The generators of λ- and µ-systems are given by
x / a − y / b = λz , x / a + y / b = 2 / λ …(2)
and x / a − y / b = 2 / µ , x / a + y / b = µz. …(3)
Their d.r.’s are given by [See article 9]
l m n l m n
= = and = = ⋅
a −b 2 / λ a b 2 /µ
If the λ-generator passes through the point (1), then
x / a + y / b = 2 / λ gives 2 / λ = r (cos θ + sin θ).
And if the µ-generator passes through the point (1), then
x / a − y / b = 2 / µ gives 2 / µ = r (cos θ − sin θ).
Hence the direction ratios of the two generators of the opposite systems passing
through the point (1) are
a, − b, r (cos θ + sin θ) and a, b, r (cos θ − sin θ).
∴ The equations of the two generators through the point “r, θ” given by (1) are
G-255

1
z − r2 cos 2θ
x − ar cos θ y − br sin θ 2
= =
a −b r (cos θ + sin θ)
1
z − r2 cos 2θ
x − ar cos θ y − br sin θ 2
and = = ⋅
a b r (cos θ − sin θ)
Problem 4: Show that the angle between generating lines of x2 / a2 − y2 / b2 = 2z through
1 /2 −1
 x2 y2   2 2
( x, y, z ) is given by tan θ = ab 1 + + z + a − b  ⋅
 a4 b4   2 
 
Solution: Proceeding as in article 8, part III, the point of intersection ( x, y, z ) of the
two generators of λ- and µ-systems is given by
λ + µ µ − λ 2
x=a , y=b , z= ⋅ …(1)
 λµ   λµ  λµ
Also their d.r.’s are aλ, − bλ, 2 and aµ , bµ , 2. [See article 9]
If θ is the angle between these generators, then
[ Σ ( m1n2 − m2 n1 )2 ]1 /2
tan θ =
l1l2 + m1m2 + n1n2
[( − 2bλ − 2bµ )2 + (2aµ − 2aλ )2 + (2abλµ )2 ]1 /2
or tan θ =
a2 λµ − b2 λµ + 4
1 /2
 λ + µ
2 2 
2 µ − λ
4b2   + 4a 
2 2
 + 4a b 
  λµ   λµ  
 
=
(a2 − b2 ) + 4 / λµ
[4b ( x / a ) + 4a2 ( y2 / b2 ) + 4a2 b2 ]1 /2
2 2 2
= , using (1)
( a2 − b2 ) + 2z
= 2ab [ x2 / a4 + y2 / b4 + 1]1 /2 / {( a2 − b2 ) + 2 z}
1 /2 −1
 x2y2   2 2
= ab 1 + + z + a − b  .
 a4 b4   2 
 
Problem 5: Find the equations to the generators of the paraboloid
( x + y + z ) (2 x + y − z ) = 6z,
which pass through the point (1, 1, 1).
Solution: The two generators of the given surface belonging to the λ- and µ-systems
are given by
x + y + z = λz , 2 x + y − z = 6 / λ …(1)
and x + y + z = 6 / µ , 2 x + y − z = µz. …(2)
If these pass through (1, 1, 1), we have λ = 3, µ = 2.
Hence the two generators of the opposite systems are
G-256

x + y − 2z = 0, 2 x + y − z = 2 …(3)
and x + y + z = 3, 2 x + y − 3z = 0. …(4)
The d.r.’s of (3) and (4) are respectively given by −1, 3, 1 and −4, 5, 1. Since (3) and (4)
both pass through the point (1, 1, 1), their equations in symmetrical forms are given by
x −1 y −1 z −1 x −1 y −1 z −1
= = and = = ⋅
−1 3 1 −4 5 1

H ints to O bjective T ype Q uestions

Multiple Choice Questions


1. See article 4.
2. See Problem 3 of Comprehensive Problems 1.
3. See Problem 6 of Comprehensive Problems 1.
4. See article 10.

Fill in the Blank(s)


1. See article 3(I).
2. See article 3, Corollary.
3. See article 5.
4. See article 10.

True or False
1. See article 3(III).
2. The angle is tan−1 (2r / a). See Example 5.
❍❍❍
G-257

Chapter-12
The Plane Sections of Conicoids

Comprehensive Problems 1
Problem 1: Prove that the axes of the section of the conicoid ax2 + by2 + cz 2 = 1 by the plane
lx + my + nz = 0 lie on the cone
( b − c ) l / x + ( c − a) m / y + ( a − b ) n / z = 0 . (Avadh 2006)
Solution: Let λ, µ , ν be the d.r.’s of the axes of the section, then proceeding as in article
3, we have [See equation (8)]
λ ( ar2 − 1) / l = µ ( br2 − 1) / m = ν ( cr2 − 1) / n = k (say).
∴ lk / λ = ( ar 2 − 1), mk / µ = ( br2 − 1), nk / ν = cr2 − 1.
Multiplying the above relations by ( b − c ), ( c − a) and ( a − b ) respectively and adding,
we get
l ( b − c ) / λ + m ( c − a) / µ + n ( a − b ) / ν = 0 .
∴ The axes x / λ = y / µ = z / ν lie on the cone
l ( b − c ) / x + m ( c − a) / y + n ( a − b ) / z = 0 .

Problem 2: Prove that the central section of an ellipsoid whose area is constant envelopes a cone
of second degree.
Solution: Let the area of the section of the ellipsoid
x2 / a2 + y2 / b2 + z 2 / c2 = 1
by the plane lx + my + nz = 0 be constant, say equal to πabc / d.
By equation (13) of article 3, we have
1 /2
 l2 + m2 + n2  πabc
πabc  = ⋅
22 2 2 2 2
a l + b m + c n  d
Squaring, a2 l2 + b2 m2 + c2 n2 = d2 ( l2 + m2 + n2 )
or l2 ( a2 − d2 ) + m2 ( b2 − d2 ) + n2 ( c 2 − d2 ) = 0.
Hence the plane lx + my + nz = 0 envelopes the cone
x2 /( a2 − d2 ) + y2 /( b2 − d2 ) + z 2 /( c2 − d2 ) = 0.

Problem 3: If A1, A2, A3 are the areas of the sections of the ellipsoid
2 2 2 2 2 2
x / a + y / b + z / c = 1 by the diametral planes of three mutually perpendicular
semi-diameters of lengths r1 , r2 , r3 , then show that
G-258

A12 A22 A32  b2 c 2 c2 a2 a2 b2 


+ + = π2  + +  ⋅
r12 r22 r32  a 2
b 2
c2 
Solution: The given ellipsoid is
x2 / a2 + y2 / b2 + z 2 / c2 = 1. …(1)
Let x / lk = y / mk = z / nk ( k = 1, 2 , 3) be the equations and rk , k = 1, 2 , 3 be the lengths of
three mutually perpendicular semi-diameters of (1) so that
l12 + l22 + l32 = m12 + m22 + m32 = n12 + n22 + n32 = 1. …(2)
The extremity ( l1r1, m1r1, n1r1 ) of the first semi-diameter will lie on (1) if
( l12 / a2 + m12 / b2 + n12 / c2 ) r12 = 1. …(3)
The diametral plane of the semi-diameter x / l1 = y / m1 = z / n1 is
xl1 / a2 + ym1 / b2 + zn1 / c2 = 0. …(4)
∴ A1 = the area of the section of (1) by the plane (4)
√ ( l12 / a4 + m12 / b4 + n12 / c4 )
= π abc ⋅
√ ( l12 / a2 + m12 / b2 + n12 / c2 )
A12 l 2 m2 n2
∴ = π2 a2 b2 c2  1 + 1 + 1 , [Using (3)]
r12  a4
b4
c4 
with similar expressions for A22 / r22 and A32 / r32.
A12 A22 A32 1 1 1 
∴ + + = π2 a2 b 2 c2  Σl12 + Σ m12 + Σn12 
r12 r22 r32 a4
b4
c 4

1 1 1
= π2 a2 b 2 c2  + +  [Using (2)]
 a4 b4 c4 
 b 2 c2 c2 a2 a2 b 2 
= π2  + +  ⋅
2 2
 a b c2 

Problem 4: Prove that the axes of the sections of the conicoid ax2 + by2 + cz 2 = 1 which pass
through the line x / l = y / m = z / n lie on the cone
 b − c  ( mz − ny ) +  c − a  ( nx − lz ) +  a − b  ( ly − mx) = 0.
     
 x   y   z 

Solution: Let x / l1 = y / m1 = z / n1 …(1)


and x / l2 = y / m2 = z / n2 …(2)
be the equations of the principal axes of the section through the given line
x / l = y / m = z / n. …(3)
Now proceeding as in Example 4, we have
al1l2 + bm1m2 + cn1n2 = 0 …(4)
and l1l2 + m1m2 + n1n2 = 0. …(5)
Since the lines (1), (2) and (3) are coplanar, we have
G-259

 l1 m1 n1 
l m2 n2 = 0
2
 
l m n
or l1 ( m2 n − n2 m) + m1 ( n2 l − l2 n) + n1 ( l2 m − lm2 ) = 0. …(6)
Eliminating l1, m1, n1 from (5), (4) and (6), we have
 l2 m2 n2 
 
al2 bm2 c n2 = 0. …(7)

m n − n m 

 2 2 n2 l − l2 n l2 m − m2 l
Again eliminating l2, m2 , n2 from (2) and (7), the equation of the required cone is
 x y z 
 ax by cz  = 0
 
 yn − zm zl − xn xm − yl 
 b − c  ( mz − ny ) +  c − a  ( nx − lz ) +  a − b  ( ly − mx) = 0,
or      
 x   y   z 
expanding the determinant along the third row.

Problem 5: One axis of a central section of the conicoid ax2 + by2 + cz 2 = 1 lies in the plane
lx + my + nz = 0. Show that the other line lies on the cone
( b − c ) lyz + ( c − a) mzx + ( a − b ) nxy = 0.
Solution: Let the equations of the two axes of a central plane section be
x / l1 = y / m1 = z / n1 …(1)
and x / l2 = y / m2 = z / n2. …(2)
Now proceeding as in Example 4, we have
l1l2 + m1m2 + n1n2 = 0, …(3)
and al1l2 + bm1m2 + cn1n2 = 0. …(4)
As given let the axis (1) lie in the given plane lx + my + nz = 0, so that
ll1 + mm1 + nn1 = 0. …(5)
Eliminating l1, m1, n1 between (5), (3) and (4), we get
 l m n 
 
l2 m2 n2 = 0

al 

 2 bm 2 cn 2 
or l m2 n2 ( b − c ) + m n2 l2 ( c − a) + nm2 l2 ( a − b ) = 0. …(6)
∴ The locus of the axis (2) is the cone given by
lyz ( b − c ) + mzx ( c − a) + nxy ( a − b ) = 0.

Problem 6: Prove that all plane sections of ax2 + by2 + cz 2 = 1 which are rectangular
hyperbolas and which pass through the point (α, β, γ ) touch the cone
( x − α )2 ( y − β )2 ( z − γ )2
+ + = 0.
b+ c c+ a a+ b
G-260

Solution: If the central plane section of ax 2 + by2 + cz 2 = 1 by the plane


lx + my + nz = 0 is a rectangular hyperbola then (as proved in article 3, Corollary 3), we
have
( b + c ) l2 + ( c + a) m2 + ( a + b ) n2 = 0. …(1)
Again the equation of the plane parallel to the plane lx + my + nz = 0 and passing
through the point (α, β, γ ) is
l ( x − α ) + m ( y − β ) + n ( z − γ ) = 0. …(2)
Since the plane (2) is parallel to the plane lx + my + nz = 0, therefore (1) is also the
condition for the section of the given conicoid by the plane (2) to be a rectangular
hyperbola.
Shifting the origin to the point (α, β, γ ), the equation (2) becomes
lx + my + nz = 0. …(3)
∴ (1) shows that the plane (3) envelopes the cone
x2 /( b + c ) + y2 /( c + a) + z 2 /( a + b ) = 0. …(4)
Again shifting the origin back from (α, β, γ ) to its original position the plane (2)
envelopes the cone
( x − α )2 /( b + c ) + ( y − β )2 /( c + a) + ( z − γ )2 / ( a + b ) = 0.
Problem 7: Prove that the normals to the central sections of the ellipsoid
x2 / a2 + y2 / b2 + z 2 / c2 = 1
which are of given eccentricity e lie on the cone
a2 b2 c2 ( e2 − 2)2 ( x2 + y 2 + z 2 ) ( a2 x2 + b2 y2 + c2 z 2 )
= (1 − e2 ) [ a2 ( b 2 + c2 ) x2 + b2 ( c2 + a2 ) y2 + c2 ( a2 + b2 ) z 2 ]2.
Solution: Consider the section of x2 / a2 + y2 / b2 + z 2 / c2 = 1 by the plane
lx + my + nz = 0. Proceeding as in article 3, Corollary 2 , the squares of the semi-axes are
given by
r4 ( a2 l2 + b2 m2 + c2 n2 ) − r 2 {a2 ( b2 + c2 ) l2 + b2 ( c2 + a2 ) m2
+ c2 ( a2 + b2 ) n2 } + a2 b2 c2 ( l2 + m2 + n2 ) = 0. …(1)
If r1 and r2 be the lengths of the semi-axes of the section, then r12 and r22 are the roots of
2
the quadratic (1) in r and so we have
Σ a2 ( b2 + c2 ) l2
r12 + r22 = , …(2)
Σ ( a2 l2 )
a2 b2 c2 ( l2 + m2 + n2 )
and r12 r22 = ⋅ …(3)
Σ ( a2 l2 )
Now in the two dimensional geometry, the eccentricity e of an ellipse is given by the
relation b2 = a2 (1 − e2 ). Therefore if e is the eccentricity of the section, then
r22 / r12 = 1 − e 2

r22 1 − e2 r12 + r22 2 − e2


or = or = ⋅
r12 1 r22 − r12 − e2
G-261

Squaring, e4 ( r12 + r22 )2 = (2 − e2 )2 {( r12 + r22 )2 − 4r12 r22}

or ( r12 + r22 )2 {(2 − e2 )2 − e4 } = 4 r12 r22 (2 − e2 )2

or ( r12 + r22 )2 (1 − e2 ) = r12 r22 (2 − e2 )2. …(4)


Putting the values from (2) and (3) in (4), we get
2
a2 b2 c2 ( l2 + m2 + n2 )  Σ a2 ( b 2 + c2 ) l2 
( e2 − 2)2 ⋅ = (1 − e2 )  
Σ ( a2 l2 )  Σ a2 l2 
or ( e − 2) a b c ( l2 + m2 + n2 ) Σ ( a2 l2 ) = (1 − e2 ) [ Σ a2 ( b2 + c2 ) l2 ]2.
2 2 2 2 2

Hence the locus of the normal x / l = y / m = z / n to the central section lx + my + nz = 0 is


given by
a2 b2 c2 ( e2 − 2)2 ( x2 + y2 + z 2 ) ( a2 x2 + b2 y2 + c2 z 2 )
= (1 − e2 ) [ a2 ( b2 + c2 ) x2 + b2 ( c2 + a2 ) y2 + c2 ( a2 + b2 ) z 2 ]2.

Problem 8: Find the angle between the asymptotes of any plane section of a conicoid and hence
prove that if θ be the angle between the asymptotes of the section of ax2 + by2 + cz 2 = 1 by the
plane lx + my + nz = 0 then
tan2 θ [ l 2 ( b + c ) + m2 ( c + a) + n2 ( a + b )]2
+ 4 ( bcl 2 + cam2 + abn2 ) = 0.
Solution: We know that the section of a conicoid by any plane is a conic. Let the
lengths of the semi-axes of this conic section be α and β. Then by a suitable
transformation of co-ordinate axes the equation of this conic section can be expressed in
the form
x2 / α2 + y2 / β2 = 1. …(1)
Again we know that the equation of the asymptotes differs from the equation of the
conic by a constant and so the equation of the asymptotes can be given by
x2 / α2 + y2 / β2 = k,
where k is a constant.
The angle θ between the asymptotes is given by
2 √ {0 − (1 / α2 )(1 / β2 )}
tan θ =
1 / α2 + 1 / β2
 2 √ ( h 2 − ab ) 
 ∵ tan θ = by plane geometry 
 a + b 
1
2 tanθ
2 2iα β 2i ( β / α )
or = = ⋅
2 1 2 2
1 − ( iβ / α )2
1 − tan θ α +β
2
1
∴ tan θ = iβ / α or θ = 2 tan−1 ( iβ / α ).
2
Also from above,
tan2 θ = − 4α2 β2 / (α2 + β2 )2 . …(2)
G-262

Second part : If r1 and r2 are the lengths of the semi-axes of the plane section of the
2
conicoid a x + by 2 + c z 2 = 1by the plane lx + my + nz = 0, then from (2), we have
−4r12 r22 −4 {Σ ( bcl 2 )} {Σl 2 }
tan2 θ = =
( r12 + r22 )2 [ Σ ( b + c ) l 2 ]2
[From equation (6) of article 3]
or tan2 θ [ Σ ( b + c ) l 2 ]2 + 4 ( bcl 2 + cam 2 + abn2 ) = 0,
if l 2 + m2 + n2 = 1.

Comprehensive Problems 2
Problem 1: If OP, OQ, OR are conjugate semi-diameters of an ellipsoid, prove that the area of
the section of the ellipsoid by the plane PQR is two thirds the area of the parallel central section.
Solution: Let the equation of the ellipsoid be
x2 / a2 + y2 / b2 + z 2 / c2 = 1. …(1)
Let the co-ordinates of the extremities P, Q, R of conjugate semi-diameters be
( xr , yr , z r ), r = 1, 2, 3. The equation of the plane PQR is [See Example11, Chapter - The
Central Conicoids after article 23.]
x y z
( x + x2 + x3 ) + ( y1 + y2 + y3 ) + ( z1 + z2 + z3 ) = 1 …(2)
2 1
a b2 c2
or lx + my + nz = 1, where l = ( x1 + x2 + x3 ) / a2 , etc.
Let A and A0 be the areas of the sections of (1) by the plane (2) and the central plane
parallel to the plane (2). We have
A = A0 (1 − p2 / p0 2 )
where p = 1 or p2 = 1
and p0 2 = a2 l 2 + b2 m2 + c2 n2 in case of ellipsoid
a2 b2 c2
= ( Σ x1)2 + ( Σ y1)2 + ( Σ z1)2
4 4 4
a b c
Σ x12 Σy12 Σz12 2Σ x1 x2
2Σ x2 x3 2Σ x3 x1
= + + + + +
a2 b2 c2 a2 a2 a2
= 1 + 1 + 1 + 0 + + 0 + 0 = 3.
[See article 22, Chapter - The Central Conicoids]
2
Putting the above values in (3), we get A = A0 .
3

Problem 2: Show that the area of the section of the ellipsoid x2 / a2 + y2 / b2 + z 2 / c2 = 1 by


the plane x / a + y / b + z / c = 1 is 2 π ( b2 c2 + c2 a2 + a2 b2 )1 /2 /(3 √ 3).
a2 b2 c2
Solution: Here p = 1, p0 2 = a2 l2 + b2 m2 + c2 n2 = + + = 3.
a2 b2
c2
G-263

( l2 + m2 + n2 )1 /2
We have A0 = πabc
√ ( a2 l2 + b2 m2 + c 2 n2 )
πabc √ {1 / a2 + 1 / b2 + 1 / c2}
=
√3
= π √ ( b2 c 2 + c2 a2 + a2 b2 ) / √ 3.
 p2  π √ ( b2 c2 + c2 a2 + a2 b2 )  1
∴ A = A0 1 −  = 1 − 
  √3  3
 p0 2 
2 π √ ( b2 c2 + c2 a2 + a2 b2 )
= ⋅
3 √3

Problem 3: Find the locus of the centres of the sections of the ellipsoid
x2 / a2 + y2 / b2 + z 2 / c2 = 1 whose area is constant = πk2 . (Avadh 2010, 13)
Solution: The ellipsoid is
x2 / a2 + y2 / b2 + z 2 / c2 = 1. …(1)
Let ( x1, y1, z1 ) be the centre of the section of (1). By the formula ‘T = S1’ the
equation of the plane giving this section is
xx1 yy1 x2 y2 c2
zz1
+ +
= 1 + 1 + 1 ⋅ …(2)
a2 b2 c2 a2 b2 c2
Let A and A0 be the areas of the sections of (1) by the plane (2) and the central plane
parallel to (2) respectively.
We have A = A0 (1 − p2 p0 2 ) …(3)

where 2
p = ( x12 / a2 + y12 2
/b + z12 2 2
/c ) ,
2 22 2 2
p0 = a l + b m + c n = 2 2
x12 / a2 + y12 / b2 + z12 / c2.

∴ p2 / p0 2 = x12 / a2 + y12 / b2 + z12 / c2 .

πabc √ ( l 2 + m2 + n2 )
Now A0 =
√ ( a2 l2 + b2 m2 + c2 n2 )
πabc √ { x12 / a4 + y12 / b4 + z12 / c4 }
= ⋅
√ ( x12 / a2 + y12 / b2 + z12 / c2 )
Also A = πk2 , (given).
Putting the above values in (3) and squaring, we get
2
π2 a2 b2 c2 {x12 / a4 + y12 / b4 + z12 / c4 }  x2 y 2 z 2
π2 k4 = ⋅ 1 − 1 − 1 − 1  .
2 2 2 2
2 2
( x1 / a + y1 / b + z1 / c ) 2
 a b2 c2 
∴ The required locus of ( x1, y1, z1 ) is
2
 x2 y2 z2   x2 y2 z2   x2 y2 z2 
a2 b2 c2  + +  1 − − − = k4  + +  .
4 4 2
a b c4   a2 b2 c  a 2
b2
c2 
G-264

Problem 4: Prove that the section of the ellipsoid x2 / a2 + y2 / b2 + z 2 / c2 = 1 whose centre


 1 1 1 
is the point  a, b, c  passes through the three of the extremities of its principal axes.
 3 3 3 

Solution: By the formula ‘T = S1’ the equation of the plane section with  a, b, c 
1 1 1
3 3 3 
as its centre is
1 1 1
ax bx cx
3 3 a2 b2 c2
+ + 3 = + +
2 2 2 2 2
a b c 9a 9b 9 c2
or x /a + y /b + z /c =1
which clearly passes through ( a, 0, 0 ), (0, b, 0 ), (0, 0, c ), the three of the extremities
of the principal axes of the given ellipsoid.

Problem 5: Prove that the areas of the sections of the greatest and least areas of the ellipsoid
x2 / a2 + y2 / b2 + z 2 / c2 = 1 which pass through the fixed line x / l = y / m = z / n are
πabc / r1, πabc / r2 where r1 and r2 are the axes of the section by the plane
lx / a + my / b + nz / c = 0.
Solution: The equation of any plane through x / l = y / m = z / n is
Lx + My + Nz = 0, …(1)
where Ll + Mm + Nn = 0. …(2)
We know that the area of the section of
x2 / a2 + y2 / b2 + z 2 / c2 = 1
by the plane (1) is πabc / p,
where p2 = a2 L2 + b2 M 2 + c2 N2 , [L, M , N being the d.c.’s].
The area πabc / pwill be maximum or minimum according as pis minimum or maximum.
We shall discuss the maximum and minimum of p (or of p2 ) by Lagrange’s method of
undetermined multipliers. We have
p2 = a2 L2 + b2 M 2 + c2 N2 …(3)
subject to the conditions
Ll + Mm + Nn = 0, …(4)
and L2 + M 2 + N2 = 1. …(5)
Differentiating the above relations (3), (4), (5), we get
a2 L d L + b2 M d M + c2 N d N = 0 ,

l d L + md M + nd N = 0 , 
L d L + M d M + N d N = 0. 

Multiplying the above relations by 1, λ1, λ2 respectively and adding and then equating
to zero the coefficients of dL, dM and dN, we get
a2 L + λ1l + λ2 L = 0, b2 M + λ1m + λ2 M = 0, c2 N + λ1n + λ2 N = 0. …(6)
Multiplying the relations (6) by L, M , N in that order and adding, we get
G-265

( a2 L2 + b2 M 2 + c2 N2 ) + λ1 ( lL + mM + nN )
+ λ2 ( L2 + M 2 + N2 ) = 0
or p2 + λ1 (0 ) + λ2 (1) = 0, using (3), (4) and (5)
or λ2 = − p2 .
Putting λ2 = − p2 in (6), we get
L = lλ1 / ( p2 − a2 ), M = mλ1 / ( p2 − b2 ), N = nλ1 / ( p2 − c2 ).
Putting these values of L, M , N in (2), we get
l2 λ1 / ( p2 − a2 ) + m2 λ1 / ( p2 − b2 ) + n2 λ1 / ( p2 − c2 ) = 0
or l2 / ( p2 − a2 ) + m2 / ( p2 − b2 ) + n2 / ( p2 − c 2 ) = 0 [ as λ1 ≠ 0 ]. …(7)
The equation (7) gives maximum and minimum values of p . Let 2
p12 and p22 be these
2
values of p .
Now the axes of the section of the conicoid ax2 + by2 + cz 2 = 1 by the plane
lx + my + nz = 0 are given by
l2 / ( ar2 − 1) + m2 / ( br2 − 1) + n2 / ( cr2 − 1) = 0. [See article 3]
∴ The axes of the section of the ellipsoid
x2 / a2 + y2 / b2 + z 2 / c2 = 1
by the plane x ( l / a) + y ( m / b ) + z ( n / c ) = 0 are given by
( l / a)2 ( m / b )2 ( n / c )2
+ + =0
2 2 2 2
r / a −1 r / b −1 r / c2 − 1
2

or l2 / ( r2 − a2 ) + m2 / ( r2 − b2 ) + n2 / ( r2 − c2 ) = 0. …(8)
Let its roots be r12 and r22. Comparing (7) and (8) we see that the values of p2 and r2 are
the same. Hence p12 = r12 and p22 = r22 . Therefore the maximum and minimum areas
are πabc / r1 and πabc / r2 .

Problem 6: The normal section of an enveloping cylinder of the ellipsoid


x2 / a2 + y2 / b2 + z 2 / c2 = 1

has a given area πk2. Prove that the planes of contact of the cylinder and ellipsoid touch the cone
x2 y2 z2
+ + = 0.
4 2 2 4 4 2 2 4
a (b c − k ) b (c a − k ) c ( a b − k4 )
4 2 2

Solution: The equation of the enveloping cylinder of


x2 / a2 + y2 / b2 + z 2 / c2 = 1
2
 x2 y2 z2   l2 m2 n2   lx my nz 
is  + + − 1  + +  =  + +  , …(1)
a2
b2
c 2
a 2
b2
c2   a2 b2 c2 
its generating lines being parallel to
x / l = y / m = z / n. …(2)
G-266

The plane of contact is the diametral plane of (2) and is given by


lx / a2 + my / b2 + nz / c2 = 0. …(3)
2 2 2 2 2 2
The area A of the section of x / a + y / b + z / c = 1by the plane (3) is given by
πabc √ {( l / a ) + ( m / b ) + ( n / c4 )}
2 4 2 4 2
A= ⋅ …(4)
√ ( l2 / a2 + m2 / b2 + n2 / c2 )
Now all the normal sections of a cylinder are equal and the central normal section being
perpendicular to the generating line (2) is given by
lx + my + nz = 0. …(5)
If θ be the angle between (3) and (5), we have

 l2 / a2 + m2 / b2 + n2 / c 2 
cos θ =  ⋅
2 4 2 4 2 4 2 2 
 √ {( l / a + m / b + n / c ) ( l + m + n )} 
…(6)
Hence the area πk2 of the normal section = A cos θ
πabc √ ( l2 / a2 + m2 / b2 + n 2 / c2 )
or πk2 = ⋅
√ ( l2 + m2 + n2 )
[Putting for A and cos θ from (4) and (6)]
 l2 m2 n2 
Squaring, k4 ( l2 + m2 + n2 ) = a2 b2 c2  + + 
2 2
a b c2 
or ( b2 c2 − k4 ) l2 + ( c2 a2 − k4 ) m2 + ( a2 b2 − k4 ) n2 = 0.
x y z
∴ The normal namely = = to the plane of contact (3) generates the
2 2
l /a m/b n / c2
cone
a4 ( b2 c2 − k 4 ) x2 + b4 ( c2 a2 − k4 ) y2 + c4 ( a2 b2 − k4 ) z 2 = 0.
…(7)
∴ The plane of contact (3) will touch the reciprocal cone of (7) i. e. the cone
x2 y2 z2
+ + = 0.
4 2 2 4 4 2 2 4
a (b c −k ) b (c a − k ) c ( a b − k4 )
4 2 2

Comprehensive Problems 3
Problem 1: Planes are drawn through a fixed point (α, β, γ ) so that their sections of the
paraboloid ax + by2 = 2z are rectangular hyperbolas. Prove that they touch the cone
2

( x − α )2 ( y − β )2 ( z − γ )2
+ + = 0.
b a a+ b
Solution: The equation of any plane through (α, β, γ ) is
l ( x − α ) + m ( y − β ) + n (z − γ ) = 0 …(1)
G-267

or lx + my + nz = lα + mβ + nγ = p (say).
The lengths of the axes of the section of ax2 + by2 = 2z by the plane (1) are given by [See
equation (15) article 6]
abn6 r4 − n 2 r2 p0 2 [( a + b ) n2 + am2 + bl2 ] + p0 4 ( l2 + m2 + n2 ) = 0

l2 m2
where p0 2 = + + 2np, (since c = 1)
a b
l2 m2
= + + 2n ( lα + mβ + nγ ).
a b
The section will be a rectangular hyperbola if
r12 + r22 = 0 i. e. if ( a + b ) n2 + am2 + bl2 = 0. …(2)
The equations of the normal to the plane (1) through (α, β, γ )
x−α y −β z−γ
are = = ⋅ …(3)
l m n
Eliminating l, m, n between (2) and (3), the normal (3) is a generator of the cone
( a + b ) ( z − γ )2 + a ( y − β )2 + b ( x − α )2 = 0. …(4)
Hence the plane (1) will touch the cone reciprocal to (4) i. e., the cone
( x − α )2 / b + ( y − β )2 / a + ( z − γ )2 /( a + b ) = 0.

Problem 2: Show that the section of the paraboloid ax2 + by2 = 2z by a tangent plane to the
x2 y2 z2
cone + + = 0 is a rectangular hyperbola.
b a a+ b
Solution: Let the equation of the tangent plane to the given cone be lx + my + nz = 0.
Hence its normal namely x / l = y / m = z / n will be the generator of the reciprocal cone
bx2 + ay2 + ( a + b ) z 2 = 0. …(1)
Since x / l = y / m = z / n is a generator of the cone (1), therefore
( a + b ) n2 + am2 + bl 2 = 0
which is the condition for the section of ax2 + by2 = 2z to be a rectangular hyperbola.
[See Corollary 3, article 6.]

Problem 3: Find the radius of curvature at the origin of the conic


ax2 + by2 = 2z, lx + my + nz = 0.
Solution: Let C (α, β, γ ) be the centre of the section (conic), then the plane
lx + my + nz = 0 is also given by T = S1
i. e., aαx + bβy − ( z + γ ) = aα2 + bβ2 − 2γ
or aαx + bβy − z = a α2 + bβ2 − γ.
aα bβ −1 aα2 + bβ2 − γ
∴ = = = ⋅
l m n 0
G-268

l −m l2 m2
∴ α=− , β= , γ = aα2 + bβ2 = + ⋅
an bn an2 bn2
2
l2 m2  l2 m2 
∴ CO2 = α2 + β2 + γ 2 = + +  +  ⋅
2 2 2 2 2
a n b n  an bn2 
The lengths of the semi-axes are given by
abn6 r4 − n2 r2 p0 2 {( a + b ) n2 + am2 + bl2 } + p0 4 ( l2 + m2 + n2 ) = 0.
…(1)
∴ r12 + r22 = n2 p0 2 [( a + b ) n2 + am2 + bl2 ] / ( abn6 ) …(2)

r12 r22 = p0 4 ( l2 + m2 + n2 ) / ( abn6 ) …(3)


2 2 2
where p0 = l / a + m / b. …(4)
[ ∵ p = 0]
Now let CO and CD be the conjugate semi-diameters of the conic.
Then CO2 + CD2 = r12 + r22
or CD2 = r12 + r22 − CO2
n2 p0 2 [( a + b ) n2 + am2 + bl2 ]
or CD2 =
abn6
 2 
l2 m2  l2 m2  
− + +  + 
 a2 n 2 b2 n2  an2 bn2  
 
 
n2 p0 2 [( a + b ) n2 + am2 + bl2 ]  l2 m2 p4
= − + + 0 ,
abn 6  a2 n2 b2 n2 n4 
 
using (4)
[( a + b ) n 2 2 2 2 2
+ am + bl − bl − am ] 1 l 2
m2 
= p0 2 ⋅ − 
 + 
abn4 n2  a2 b2
a + b  1  l2 m2  1  l2 m2 
=    +  −  + 
 ab  n2  a 2
b  n a 2
b2 
l2 + m2
= ⋅ …(5)
abn2
When know that the radius of curvature ρ of a central conic at any point P is CD3 / (αβ ),
where CP and CD are the conjugate semi-diameters, C is the centre and α, β are the
lengths of the semi-axes. Hence if ρ0 be the radius of curvature at the origin O, then
G-269

3 /2
 l2 + m2 
 
CD3  abn2  ( l2 + m2 )3 /2
ρ0 = = = ⋅
r1r3 1 /2
p0 2  l2 + m2 + n2   l2 m2  2
  ab  +  ( l + m2 + n2 )1 /2
n3  ab  a b 

Comprehensive Problems 4
Problem 1: Show that the radius of the circle in which the plane
( x / a) √ ( a − b2 ) + ( z / c ) √ ( b2 − c2 ) = λ cuts the ellipsoid x2 / a2 + y2 / b 2 + z 2 / c2 = 1 is
2

b √ {1 − λ2 /( a2 − c2 )}.
Solution: The equation of the sphere through the intersection of the ellipsoid and the
given plane is [See article 9 equation (5)]
x2 + y2 + z 2 − λ √ ( a2 − b2 ) ( x / a) + λ √ ( b2 − c2 ) ( z / c ) − b2 = 0.
…(1)
The co-ordinates of the centre C of the sphere (1) are
( λ √ ( a2 − b2 ) /(2a), 0, − λ √ ( b2 − c2 ) /(2c )).
If R be the radius of the sphere (1), it is given by
λ2 ( a2 − b2 ) λ2 b2 ( a2 − c2 )
λ2 ( b2 − c2 )
R2 = + + b2 = b2 + ⋅
2 2
4a 4c 4a2 c2
Also p = the length of the perpendicular from the centre C of the sphere to the given
plane
λ ( a2 − b2 ) λ ( b2 − c2 )
− −λ
2a2 2 c2 − bλ ( a2 + c2 )
= = ⋅
√ {( a2 − b2 ) / a2 + ( b2 − c2 ) / c2 } 2ca √ ( a2 − c2 )
If r is the radius of the circular section, then
λ2 b2 ( a2 − c 2 ) ( a2 + c2 )2 
r2 = R2 − p2 = b2 +  − 
4a2 c2  1 a2 − c2 
λ2 b2 ( a2 − c2 )2 − ( a2 + c2 )2  2  λ2 
= b2 +  = b 1 − 2  ⋅
2 2  2 2
4a c  (a − c )   a − c2 
 λ2 
∴ r = b 1 −  ⋅
 a − c 2
2

Problem 2: Prove that the radius of the circular section of the ellipsoid at a distance p from the
centre is b √ {1 − ( p2 b2 ) /( a2 c2 )}. (Gorakhpur 2005)
Solution: Let r be the radius of the circular section
( x / a) √ ( a2 − b2 ) + ( z / c ) √ ( b2 − c2 ) = λ. …(1)
From Problem 1,
G-270

 λ2 
r = b 1 −  . …(2)
 a − c2 
2

We are given that p is the length of the perpendicular from the centre (0, 0, 0 ) of the
ellipsoid to the plane (1).
−λ − λac
∴ p= = ⋅
√ {( a − b ) / a + ( b − c ) / c } b √ ( a2 − c2 )
2 2 2 2 2 2

λ2 a2 c2 λ2 b2 p2
Squaring, p2 = or = ⋅
2 2 2 2 2
b (a − c ) a −c a2 c2
∴ From (2), r = b √ {1 − ( b2 p2 ) / ( a2 c 2 )}.

Problem 3: Show that the circular sections of the ellipsoid passing through one extremity of
r2 b2 − c2
x-axis are both of radius r where = ⋅
b2 a2 − c2
Solution: The real circular sections of the ellipsoid are
( x / a) √ ( a2 − b2 ) + ( z / c ) √ ( b2 − c2 ) = λ, …(1)
2 2 2 2
and ( x / a) √ ( a − b ) − ( z / c ) √ ( b − c ) = µ . …(2)
Let r be the radius of the circular section in which the plane (1) cuts the ellipsoid. Then
proceeding as in Problem 1 above, we have
r = b √ {1 − λ2 /( a2 − c2 )}. …(3)
Similarly the radius of the circular section in which the plane (1) cuts the ellipsoid is
b √ {1 − µ 2 /( a2 − c2 )}. [Replacing λ by µ and c by −c in (3)].
Now as given if the plane (1) passes through ( a, 0, 0 ), an extremity of the x-axis, then (1)
gives λ = √ ( a2 − b2 ).
∴ From (3), we have
 a2 − b2  r2 b2 − c2
r2 = b2 1 − or = ⋅
2 2 2
 a −c  b a2 − c2
Again if the plane (2) also passes through ( a, 0, 0 ), then (2) gives
µ = √ ( a2 − b2 ).
Hence in this case also
r2 b 2 − c2
= ⋅
2
b a2 − c2

Problem 4: If p1, p2 , p3 and π1, π2 , π3 be the perpendiculars from the extremities P1, P2 , P3 of
conjugate semi-diameters on the two central circular sections of the ellipsoid
x2 / a2 + y2 / b 2 + z 2 / c2 = 1 then prove that
p1 π1 + p2 π2 + p3 π3 = a2 c2 ( a2 + c2 − 2b2 ) / {( a2 − c2 ) b2}.
Solution: We know that the two real central circular sections of the given ellipsoid are
( x / a) √ ( a2 − b2 ) + ( z / c ) √ ( b2 − c2 ) = 0, …(1)
G-271

and ( x / a) √ ( a2 − b2 ) − ( z / c ) √ ( b2 − c2 ) = 0. …(2)
Proceeding as in Example 10, we have
( x1 / a) √ ( a2 − b2 ) + ( z1 / c ) √ ( b2 − c2 )
p1 = ,
√ {( a2 − b2 ) / a2 + ( b2 − c2 ) / c2 }
π1 = the length of the perpendicular from P1 ( x1, y1, z1 ) on (2)
( x1 / a) √ ( a2 − b2 ) − ( z1 / c ) √ ( b2 − c2 )
= ⋅
√ {( a2 − b2 ) / a2 + ( b2 − c2 ) / c2}
( x12 / a2 ) ( a2 − b2 ) − ( z12 / c2 ) ( b2 − c2 )
∴ p1 π1 =
b2 ( a2 − c2 ) /( a2 c2 )
 a2 − b2 
a2 c2  b2 − c2  
=   ⋅ x12 −   ⋅ z12  ⋅
2 
b ( a − c )  a2 
2 2 2
 c  
Similar expressions can be obtained for p2 π2 and p3 π3 .
 a − b  Σx 2 −  b − c  Σz 2 
2 2
a2 c2 2 2
∴ p1 π1 + p2 π2 + p3 π3 =    1   1 
b2 ( a2 − c 2 )  a2   c2  
a2 c2   a2 − b2  2  b2 − c2  2 
=    ⋅ a −   ⋅ c 
b2 ( a2 − c2 )   a2   c2  

a2 c2 ( a2 − 2b2 + c2 )
= ⋅
b2 ( a2 − c2 )
Problem 5: A cone is drawn with its vertex at the centre of the ellipsoid
x / a + y / b2 + z 2 / c2 = 1 and its base is a circular section of the ellipsoid. If the cone
2 2 2

contains three mutually perpendicular generators, show that the distance from the centre of the
ellipsoid to the section is abc / √ ( b2 c2 + c2 a2 + a2 b2 ).
Solution: The ellipsoid is
x2 / a2 + y2 / b2 + z 2 / c2 = 1. …(1)
Any real circular section of (1) is
( x / a) √ ( a2 − b2 ) + ( z / c ) √ ( b2 − c2 ) = λ. …(2)
Making (1) homogeneous with the help of (2), the equation of the cone with its vertex at
the centre (0, 0, 0) of (1) and having its base as the above circular section is given by
2
x2 y2 1 x z22 2 z 2 2 
+ + =
 √ ( a − b ) + √ ( b − c ) ⋅
a 2
b c 2
λ2  a 2 c 
The above cone has three mutually perpendicular generators.
∴ the coeff. of x2 + the coeff. of y2 + the coeff. of z 2 = 0
1 a2 − b2  1 1 b2 − c2 
or  −  + +  −  = 0
 a2 a2 λ2  b2  c2 c2 λ2 
G-272

1 1 1  a2 − b2 b 2 − c2
or λ2  + +  = + ⋅ …(3)
 a2 b2 c2  a2 c2
Now the length of the perpendicular (i. e. distance) from the centre (0, 0, 0) of (1) to the
plane is
λ 1
= = , from (3)
√ {( a2 − b2 ) / a2 + ( b2 − c 2 ) / c2} √ (1 / a2 + 1 / b2 + 1 / c2 )
= abc / √ ( b2 c2 + c2 a2 + a2 b2 ).

Problem 6: Find the central circular sections of the conicoid


3 x2 + 5 y2 + 3 z 2 + 2zx − 4 = 0.
Also find any circular sections.
Solution: The equation of the given conicoid can be rewritten as
[3 x2 + 5 y2 + 3z 2 + 2zx − λ ( x2 + y2 + z 2 )]
+ λ ( x2 + y2 + z 2 − 4 / λ ) = 0. …(1)
The equation (1) will represent a surface passing through the intersection of a sphere
and a pair of planes if
3 x2 + 5 y2 + 3 z 2 + 2zx − λ ( x 2 + y2 + z 2 ) = 0 …(2)
represents a pair of planes. The condition for this is
3 − λ 0 1 
 0 5−λ 0 = 0
 
 1 0 3 − λ
or (3 − λ )(5 − λ )(3 − λ ) − (5 − λ ) = 0
or λ3 − 11λ2 + 38λ − 40 = 0
or ( λ − 2)( λ − 4)( λ − 5) = 0
or λ = 2, 4, 5.
The mean value of λ i. e. λ = 4 will give real circular sections. Putting λ = 4 in (2), the real
circular sections are given by
− x2 + y2 − z 2 + 2zx = 0
or ( x − z )2 − y2 = 0
or x + y − z = 0 and x − y − z = 0. …(3)
Also any circular sections are those which are parallel to (3) and hence they are given by
x + y − z = λ and x − y − z = µ.

Problem 7: Find the real circular sections of the following :


(i) 5 y − 8z 2 + 18 yz − 14zx − 10 xy + 27 = 0
2

(ii) x2 + 2 y2 + 6z 2 = 8 (Gorakhpur 2006)


2 2 2
(iii) 15 x − y − 10 z + 4 = 0.
Solution: Proceeding as in Example 11, the real circular sections are given by
G-273

(i) 3 x − 4 y + z = λ, x − 2 y − 5z = µ
(ii) x + 2z = λ, x − 2z = µ
(iii) 4 x + 3z = λ, 4 x − 3z = µ .
If we choose λ = µ = 0 the central circular sections are obtained.
Problem 8: Show that the real central circular sections of the hyperboloids
x / a + y / b2 − z 2 / c2 = 1 and x2 / a2 − y2 / b2 − z 2 / c2 = 1 are given by the planes
2 2 2

( y / b ) √ ( a2 − b2 ) ± ( z / c ) √ ( a2 + c 2 ) = 0 and ( x / a) √ ( a2 + b2 ) ± ( z / c ) √ ( b2 − c2 ) = 0.
Solution: The equation
x2 / a2 + y2 / b2 − z 2 / c2 = 1 …(1)
can be rewritten in the following three forms :
 x2 + y2 + z 2 − a2  1 1 1 1
  + y2  − − z2  +  = 0,
2 
…(2)
 a2
  b2
a  c 2
a2 
 x2 + y2 + z 2 − b2  1 1 2  1 1
  + x2  −  − z  2 + 2  = 0, …(3)
 b2   a2 b2  c b 
 x2 + y 2 + z 2 − c2  1 1 1 1
  + x2  + + y2  2 + 2  = 0.
2 
and …(4)
 c 2
  a2
c b c 
In case a > b > c , we have 1 / a < 1 / b < 1 / c. Then the only real central circular sections of
(1) are given by the planes
 1 1 2  1 1
y2  −  − z  2 + 2 =0
 b 2 a2  c a 
y2 z2 2
or ( a2 − b2 ) − ( a + c2 ) = 0
2
b c2
or ( y / b ) √ ( a2 − b 2 ) ± ( z / c ) √ ( a2 + c2 ) = 0.
Again the equation of the hyperboloid x2 / a2 − y2 / b2 − z 2 / c2 = 1may be rewritten as
 x2 + y 2 + z 2 + b2  1 1 2  1 1
−   + x2  +  − z  2 − 2  = 0.
 b2   a2 b2  c b 
Hence discussing as above the real central circular sections are
( x / a) √ ( a2 + b2 ) ± ( z / c ) √ ( b2 − c2 ) = 0.

Problem 9: Prove that the planes 2 x + 3z − 5 = 0, 2 x − 3z + 7 = 0 meet the hyperboloid


− x2 + 3 y2 + 12 z 2 = 75 in circles which lie on the sphere
3 ( x2 + y2 + z 2 ) + 4 x + 36z − 110 = 0.
Solution: The equation of any surface passing through the sections of the given
hyperboloid by the two given planes is given by
( − x2 + 3 y2 + 12 z 2 − 75) + k (2 x + 3z − 5)(2 x − 3z + 7) = 0. …(1)
2 2 2
The equation (1) will be that of a sphere if the coefficients of x , y and z are equal
i. e. if −1 + 4k = 3 = 12 − 9k i. e. if k = 1.
G-274

Putting k = 1 in(1), the circular sections lie on the sphere


3 ( x2 + y2 + z 2 ) + 4 x + 36z − 110 = 0.
Problem 10: If the section of the cone whose vertex is P (α, β, γ ) and base z = 0, ax2 + by2 = 1,
by the plane x = 0 is a circle, then P lies on y = 0, ax2 − bz 2 = 1 and the section of the cone by the
plane ( a − b ) γ x − 2axz = 0 is also a circle.
Solution: It can be easily shown that the equation of the cone is given by
a (αz − γx )2 + b ( βz − γy )2 = ( z − γ )2 . …(1)
The section of (1) by the plane x = 0 is given by
aα2 z 2 + b ( βz − γy )2 = ( z − γ )2 , x = 0. …(2)
The equations (2) will represent a circle if
the coeff. of y2 = the coeff. of z 2 and the coeff. of yz = 0
i. e. if bγ 2 = aα2 + bβ2 − 1 and bβγ = 0. …(3)
But b ≠ 0 and also γ ≠ 0. Note that if we take γ = 0 then the vertex (α, β, γ )of the cone will
lie in its base which is impossible. Hence we must have β = 0. Then the first relation of
(3) provides
bγ 2 = aα2 − 1, β = 0. …(4)
2 2
∴ The vertex P (α, β, γ ) lies on ax − bz = 1, y = 0. Again putting β = 0 in (1), the
equation of the cone becomes
a (αz − γx )2 + bγ 2 y2 = ( z − γ )2
or aα2 z 2 + aγ 2 x2 − 2aαγ zx + bγ 2 y2 − z 2 + 2γz − γ 2 = 0
or aγ 2 x2 + ( bγ 2 + 1) z 2 − z 2 + bγ 2 y2 − 2aαz x + 2γz + γ 2 = 0
[Putting aα2 = bγ 2 + 1 from (4)]
or {bγ 2 ( x2 + y2 + z 2 ) + 2γz − γ 2} + γx [( a − b ) γx − 2aαz ] = 0. …(5)
The equation (5) will represent a sphere if
γx [( a − b ) γx − 2aα z ] = 0
i. e. x = 0, ( a − b ) γx − 2aαz = 0.
Therefore the circular sections are given by
( a − b ) γx − 2aαz = 0, x = 0.
Problem 11: The normals to the ellipsoid x2 / a2 + y2 / b2 + z 2 / c2 = 1 at all points of a
 ac 
central circular section are parallel to a plane that makes an angle cos −1 
2 2 2 
 b √ ( a − b + c )
with the section.
Solution: One of the real central circular sections is
( x / a) √ ( a2 − b2 ) + ( z / c ) √ ( b2 − c2 ) = 0. …(1)
Let the generators of the enveloping cylinder which touch along (1), be parallel to the
line x / l = y / m = z / n.
∴ The diametral plane
G-275

lx / a2 + my / b2 + nz / c2 = 0 …(2)
and the plane (1) should represent the same plane.
Hence on comparing (1) and (2), we get
l m n
= = ⋅
2 2
a √ (a − b ) 0 c √ (b − c 2 )
2

Also the normals to the ellipsoid will be parallel to the normal section of the cylinder i. e.
to the plane lx + my + nz = 0
or a √ ( a2 − b2 ) x + c √ ( b2 − c2 ) z = 0. …(3)
Now if θ be the angle between the planes (1) and (3), then we have
( a2 − b2 ) + ( b2 − c2 )
cos θ =
1 /2
  a2 − b2 b2 − c2  2 2 
  +  { a ( a − b2 ) + c2 ( b2 − c2 )}
2 2
  a c  
ac ( a2 − c2 )
= ⋅
b [( a − c ) ( a2 − c2 ) ( a2 + c2 − b2 )]1 /2
2 2

ac ( a2 − c2 )
∴ θ = cos −1 ⋅
b √ ( a2 − b2 + c2 )

Problem 12: Find the real circular sections of the following paraboloids :
(i) x2 + 10 z 2 = 2 y, (Gorakhpur 2006)
2 2
(ii) 13 y + 4z = 2 x.
Solution: (i) The equation x2 + 10 z 2 = 2 y of the paraboloid may be rewritten as
( x2 + y2 + z 2 − 2 y ) + (9z 2 − y2 ) = 0. …(1)
The equation (1) will represent a surface passing through the intersection of a sphere
and a pair of planes if the equation 9z 2 − y2 = 0 represents a pair of planes. Clearly
9z 2 − y2 = 0 gives y ± 3z = 0 as two planes. Hence the real central circular sections of
the given paraboloid are y ± 3z = 0.
Hence the equations of any circular sections are
y + 3z = λ and y − 3z = µ.
(ii) Writing the given equation as
4 ( x2 + y2 + z 2 − x / 2) + (9 y2 − 4 x 2 ) = 0
and proceeding as in part (i) above the real circular sections are given by
2 x + 3 y = λ, 2 x − 3 y = µ .

Problem 13: Prove that the radius of the circle in which the plane
( x / a) √ ( a2 − b2 ) + ( z / c ) √ ( b2 − c2 ) = λ cuts the ellipsoid
 λ2 
x2 / a2 + y2 / b2 + z 2 / c 2 = 1 is b 1 −  ⋅
 a − c2 
2
G-276

Deduce the co-ordinates of the umbilics of the ellipsoid.


Solution: For the first part see Problem1 of Comprehensive Problems 4. There we
have proved that the radius r of the circular section
( x / a) √ ( a2 − b2 ) + ( z / c ) √ ( b2 − c2 ) = λ …(1)
2 2 2
is r = b √ {1 − λ /( a − c )}.
For an umbilic, the radius r of the circle is zero.
Hence if r = 0, we have λ = ± √ ( a2 − c2 ) and the equation of the plane (1) becomes
( x / a) √ ( a2 − b2 ) + ( z / c ) √ ( b2 − c2 ) = ± √ ( a2 − c2 ). …(2)
Now let (α, β, γ ) be the co-ordinates of the umbilic. The tangent plane at (α, β, γ ) to the
ellipsoid is
αx / a2 + βy / b2 + γz / c2 = 1. …(3)
The planes (2) and (3) should be the same.
α / a2 β / b2 γ / c2 1
∴ = = =
√ ( a2 − b2 ) / a 0 √ ( b − c ) / c ± √ ( a2 − c2 )
2 2

± a √ ( a2 − b 2 ) ± c √ ( b2 − c2 )
or α= , β = 0, γ = ⋅
2 2
√ (a − c ) √ ( a2 − c2 )

H ints to O bjective T ype Q uestions

Multiple Choice Questions


1. See article 3, Corollary 2.
2. The plane lx + my + nz = p is parallel to the plane lx + my + nz = 0.
We know that the sections of a conicoid by parallel planes are similar and
similarly situated conics.
So if the section by the plane lx + my + nz = 0 is a hyperbola, then the
section by the parallel plane lx + my + nz = p is also a hyperbola.
3. The plane lx + my + nz = 0 cuts the conicoid ax2 + by2 + cz 2 = 1 in a
hyperbola if bcl2 + cam2 + ab n2 < 0, in an ellipse if bcl2 + cam2 + abn2 > 0 and
in a parabola if bcl2 + cam2 + abn2 = 0. See article 2.
4. See article 2.
5. See article 2.
6. If the section of the conicoid ax2 + by2 + cz 2 = 1 by the central plane
lx + my + nz = 0 is an ellipse, then the area of the ellipse is
1 /2
 l2 + m2 + n2 
π ⋅
2 2 2
 bcl + cam + abn 
See article 3, corollary 1.
G-277

7. See article 3, equation (6).


Fill in the Blank(s)
1. The section of a conicoid by a plane passing through its centre is called a
central plane section. See article 1.
2. All plane sections of a conicoid are conics. See article 1.
3. Sections of a conicoid by parallel planes are similar and similarly situated
conics. See article 1.
4. If the section of the conicoid ax2 + by2 + cz 2 = 1 by the central plane
lx + my + nz = 0 is an ellipse whose area is A0 , then the area A of the ellipse
in which the plane lx + my + nz = p cuts the conicoid ax2 + by2 + cz 2 = 1 is
given by
 p2  l2 m2 n2
A = A0 1 −  , where p0 2 = + + ⋅
 2
 p0  a b c

See article 4, equation (15) and the value of p0 2 given just below equation (5).
5. If a2 > b2 > c2 , then the central circular sections of the ellipsoid
x2 y2 z2
+ + =1
2 2
a b c2
are given by the planes
x z
√ ( a2 − b2 ) + √ ( b2 − c2 ) = 0
a c
x z
and √ ( a2 − b2 ) − √ ( b2 − c2 ) = 0.
a c
See article 8.

True or False
1. See article 13, equation (6).
2. See article 9.
3. See article 3, Corollary 2.
4. The section of the conicoid ax2 + by2 + cz 2 = 1 by the plane
lx + my + nz = 0
is a hyperbola if bcl2 + cam2 + abn2 < 0. See article 2.
❍❍❍
G-278

Chapter-13
Reduction of General Equation
of Second Degree
Comprehensive Problems 1
Problem 1: Reduce the following equation to the standard form :
3 x2 + 5 y2 + 3z 2 + 2 yz + 2zx + 2 xy − 4 x − 8z + 5 = 0 (Kanpur 2007)
Solution: Proceed as in Example 1. Ans. 3 x2 + 2 y2 + 6z 2 = 1, ellipsoid.
Problem 2: Reduce the following equation to the standard form :
3 x2 − y2 − z 2 + 6 yz − 6 x + 6 y − 2z − 2 = 0
(Kumaun 2007; Avadh 13)
Solution: Proceed as in Example 1. Ans. 3 x2 + 2 y2 − 4z 2 = 4, hyperboloid of one
sheet.
Problem 3: Reduce the following equation to the standard form :
3 x2 + 7 y2 + 3z 2 + 10 yz − 2zx + 10 xy + 4 x − 12 y − 4z + 1 = 0
(Kanpur 2013)
Solution: Proceed as in Example 1 Ans. 3 x2 − 4 y2 − 12z 2 = 1, hyperboloid of two
sheets.
Problem 4: Reduce the following equation to the standard form :
x2 + 3 y2 + 3z 2 − 2 yz − 2 x − 2 y + 6z + 3 = 0 (Kanpur 2006, 08, 15)
Solution: Proceed as in Example 1. Ans. x2 + 2 y2 + 4z 2 = 1 ellipsoid.

Comprehensive Problems 2
Problem 1: Reduce the following equation to the standard form :
x2 + 2 yz − 4 x + 6 y + 2z = 0.
Solution: Proceed as in Example 3. Ans. x2 + y2 − z 2 = 10, hyperboloid of
revolution.
Problem 2: Show that the equation x2 + y2 + z 2 + yz + zx + xy = 1 represents an
1
ellipsoid the squares of whose semi-axes are 2, 2, ⋅ Show also that the equations of its axis of
2
revolution are x = y = z.
Solution: Multiplying by 2, the given equation becomes
2 x2 + 2 y2 + 2z 2 + 2 yz + 2 zx + 2 xy − 2 = 0.
The discriminating cubic is
 2−λ 1 1 
 1 2−λ 1 = 0
 
 1 1 2 − λ
3 2
or λ − 6λ + 9λ − 4 = 0
G-279

or ( λ − 1)2 ( λ − 4) = 0.
∴ λ = 1, 1, 4.
The centre (α, β, γ ) is given by [As in Example 3]
2α + β + γ = 0, α + 2β + γ = 0, α + β + 2γ = 0.
Solving these equations, the centre is (0, 0, 0).
Also µ = − ( uα + vβ + wγ + d) = 2, as α = β = γ = 0.
Hence the reduced equation (i. e., the equation in standard form) is
λ1 x2 + λ2 y2 + λ3 z 2 = µ
or x2 + y2 + 4z 2 = 2
x2 y2 z2
or + + =1
2 2 1
2
1
which represents an ellipsoid of revolution, the squares of whose semi-axes are 2, 2, .
2
The d.c.’s l, m, n of the axis of revolution i. e., of the principal axis corresponding to the
non-repeated root λ = 4 are given by
al + hm + gn hl + bm + fn gl + fm + cn
= = =4
l m n
2l + m + n l + 2m + n l + m + 2n
or = = =4
l m n
or − 2l + m + n = 0, l − 2m + n = 0, l + m − 2n = 0.
Solving, we have
l m n 1 1
= = = = ⋅
1 1 1 √ (12 + 12 + 12 ) √ (3)
∴ The equations of the axis of revolution are
x−0 y −0 z −0
= = i. e., x = y = z.
1 1 1
Problem 3: Discuss the nature of the following surface :
2 x2 + 5 y2 + 10 z 2 + 12 yz + 6zx + 4 xy − 1 = 0.
Solution: The given equation is
2 x2 + 5 y2 + 10 z 2 + 12 yz + 6zx + 4 xy − 1 = 0. …(1)
Compare the equation with the general equation F ( x , y , z ) = 0 of second degree, we
get
a = 2, b = 5, c = 10 , f = 6, g = 3, h = 2, u = 0, v = 0 , w = 0 , d = − 1.
The discriminating cubic is
a−λ h g
λ b−λ f =0
g f c−λ
2−λ 2 3
or 2 5 −λ 6 =0
3 6 10 − λ
or (2 − λ ){(5 − λ ) (10 − λ ) − 36} − 2 {2(10 − λ ) − 18}
+ 3 {12 − 3(5 − λ )} = 0
G-280

or (2 − λ )( λ2 − 15λ + 14) − 2(2 − 2λ ) + 3(3λ − 3) = 0


or − λ3 + 15λ2 − − 14λ + 2λ2 − 30 λ + 28 − 4 + 4λ + 9λ − 9 = 0
or − λ3 + 17λ2 − 31λ − 15 = 0
or λ3 − 17λ2 + 31λ + 15 = 0
or λ2( λ − 1) − 16λ ( λ − 1) − 15( λ − 1) = 0
or ( λ2 − 16λ − 15)( λ − 1) = 0
or ( λ2 − 15λ − λ − 15)( λ − 1) = 0
or ( λ − 1)( λ − 15)( λ − 1) = 0.
Here λ = 1, 1, 15 .
Centre (α , β , γ ) is given by the equation
1 ∂F
= 2α + 2β + 3γ + 0 = 0 …(2)
2 ∂α
1 ∂F
= 2α + 5β + 6γ + 0 = 0 …(3)
2 ∂β
1 ∂F
and = 3α + 6β + 10 γ + 0 = 0. …(4)
2 ∂γ
Solving these equations. The centre is (0 , 0 , 0 ).
Also µ = − ( uα + vβ + wγ + d)
or µ = − ( −1) = 1.
Hence the reduced equation is
λ1 x2 + λ2 y2 + λ3 z 2 = µ
or x2 + y2 + 15z 2 = 1,
which is an ellipsoid of revolution.
Let l , m, n be the d.c.’s of the principal direction corresponding to λ = 15. These are given
by
( a − λ ) l + gm + gn = 0  −13l + 2m + 3n = 0 

hl + ( b − λ ) m + fn = 0  or 2l − 19m + 6n = 0  .
gl + fm + ( c − λ )n = 0  3l + 6m − 5n = 0 
Solving any two we have
l m n
= =
12 + 30 6 + 78 130 − 4
l m n
or = =
42 84 126
l m n
or = = ⋅
1 2 3
Hence the equation of the axis through the centre (0, 0, 0) and d.c.’s are proportional to
1, 2, 3 is
x y z
= = ⋅
1 2 3
Problem 4: Discuss the nature of the following surface :
7 x2 + y2 + z 2 + 16 yz + 8zx − 8 xy + 2 x + 4 y − 40 z − 14 = 0.
Solution: Proceed as in Example 3. Ans. Here λ = 9, 9, − 9; centre is (1, 2, 0).
Reduced equation is x2 + y2 − z 2 = 1, which is a hyperboloid of revolution, its axis of
G-281

x −1 y −2 z −0
revolution being the line = = ⋅
1 2 −2
Problem 5: Discuss the nature of the following surface :
2 y2 + 4zx − 6 x − 8 y + 2 z + 5 = 0.
Solution: Proceed as in Example 3. Ans. Here λ = − 2, 2, 2. The reduced equation is
x2 = y2 + z 2 , representing a cone with its vertex (i. e., centre) as the point  − 1, 2,  , its
3
 2 
axis of revolution parallel to the line x + z = 0, y = 0.

Comprehensive Problems 3
Problem 1: Reduce to standard form the equation
x2 − y2 + 2 yz − 2zx − x − y + z = 0.
Solution: The given equation
x2 − y2 + 2 yz − 2zx − x − y + z = 0.
Comparing the given equation with the general equation F ( x , y , z ) = 0 os second
1 1 1
degree, we get a = 1, b = − 1, c = 0 , f = 1, g = − 1, h = 0 , u = − , v = − , w = , d = 0.
2 2 2
a h g 1 0 −1
We have D = h b f = 0 −1 1 = 0.
g f c −1 1 0
∴ The discriminating cubic is
λ3 − ( a + b + c ) λ2 + ( bc + ca + ab − f 2 − g2 − b2 ) λ − D = 0
or λ3 − 0 λ2 + ( −3)λ − 0 = 0
or λ3 − 3λ = 0
or λ ( λ2 − 3) = 0
or λ = 0 , λ = ± 3.
The d.c.’s l, m, n of the axis corresponding to λ = 0 are given by
al + hm + gn = 0 λl + 0 m − n = 0 …(1)
hl + bm + fn = 0 or 9l − m + 1n = 0 …(2)
gl + fm + cn = 0 − l + m + on = 0. …(3)
Solving (1) and (2), we get
l m n 1
= = = ⋅
−1 −1 −1 3
∴ K = ul + vm + wn
= −  −
1 1  1 1  1  1 
 − −  + − 
2 3 2 3 2  3
1
=− ≠ 0.
2 3
The reduced equation is
λ1 x2 + λ2 y2 + 2Kz = 0
3 x2 − 3 y2 + 2  −
1 
or  z =0
 2 3
G-282

1
3 ( x2 − y2 ) = z
3
or 3( x2 − y2 ) = z ,
which is a hyperbolic paraboloid.
Problem 2: Show that the surface whose equation is
16 x2 + 4 y2 + 4z 2 + 4 yz − 8 zx + 8 xy + 4 x + 4 y − 16z − 24 = 0
is an elliptic paraboloid. Find the co-ordinates of its vertex, and the equations to its axis.
(Kumaun 2013)
Solution: Proceed as in Example 4.
Ans. λ = 0, 18, 6; reduced equation is 6 x2 + 18 y2 − 12z = 0 i. e., x2 + 3 y2 = 2z.
x+1 y −2 z +1
Vertex is ( − 1, 2, − 1); the equations of the axis are = = ⋅
1 −2 2
Problem 3: Reduce the following equation of a hyperbolic paraboloid to standard form :
5 x2 − 16 y2 + 5z 2 + 8 yz − 14zx + 8 xy + 4 x + 20 y + 4z − 24 = 0.
Find also the co-ordinates of the vertex and the equations to its axis.
Solution: Proceed as in Example 4.
Ans. λ = 0, 12, − 18; reduced equation is 2 x2 − 3 y2 + 2z = 0, vertex is (1, 1, 1); the
x −1 y −1 z −1
equations of the axis are = = ⋅
2 1 2
Problem 4: Show that the surface whose equation is
4 x2 − y2 − z 2 + 2 yz − 8 x − 4 y + 8z − 2 = 0
is a hyperbolic paraboloid. Find its vertex and axis.
Solution: Proceed as in Example 4.
Ans. λ = 0, 4, − 2 ; reduced equation is 2 x2 − y2 + √ 2z = 0, which is a hyperbolic
paraboloid. Vertex is 1, − ,  ⋅ The equations of the axis are
2 3
 4 4 
x −1 y + (9 / 4) z − (3 / 4)
= = ⋅
0 1 1
Problem 5: Reduce to standard form the equation
6 y2 − 18 yz − 6zx + 2 xy − 9 x + 5 y − 5z + 2 = 0. Find its nature.
Solution: Proceed as in Example 4.
Ans. λ = 0, 26, − 14; reduced equation is 14 x2 − 26 y2 = 2 √(19) z, which is a
hyperbolic paraboloid.
Problem 6: Reduce to standard form the equation
4 y2 + 4z 2 + 4 yz − 2 x − 14 y − 22 z + 33 = 0. Give its vertex and axis.
Solution: Proceed as in Example 4.
Ans. λ = 0, 2, 6; reduced equation is x2 + 3 y2 = z, which is an elliptic paraboloid.
1
y−
 1 5  x −1 2 = z − (5 / 2) .
Vertex is 1, ,  ; axis is the line =
 2 2  1 0 0
Problem 7: Reduce to standard form the equation
2 x2 + 2 y2 − 4z 2 − 2 yz − 2 zx − 5 xy − 2 x − 2 y + z = 0.
Give its vertex and axis.
G-283

Solution: Proceed as in Example 4.


Ans. λ = 0, 9, − 9; reduced equation is 3 ( x2 − y2 ) = 2z, which is a hyperbolic
x y z
paraboloid. Vertex is (0, 0, 0); axis is = = ⋅
2 2 −1

Comprehensive Problems 4
Problem 1: Reduce the equation to the standard form
x2 + 6 y2 − z 2 − yz + 5 xy + 2 x + 5 y = 0.
Solution: Proceed as in Example 5. λ = 0, 6 ± √(66); the line of centres passes through
(− 1, 0, 0). The reduced equation is
{3 + √ (33 / 2)} x2 − { √ (33 / 2) − 3} y2 = 1,
which is a hyperbolic cylinder.
Problem 2: Reduce the equation to the standard form
3 x2 − 24 y2 + 8z 2 + 16 yz − 10 zx − 14 xy + 22 y + 2z − 4 = 0.
Solution: Proceed as in Example 5. λ = 0, 14, − 27; the line of centres passes through
 0, 5 , − 7  . The reduced equation is14 x2 − 27 y2 = 1, which is a hyperbolic cylinder.
 16 16 
Problem 3: Prove that
5 x2 + 5 y2 + 8z 2 + 8 yz + 8zx − 2 xy + 12 x − 12 y + 6 = 0
represents a cylinder whose cross section is an ellipse of eccentricity1 / √ 2 and find the equations to
its axis.
Solution: Here a = 5, b = 5, c = 8, f = 4, g = 4, h = − 1,
u = 6, v = − 6, w = 0, d = 6.
The discriminating cubic is
5 − λ −1 4 
 −1 5−λ 4 = 0
 
 4 4 8 − λ
or λ3 − 18λ2 + 72 λ = 0
or λ ( λ − 12) ( λ − 6) = 0.
∴ λ = 0, 12, 6.
The d.c.’s l, m, n of the axis corresponding to λ = 0 are given by
al + hm + gn = 0  5l − m + 4n = 0 …(1)
hl + bm + fn = 0  or − l + 5m + 4n = 0 …(2)

gl + fm + cn = 0  4l + 4m + 8n = 0. …(3)
Solving (1) and (2), we get
l m n
= =
− 24 − 24 24
l m n 1
or = = = ⋅ …(4)
1 1 − 1 √3
∴ k = ul + vm + wn
= (6 − 6 + 0 ) / √ 3 = 0.
G-284

Since k = 0, the given surface has no unique centre.


It has a line of centres given by any two of the following equations :
1 ∂F …(5)
= 5 x − y + 4z + 6 = 0
2 ∂x
1 ∂F
= − x + 5 y + 4z − 6 = 0 …(6)
2 ∂y
1 ∂F
= 4 x + 4 y + 8z = 0. …(7)
2 ∂z
The sum of (5) and (6) gives (7). Hence there are only two independent equations.
Assuming z = 0, from (5) and (6), we get x = − 1 , y = 1.
∴ Any centre (α, β, γ ) on the line of centres is (− 1, 1, 0).
∴ µ = − ( uα + vβ + wγ + d)
= − [6 ( − 1) + ( − 6) . 1 + 0 + 6] = 6.
Hence the reduced equation is λ1 x2 + λ2 y2 = µ
x2 y2
or 6 x2 + 12 y2 = 6 or + =1 …(8)
1 1
2
which represents an elliptic cylinder with its generators parallel to z-axis.
The cross section of (8) by the plane z = 0 is the ellipse
x2 y2
+ = 1, z = 0.
1 1
2
Its eccentricity e is given by
1
= 1 (1 − e2 ) ∵ b2 = a2 (1 − e2 ), a2 = 1, b2 = 1 
2  2 
or e = 1 / √ 2.
The equations of the axis of cylinder are given by
x+1 y −1 z
= =
1 1 −1
because the axis is a line through any centre (− 1, 1, 0) and having its d.c.’s l, m, n given by
the equations (4).
Also (as explained in Example 5) the equations of the axis in general form are given by
the equations giving the line of centres namely the equations
5 x − y + 4z + 6 = 0 = − x + 5 y + 4z − 6.
Problem 4: Discuss the nature of the surface
( cy − bz )2 + ( az − cx )2 + ( bx − ay )2 = 1.
Solution: The equation of the given surface may be rewritten as
x2 ( b2 + c2 ) + y2 ( c2 + a2 ) + z 2 ( a2 + b2 ) − 2bcyz − 2cazx
− 2abxy − 1 = 0. …(1)
Here a = b2 + c2, b = c2 + a2, c = a2 + b2, f = − bc, g = − ca, h = − ab,
u = v = w = 0, d = − 1.
The discriminating cubic is
G-285

 (b + c ) − λ
2 2 − ab − ca 
 − ab ( c2 2
+ a )− λ − bc  = 0.

 

− ca − bc ( a2 + b2 ) − λ 

Simplifying, we get
λ [ λ2 − 2λ ( a2 + b2 + c2 ) + ( a2 + b2 + c2 )2 ] = 0
or λ [ λ − ( a2 + b2 + c2 )]2 = 0.
∴ λ = 0, ( a2 + b2 + c2 ), ( a2 + b2 + c2 ).
Since two roots of the discriminating cubic are equal, the given quadric is a surface of
revolution.
The d.c.’s l, m, n of the axis corresponding to λ = 0 are given by
( b2 + c2 ) l − abm − can = 0 …(1)
− abl + ( c2 + a2 ) m − bcn = 0 …(2)
− cal − bcm + ( a2 + b2 ) n = 0. …(3)
Solving (1) and (2), we get
l m
=
ab2 c + ca ( c2 + a2 ) a2 bc + bc ( b2 + c2 )
n
=
( b2 + c2 ) ( c2 + a2) − a2 b2
l m n
or = =
ac ( b2 + c2 + a2 ) bc ( a2 + b2 + c2 ) c2 ( a2 + b2 + c2 )
l m n 1
or = = = ⋅ …(4)
a b c √ ( a2 + b2 + c2 )
∴ k = ul + vm + wn = 0.
Since k = 0, the given surface has no unique centre. It has a line of centres given by any
two of the following equations :
1 ∂F
= ( b2 + c2 ) x − aby − caz = 0 …(5)
2 ∂x
1 ∂F
= − abx + ( c2 + a2 ) y − bcz = 0 …(6)
2 ∂y
1 ∂F
= − cax − bcy + ( a2 + b2 ) z = 0. …(7)
2 ∂z
These give only two independent equations.
Assuming z = 0, (5) and (6) clearly give x = 0, y = 0.
∴ Any centre (α, β, γ ) on the line of centres is (0, 0, 0).
∴ µ = − ( uα + vβ + wγ + d) = − [0 + 0 + 0 − 1] = 1.
Hence the reduced equation is
λ1 x2 + λ2 y2 = µ
or ( a2 + b2 + c2 ) ( x2 + y2 ) = 1
which represents a right circular cylinder with its generators parallel to z-axis and having
the base radius
= 1 / √ ( a2 + b2 + c2 ).
G-286

The equations of the axis of the cylinder (as explained in Example 5) are
x−0 y −0 z −0
= =
a b c
x y z
or = = ⋅
a b c
Problem 5: Reduce to standard form and find the axis of
26 x2 + 20 y2 + 10 z 2 − 4 yz − 16zx − 36 xy + 52 x − 36 y − 16z + 25 = 0.
Solution: Proceed as in Example 5.
Ans. λ = 0, 14, 42; the line of centres passes through ( − 1, 0, 0 ). Standard form is
14 x2 + 42 y2 = 1 which is an elliptic cylinder. Axis is given by x + 1 = y = z .
Problem 6: Reduce x2 − y2 + 4 yz + 4zx − 6 x − 2 y − 8z + 5 = 0. Give the equations of
its axis.
Solution: Proceed as in Example 5.
Ans. λ = 0, 3, − 3; the line of centres passes through (1, 1, 1).
Standard form is x2 − y2 = 1, which is a hyperbolic cylinder.
x −1 y −1 z −1
The axis is = = ⋅
−2 2 1
Problem 7: Reduce to standard form
4 x2 + 3 y2 + 2z 2 + 4 yz − 4 xy − 4 x − 6 y − 8z + 6 = 0.
Also find the equations to its axis.
Solution: Proceed as in Example 5.
Ans. λ = 0, 3, 6 ; the line of centres passes through (1, 1, 1).
The reduced equation is x2 + 2 y2 = 1, which is an elliptic cylinder.
x −1 y −1 z −1
The equations to its axis are = = ⋅
1 2 −2

Comprehensive Problems 5
Problem 1: Reduce the equation to the standard form
(3 x − 4 y + z )2 + 9 x − 12 y + 3z − 10 = 0.
Solution: Write the given equation as
(3 x − 4 y + z )2 + 3 (3 x − 4 y + z ) − 10 = 0
or (3 x − 4 y + z − 2) (3 x − 4 y + z + 5) = 0.
Choose 3 x − 4 y + z = 0 as the plane X = 0, so that the reduced equation is
26 X2 + 3 √ (26) X − 10 = 0, pair of parallel planes.
Problem 2: Reduce the equation to the standard form
(3 x + 4 y + z )2 + 16 x − 27 y + 18 = 0.
Solution: Proceed as in Example 7.
Ans. 26Y 2 = 7 X, which is a parabolic cylinder.
Problem 3: Reduce the equation to the standard form
9 x2 + 4 y2 + 4z 2 + 8 yz + 12zx + 12zy + 4 x + y + 10 z + 1 = 0.
Solution: Proceed as in Example 7.
Ans. Y 2 = (7 / 17) X, which is a parabolic cylinder.
G-287

Problem 4: Reduce the equation to the standard form


(2 x − 2 y − 3z )2 + 19 x − 34 y + 50 = 0.
Solution: Proceed as in Example 7.
Ans. Y 2 = (11 / 17) X, which is a parabolic cylinder.
Problem 5: Reduce the equation to the standard form
4 x2 + y2 + 4z 2 − 4 yz + 8zx − 4 xy + 2 x − 4 y + 5z + 1 = 0.
Solution: Proceed as in Example 7.
1
Ans. Y 2 = X, which is a parabolic cylinder.
3
1
The length of the latus rectum of a normal section is ⋅
3

H ints to O bjective T ype Q uestions

Multiple Choice Questions


1. If A, B, C are all positive, then the surface represented by the equation
Ax2 + By2 − Cz 2 = 1 is a hyperboloid of one sheet.
2. The equation 4 ( x2 + y2 ) = − 9z is of the form A ( x2 + y2 ) = 2Cz.
So the surface represented by this equation is a paraboloid of revolution.
See article 1.
3. The equation 3 x2 + 4 y2 + 5z 2 = 1 is of the form Ax2 + By2 + Cz 2 = 1, where
A, B, C are all positive. Hence, the surface represented by this equation is an
ellipsoid.
4. See article 1. 5. See article 1.
6. See article 1. 7. See article 1.
8. See article 1. 9. See article 1.
10. See Problem 4 of Comprehensive Problems 4.
11. See article 1. 12. See article 1.
13. See article 2.
14. See Example 3 after article 5, Case II.
15. The equation y2 + z 2 − x2 = 1 is of the form A ( y2 + z 2 ) − Bx2 = 1.
So it represents a hyperboloid of revolution. See article 1.
16. The equation x2 + 3 y2 = − z is of the form Ax2 + By2 = 2Cz in which A and B
areboth positive. So it represents an elliptic paraboloid.
See article 1.
17. See article 1.
G-288

Fill in the Blank(s)


a − λ h g 
1. The discriminating cubic is  h b−λ f  = 0.
 
 g f c − λ

2. If in the general equation of second degree


ax2 + by2 + cz 2 + 2 fyz + 2 gzx + 2hxy + 2ux + 2vy + 2wz + d = 0,
the second degree terms form a perfect square, then it represents a parabolic
cylinder or a pair of parallel planes.
3. See article 1.
4. See article 1.
5. See article 1.

True or False
1. The equation (3 x + 4 y − 9z − 1)2 − (4 x + 5 y + 7z − 2)2 = 0 is equivalent to
(3 x + 4 y − 9z − 1 + 4 x + 5 y + 7z − 2)
(3 x + 4 y − 9z − 1 − 4 x − 5 y − 7z + 2) = 0
i. e., (7 x + 9 y − 2z − 3) ( − x − y − 16z + 1) = 0
which represents two planes
7 x + 9 y − 2z − 3 = 0 and x + y + 16z − 1 = 0
which are not parallel. Thus it represents a pair of intersecting planes.
2. Proceed as in Question 1 above of True or False.
3. The equation (3 x − 4 y + 5z − 1)2 + 4 (3 x − 4 y + 5z − 1) + 1 = 0 represents two
−4 ± √ (16 − 4)
planes given by 3 x − 4 y + 5z − 1 = ⋅
2
These two planes are parallel planes.
4. Here a = 2, b = 2, c = 0, f = 0, g = 2, h = 0, u = 1, v = −2, w = 3, d = 5.
0 − λ 0 2 
The discriminating cubic is  0 2−λ 0 = 0
 
 2 0 0 − λ
or (2 − λ )( λ2 − 4) = 0 or λ = 2, 2, − 2 .
2 2
5. The equation 4 x − 3 y = 6 represents a hyperbolic cylinder.
6. The latus rectum of the paraboloid of revolution x2 + y2 = 16z is 16.
❍❍❍

You might also like